Download as pdf or txt
Download as pdf or txt
You are on page 1of 273

ACLS

STUDY GUIDE
Barbara Aehlert, MSEd, BSPA, RN

SIXTH EDITION
Elsevier
3251 Riverport Lane
St. Louis, Missouri 63043

ACLS STUDY GUIDE, SIXTH EDITION ISBN: 978-0-323-71191-3

Copyright © 2022 by Elsevier, Inc. All rights reserved.


Previous editions copyrighted 2017, 2012, 2007, 2002.

No part of this publication may be reproduced or transmitted in any form or by any means, electronic or mechanical, includ-
ing photocopying, recording, or any information storage and retrieval system, without permission in writing from the pub-
lisher. Details on how to seek permission, as well as further information about the Publisher’s permissions policies and our
arrangements with organizations such as the Copyright Clearance Center and the Copyright Licensing Agency, can be found
at our website: www.elsevier.com/permissions.

This book and the individual contributions contained in it are protected under copyright by the Publisher (other than as may
be noted herein).

Notices

Knowledge and best practice in this field are constantly changing. As new research and experience broaden our under-
standing, changes in research methods, professional practices, or medical treatment may become necessary. Practitioners
and researchers must always rely on their own experience and knowledge in evaluating and using any information, meth-
ods, compounds or experiments described herein. Because of rapid advances in the medical sciences, in particular, inde-
pendent verification of diagnoses and drug dosages should be made. To the fullest extent of the law, no responsibility is
assumed by Elsevier, authors, editors or contributors for any injury and/or damage to persons or property as a matter of
products liability, negligence or otherwise, or from any use or operation of any methods, products, instructions, or ideas
contained in the material herein.

Library of Congress Control Number: 2021938645

Senior Content Strategist: Sandra Clarke


Senior Content Development Manager: Lisa Newton
Senior Content Development Specialist: Danielle Frazier
Publishing Services Manager: Deepthi Unni
Project Manager: Janish Ashwin Paul

Printed in India

Last digit is the print number:  9  8  7  6  5  4  3  2  1


PREFACE TO THE SIXTH EDITION
I took my first Advanced Cardiac Life Support (ACLS) class many years ago. I was terrified (and lost)
throughout the entire course. Although I spent weeks studying before the course, the information I
read seemed to be written in a foreign language. I could find no resources to “translate” the information
into something useful to me. The course consisted of very long lectures by instructors who read slides
and offered little useful insight. The most memorable part of the course was the “Patient Management”
station in which each course participant was evaluated one-on-one by an instructor. (For those of you
who have been around awhile, you are probably having flashbacks of those days). I will never forget that
experience.
Despite the time spent studying, I was a mental wreck as soon as the door closed behind me. The
instructor proceeded to methodically strip away any self-confidence I might have had in treating a
patient who had a cardiac-related emergency. I answered the questions asked of me until I was pre-
sented with a patient who had symptomatic bradycardia. Atropine had not worked (transcutaneous
pacing was not a readily available option that many years ago), and the next drug that was recom-
mended at that time was isoproterenol. I knew that I could not recall whether isoproterenol was given
in mcg/min (correct) or mg/min. I took a “50/50” guess and said mg/min. Because that was the wrong
decision, I was told I had failed the course and would need to schedule myself to attend another two-
day class.
Before driving home, I sat outside for a few minutes, contemplating what had happened and what
I could have done to change the outcome. On that day, I promised myself that I would become an
ACLS instructor someday and find a way to teach the information in a more user-friendly atmosphere.
I also promised myself that I would be a part of teaching courses useful for practicing healthcare pro-
fessionals and delivered in an environment where the participants looked forward to the class—instead
of dreading it.
As the years passed, I did become an ACLS instructor, and I loved it. After each course, participants
would often write on their course evaluations that a study guide would have helped them prepare for
class. Those suggestions resulted in writing a few pages of information and ultimately became a book—
this book.
The ACLS Study Guide is designed for paramedic, nursing, and medical students; electrocardio-
gram monitor technicians; nurses; and other allied health personnel working in emergency depart-
ments, critical care units, postanesthesia care units, operating rooms, and telemetry units preparing for
an ACLS course. This edition is based on current science, treatment recommendations, and guidelines
cited in the reference section of relevant chapters. Medicine is a dynamic field, and resuscitation
guidelines change, new medications and technology are being developed, and medical research is
ongoing. As a result, be sure to learn and follow local protocols as defined by your medical advisors.
The author and publisher assume no responsibility or liability for loss or damage resulting from using
the information contained within.
I genuinely hope you find the information in the pages that follow helpful and wish you success in
your ACLS course and clinical practice.

Sincerely,
Barbara Aehlert

iii
ACKNOWLEDGMENTS

My sincerest thanks to Danielle Frazier for her development of this text and a special thanks to all
instructors who share the same philosophy about teaching ACLS as I do.

iv
ABOUT THE AUTHOR

Barbara Aehlert, MSEd, BSPA, RN, has been a registered nurse for more than 40 years with clinical
experience in medical/surgical nursing, critical care nursing, prehospital education, and nursing educa-
tion. Barbara is an active CPR and ACLS instructor who enjoys teaching dysrhythmia recognition and
ACLS to nurses and paramedics.

v
CONTENTS

  1.  Chain of Survival and High-Quality Cardiopulmonary Resuscitation 1


Introduction 1
PhasesofCardiacArrest 2
ChainofSurvival 3
Out-of-Hospital Chain of Survival 3
In-Hospital Chain of Survival 6
High-QualityCardiopulmonaryResuscitation 7
Barriers to Effective Cardiopulmonary Resuscitation 9
Techniques and Adjuncts to Improve the Quality
of Cardiopulmonary Resuscitation 9
MonitoringDuringCardiopulmonaryResuscitation 10
Monitoring the Patient’s Response 10
Monitoring the Rescuer’s Performance 11

  2.  Teams 17
Introduction 17
RapidResponseSystems 18
TheResuscitationTeam 20
Pit Crew Model 21
Crew Resource Management 21
Team Arrival 21
Team Roles and Responsibilities 22
ElementsofEffectiveTeamPerformance 23
Teamwork 23
Task Management 24
Team Communication 24
Situation Awareness 25
Decision Making 26
Debriefing 26
FamilyNotification 27
Conveying Bad News 27
HelpingtheCaregivers 29

  3.  Patient Assessment and Resuscitation Therapies 33


Introduction 33
PartI:PatientAssessment 35
BasicLifeSupportAssessment 35
PrimaryAssessment 36
Airway 36
Breathing 37
Circulation 38
Disability 38

vi
Contents vii

Exposure 38
Vital Signs 38
SecondaryAssessment 39
PartII:ResuscitationTherapies 40
AirwayandBreathingManagement 40
Tools for Assessing Oxygenation and Ventilation 40
Manual Airway Maneuvers 40
SUCTIONING 43
Pharyngeal Airways 44
Positive Pressure Ventilation 48
Advanced Airways 54
Oxygen Delivery Devices 56
ELECTROCARDIOGRAM MONITORING 60
Conduction System 60
Leads 61
Electrocardiography Paper 63
Electrocardiogram Components 63
ElectricalTherapies 64
Defibrillation 64
Synchronized Cardioversion 68
Possible Complications 70
Transcutaneous Pacing 70
Alternative Therapies 73
VascularAccess 73
MedicationAdministration 73

  4.  The Patient With Respiratory Compromise 87


Introduction 87
RespiratoryCompromise 88
Respiratory Distress 88
Respiratory Failure 89
Respiratory Arrest 89
PatientAssessment 89
TherapeuticInterventions 90

  5.  Bradycardias 97
Introduction 97
Overview 98
RhythmReview 98
Sinus Bradycardia 98
Junctional Escape Rhythm 99
Ventricular Escape Rhythm 100
Atrioventricular Blocks 100
PatientAssessment 103
TherapeuticInterventions 104

  6.  Tachycardias 115


Introduction 115
Overview 116
Narrow-QRStachycardias 116
Sinus Tachycardia 116
Supraventricular Tachycardias 117
Wide-QRSTachycardias 121
Ventricular Tachycardia 121
viii Contents

IrregularTachycardias 122
Multifocal Atrial Tachycardia 122
Atrial Flutter 123
Atrial Fibrillation 123
Polymorphic Ventricular Tachycardia 124
PatientAssessment 125
TherapeuticInterventions 125
Narrow-QRS Tachycardias 127
Wide-QRS Tachycardias 128
Irregular Tachycardias 129

  7.  Cardiac Arrest Rhythms 145


Introduction 145
CardiacArrestRhythms 146
Ventricular Tachycardia 146
Ventricular Fibrillation 147
Asystole 147
Pulseless Electrical Activity 148
Out-of-HospitalCardiacArrest 149
In-HospitalCardiacArrest 150
TheCardiacArrestAlgorithm 150
Shockable Rhythms 150
Nonshockable Rhythms 155
Refractory Cardiac Arrest 156
Return of Spontaneous Circulation 156
Patient Transfer 156
SpecialResuscitationSituations 156
Known or Suspected Opioid Overdose 156
Known or Suspected COVID-19 157
Cardiac Arrest and Pregnancy 157
PostcardiacArrestCare 158
Oxygenation and Ventilation 158
Cardiovascular Care 159
Neurologic Care 162
Other Considerations 162

  8.  Acute Coronary Syndromes 173


Introduction 173
CoronaryHeartDisease 174
Obstructive Atherosclerosis With Systemic Inflammation 176
Obstructive Atherosclerosis Without Systemic Inflammation 177
Nonobstructive Atherosclerosis 177
IschemicHeartDisease 178
Clinical Features 178
Diagnostic Studies 178
Stable Angina 179
Variant Angina 180
Microvascular Angina 180
AcuteCoronarySyndromes 181
Clinical Features 182
Diagnosis 185
Unstable Angina and Non–ST-Elevation Myocardial Infarction 187
ST-Elevation Myocardial Infarction 187
Imaging Studies 198
InitialManagementofAcuteCoronarySyndromes 198
Prehospital Management 198
Contents ix

Emergency Department Management 199


Pharmacologic Therapies 201
Reperfusion Therapies 207

  9.  Acute Ischemic Stroke 219


Introduction 219
AnatomyReview 220
StrokeTypes 220
Subarachnoid Hemorrhage 221
Intracerebral Hemorrhage 222
Ischemic Stroke 223
Transient Ischemic Attack 224
StrokeSystemsofCare 224
Stroke Centers 225
Public Education 226
Emergency Medical Services 226
Hospital Initial Evaluation and Treatment 227

10.  Posttest 239


Questions1through7pertaintothefollowingscenario. 239
Questions8through21pertaintothefollowingscenario. 240
Questions22through26pertaintothefollowingscenario. 243
Questions27through43pertaintothefollowingscenario. 244
Questions44through46pertaintothefollowingscenario. 247
Questions47and48pertaintothefollowingscenario. 248
Questions49and50pertaintothefollowingscenario. 248

Glossary 255
Index 257
This page intentionally left blank
CHAPTER 1
Chain of Survival and
High-Quality Cardiopulmonary
Resuscitation

INTRODUCTION
Heart disease is the leading cause of death for both men and women in the United States (Benjamin
et al., 2019) (Fig. 1.1). As a result, the likelihood of encountering a patient who requires basic life
support (BLS) or advanced cardiac life support (ACLS) care is high. ACLS is an orderly approach
to providing advanced emergency care to a patient who is experiencing a cardiac-related problem.
This chapter discusses the phases of cardiac arrest, the Chain of Survival, the elements of high-quality
cardiac pulmonary resuscitation (CPR), common barriers to effective CPR, techniques and adjuncts
to improve the quality of CPR, and monitoring the patient’s response and rescuer performance
during CPR.

1
Fig. 1.1  ​Heart disease is the leading cause of death for men and women in the United States. (From istock.com.) 1
2 CHAPTER 1  Chain of Survival and High-Quality Cardiopulmonary Resuscitation

LEARNING OBJECTIVES
After completing this chapter, you should be able to:
1. Define cardiovascular collapse, cardiac arrest, sudden cardiac death, and sudden
cardiac arrest.
2. Discuss the phases of a cardiac arrest.
3. Describe the links in the Chain of Survival.
4. Discuss the elements of high-quality CPR.
5. Consider common barriers to effective CPR and possible actions that can be taken to
overcome them.
6. Explore the use of continuous end-tidal carbon dioxide (EtCO2) monitoring during
resuscitation efforts.
7. Examine the use of feedback devices during CPR.

LEARNING PLAN
• Whether you are preparing for your first ACLS course or your 10th, schedule time to
study and review before the course. Studying in half-hour intervals with 10-minute
breaks allows a reasonable period for both learning and relaxation.
• Read this chapter before class. Take the time to highlight essential concepts as you
read.
• Develop and use flashcards, flowcharts, and mnemonics to help enhance your
retention of the information presented.
• Complete the chapter quiz and review the quiz answers provided.

KEY TERMS
Automated external defibrillator (AED)  A machine with a sophisticated computer
system that analyzes a patient’s heart rhythm using an algorithm to distinguish
shockable rhythms from nonshockable rhythms and provide visual and auditory
instructions to the rescuer to deliver an electrical shock if a shock is indicated.
Cardiopulmonary (cardiac) arrest  The absence of cardiac mechanical activity, which is
confirmed by the absence of a detectable pulse, unresponsiveness, and apnea or
agonal, gasping breathing.
Cardiovascular collapse  A sudden loss of effective blood flow that is caused by cardiac
and/or peripheral vascular factors that may reverse spontaneously (e.g., syncope) or
require interventions (e.g., cardiac arrest).
Chain of Survival  The essential elements of a system of care that are necessary to link
the victim of sudden cardiac arrest with survival.
Sudden cardiac death (SCD)  A natural death of cardiac cause that is preceded by an
abrupt loss of consciousness within 1 hour of the onset of an acute change in
cardiovascular status; sudden cardiac arrest is a term commonly applied to such an
event when the patient survives.

PHASES OF CARDIAC ARREST


Cardiovascular collapse is a sudden loss of effective blood flow that is caused by cardiac factors,
peripheral vascular factors, or both, that may reverse spontaneously (e.g., syncope) or require interven-
tions (e.g., cardiac arrest) (Myerburg & Goldberger, 2019). Cardiopulmonary (cardiac) arrest is
the absence of cardiac mechanical activity, which is confirmed by the absence of a detectable pulse,
unresponsiveness, and apnea or agonal, gasping breathing. Gasping is abnormal breathing, is common
during the first few minutes of primary cardiac arrest, and is a sign of adequate blood flow to the
CHAPTER 1  Chain of Survival and High-Quality Cardiopulmonary Resuscitation 3

TABLE 1.1 Phases of Cardiac Arrest and Resuscitation


Phase Interval Focus of Care
Prearrest Period before the arrest Identify, anticipate, and manage factors that may result ​
in cardiac arrest (e.g., use of rapid response teams to
recognize and treat patients at risk of deterioration)
No flow Untreated cardiac arrest Prompt initiation of BLS upon recognition of the arrest by ​
a bystander or healthcare professional
Low flow Onset of CPR Delivery of high-quality chest compressions to optimize
myocardial and cerebral perfusion; consider ECMO for
selected patients if conventional CPR is not promptly
successful
Postresuscitation Return of spontaneous ​ Identify and treat the cause of the arrest, optimize cardiac
circulation output and cerebral blood flow, support end-organ ​
perfusion and function

BLS, Basic life support; CPR, cardiopulmonary resuscitation; ECMO, extracorporeal membrane oxygenation.

brainstem (Ewy, 2012). Respiratory efforts can persist for 1 minute or longer after the onset of a
cardiac arrest (Myerburg & Goldberger, 2019).
It is estimated that one of every 7.5 people in the United States will die of sudden cardiac death
(SCD) (Benjamin et al., 2019). SCD is a natural death of cardiac cause that is preceded by an abrupt
loss of consciousness within 1 hour of the onset of an acute change in cardiovascular status (Myerburg
& Goldberger, 2019). After the heart stops, reestablishing circulation as quickly as possible is critical
because survival decreases by about 10% per minute (Institute of Medicine et al., 2015). The phrase
sudden cardiac arrest is used when the patient survives such an event. Four phases of cardiac arrest have
been described, each with unique physiology and treatment strategies (Kilbaugh et al., 2020; Topijan
et al., 2013) (Table 1.1).

CHAIN OF SURVIVAL
The Chain of Survival represents the essential elements of a system of care that are necessary to link the
victim of sudden cardiac arrest with survival. Links of the Chain depict the interrelated steps needed in
an adult cardiac arrest both outside and inside the hospital setting. Two separate chains exist because there
are differences in these systems of care. Time is critical when dealing with a victim of sudden cardiac
arrest; a weak or missing link in either Chain of Survival can reduce the likelihood of a positive outcome.

Out-of-Hospital Chain of Survival


The links in the out-of-hospital Chain of Survival for adults include early recognition, emergency re-
sponse activation, and prevention; delivery of high-quality CPR; rapid defibrillation; effective advanced
life support (ALS); postcardiac arrest care; and recovery.

Early Recognition, Emergency Response Activation, and Prevention


The first link in the out-of-hospital Chain of Survival is recognition of the event and activation of the
emergency medical services system (EMSS). When a cardiac emergency occurs, the patient (or a fam-
ily member or bystander) must identify their signs and symptoms, recognize that they are related to a
heart condition, and seek medical assistance in the hope of preventing cardiac arrest. Because delays in
seeking assistance and delays in the arrival of assistance affect patient outcomes, prevention measures
include increasing public awareness in recognizing the signs and symptoms of acute coronary syn-
dromes and cardiac arrest (Berg et al., 2020).
Emergency dispatchers, who are located at public service access points, are the link between the call
for help and the arrival of medical assistance. Dispatchers are trained to recognize the caller’s descrip-
tion of a potential heart attack or cardiac arrest and to provide real-time CPR instructions over
the phone if necessary while quickly sending appropriately trained and equipped emergency medical
4 CHAPTER 1  Chain of Survival and High-Quality Cardiopulmonary Resuscitation

services (EMS) personnel to the scene. Dispatcher-assisted CPR is also referred to as dispatcher-
assisted bystander CPR (DA-CPR), just-in-time instruction, and telecommunicator CPR (TCPR).
Research has shown that the availability of DA-CPR is associated with a greater than fivefold increase
in the likelihood of bystander CPR (Panchal et al., 2019).
To minimize the risk for transmission of coronavirus disease 2019 (COVID-19), dispatchers should
query callers to determine if the patient has signs or symptoms (e.g., fever, cough, shortness of breath) or
risk factors for COVID-19 (e.g., known infection in the victim or any recent contacts, including family
members). When determining the patient’s address, dispatchers should determine if the address been
identified as a known COVID-19 location. When CPR is indicated, dispatchers should provide lay rescu-
ers with information about the risk of exposure to COVID-19 and instructions for providing compression-
only CPR (Edelson et al., 2020). If there is a suspicion for COVID-19 infection, dispatchers should alert
responding EMS personnel to enable the use of appropriate personal protective equipment (PPE).
Bystander CPR can markedly improve cardiac arrest survival rates, neurologic outcomes, and the result-
ing quality of life (Institute of Medicine et al., 2015). Strategies, such as placing automated external defibril-
lators (AEDs) in public locations, have been used to increase bystander response to cardiac arrests. Despite
such efforts, barriers to bystander response persist and can be divided into four categories: (1) an inability to
recognize an out-of-hospital cardiac arrest (OHCA) followed by delayed activation of EMS, (2) a lack of
adequate CPR training, (3) concerns about possible harm when delivering CPR and possible subsequent
liability, and (4) psychologic factors, rescuer confusion, and health concerns (Institute of Medicine et al.,
2015).

ACLS Pearl
Performing CPR, providing positive pressure ventilation, and inserting an advanced airway are
examples of aerosol-generating procedures that can expose healthcare workers and lay rescuers
to pathogens that cause acute respiratory infections (Tran et al., 2012). The fear of contracting an
infectious respiratory disease is often a factor in a bystander’s reluctance to provide CPR. This
fear is increased when a lay rescuer or healthcare worker does not have access to adequate PPE,
increasing their exposure risk. Before performing CPR, all rescuers should wear appropriate PPE
to limit exposure to airborne and droplet particles (Edelson et al., 2020).

Bystanders may have difficulty recognizing the signs of cardiac arrest. For example, abnormal
breathing, such as gasping, can continue for several minutes after a cardiac arrest and may be confused
with difficulty breathing (or even interpreted by some as adequate breathing), resulting in miscom-
munication with dispatchers and delays in the start of CPR (Berdowski et al., 2009). Research shows
that agonal breathing is common and present in about 40% to 60% of OHCA victims (Panchal et al.,
2020). Oxygen depletion that accompanies cardiac arrest can result in anoxic seizures. Bystanders may
confuse the resulting brief seizure-like activity (i.e., shaking) with seizures, resulting in delays in cardiac
arrest recognition by bystanders and actions taken by dispatchers (Clawson et al., 2008; Institute of
Medicine et al., 2015).
In some areas, public safety agencies are using mobile apps with increasing frequency to summon
nearby volunteers to the scene of a cardiac arrest in a public location to provide bystander CPR until
the arrival of EMS providers. In addition to providing the victim’s location, some apps also provide the
location of nearby AEDs.

Delivery of High-Quality Cardiopulmonary Resuscitation


After recognizing that an emergency exists, assess the scene to ensure that it is safe to enter. If dispatch
information indicates that the patient has a possible respiratory infection, be sure to don appropriate
PPE before entering the scene. If the scene is safe, quickly assess the patient for life-threatening
conditions and determine the nature of the emergency.
CPR is a part of BLS. BLS includes the recognition of signs of cardiac arrest, heart attack, stroke,
and choking; the relief of choking; CPR; and defibrillation with an AED. Provide BLS until advanced
medical help arrives, and the providers assume responsibility for the patient’s care. Emergency inter-
ventions may include the following:
• Patient positioning
• CPR for victims of cardiac arrest
• Defibrillation with an AED
CHAPTER 1  Chain of Survival and High-Quality Cardiopulmonary Resuscitation 5

• Rescue breathing for victims of respiratory arrest


• Recognition and relief of choking
• Early administration of naloxone for opioid-associated respiratory arrest

Rapid Defibrillation
When an individual experiences a cardiac arrest, the likelihood of successful resuscitation is affected by
the speed with which CPR and defibrillation are performed. The goal for providing the first shock for
sudden cardiac arrest resulting from ventricular fibrillation (VF) or pulseless ventricular tachycardia
(pVT) is within 2 minutes of collapse (Institute of Medicine et al., 2015).
The American Heart Association has promoted the development of AED programs to improve
survival from sudden cardiac arrest since 1995. An AED is a machine with a sophisticated computer
system that analyzes the patient’s heart rhythm. The AED uses an algorithm to distinguish shockable
rhythms from nonshockable rhythms. If the AED detects a shockable rhythm, it provides visual and
auditory instructions to the rescuer to deliver an electrical shock. Defibrillation performed by citizens
(such as flight attendants, casino security officers, athletic or golf club employees, and ushers at sport-
ing events) at the scene is called public access defibrillation. Defibrillation is discussed in more detail
in Chapter 3.
The use of an “ambulance drone” is an innovative approach being used to reduce the interval from
patient collapse to defibrillation time (Institute of Medicine et al., 2015). This strategy uses a drone
equipped with a camera, microphone, two-way speakers, AED, and a built-in global positioning system
(GPS) (Fig. 1.2). The drone can deliver the AED within 4.6 square miles, using GPS coordinates, in
less than 1 minute. Once the drone is at the scene, a “remote” operator can provide instructions to those
at the scene with the help of the drone’s cameras and speakers.

Effective Advanced Life Support


Outside the hospital, early advanced care is provided by paramedics (and/or nurses) arriving on the
scene. Prehospital providers work quickly to stabilize the patient by providing ventilation support,
vascular access, and giving emergency medications, among other interventions.

Postcardiac Arrest Care


Prehospital providers transport the patient for definitive care. Initiate contact with the receiving hospital
as early as possible when transporting a patient with known or suspected COVID-19.
Some studies suggest the postarrest patient should be transported to the closest most appropriate
emergency department (ED) for initial stabilization with subsequent transfer to a higher acuity care
facility for further care. Other studies suggest that postarrest patients should be triaged in the field and
transported directly to specialized centers that offer invasive coronary care to ensure the best possible
outcome (Geller & Abella, 2018; Institute of Medicine et al., 2015). Terms used to identify these spe-
cialized centers include cardiac arrest centers (CACs), cardiac arrest receiving centers, comprehensive

Fig. 1.2  ​A drone carrying an automated external defibrillator can reduce the interval from patient collapse to defibrillation
time. (From istock.com.)
6 CHAPTER 1  Chain of Survival and High-Quality Cardiopulmonary Resuscitation

cardiac centers, and cardiac resuscitation centers. Current guidelines consider the transport of resusci-
tated patients directly to specialized CACs to be reasonable when comprehensive postcardiac arrest
care is not available at local facilities (Berg et al., 2020).

ACLS Pearl
If return of spontaneous circulation (ROSC) has not been achieved after appropriate resuscitation
efforts in the field for a patient with known or suspected COVID-19, experts advise considering not
transferring the patient to the hospital because of the low likelihood of patient survival, balanced
against the added risk of additional exposure to EMS and hospital providers (Edelson et al., 2020).

Several progressive EMS systems are collaborating with hospital systems to deliver evidence-based
bundles of care designed to improve the ROSC and survival to hospital discharge. Components of the
bundles of care vary; examples include implementation of a high-quality DA-CPR program, use of
defibrillators equipped with feedback functions, minimally interrupted chest compressions, passive
ventilation, delayed advanced airway insertion, or use of mechanical CPR devices, among many other
possibilities. A bundle of care approach that has resulted in improved ROSC rates implemented by
Palm Beach County Fire Rescue in Florida includes passive ventilation, delayed positive pressure ven-
tilation, use of an impedance threshold device, a focus on continuous chest compressions with an early
transition to CPR using a mechanical chest compression device, and patient transport with the head
and thorax in an elevated position to allow a drop in intracranial pressure and a potential improvement
in cerebral perfusion (Scheppke, 2019).

Recovery
Recovery has been added to the Chain of Survival to acknowledge the importance of supporting
survivors and their caregivers during short- and long-term rehabilitation after a cardiac arrest.

In-Hospital Chain of Survival


The links in the in-hospital Chain of Survival for adults include early recognition and prevention of cardiac
arrest, prompt notification and response when a cardiac arrest occurs, the performance of high-quality
CPR, prompt defibrillation, intraarrest and postcardiac arrest care, and recovery (Panchal et al., 2020).

ACLS Pearl
Studies show that survival after in-hospital cardiac arrest is lower during nights and weekends
compared with daytime during weekdays (Ofoma et al., 2018).

Early Recognition and Prevention


A cardiac arrest experienced by a hospitalized adult is often preceded by signs and symptoms that
suggest physiologic deterioration, such as abnormal vital signs (Andersen et al., 2016; Douw et al.,
2015; Lyons et al., 2018). The concept of a Rapid Response System emerged after recognizing that
early detection and treatment of the patient who demonstrates signs of clinical deterioration may
prevent cardiac arrest and improve patient outcomes. Chapter 2 discusses these systems in more detail.

Notification and Response


Every member of the hospital staff should know how to recognize a cardiac arrest and know how to
summon assistance when such an event occurs. Prompt notification and activation of the code team may
include pressing a “code button” at the patient’s bedside, calling a specific phone extension, or use of a
“quick-dial button” located on telephones within the facility. The type of emergency and its location are
stated when the operator is reached. Once the operator is notified of the emergency, members of the
code team typically are activated using cell phones, pagers, and/or a hospital-wide public address system.

Performance of High-Quality Cardiopulmonary Resuscitation


Although cardiac arrests and the performance of CPR are relatively uncommon in in-hospital environ-
ments (Kronick et al., 2015), hospital staff must be able to perform high-quality CPR immediately
CHAPTER 1  Chain of Survival and High-Quality Cardiopulmonary Resuscitation 7

upon recognition of a cardiac arrest. Because training may not be adequate to ensure optimal perfor-
mance, strategies such as timely access to equipment, visual reminders, regular testing, and point-
of-care feedback have been suggested as methods to improve the translation of resuscitation guidelines
into practice during cardiac arrest (Morrison et al., 2013).

Prompt Defibrillation
It is estimated that about half of all in-hospital cardiac arrests (IHCAs) occur outside the intensive care
unit (ICU) (Morrison et al., 2013). Because it can take several minutes for code team members to ar-
rive with a defibrillator, the strategic deployment of AEDs throughout the facility can aid in achieving
prompt defibrillation, with the goal being the delivery of the first shock within 3 minutes of patient
collapse (Link et al., 2010).

Intraarrest and Postcardiac Arrest Care


During the arrest, and under the direction of a team leader, the code team works to stabilize the patient
by continuing high-quality CPR, performing defibrillation for pVT/VF, obtaining vascular access
and giving medications, performing advanced airway management procedures when warranted, and
providing ventilation support, among other interventions.
If a ROSC is achieved, postcardiac arrest care, including advanced monitoring and targeted tem-
perature management, is provided by a multidisciplinary team in an ICU. After a resuscitation effort,
a debriefing of the resuscitation team is recommended to discuss areas such as psychomotor skill issues,
cognitive issues, team issues, family emotional issues, and professional staff emotional issues (Kronick
et al., 2015).

Recovery
The recovery link recognizes the importance of providing effective support for the physical, cognitive,
and emotional needs of cardiac arrest survivors and their caregivers as the patient transitions from
hospital care to home and returns to their role and social function (Berg et al., 2020; Panchal et al., 2020).

HIGH-QUALITY CARDIOPULMONARY
RESUSCITATION
The terms high-quality CPR and high-performance CPR are often used interchangeably. In its 2015
report, Strategies to Improve Cardiac Arrest Survival: A Time to Act, the Institute of Medicine explained
the difference between these terms. “High-quality CPR is CPR that meets performance standards along
a number of fundamental measures identified as clinically relevant in national and international guide-
lines. High-performance CPR emphasizes the importance of team-related factors (e.g., communication,
collaboration, teamwork, and leadership) in attaining high-quality CPR, as well as other aspects of
performance that affect patient outcomes. Although the scientific literature is inconsistent in its use of
these terms, the committee believes this distinction is useful for highlighting the different skillsets req-
uisite to the performance of effective CPR. Optimally, these two concepts will be employed in tandem
by any resuscitation team” (Institute of Medicine et al., 2015, pp. 208-209).

ACLS Pearl
Even when CPR is delivered following recommended resuscitation guidelines, experts say that it
provides only 10% to 30% of normal blood flow to the heart and 30% to 40% of normal blood
flow to the brain (Meaney et al., 2013).

After recognizing that CPR is indicated, the patient should be positioned supine on a firm, flat
surface when feasible, and chest compressions should be started. If the patient cannot be placed in the
supine position, current guidelines note that it may be reasonable for rescuers to provide CPR with the
victim in the prone position, particularly in hospitalized patients with an advanced airway in place
(Panchal et al., 2020).
High-quality CPR consists of several elements, which are shown in Box 1.1. When chest compres-
sions are stopped during cardiac arrest, no blood flow is generated; this is referred to as no flow time.
8 CHAPTER 1  Chain of Survival and High-Quality Cardiopulmonary Resuscitation

BOX 1.1 Elements of High-Quality Cardiopulmonary Resuscitation


• Maximize the chest compression fraction (i.e., at least 60%) by minimizing pauses in chest
compressions (keep pauses to 10 seconds or less)
• Perform chest compressions at a rate of 100 to 120 compressions per minute
• Compress the adult chest at a depth of at least 2 inches (5 cm), not to exceed 2.4 inches
(6 cm)
• Avoid leaning on the chest between compressions to allow full chest wall recoil
• To avoid fatigue, change chest compressors about every 2 minutes or earlier if warranted
(change should take about 5 seconds)
• Avoid excessive ventilation

As a result, cerebral and coronary perfusion pressures fall quickly and dramatically, thereby reducing
blood flow to the brain and heart. Even after compressions are resumed, several chest compressions are
needed to restore coronary perfusion pressure. Research shows that an increased chest compression
fraction (CCF), which is the proportion of time spent performing chest compressions during a cardiac
arrest, is predictive of improved survival. Although a 2013 consensus statement about CPR quality
recommended a CCF of at least 80% for optimal outcomes (Meaney et al., 2013), current resuscitation
guidelines note that in adult cardiac arrest, it may be reasonable to perform CPR with a CCF of
at least 60% (Panchal et al., 2020). Interruptions during compression delivery resulting from pauses
during vascular access procedures, intubation, compression provider switching, and pauses before the
delivery of a shock can significantly decrease CCF (Bhardwaj & Abella, 2015). In adult patients in
cardiac arrest with a shockable rhythm, research shows that perishock pauses (i.e., pauses in chest
compressions before and after a defibrillatory shock) of 40 seconds or longer are associated with
decreased odds of survival as compared with pauses of 20 seconds or less in duration (Cheskes et al.,
2011).

ACLS Pearl
Interruptions in chest compressions must be kept to a minimum to maximize the amount of time
that chest compressions generate blood flow during a cardiac arrest. When pauses in CPR are
necessary (e.g., defibrillation, rotation of rescuers performing chest compressions, placement of
a backboard), keep them to less than 10 seconds if possible. In some areas, when there is any
pause in CPR for a pulse check, the rescuer performing chest compressions begins a 10-second
countdown and resumes CPR automatically unless it is announced that a pulse is detected
(Hopkins et al., 2016).

High-quality chest compressions require pushing hard and fast on the victim’s chest, which means
compressing the adult chest a depth of at least 2 inches (5 cm), not to exceed 2.4 inches (6 cm) at a
rate of 100 to 120 compressions per minute. During the compression (systolic) phase of chest compres-
sion, the compressions must be of sufficient depth to provide adequate stroke volume and cerebral
perfusion pressure (Benner et al., 2011). During the release (diastolic) phase of chest compression,
intrathoracic pressure is low, which helps increase the return of venous blood into the chest. If intra-
thoracic pressure is too high, venous return is inhibited.
During a cardiac arrest, be mindful of the rate and volume used when ventilating the patient. Ven-
tilating the patient too fast or with too much volume is common during resuscitation efforts and results
in excessive intrathoracic pressure, which results in decreased venous return into the chest, decreased
coronary and cerebral perfusion pressures, diminished cardiac output, and decreased rates of survival
(Meaney et al., 2013; O’Neill & Deakin, 2007).

ACLS Pearl
It is important to ventilate a patient in cardiac arrest with just enough volume to see the patient’s
chest rise gently.
CHAPTER 1  Chain of Survival and High-Quality Cardiopulmonary Resuscitation 9

Barriers to Effective Cardiopulmonary Resuscitation


Several studies have shown that the quality of CPR during actual resuscitation often falls short of
established resuscitation guidelines in both out-of-hospital and in-hospital settings. Research indicates
that compressions are often not continued for long-enough periods during resuscitation attempts.
Compressions administered are often too slow, too fast, or too shallow and have excessively long pauses
around defibrillation attempts, and ventilations are too fast (Institute of Medicine et al., 2015). As a
result, providers often fall short of delivering CPR within target ranges for chest compression depth,
rate, recoil, and chest compression fraction during both simulated and real cardiac arrest events
(Anderson et al., 2019). Possible factors influencing these deficiencies include infrequent training,
lack of awareness of the quality of CPR during resuscitation, and ineffective team leadership during
resuscitation efforts (Abella & Becker, 2019).
Rescuer fatigue has been identified as a significant potential contributor to poor CPR quality
(Brooks et al., 2014). Rescuer fatigue contributes to insufficient depth of compressions, compro-
mises coronary perfusion pressure, and leads to inadequate chest recoil (Reynolds et al., 2012).
Research has shown that the depth of compressions is compromised after just 1 minute of perform-
ing CPR (Hightower et al., 1995; Zhang et al., 2013), and rescuers tend not to recognize their
fatigue until after about 5 minutes of CPR (Reynolds et al., 2012). To minimize fatigue, rescuers
delivering chest compressions should rotate every 2 minutes. Ideally, the switch should be accom-
plished in less than 5 seconds and should be done while another intervention is being performed
(e.g., defibrillation).
The brain and heart are sensitive to ischemic injury. Because it takes time to build up cerebral and
coronary perfusion pressures, even short pauses (4 to 5 seconds) in chest compressions have resulted in
a dramatic drop-off in cerebral and coronary perfusion pressures, thereby reducing blood flow to the
brain and heart (Ewy, 2005; Wik et al., 2005). When chest compressions are stopped during cardiac
arrest, no blood flow is generated. Even after compressions are resumed, several chest compressions are
needed to restore coronary perfusion pressure.
It is essential to allow the chest wall to rebound to its normal position after each compression. In-
complete chest wall recoil is common when performing CPR, especially when rescuers are fatigued,
and can occur when a rescuer leans over the patient’s chest, hindering full chest expansion (Meaney et
al., 2013). Incomplete recoil results in higher intrathoracic pressure and decreased coronary perfusion
pressure, decreased myocardial blood flow, decreased cerebral perfusion, and decreased cardiac output
(Rajab et al., 2011; Reynolds et al., 2012).

Techniques and Adjuncts to Improve the Quality


of Cardiopulmonary Resuscitation
Several techniques and adjuncts have been studied as alternatives to standard CPR. Active compres-
sion-decompression CPR (ACD-CPR) is performed using a handheld or mechanical suction device
positioned midsternum on the patient. The device actively lifts the patient’s anterior chest during
the recoil phase of CPR. The negative intrathoracic pressure that results enhances venous return to
the heart.
An impedance threshold device (ITD) is a simple tool that is placed between a face mask and a
ventilation bag or between an advanced airway and a ventilation bag. The ITD limits air entry into the
lungs during the recoil phase of chest compressions without hindering active ventilation. The negative
intrathoracic pressure generated improves venous return to the right heart and cardiac output with
subsequent chest compressions. The ITD is removed when ROSC is achieved. Current resuscitation
guidelines observe that the combination of ACD-CPR and the use of an ITD may be a reasonable
alternative to standard CPR in settings with available equipment and adequately trained personnel;
however, the routine use of the ITD as an adjunct during conventional CPR is not recommended
(Panchal et al., 2020).
The use of mechanical chest compression devices has been proposed as an alternative to manual
compressions to improve compression depth, rate, and consistency. There are conflicting data
about whether mechanical chest compression devices perform high-quality CPR, including com-
pression rate, depth, and overall chest compression fraction, more reliably than humans (Nordeen,
2018). Researchers note that the optimum time during a cardiac arrest at which a mechanical
10 CHAPTER 1  Chain of Survival and High-Quality Cardiopulmonary Resuscitation

device should be deployed is uncertain, particularly in patients who have a shockable rhythm
(Poole et al., 2018). Some studies show that the quality of mechanical compressions can be better
than human compressions once the device has been appropriately applied; however, consideration
must be given to the fact that application of the device requires interruptions in the performance
of chest compressions, which could be harmful if the hands-off time is prolonged (Nordeen,
2018).
Although manual chest compressions remain the standard of care for the treatment of cardiac arrest,
the use of mechanical chest compression devices may be a reasonable alternative for use by properly
trained personnel and “may be considered in specific settings where the delivery of high-quality
manual compressions may be challenging or dangerous for the provider (e.g., limited rescuers available,
prolonged CPR, during hypothermic cardiac arrest, in a moving ambulance, in the angiography suite,
during preparation for extracorporeal CPR), provided that rescuers strictly limit interruptions in CPR
during deployment and removal of the devices” (Panchal et al., 2020). When mechanical devices
are used, training should be provided to reduce the time needed for device deployment and also stress
the importance of minimizing interruptions in chest compressions while the device is in use (Brooks
et al., 2014).

ACLS Pearl
To reduce rescuer exposure during the resuscitation of an adult or adolescent with known or sus-
pected COVID-19, consider replacing manual chest compressions with a mechanical CPR device
if the patient meets the manufacturer’s height and weight criteria, and protocols and expertise are
in place for the use of such devices (Edelson et al., 2020).

MONITORING DURING CARDIOPULMONARY


RESUSCITATION
Experts recognize that the quality of CPR delivered is just as critical to survival from cardiac arrest
as early recognition of the event and immediate activation of the emergency response system (Meaney
et al., 2013). Monitoring of CPR quality includes an assessment of the patient’s response and rescuer
performance.

Monitoring the Patient’s Response


After inserting an advanced airway, use continuous EtCO2 monitoring to monitor the quality of com-
pressions during resuscitation efforts (Fig. 1.3). EtCO2 falls sharply with the onset of cardiac arrest,
increases when effective CPR is delivered (generally 10 to 20 millimeters of mercury [mm Hg]), and
returns to physiologic levels (35 to 45 mm Hg) with the ROSC (Abella et al., 2019). Low EtCO2
values (i.e., less than 10 mm Hg) during resuscitation efforts indicate the need to explore factors that
are hindering effective CPR (e.g., rescuer fatigue, cardiac tamponade, pneumothorax, bronchospasm,
mucus plugging of the endotracheal tube [ETT], kinking of the ETT, alveolar fluid in the ETT, an
airway with an air leak, hyperventilation) (Kodali & Urman, 2014; Link et al., 2015). Experts note that
failure to maintain EtCO2 at levels higher than 10 mm Hg during adult CPR reflects poor cardiac
output and strongly predicts unsuccessful resuscitation (Link et al., 2015; Meaney et al., 2013).
As the chest compressor tires, a gradual decrease in waveform height can be observed on the
monitor screen, indicating the need to change rescuer positions. A sudden sustained increase in
EtCO2 during CPR is an indicator of ROSC. In addition to improving the quality of CPR delivered,
EtCO2 monitoring allows clinicians to perform chest compressions without pausing for pulse checks
unless a sudden increase in EtCO2 is observed, at which time ROSC can be verified (Cunningham
et al., 2012).
In the hospital setting, if arterial and venous catheters are in place at the time of an adult cardiac
arrest, arterial waveforms can be used to assess the adequacy of chest compressions during the resuscita-
tion effort. Experts observe that an adult resuscitation is more likely to be successful when coronary
perfusion pressure is greater than 20 mm Hg and when the diastolic pressure is greater than 25 mm Hg
(Meaney et al., 2013).
CHAPTER 1  Chain of Survival and High-Quality Cardiopulmonary Resuscitation 11

ET CO2
40

CO2 mm Hg

0
Exhalation Inhalation
A Time (sec)

0
Compressions
(mm)

30

60

50
mm Hg
CO2

25

0
B
Fig. 1.3  ​Waveform capnography during cardiac arrest. (A) End-tidal carbon dioxide partial pressure (PETCO2) diagram
showing a typical ventilation cycle and CO2 waveform. The point that represents PETCO2 is marked with an arrow. (B) PETCO2
recording during cardiopulmonary resuscitation. This image demonstrates the use of capnography during ongoing resusci-
tation. The chest compression waveform is shown in red (top panel), and the PETCO2 waveform is shown in blue (bottom
panel). EtCO2, End-tidal CO2.  (From Roberts, J.R., Custalow, C.B., & Thomsen, T.W. [2019]. Roberts and Hedges’ clinical
procedures in emergency medicine and acute care, ed 7, Philadelphia, Elsevier.)

Monitoring the Rescuer’s Performance


Feedback devices measure aspects of CPR performance that affect CPR quality (e.g., chest compres-
sion rate, depth, hand position, chest wall recoil, chest compression fraction). The measured data is then
relayed to the rescuer by means of audio or visual feedback (or both), enabling the rescuer to make
immediate self-corrections to their performance. Some feedback devices are stand-alone systems that
are applied to the patient’s chest, some are incorporated into defibrillators, and others use apps on a
wearable product, such as a smartwatch.
With some feedback-enabled defibrillators, audible voice prompts and visual messages on the
monitor screen are triggered when measured chest compressions or ventilations are interrupted or
when they deviate from preprogrammed resuscitation guideline parameters (Fig. 1.4). The chest com-
pressor must have an unobstructed view of the monitor screen throughout a resuscitation effort to
enhance the effectiveness of audiovisual feedback (Bobrow et al., 2013). Some defibrillators also possess
12 CHAPTER 1  Chain of Survival and High-Quality Cardiopulmonary Resuscitation

Fig. 1.4  ​Several defibrillators, such as the MRx-QCPR shown here, are equipped with a chest compression pad that
enables monitoring of the quality of chest compressions and provides corrective feedback to rescuers. (Courtesy Philips
Healthcare. All rights reserved.)

Fig. 1.5  ​This Zoll R Series Monitor defibrillator filters cardiopulmonary resuscitation artifact, enabling the rescuer to
analyze a patient’s cardiac rhythm without interrupting chest compressions.  (Courtesy Zoll Medical Corporation,
Chelmsford, MA.)

technology that filters CPR artifact, allowing the rescuer to analyze a patient’s cardiac rhythm without
interrupting CPR (Fig. 1.5). Specialized software is used with many defibrillators to provide summary
data about CPR metrics, including the average chest compression rate and depth, CCF, preshock and
postshock pauses, the proportion of all compressions that were within current guidelines for both rate
and depth, ventilation frequency, analysis of the rhythm interpretation and treatment decisions, and
suggestions for improvement (Hopkins et al., 2016). Current resuscitation guidelines reflect that it may
be reasonable to use audiovisual feedback devices during CPR for real-time optimization of CPR
performance and the use of physiologic parameters, such as arterial blood pressure or EtCO2 to
monitor and optimize CPR quality, is reasonable when feasible (Panchal et al., 2020).
CHAPTER 1  Chain of Survival and High-Quality Cardiopulmonary Resuscitation 13

STOP AND REVIEW


Identify one or more choices that best complete the statement or answer the question.

_ ___ 1. During which phase of a cardiac arrest is CPR performed?


a. No flow phase
b. Prearrest phase
c. Low flow phase
d. Postresuscitation phase

____ 2. Which of the following are essential elements of high-quality CPR? Select all that
apply.
a. Push hard, push fast
b. Compress the chest at a rate of at least 120/min
c. Minimize interruptions of chest compressions
d. Allow full chest recoil between compressions

_ ___ 3. To minimize fatigue, rescuers delivering chest compressions should rotate every:
a. 30 seconds.
b. 2 minutes.
c. 5 minutes.
d. 10 minutes.

____ 4. Which of the following is a common cause of excessive intrathoracic pressure


during CPR?
a. Hyperventilation
b. Inability to open the victim’s airway
c. Inadequate rate of chest compressions
d. Frequent interruptions for rhythm/pulse checks

_ ___ 5. From the subsequent choices, select the links in the in-hospital Chain of Survival.
a. Prompt defibrillation
b. Early high-quality CPR
c. Surveillance and prevention
d. Intraarrest and postcardiac arrest care

_ ___ 6. Experts recommend a CCF of at least __ for optimal outcomes.


a. 25%
b. 60%
c. 75%
d. 90%

____ 7. When a cardiac arrest occurs in the hospital, the recommended goal is to deliver
the first shock within __ of patient collapse.
a. 2 minutes
b. 3 minutes
c. 5 minutes
d. 10 minutes

_ ___ 8. A ROSC is associated with which phase of a cardiac arrest?


a. No flow phase
b. Prearrest phase
c. Low flow phase
d. Postresuscitation phase
14 CHAPTER 1  Chain of Survival and High-Quality Cardiopulmonary Resuscitation

____ 9. Select the statements that are true about feedback devices used to improve the
quality of CPR.
a. Although useful in guiding the rate at which chest compressions should be
performed, the rhythm of a song is not considered a feedback device.
b. Chest compression rate, depth, hand position, chest wall recoil, and chest
compression fraction are examples of data that can be measured with some
feedback devices.
c. A metronome is an example of an inexpensive, but effective, feedback device.
d. Currently available devices can provide audio, but not video, feedback to the
user.

____ 10. Select the correct statements about EtCO2 monitoring after insertion of an advanced
airway during cardiac arrest.
a. Search for causes that may be hindering effective CPR if EtCO2 values are
consistently less than 20 mm Hg during resuscitation efforts.
b. EtCO2 values of 10 to 20 mm Hg are reasonable to expect when effective CPR
is delivered.
c. Failure to maintain EtCO2 at levels higher than 20 mm Hg during adult CPR
is a strong predictor of unsuccessful resuscitation.
d. EtCO2 values of 35 to 45 mm Hg are typical with ROSC.

STOP AND REVIEW  ANSWERS


1. C. The low flow phase of a cardiac arrest begins with the onset of CPR.

2. A, C, D. Essential elements of high-quality CPR are to push hard, push fast (that is, compress the
adult chest at a depth of at least 2 inches and perform compressions at a rate of 100 to 120 compres-
sions per minute), allow full chest recoil between compressions, maximize the chest compression
fraction (i.e., minimize interruptions of chest compressions), and avoid excessive ventilation.

3. B. To minimize fatigue, rescuers delivering chest compressions should rotate every 2 minutes.
Ideally, the switch should be accomplished in less than 5 seconds and should be done while another
intervention is being performed (e.g., defibrillation).

4. A. Hyperventilation is a common cause of excessive intrathoracic pressure during CPR. It is es-


sential to ventilate a patient in cardiac arrest at an age-appropriate rate and with just enough volume
to see the patient’s chest rise gently. Ventilating a cardiac arrest patient too fast or with too much
volume results in excessive intrathoracic pressure, which results in decreased venous return into the
chest, decreased coronary and cerebral perfusion pressures, diminished cardiac output, and decreased
rates of survival.

5. A, B, C, D. The links in the in-hospital Chain of Survival for adults include surveillance and preven-
tion of cardiac arrest, prompt notification and response when a cardiac arrest occurs, the performance
of high-quality CPR, prompt defibrillation, and intraarrest and postcardiac arrest care.

6. B. The CCF is the proportion of time spent performing chest compressions during a cardiac arrest.
Although a 2013 consensus statement about CPR quality recommended a CCF of at least 80% for
optimal outcomes, 2020 resuscitation guidelines note that in adult cardiac arrest, it may be reason-
able to perform CPR with a CCF of at least 60%.

7. A. Because it can take several minutes for code team members to arrive with a defibrillator, the
strategic deployment of AEDs throughout the facility can aid in achieving prompt defibrillation,
with the goal being the delivery of the first shock within 2 minutes of patient collapse.

8. D. The postresuscitation phase of a cardiac arrest begins with the ROSC.


CHAPTER 1  Chain of Survival and High-Quality Cardiopulmonary Resuscitation 15

9. A, B. Feedback devices are tools used to measure aspects of CPR performance that affect CPR
quality (e.g., chest compression rate, depth, hand position, chest wall recoil, chest compression
fraction). The measured data is then relayed to the rescuer by means of audio or visual feedback (or
both), enabling the rescuer to make immediate self-corrections to their performance. Some feed-
back devices are stand-alone systems that are applied to the patient’s chest, some are incorporated
into defibrillators, and others use apps on a wearable product, such as a smartwatch. Although a
metronome, or even the rhythm of a popular song, is useful in guiding the rate at which chest
compressions should be performed, it is not considered a feedback device because it does not pro-
vide the rescuer with real-time feedback that would allow correction of their performance.

10. B, D. EtCO2 falls sharply with the onset of cardiac arrest, increases when effective CPR is deliv-
ered (generally 10 to 20 mm Hg), and returns to physiologic levels (35 to 45 mm Hg) with the
ROSC. Low EtCO2 values (i.e., less than 10 mm Hg) during resuscitation efforts indicate the
need to explore factors that are hindering effective CPR. Experts note that failure to maintain
EtCO2 at levels higher than 10 mm Hg during adult CPR reflects poor cardiac output and
strongly predicts unsuccessful resuscitation.

REFERENCES
Abella, B. S., & Becker, L. B. (2019). Cardiopulmonary resuscitation and artificial perfusion during cardiac arrest.
In J. R. Roberts, C. B. Custalow, & T. W. Thomsen (Eds.), Roberts and Hedges’ clinical procedures in emergency
medicine and acute care (7th ed., pp. 332–337). Philadelphia, PA: Elsevier.
Andersen, L. W., Kim, W. Y., Chase, M., et al. (2016). The prevalence and significance of abnormal vital signs prior
to in-hospital cardiac arrest. Resuscitation, 98, 112–117.
Anderson, R., Sebaldt, A., Lin, Y., & Cheng, A. (2019). Optimal training frequency for acquisition and retention
of high-quality CPR skills: A randomized trial. Resuscitation, 135, 153–161.
Benjamin, E. J., Muntner, P., Alonso, A., et al. (2019). Heart disease and stroke statistics-2019 update: A report
from the American Heart Association. Circulation, 139(10), e56–e528.
Benner, J. P., Morris, S., & Brady, W. J. (2011). A phased approach to cardiac arrest resuscitation involving ven-
tricular fibrillation and pulseless ventricular tachycardia. Emerg Med Clin North Am, 29(4), 711–719.
Berdowski, J., Beekhuis, F., Zwinderman, A. H., Tijssen, J. G., & Koster, K. W. (2009). Importance of the first
link: Description and recognition of an out-of-hospital cardiac arrest in an emergency call. Circulation, 119(15),
2096–2102.
Berg, K. M., Cheng, A., Panchal, A. R., et al. (2020). Part 7: Systems of care: 2020 American Heart Association
guidelines for cardiopulmonary resuscitation and emergency cardiovascular care. Circulation, 142(16 suppl 2),
S580–S604.
Bhardwaj, A., & Abella, B. S. (2015). Does chest compression fraction matter, after all? Resuscitation, 97,
A5–A6.
Bobrow, B. J., Vadeboncoeur, T. F., Stolz, U., et al. (2013). The influence of scenario-based training and real-time
audiovisual feedback on out-of-hospital cardiopulmonary resuscitation quality and survival from out-of-
hospital cardiac arrest. Ann Emerg Med, 62(1), 47–56.
Brooks, S. C., Hassan, N., Bigham, B. L., & Morrison, L. J. (2014). Mechanical versus manual chest compressions
for cardiac arrest. Cochrane Database Syst Rev, 2014(2), CD007260.
Cheskes, S., Schmicker, R. H., Christenson, J., et al. (2011). Perishock pause: An independent predictor of survival
from out-of-hospital shockable cardiac arrest. Circulation, 124(1), 58–66.
Clawson, J., Olola, C., Scott, G., Heward, A., & Patterson, B. (2008). Effect of a medical priority dispatch system
key question addition in the seizure/convulsion/fitting protocol to improve recognition of ineffective (agonal)
breathing. Resuscitation, 79(2), 257–264.
Cunningham, L. M., Mattu, A., O’Connor, R. E., & Brady, W. J. (2012). Cardiopulmonary resuscitation for cardiac
arrest: The importance of uninterrupted chest compressions in cardiac arrest resuscitation. Am J Emerg Med,
30(8), 1630–1638.
Douw, G., Schoonhoven, L., Holwerda, T., et al. (2015). Nurses’ worry or concern and early recognition of dete-
riorating patients on general wards in acute care hospitals: A systematic review. Crit Care, 19(1), 230.
Edelson, D. P., Sasson, C., Chan, P. S., et al. (2020). Interim guidance for basic and advanced life support in adults,
children, and neonates with suspected or confirmed COVID-19. Circulation, e20201405 [online ahead of print].
Ewy, G. A. (2005). Cardiocerebral resuscitation: the new cardiopulmonary resuscitation. Circulation, 111(16),
2134–2142.
Ewy, G. A. (2012). The cardiocerebral resuscitation protocol for treatment of out-of-hospital primary cardiac
arrest. Scand J Trauma Resusc Emerg Med, 20(65), 1–6.
16 CHAPTER 1  Chain of Survival and High-Quality Cardiopulmonary Resuscitation

Geller, B. J., & Abella, B. S. (2018). Evolving strategies in cardiac arrest management. Cardiol Clin, 36(1), 73–84.
Hightower, D., Thomas, S. H., Stone, C. K., Dunn, K., & March, J. A. (1995). Decay in quality of closed-chest
compressions over time. Ann Emerg Med, 26(3), 300–303.
Hopkins, C. L., Burk, C., Moser, S., Meersman, J., Baldwin, C., & Youngquist, S. T. (2016). Implementation of pit
crew approach and cardiopulmonary resuscitation metrics for out-of-hospital cardiac arrest improves patient
survival and neurological outcome. J Am Heart Assoc, 5(1), 1–10.
Institute of Medicine, Graham, R., McCoy, M. A., & Schultz, A. M. (Eds.). (2015). Strategies to improve cardiac
arrest survival: a time to act. Washington, DC: National Academies Press.
Kilbaugh, T. J., Zwass, M. S., & Ross, P. (2020). Pediatric and neonatal critical care. In M. A. Gropper, R. D.
Miller, N. H. Cohen, L. I. Eriksson, L. A. Fleisher, K. Leslie, & J. P. Wiener-Kronish (Eds.), Miller’s anesthesia
(9th ed., pp. 2513–2584). Philadelphia, PA: Elsevier.
Kodali, B. S., & Urman, R. D. (2014). Capnography during cardiopulmonary resuscitation: Current evidence and
future directions. J Emerg Trauma Shock, 7(4), 332–340.
Kronick, S. L., Kurz, M. C., Lin, S., et al. (2015). Part 4: Systems of care and continuous quality improvement:
2015 American Heart Association guidelines update for cardiopulmonary resuscitation and emergency cardio-
vascular care. Circulation, 132(suppl 2), S397–S413.
Link, M. S., Atkins, D. L., Passman, R. S., et al. (2010). Part 6: Electrical therapies: Automated external defibrillators,
defibrillation, cardioversion, and pacing: 2010 American Heart Association guidelines for cardiopulmonary
resuscitation and emergency cardiovascular care. Circulation, 122(suppl 3), S706–S719.
Link, M. S., Berkow, L. C., Kudenchuk, P. J., et al. (2015). American Heart Association Guidelines for CPR & ECC.
Retrieved from American Heart Association. In Web-based integrated guidelines for cardiopulmonary resus-
citation and emergency cardiovascular care – Part 7: Adult advanced cardiovascular life support: Eccguidelines.
heart.org
Lyons, P. G., Edelson, D. P., & Churpek, M. M. (2018). Rapid response systems. Resuscitation, 128, 191–197.
Meaney, P. A., Bobrow, B. J., Mancini, M. E., et al. (2013). Cardiopulmonary resuscitation quality: [corrected]
Improving cardiac resuscitation outcomes both inside and outside the hospital: A consensus statement from
the American Heart Association. Circulation, 128(4), 417–435.
Morrison, L. J., Neumar, R. W., Zimmerman, J. L., et al. (2013). Strategies for improving survival after in-hospital
cardiac arrest in the United States: 2013 consensus recommendations. Circulation, 127(14), 1538–1563.
Myerburg, R. J., & Goldberger, J. J. (2019). Cardiac arrest and sudden cardiac death. In D. P. Zipes, P. Libby, R.
O. Bonow, D. L. Mann, G. F. Tomaselli, & E. Braunwald (Eds.), Braunwald’s heart disease: A textbook of cardio-
vascular medicine (11th ed., pp. 807–847). Philadelphia, PA: Elsevier.
Nordeen, C. A. (2018). Manual versus mechanical cardiopulmonary resuscitation. Cardiol Clin, 36(3), 376–386.
Ofoma, U. R., Basnet, S., Berger, A., Kirchner, H. L., & Girotra, S. (2018). Trends in survival after in-hospital
cardiac arrest during nights and weekends. J Am Coll Cardiol, 71(4), 402–411.
O’Neill, J. F., & Deakin, C. D. (2007). Do we hyperventilate cardiac arrest patients? Resuscitation, 73(1), 82–85.
Panchal, A. R., Bartos, J. A., Cabañas, J. G., et al. (2020). Part 3: Adult basic and advanced life support: 2020
American Heart Association guidelines for cardiopulmonary resuscitation and emergency cardiovascular care.
Circulation, 142(16 suppl 2), S366–S468.
Panchal, A. R., Berg, K. M., Cabañas, J. G., et al. (2019). 2019 American Heart Association focused update on
systems of care: dispatcher-assisted cardiopulmonary resuscitation and cardiac arrest centers. Circulation,
140(24), e895–e903.
Poole, K., Couper, K., Smyth, M. A., Yeung, J., & Perkins, G. D. (2018). Mechanical CPR: Who? When? How?
Crit Care, 22(1), 140.
Rajab, T. K., Pozner, C. N., Conrad, C., Cohn, L. H., & Schmitto, J. D. (2011). Technique for chest compressions
in adult CPR. World J Emerg Surg, 6(41), 1–5.
Reynolds, J. C., Bond, M. C., & Shaikh, S. (2012). Cardiopulmonary resuscitation update. Emerg Med Clin North
Am, 30(1), 35–49.
Scheppke, K. A. (2019). Palm Beach County sees results after instituting a bundle of care approach to resuscitation.
JEMS. Retrieved from https://www.jems.com/2019/02/27/palm-beach-county-sees-results-after-instituting-
a-bundle-of-care-approach-to-resuscitation/.
Topijan, A. A., Berg, R. A., & Nadkarni, V. M. (2013). Advances in recognition, resuscitation, and stabilization of
the critically ill child. Pediatr Clin North Am, 60(3), 605–620.
Tran, K., Cimon, K., Severn, M., Pessoa-Silva, C. L., & Conly, J. (2012). Aerosol generating procedures and risk
of transmission of acute respiratory infections to healthcare workers: A systematic review. PLoS One, 7(4),
e35797.
Wik, L., Kramer-Johansen, J., Myklebust, H., Sørebø , H., Svensson, L., Fellows, B., & Steen, P. A. (2005). Quality
of cardiopulmonary resuscitation during out-of-hospital cardiac arrest. JAMA, 293(3), 299–304.
Zhang, F. L., Yan, L., Huang, S. F., & Bai, X. J. (2013). Correlations between quality indexes of chest compression.
World J Emerg Med, 4(1), 54-58.
CHAPTER 2
Teams

INTRODUCTION
A team is defined as two or more people who interact dynamically, interdependently, and adaptively
toward a common and valued goal, have specific roles or functions, and have a time-limited member-
ship (Agency for Healthcare Research and Quality [AHRQ], 2014). Effective teamwork requires team
members to anticipate the needs of each other, adjust to each other’s actions and environmental
changes, and have a shared understanding of how a procedure or plan of care should occur (AHRQ,
2014). Team effectiveness can be enhanced using team training sessions, simulation exercises, and
interactive debriefings.
Rapid response or medical emergency teams respond when a patient shows signs of acute clinical
deterioration. A resuscitation team, also called a code team, responds when a respiratory or cardiac
arrest occurs. In some institutions, members of a rapid response team are also part of the code team.
This chapter discusses these teams, the roles and responsibilities of each member of the resuscitation
team, the importance of effective communication among team members, and communicating with
families.

L E A R N I N G OBJECTIVES
After completing this chapter, you should be able to:
1. Differentiate between Rapid Response Systems and resuscitation teams.
2. Describe the role of each member of a resuscitation team.
3. Discuss the key elements of effective team performance.
4. Recognize the opportunities provided when conducting a postevent debriefing.
5. Discuss the use of the SPIKES protocol when conveying bad news.

L E A R N I N G PLAN
• Read this chapter before class. Take the time to highlight important concepts as
you read.
• Master the following skills:
n Assign team member roles or perform as a team member in a simulated patient
situation.
n Review your performance as a team leader or team member during a postevent
debriefing.

17
18 CHAPTER 2  Teams

RAPID RESPONSE SYSTEMS


Severe adverse events (including cardiac arrest) experienced by hospitalized adults are often preceded
by warning signs and symptoms that suggest physiologic deterioration for hours before the event
(Andersen et al., 2016; Mathukia et al., 2015). Recognizing that early detection and treatment of the
patient who demonstrates signs of clinical deterioration may prevent cardiac arrest and improve patient
outcomes, the concept of a Rapid Response System (RRS) emerged. The RRS is mobilized by hospital
staff based on predetermined criteria for activation of the team.
Several types of response teams exist, and team composition may vary based on institutional re-
sources, preferences, and goals (Lyons et al., 2018) (Fig. 2.1). Team activation brings several clinicians
to the patient’s location. Each RRS member knows and understands the roles and responsibilities of
all other team members. Upon the arrival of the team, an assessment is performed, and initial interven-
tions are begun to stabilize the patient.

ACLS Pearl
SBAR is an acronym for Situation, Background, Assessment, and Recommendation that is often
used by healthcare personnel as a tool to ensure rapid, effective communication when exchang-
ing information about a patient’s condition (e.g., bedside clinician to response team). ISBARR,
an expanded form of this communication tool, is used in some areas: Introduction, Situation,
Background, Assessment, Recommendation, and Repeat (or Read back).

A medical emergency team (MET) is a physician-led team that can prescribe critical care interven-
tions, obtain central vascular access, and perform or facilitate advanced airway management (Lyons
et al., 2018). The term rapid response team (RRT) is used to describe a team without all of those abilities
that performs an initial evaluation of a patient and mobilizes additional help or facilitates patient
transfer to a higher level of care if warranted (Lyons et al., 2018; McCurdy & Wood, 2012).
A nurse often leads RRTs, and additional team members may include physicians (e.g., critical care or
hospitalist), additional nurses, respiratory therapists, and pharmacists.

ACLS Pearl
Researchers observe that the most critical component of any successful RRS is early event detec-
tion by the bedside nurse because they are usually the first to detect any significant changes in
the patient’s condition and determine what, if any, additional resources are needed (Sebat
et al., 2017).

Several scoring systems for detecting warning signs of patient deterioration exist, and they are used
as tools to assist in determining when the RRS should be activated. Box 2.1 shows examples of typical
RRS calling criteria. Most scoring systems identify physiologic deviations from normal, such as vital
sign changes and laboratory values. These values are then weighted and provided as a total risk score,

Fig. 2.1  ​Rapid Response Systems can be useful in reducing the incidence of cardiac arrest. (From iStock.)
CHAPTER 2  Teams 19

BOX 2.1 Rapid Response System Calling Criteria


• Acute change in mental status
• Heart rate greater than 140 beats per minute or less than 40 beats per minute
• Oxygen saturation less than 90% despite supplementation
• Progressive lethargy
• Staff concern about the patient’s condition
• Systolic blood pressure greater than 180 mm Hg or less than 90 mm Hg
• Urine output less than 50 mL over 4 hours
• Ventilatory rate greater than 28 breaths per minute or less than 8 breaths per minute
Additional criteria used at some institutions:
• Chest pain or discomfort unrelieved by nitroglycerin
• Seizure
• Threatened airway
• Uncontrolled pain
(Modified from Rapid Response Systems, Agency for Healthcare Research and Quality Patient Safety Network, 2019.)

which can help identify patients at risk for acute clinical deterioration ( Jackson, 2017). Recognizing
that physiologic changes may not be the only sign of patient deterioration, some institutions include
the use of subjective reasons (e.g., “worried/concerned” criteria) in their response team activation
criteria. Adding subjective criteria provides an opportunity for nurses to activate the team when
they intuitively feel that something is wrong with a patient, even when vital signs do not (yet) meet
RRS calling criteria (Douw et al., 2015). Examples of possible worry or concern indicators include the
following (Douw et al., 2015; Jackson, 2017):
• Patient agitation
• New or increasing pain
• A patient indicating that they are is not feeling well
• Changes in breathing (e.g., noisy breathing, accessory muscle use)
• Changes in circulation (e.g., sweating, skin color change from patient’s normal)
• A nurse’s gut feeling that something is wrong
Regardless of the criteria used, the decision to activate the RRS is ultimately the responsibility of
the bedside clinician (McCurdy & Wood, 2012).
Experts observe that implementing an RRS necessitates the following (Yeung et al., 2020):
• Staff education about the signs of patient deterioration
• Appropriate and regular vital signs monitoring of patients
• Clear guidance (e.g., alert systems or early warning scores) to assist staff in the early detection of
patient deterioration
• A clear, uniform system of tiered clinical response
• A clinical response to calls for assistance
Barriers to activation of the RRS have been identified and appear in Box 2.2.

BOX 2.2 Barriers to Rapid Response System Activation


• High nurse-to-patient staffing ratios
• Other patient care needs
• Trajectory of the patient’s disease
• Communication barriers between rapid responders and other hospital staff
• The bedside nurse may lack confidence or be inexperienced in recognizing a deteriorating patient
• The bedside nurse may not know whom to contact when a patient’s condition deteriorates
• Lack of empowerment of the bedside nurse to activate the Rapid Response System (RRS)
• The bedside nurse may fear blame if activation of the RRS is later deemed unnecessary
• Nurses often observe patients who briefly exhibit abnormal vital signs that spontaneously
normalize
• Physician concerns about RRSs leading to a loss of patient care autonomy
(From Lyons, P.G., Edelson, D.P., Churpek, M.M. [2018]. Rapid response systems. Resuscitation, 128, 191–197; McCurdy,
M.T., Wood, S.L. [2012]. Rapid response systems: Identification and management of the “prearrest state.” Emerg Med Clin
North Am, 30[1], 141–152; Sebat, C., Sinigayan, V., Albertson, T. [2017]. Hospital rapid response systems. Hosp Med Clin,
6[4], 480–491.)
20 CHAPTER 2  Teams

RRSs are recommended by the Institute for Healthcare Improvement and other national patient
safety initiatives to reduce cardiopulmonary arrests and other serious adverse events (Mathukia et al.,
2015). However, studies show considerable variation in patient outcome data (e.g., improved patient
survival, reduced number of cardiac arrests) regarding RRS use. In adults, some studies demonstrate
reductions in both in-hospital cardiac arrest (IHCA) and mortality, others demonstrate reductions in
IHCA without a significant change in mortality, and still others show no significant differences in
either IHCA or mortality (McCurdy & Wood, 2012; Sebat et al., 2017). Possible reasons why outcome
studies have shown inconsistent results include variations among hospitals in the composition of the
responding teams, the consistency of team activation and response, and the elements of the early warn-
ing scoring systems used (Berg et al., 2020). Current resuscitation guidelines note that RRSs can be
effective in reducing the incidence of cardiac arrest in hospitalized adults, particularly in general care
wards, and that the use of early warning scoring systems may be considered for hospitalized adults
(Berg et al., 2020).

ACLS Pearl
A bedside clinician is likely to activate the RRS when a patient with known or suspected corona-
virus-19 (COVID-19) shows signs of clinical deterioration. It is essential to communicate a
patient’s COVID-19 status to responding team members before their arrival and when patient
care is transferred (Edelson et al., 2020). Consider proactively moving patients at risk for cardiac
arrest to a negative pressure room or unit, if it is available, to minimize the risk of rescuer exposure
during a resuscitation (Edelson et al., 2020).

THE RESUSCITATION TEAM


The primary goals of resuscitation are to restore spontaneous circulation and meaningful neurologic
recovery and to preserve vital organ function. During a resuscitation effort, an interdisciplinary team
works together to provide coordinated patient care. Teamwork helps to ensure that the patient’s many
needs are met throughout the resuscitation effort. Each team member has a preassigned role with
specific patient management responsibilities. The person in charge of a resuscitation effort is typically
called the code director or team leader. Because the team members are experienced in their roles, they
perform their assigned tasks expertly and efficiently, limiting interruptions during the resuscitation
effort. Team members avoid sidebar conversations because each person recognizes that such commu-
nication can be distracting to other team members.

ACLS Pearl
Just as it is vital to know how to use a piece of equipment before using it in an emergency, you
must know your facility’s procedure for activating the code team. It is essential to know, learn, and
practice your facility’s code procedure and to learn what is expected of you as a member of the
resuscitation team. Frequent (e.g., monthly) practice using methods such as simulation-based
mock codes is needed to minimize errors, maintain skills, and optimize patient outcomes
(Morrison et al., 2013).

In the prehospital setting, a paramedic or nurse, operating under standing physician orders, local
protocols, or both, usually leads the resuscitation effort. In the hospital setting, the team leader is
usually a physician who is experienced in cardiac arrest management. In most institutions, advanced
cardiac life support (ACLS) is considered the standard of care in a cardiac arrest situation, and, in
the absence of a physician, emergency care may be initiated by appropriately trained nurses per that
institution’s policy.
Research shows consistently higher performance when using dedicated or designated resuscita-
tion teams rather than ad hoc teams (Hunziker et al., 2011; Nallamothu et al., 2018). A dedicated
team refers to the presence of a core group of nursing or respiratory therapy staff with no clinical
responsibilities during a given shift that supersede their roles on a resuscitation team. In contrast,
CHAPTER 2  Teams 21

a designated team refers to nursing and respiratory therapy staff who may have other clinical respon-
sibilities that involve direct patient care, but systems are in place for shedding these quickly when an
IHCA occurs (Nallamothu et al., 2018). Some facilities with dedicated resuscitation teams also use
the expertise of specific team members as part of rapid response, sepsis, or difficult airway response
teams.

Pit Crew Model


Team performance during a cardiac arrest has been compared with the “pit crew” model used in For-
mula One motor racing. This model is an example of how a team of professionals comes together as
a single unit to effectively perform a complex task under significant time pressure with minimal error
(Catchpole et al., 2007). The pit crew approach focuses on effective leadership, skill competence,
teamwork, and communication. Team members can perform some resuscitation therapies without
specific instructions from the team leader (e.g., attaching the patient to a cardiac monitor, applying
adhesive pads to the patient’s chest, preparing medications) (Hopkins et al., 2016). Debriefing is a
key element of the pit crew model, enabling team members to recognize and understand where,
when, and how tasks and processes could (or did) fail and what the performance strengths and
weaknesses were, both for individual members and the team as a whole (Catchpole et al., 2007;
Cochrane et al., 2018).

Crew Resource Management


Crew resource management (CRM), also called crisis resource management, is another popular model
of team training that has been used for many years to improve teamwork, communication, and safety
in the aviation industry and has been more recently applied to healthcare. Key elements of CRM in-
clude developing and understanding situation awareness; teamwork (including leadership), decision
making, and task management in an atmosphere of mutual trust and respect; and effective interpersonal
communication to reduce the risk of errors and enhance patient safety. Closed-loop communication and
debriefing are fundamental CRM principles.

Team Arrival
While wearing appropriate personal protective equipment, several tasks are performed simultane-
ously as the members of the code team converge and position themselves around the patient
to begin or continue resuscitation efforts. For example, the code cart is placed at the patient’s bed-
side for easy access to the defibrillator, oxygen, suction equipment, medications, and supplies, as
well as for viewing the electrocardiogram (ECG) monitor. The patient is attached to a cardiac
monitor and to a continuous end-tidal carbon dioxide (EtCO2) monitor (if available), adhesive
pads are applied to the patient’s bare chest, an oxygen source is attached to a bag-mask device, and
suction is set up. Pertinent information is quickly obtained, such as patient age, comorbidities,
weight (this allows the team to anticipate weight-based drug dosages), the estimated time of the
arrest, the circumstances surrounding the arrest, and the presence of a do-not-attempt-resuscitation
order.
The team leader and team members, who have roles involving direct patient care activities, posi-
tion themselves within one arm’s length of the patient (Spitzer et al., 2019). When known or sus-
pected COVID-19 is a factor, it is essential to limit the number of personnel present to only those
essential for patient care (Edelson et al., 2020). In general, the team leader is positioned at the foot
of the patient, and the team member in charge of the airway is located at the patient’s head. The
team member performing chest compressions is positioned near the patient’s shoulder, and the
team member responsible for defibrillator operation is located on the opposite side at the patient’s
shoulder. The vascular access and medication administration team member is situated near the
patient’s leg. Because the size of a resuscitation team varies, the physical location of team members
relative to the patient may differ from that described here. Team members whose roles support the
team’s function but do not require direct patient care (e.g., pharmacist, timekeeper, documenter,
second chest compressor, code supervisor) are typically positioned more than arm’s length from the
patient.
22 CHAPTER 2  Teams

ACLS Pearl
Knowledge of the algorithms is essential to the successful completion of an ACLS course. During
an ACLS course, your knowledge of the ACLS algorithms is evaluated in simulated situations and
on the course posttest. An ACLS instructor evaluates the simulations (also called “cases”). The
cardiac arrest algorithms are assessed in the Cardiac Arrest Management (also called the Mega
Code) station. In this station, you work in teams of four or five persons. Each person takes a
turn as the team leader and as individual resuscitation team members, performing each of the
critical tasks of resuscitation. The team leader is evaluated on their knowledge of the ACLS algo-
rithms, the ability to manage the resuscitation team, and their decisions regarding patient man-
agement. Although the team leader is responsible for directing the overall actions of the team, a
resuscitation effort requires teamwork. Each member of the team must know their responsibilities
and should be able to anticipate the team leader’s instructions. This is true in real life, as well as
in simulated situations.

Team Roles and Responsibilities


Every resuscitation effort must have someone who assumes responsibility for overseeing the actions of
the code team. If more than one person attempts to make decisions regarding the patient’s care, confu-
sion reigns, and chaos will most likely result. The team leader guides the members of the code team
and uses rapid, dynamic reasoning that considers several things at once (Table 2.1). Research has
shown that team leaders who perform hands-on tasks in an emergency are less likely to be efficient
leaders; therefore the team leader should be able to “stand back” to view and direct the resuscitation
effort (Hunziker et al., 2011; Meaney et al., 2013).
It is likely that anyone who has been involved in, or simply observed, a resuscitation effort can recall
at least one chaotic event where the team leader shouted at everyone and the team members became
flustered, not knowing what to anticipate next. As the team leader, your manner, attitude, words, and
skills must be professional throughout the resuscitation effort. A good team leader values their team

TABLE 2.1 Essential Resuscitation Team Roles and Responsibilities


Role Key Responsibilities
Team leader • Leads the code team; coordinates and prioritizes all aspects of care per current
resuscitation guidelines
• Ensures team delivery of high-quality CPR
• Problem-solves, including evaluating possible causes of the arrest
• Directs postcardiac arrest care when there is a return of spontaneous circulation
• Decides when to terminate resuscitation efforts (in consultation with team
members), when there is no response to resuscitation efforts after a reasonable
period
• Provides an opportunity for team members to be involved in a team debriefing
or reflection on the resuscitation effort after the event
CPR team member • Provides chest compressions of adequate rate, force, and depth
• Alternates responsibility with predesignated compressor every 2 minutes
ECG/defibrillation ​ • Interprets cardiac dysrhythmias
team member • Operates an AED and/or manual defibrillator
• Problem-solves concerning equipment failure
Airway team member • Performs the head tilt–chin lift or jaw thrust maneuver as needed
• Suctions the upper airway when indicated
• Performs bag-mask ventilation; avoids excessive ventilation
• Inserts an advanced airway (if within scope of practice); if used, confirms
placement and properly secures the device
Vascular access/​ • Obtains vascular access (IV or IO)
medications ​ • Administers medications and IV fluids as directed by the team leader
team member • Repeats back each medication order received; questions any unclear order
• Announces when each medication has been given
AED, Automated external defibrillator; CPR, cardiopulmonary resuscitation; ECG, electrocardiography; IO, intraosseous; IV,
intravenous.
CHAPTER 2  Teams 23

members, fosters an environment in which team members feel comfortable speaking up, and encour-
ages a respectful exchange of ideas.
Each member of the resuscitation team must:
• Have clear roles and responsibilities
• Know their limitations
• Be knowledgeable about current resuscitation algorithms
• Be practiced in resuscitation skills
• Be prepared to question other team members if an action is about to occur that may be inappropriate
All members of the resuscitation team are responsible for ensuring the performance of high-
quality CPR.
Nurses who respond to a cardiac arrest must be familiar with the layout of the code cart, which
is also called a crash cart, and the location of all items contained therein. In the prehospital setting,
paramedics must be familiar with the location of all medications in their drug box and the resuscita-
tion-related equipment in their emergency bags and vehicles, if applicable.

Support Roles
There are many support roles in a resuscitation effort. In the hospital, a pharmacist may be a member
of the code team and be responsible for drawing up medications contained on the code cart when re-
quested by the team leader. A timekeeper should announce every 2-minute interval throughout the
resuscitation effort while a documenter enters pertinent code events in the patient’s electronic medical
record. Many organizations have added a “safety officer” to RRS and resuscitation teams. This indi-
vidual is responsible for ensuring best practices and the safety of the patient and all team members,
including the management of patients at high risk for COVID-19.
A nursing supervisor often assumes responsibility for contacting the patient’s attending physician,
limiting the number of people present to those necessary (i.e., crowd control), ensuring that a critical
care bed is available, and coordinating the transfer of the patient to the intensive care unit. Another
nurse typically assumes responsibility for referring to the patient’s electronic health record for pertinent
patient information (e.g., code status, allergies, most recent laboratory results) and relaying that infor-
mation to the team leader.
Support staff is needed to remove excess furniture or equipment from the room (e.g., overbed table,
wheelchair), to assist the patient’s roommate (if applicable), and to provide ongoing care to other pa-
tients on the ward. Pastoral care, social workers, or other nursing staff are needed for family support.
The use of a professional language interpreter may be required to explain the patient’s condition to the
family.

ELEMENTS OF EFFECTIVE TEAM PERFORMANCE


Key elements of effective team performance include teamwork, task management, verbal and nonverbal
communication, situation awareness, and decision making (Fig. 2.2).

Teamwork
Teamwork, which includes leadership, involves coordinating activities with team members, exchanging
information, using authority and assertiveness, assessing capabilities, and supporting others (Fletcher
et al., 2012). To be effective, all members of the team must share a common mental model of the team’s
goal, the tasks required to achieve it, and how the tasks fit together (Cochrane et al., 2018). When a
common mental model is shared, team members are better able to anticipate each other’s needs and
actions. When team members train and work together, mutual trust is built, enabling members to get
to know each other and their roles and responsibilities. Team members assist one another, monitor
other team members’ performance to anticipate requests for assistance, offer or ask for help as needed,
provide encouragement and feedback to each other, self-correct errors, and assist others in correcting
their mistakes (AHRQ, 2019c).
While working under stressful conditions, the entire team must be able to adapt to changes that
occur during the event. Effective team members are individuals to whom the team leader can safely
assign tasks and delegate responsibility while recognizing that their actions (or inaction) affect the
24 CHAPTER 2  Teams

Situation
awereness

Task Communication
verbal & Teamwork
management
nonverbal

Decision
making

Fig. 2.2  Key elements of effective team performance include verbal and nonverbal communication, situation awareness,
decision making, task management, and teamwork. The components are intertwined with and related to each other, indicated
by the arrows between the elements. (From Gropper, M.A. [2020]. Miller’s anesthesia, ed 9, Philadelphia, Elsevier.)

entire team. Effective team members can focus their efforts, manage themselves well, are committed to
the team’s purpose, and appreciate the goals and needs of the group (Kelley, 1988).

Task Management
Elements of task management include planning and preparing, prioritizing, providing and maintaining
standards, and identifying and using resources (Fletcher et al., 2012). Before an event, the functionality
of equipment should be regularly checked, bags or carts should be appropriately stocked, and backup
plans should be developed for use when needed. During an event, necessary equipment should be im-
mediately accessible, supplies should be available in adequate quantities (and a plan should be in place
to acquire additional supplies at a moment’s notice, if necessary), established resuscitation guidelines
should be followed by all team members, the plan of care must be clearly communicated and modified
when necessary to adapt to the patient’s response to interventions, accurate documentation must be
maintained, and additional resources should be requested if needed.
Tasks are typically delegated to team members based on technical skills, level of expertise, and
experience. Throughout the event, team members should be monitored and duties redistributed to
other members of the team if an individual becomes overloaded (AHRQ, 2019c).

Team Communication
Clear and effective communication is essential because it affects the delivery and effectiveness of the
actions of the team. Speak in a confident tone using terms that are known and shared by all team
members. Because appearing or sounding anxious can quickly affect the members of your team, speak-
ing in a composed tone generally has a calming effect on those present.
Statements and questions pertaining to time, patient status (including vital signs and clinical find-
ings), patient history, and interventions are examples of information commonly shared among rapid
response and resuscitation team members. Team members must provide the team leader with frequent
updates to enable ongoing reassessment. Updates should also be provided to all members of the team
throughout the event. Updates are often provided using call-outs, which are methods used to relay
critical information during an emergent event. Calling out critical information enables the team to
anticipate and prepare for vital next steps in patient care (AHRQ, 2019a). The team leader should also
ask team members for information as needed.
CHAPTER 2  Teams 25

To avoid information overload and to help ensure that what is said by the team leader is what is
heard by the team members, the team leader should state their instructions one at a time using terms
that are known and shared by all team members. The team member’s name should be used if known.
For example, “Aubree, please charge the defibrillator to 150 joules” or “Sawyer, please insert an oropha-
ryngeal airway.”
To avoid the need for repetitious instructions and to avoid misunderstandings, team members
must close the communication loop by clearly acknowledging when procedures and medications are
complete. For example, if a team member was directed to establish an intravenous (IV) or give a
medication, they should respond by saying something like, “IV started, left antecubital vein” or “amio-
darone 300 mg given IV” when the task is completed. This practice, commonly referred to as a “check-
back,” is a closed-loop communication strategy that allows those sending and receiving messages an
opportunity to recognize and correct errors. Check-backs also help to ensure accurate documentation
of the interventions performed, the timing of those interventions, and the patient’s response to them
by the designated event recorder.
Team members must verbalize any patient changes, highlighting the change in status in the infor-
mation shared (Hunziker et al., 2011). For example, “Dr. __, the rhythm on the monitor has changed
from ventricular fibrillation to sinus bradycardia.”

Speaking Up
Regardless of your role in a resuscitation effort or your level of certification or licensure, it is crucial to
tactfully voice your concerns and question an intervention if you know a misstep is being made or is
about to occur. Because failing to speak up is the root cause for many preventable occurrences in
healthcare, all team members must be empowered to “stop the line” if a team member senses or discov-
ers an essential safety breach (AHRQ, 2019c). Team members must be encouraged to voice their
concerns in a way that does not place blame.

ACLS Pearl
Safe practice includes the verification of orders; therefore team members must request clarification
of any unclear orders.

A strategy that can be used to enhance patient safety is the CUS tool, which stands for Concern,
Uncomfortable, and Safety (AHRQ, 2019c). For example, a team member saying, “I am concerned...”
should result in the team’s immediate attention and momentary cessation of all procedures that can be
safely halted (except for chest compressions during a cardiac arrest). The team member should then
explain why they are is uncomfortable, “I am uncomfortable because...” If the concern is not resolved,
the team member would escalate their concern, explaining, “This is a safety issue...” and identify how
the issue is related to safety.
If an expressed concern is ignored, each member of the team has a responsibility to voice their
uneaseat least 2 times to ensure the concern has been heard, understood, and acknowledged. The two
attempts, called the Two-Challenge Rule, may come from the same person or two different team
members (AHRQ, 2019c). The team member being challenged must acknowledge the concern. If the
outcome is still not acceptable, the team member should take a stronger course of action or use a su-
pervisor or the chain of command.

Situation Awareness
Elements of situation awareness (SA) include gathering information, recognizing and understanding,
and anticipating (Fletcher et al., 2012). Information is gathered from multiple sources, such as the
patient, the patient’s physical examination and history, the patient’s medical record, monitoring equip-
ment in use, and information shared by team members, among other sources. For the leader of a rapid
response or resuscitation team, SA involves assessing the current situation, examining all existing in-
formation (and requesting additional information if necessary), determining what data is relevant, and
thinking ahead about the patient’s future course and the possible consequence of actions that are being
considered. SA also includes monitoring the fatigue level, workload, task performance, skill level, and
stress level of fellow team members to prevent errors from occurring (AHRQ, 2019b). Throughout an
26 CHAPTER 2  Teams

event, team members should ensure that monitors and pump displays are clearly visible and that team
members, particularly those individuals performing chest compressions and bag-mask ventilation, are
observed for signs of tiring.

Decision Making
Effective leaders earn the trust that is required to make difficult decisions (Gillman et al., 2016).
Decision making involves the following (Fletcher et al., 2012):
• Identifying options before taking a course of action
• Considering suggestions offered by other team members when appropriate
• Considering the risks of different treatment options while weighing factors concerning the patient’s
condition
• Assessing time criticality associated with possible options
• Implementing a chosen option
• Reassessing the patient after implementing a treatment or intervention
• Revising a course of action, when warranted, as the patient’s condition evolves

Debriefing
Debriefing is a “facilitated or guided reflection in the cycle of experiential learning” (Fanning & Gaba,
2007). Although any member of a response or code team can request a postevent debriefing, the team
leader is generally responsible for making sure that a debriefing takes place. Data from arterial blood
pressure and capnography tracings, the code sheet or other resuscitation records, feedback devices, te-
lemetry and defibrillator printouts, defibrillator audio recordings, video review, chart review, and other
sources that captured data during the event should be collected and provided for feedback to the team.
“Hot” debriefings occur immediately after an event (Kessler et al., 2015). “Warm” debriefings occur
minutes to hours after an event. Hot and warm debriefings rely on team member recall and available
documentation (Mullan et al., 2017). “Cold” debriefings occur in the days to weeks after an event.
Cold debriefings may not include all original team members and may intentionally include nonteam
members, as well as supplemental data from video review, chart review, and feedback devices (Mullan
et al., 2017).
During a debriefing, each member of the team has an opportunity to engage in honest dialogue to
gain understanding and to identify lessons learned in a nonpunitive environment (Fig. 2.3). Ideally, the
individual who leads the debriefing should have training and experience as a facilitator. Their role is to
facilitate, rather than control or dominate, discussion among team members using a professional,
matter-of-fact tone.
A debriefing provides the following:
• An opportunity for each team member to reflect on what they did, when they did it, how they did
it, why they did it, and how they can improve (Phrampus & O’Donnell, 2013)

Fig. 2.3  ​Each member of the team has an opportunity to engage in honest dialogue during a debriefing. (From iStock.)
CHAPTER 2  Teams 27

• An opportunity to identify and address performance gaps (i.e., the gap between desired and actual
performance) and perception gaps (i.e., the difference between the team member’s perception
of their performance and actual performance as defined by objective measures) (Phrampus &
O’Donnell, 2013)
• An opportunity to review the clinical judgments made and actions performed during the event and
compare them with current resuscitation algorithms, professional standards, institution policies, and
local protocols
• An opportunity to address emotional responses related to the event
• An opportunity for self-reflection that can be translated to actionable knowledge to guide future
decisions and actions, and ultimately improve patient care
• An opportunity to identify and discuss the elements of the resuscitation that went well, those areas
that could be improved, and recommendations for future resuscitation efforts
Although there are many debriefing techniques, the structured and supported debriefing model is a
method that is commonly used in advanced life support courses. This model consists of the following
phases: (Phrampus & O’Donnell, 2013)
1. Gather phase. This phase is used for gauging the reaction of the team to the event, clarifying facts,
describing what happened, and creating an environment for reflective learning. During this phase
of the debriefing, the team leader is asked to provide a synopsis of what occurred, and supplemental
information is requested from team members. Using open-ended questions, the facilitator listens as
the team members describe their perceptions of their behaviors.
2. Analysis phase. During this phase, the record of the event (e.g., code sheet, data from feedback-en-
abled devices) is reviewed, and the observations of team members are reported. The facilitator asks
questions to assist with self-reflection and analysis of each team member’s actions, changes in the
patient’s condition that may have occurred during the event, and how individual and team actions
may have influenced the outcome of the event. The actions of the team can be compared with cur-
rent resuscitation algorithms, professional standards, institution policies, best evidence, and local
protocols to enhance understanding.
3. Summary phase. The debriefing concludes with a review of the lessons learned and a summary of the
main take-home messages and needed performance improvements.

FAMILY NOTIFICATION
Several surveys have revealed that most relatives of patients who require CPR would like to be offered
the possibility of being present during a resuscitation attempt. According to follow-up surveys with
family members who had witnessed a resuscitation effort, most felt that their adjustment to the death
or grieving was facilitated by their seeing the resuscitation and that their being present was beneficial
to the dying family member.
If family members are not present during the resuscitation effort, they should be told that resusci-
tation efforts have begun, and they should be periodically updated. The result of the resuscitation
effort, whether successful or unsuccessful, should be relayed to the family promptly with honesty and
compassion.
When speaking with the family, talk slowly and in a quiet, calm voice. Use simple terms rather than
medical terms. Pause every few seconds to ask if they understand what is being said. You may need to
repeat information several times. In general, you should make eye contact with the family members,
except where cultural differences may exist. Enlist the assistance of a social worker, a clergy member,
or grief support personnel, as needed.

Conveying Bad News


As a clinician, you will likely have to deliver bad news to a patient and their family members at some
time during your career. How you relay this news can affect the patient’s or survivor’s ability to under-
stand and cope with the information provided. Research shows that patients and families consider the
following factors as desirable when receiving bad news (Quest, 2018):
• Privacy when receiving news
• Ability to express emotions safely
• Receiving information that is free of unclear language or medical jargon
28 CHAPTER 2  Teams

BOX 2.3 SPIKES Protocol


S—Setting (i.e., arrange for privacy, avoid interruptions)
P—Perception of what the patient/family understand about the situation
I—Invitation from the patient/family to give information
K—Knowledge (i.e., relaying medical facts)
E—Emotions (i.e., address with empathetic responses)
S—Summary (i.e., confirm understanding)

• Empathic and caring attitude


• Allowance for hope
• Ability to ask for and receive useful medical information
Although delivering bad news is stressful for the clinician, the patient, and family members, the
use of a structured protocol, in combination with role-playing, simulations, and communication skills
training, can help you prepare for these difficult situations. SPIKES is an acronym for a six-step
protocol that is widely used for conveying distressing information to patients and families (Box 2.3)
(Baile et al., 2000).
1. Setting. Organize your thoughts about the information that you will need to convey. Anticipate
questions that family members will ask. Select a location that provides for privacy with all appropri-
ate people present (Fig. 2.4). Sit down, face the family, acknowledge everyone present, and ask their
relationship to the patient. Appear calm and attentive. Minimize interruptions by putting your
cell phone on vibrate. If language is a barrier, arrange for a translator to be present and part of the
discussion.
2. Perception. Before conveying information, use open-ended questions to find out what the family
already knows. Asking, “What have you been told so far?” or “What is your understanding of what
has happened?” allows an opportunity to gauge how the family perceives the current situation—
what it is and its seriousness (Baile et al., 2000). It also provides an opportunity to correct
misinformation.
3. Invitation. Ask the family how they prefer to receive the information that you have to share and
how much they want to know. For example, “Would you like me to tell you more about what hap-
pened?” Keep in mind that ethnic and cultural values play a significant role in the need for informa-
tion. Although families are often clear about how much information they are ready to hear, it is
possible that they may be too emotionally upset or overwhelmed to comprehend the information
that you are about to convey.
4. Knowledge. Beginning with a warning statement like, “I have some bad news to tell you” and then
pause. Pausing allows the family time to grasp what has been said. Although the use of the phrase

Fig. 2.4  ​When conveying bad news, allow time for the shock to be absorbed and as much time as necessary for questions
and discussion. (From iStock.)
CHAPTER 2  Teams 29

“I’m sorry...” can be a reflection of empathy, some emergency clinicians have suggested the use of
expressions such as “I wish things were different” instead to avoid the implication that errors have
occurred in treatment (Quest, 2018). While speaking slowly, proceed to convey the news in small
chunks and a straightforward manner. To reduce the potential for misunderstanding, use words that
the family will easily comprehend. Do not use medical jargon and avoid excessive bluntness. Assume
nothing as to how the news is going to be received. If the resuscitation effort was unsuccessful, allow
time for the shock to be absorbed and as much time as necessary for questions and discussion.
Recognize that the initial shock experienced by the family may prevent them from knowing
what questions to ask. It may be necessary to repeat answers or explanations to ensure that they are
understood.
5. Emotions. Give the family time to respond. Be sensitive and respectful of cultural differences. The
family’s reaction is often unpredictable and may be anger, shock, withdrawal, disbelief, extreme agi-
tation, guilt, or sorrow. An expected death may elicit a response of acceptance and relief. The resus-
citation efforts may have given the family time to accept the outcome. In some cases, there may be
no apparent response or the reaction may seem inappropriate. A statement such as, “You have my
(our) sincere sympathy” may be used to express your feelings. However, there are times that silence
is appropriate. Silence respects the family’s feelings and allows them to regain composure at their
own pace.
6. Summarize. Offer to contact the patient’s physician and to be available if there are further
questions. Arrange for follow-up and continued support during the grieving period. Allow the
family the opportunity to see their relative. A staff member should be available in the room
or nearby to offer support as needed. In cases involving severe traumatic cardiac arrest, viewing
the body may not be advisable. If equipment remains connected to the patient, prepare the
family for what they will see. Gown the patient before the family views the body. Some caregiv-
ers may prefer not to view the body. If this is their preference, do not attempt to force them
to do so.

HELPING THE CAREGIVERS


An unsuccessful resuscitation effort is difficult for the family, as well as the healthcare providers in-
volved in the resuscitation. Although each provider may deal with stress differently, reactions suggest-
ing a need for assistance include persistent feelings of anger, self-doubt, sadness, depression, or a desire
to withdraw from others. It is essential to recognize the warning signs of stress in yourself and others
and know how to deal with them (Fig. 2.5). Strategies for coping with stress may include engaging in
exercise, practicing relaxation techniques, talking with family or friends, or meeting with a qualified
mental health professional.

Fig. 2.5  ​Know how to recognize the warning signs of stress in yourself and know how to deal with them. (From iStock.)
30 CHAPTER 2  Teams

STOP AND REVIEW


Identify one or more choices that best complete the statement or answer the question.

____ 1. Which of the following statements are correct about team dynamics during a
resuscitation effort? Select all that apply.
a. The team leader should state their instructions one at a time.
b. The team leader should encourage a respectful exchange of ideas.
c. Team members must be knowledgeable about current resuscitation algorithms.
d. Team members should be encouraged to confer among themselves throughout
the resuscitation effort.

____ 2. In addition to objective criteria, some institutions are including subjective reasons
for activating the Rapid Response System within their facilities. Which of the
following are examples of subjective reasons for activating the response team?
a. Urine output less than 50 mL over 4 hours
b. A nurse’s gut feeling that something is wrong
c. Oxygen saturation less than 90% despite supplementation
d. Heart rate greater than 140 beats per minute or less than 40 beats per minute

____ 3. During a cardiac arrest, the team member responsible for vascular access is typically
positioned at the patient’s:
a. leg.
b. head.
c. foot.
d. shoulder.

_ ___ 4. Which of the following are key elements of effective team performance?
a. Teamwork
b. Communication
c. Decision making
d. Task management
e. Situation awareness

_ ___ 5. Which of the following are true regarding a postevent debriefing?


a. The facilitator should use open-ended questions to encourage discussion.
b. Team members are encouraged to identify lessons learned in a nonpunitive
environment.
c. The gather phase of the debriefing includes a comparison of the team’s actions
with current resuscitation algorithms.
d. Team members are given an opportunity to reflect on their performance and
how their performance can be improved.

STOP AND REVIEW  ANSWERS


1. A, B, C. Each member of the resuscitation team must have clear roles and responsibilities, must
know their limitations, must be knowledgeable about current resuscitation algorithms, must be
practiced in resuscitation skills, and must be prepared to question other team members if an action
is about to occur that may be inappropriate. The team leader should state their instructions one at
a time using terms that are known and shared by all team members. The team member’s name
should be used if known. A good team leader values their team members, fosters an environment in
which team members feel comfortable speaking up, and encourages a respectful exchange of ideas.
Team members must clearly acknowledge when procedures and medications are complete. Because
there are often a large number of persons present during a code, sidebar conversations among team
members that can be distracting to other team members must be avoided.
CHAPTER 2  Teams 31

2. B. Most early warning scoring systems identify physiologic deviations from normal, such as vital
sign changes and laboratory values. These values are then weighted and provided as a total risk score,
which can help identify patients at risk for acute clinical deterioration. Some institutions include
the use of subjective reasons (e.g., “worried/concerned” criteria) into their response team activation
criteria, providing an opportunity for nurses to activate the team when they intuitively feel that
something is wrong with a patient, even when vital signs do not (yet) meet RRS calling criteria.
Examples of possible worry or concern indicators include patient agitation, complaints of pain, a
patient indicating that they are is not feeling well, or a nurse’s gut feeling that something is wrong.

3. A. The team member responsible for vascular access and medication administration is typically
positioned near the patient’s leg.

4. A, B, C, D, E. Key elements of effective team performance include verbal and nonverbal commu-
nication, situation awareness, decision making, task management, and teamwork.

5. A, B, D. During a debriefing, each member of the team has an opportunity to engage in honest
dialogue to gain understanding and to identify lessons learned in a nonpunitive environment. An
opportunity is provided for each team member to reflect on what they did, when they did it, how
they did it, why they did it, and how they can improve. The facilitator uses open-ended questions to
encourage discussion and listens as the team members describe their perceptions of their behaviors.
The actions of the team can be compared with current resuscitation algorithms, professional stan-
dards, institution policies, best evidence, and local protocols to enhance understanding and support
discussion during the analysis phase of the debriefing.

REFERENCES
Agency for Healthcare Research and Quality. (2014, Oct). TeamSTEPPS Fundamentals Course: Module 2, Team
structure. Retrieved from TeamSTEPPS 2.0: https://www.ahrq.gov/teamstepps/instructor/fundamentals/module2/
igteamstruct.html.
Agency for Healthcare Research and Quality. (2019a, Mar). TeamSTEPPS Fundamentals Course: Module 3, Com-
munication. Retrieved from TeamSTEPPS 2.0: https://www.ahrq.gov/teamstepps/instructor/fundamentals/
module3/igcommunication.html#calloutis.
Agency for Healthcare Research and Quality. (2019b, Mar). TeamSTEPPS Fundamentals Course: Module 5, Situation
monitoring. Retrieved from TeamSTEPPS 2.0: https://www.ahrq.gov/teamstepps/instructor/fundamentals/
module5/igsitmonitor.html.
Agency for Healthcare Research and Quality. (2019c, Mar). TeamSTEPPS Fundamentals Course: Module 6, Mutual
support. Retrieved from TeamSTEPPS 2.0: https://www.ahrq.gov/teamstepps/instructor/fundamentals/module6/
igmutualsupp.html.
Agency for Healthcare Research and Quality Patient Safety Network (PSNet). (2019, Jan). Rapid response systems.
Retrieved from U.S. Department of Health and Human Services: https://psnet.ahrq.gov/primers/primer/4/
rapid-response-systems.
Andersen, L. W., Kim, W. Y., Chase, M., et al. (2016). The prevalence and significance of abnormal vital signs prior
to in-hospital cardiac arrest. Resuscitation, 98, 112–117.
Baile, W. F., Buckman, R., Lenzi, R., Glober, G., Beale, E. A., & Kudelka, A. P. (2000). SPIKES-A six-step
protocol for delivering bad news: application to the patient with cancer. Oncologist, 5(4), 302–311.
Berg, K. M., Cheng, A., Panchal, A. R., et al. (2020). Part 7: Systems of care: 2020 American Heart Association
guidelines for cardiopulmonary resuscitation and emergency cardiovascular care. Circulation, 142(16 suppl 2),
S580–S604.
Catchpole, K. R., de Leval, M. R., McEwan, A., et al. (2007). Patient handover from surgery to intensive
care: using Formula 1 pit-stop and aviation models to improve safety and quality. Paediatr Anaesth, 17(5),
470–478.
Cochrane, D. D., Muniak, A., & Kennedy, C. (2018). Teams, competence, and safety in surgery. In D. J. Guillaume,
& M. A. Hunt (Eds.), Quality and safety in neurosurgery (pp. 79–95). London, United Kingdom: Elsevier.
Douw, G., Schoonhoven, L., Holwerda, T., et al. (2015). Nurses’ worry or concern and early recognition of
deteriorating patients on general wards in acute care hospitals: A systematic review. Crit Care, 19(1), 230.
Edelson, D. P., Sasson, C., Chan, P. S., et al. (2020). Interim guidance for basic and advanced life support in adults,
children, and neonates with suspected or confirmed COVID-19. Circulation, e20201405 [online ahead of print].
32 CHAPTER 2  Teams

Fanning, R. M., & Gaba, D. M. (2007). The role of debriefing in simulation-based learning. Simul Healthc, 2(2),
115–125.
Fletcher, G., Flin, R., Glavin, R. J., & Maran, N. (2012). Framework for observing and rating anaesthetists’
nontechnical skills. In Anaesthetists’ non-technical skills (ANTS) system handbook v1.0 (pp. 1–15). Aberdeen,
Scotland: University of Aberdeen.
Gillman, L. M., Brindley, P. G., Blaivas, M., Widder, S., & Karakitsos, D. (2016). Trauma team dynamics. J Crit
Care, 32, 218–221.
Hopkins, C. L., Burk, C., Moser, S., Meersman, J., Baldwin, C., & Youngquist, S. T. (2016). Implementation of
pit crew approach and cardiopulmonary resuscitation metrics for out-of-hospital cardiac arrest improves
patient survival and neurological outcome. J Am Heart Assoc, 5(1), 1–10.
Hunziker, S., Johansson, A. C., & Tschan F., et al. (2011). Teamwork and leadership in cardiopulmonary resuscita-
tion. J Am Coll Cardiol, 57(24), 2381–2388.
Jackson, S. A. (2017). Rapid response teams: What’s the latest? Nursing, 2017, 47(12), 35–41.
Kelley, R. E. (1988). In praise of followers. Harv Bus Rev, 66, 142–148.
Kessler, D. O., Cheng, A., & Mullan, P. C. (2015). Debriefing in the emergency department after clinical events:
a practical guide. Ann Emerg Med, 65(6), 690–698.
Lyons, P. G., Edelson, D. P., & Churpek, M. M. (2018). Rapid response systems. Resuscitation, 128, 191–197.
Mathukia, C., Fan, W., Vadyak, K., Biege, C., & Krishnamurthy, M. (2015). Modified Early Warning System
improves patient safety and clinical outcomes in an academic community hospital. J Community Hosp Intern
Med Perspect, 5(2), 26716.
McCurdy, M. T., & Wood, S. L. (2012). Rapid response systems: Identification and management of the “prearrest
state.” Emerg Med Clin North Am, 30(1), 141–152.
Meaney, P. A., Bobrow, B. J., Mancini, M. E., et al. (2013). Cardiopulmonary resuscitation quality: [corrected]
Improving cardiac resuscitation outcomes both inside and outside the hospital: A consensus statement from
the American Heart Association. Circulation, 128(4), 417–435.
Morrison, L. J., Neumar, R. W., Zimmerman, J. L., et al. (2013). Strategies for improving survival after in-hospital
cardiac arrest in the United States: 2013 consensus recommendations: A consensus statement from the American
Heart Associations. Circulation, 127(14), 1538–1563.
Mullan, P. C., Cochrane, N. H., Chamberlain, J. M., et al. (2017). Accuracy of postresuscitation team debriefings
in a pediatric emergency department. Ann Emerg Med, 70(3), 311–319.
Nallamothu, B. K., Guetterman, T. C., Harrod, M., et al. (2018). How do resuscitation teams at top-performing
hospitals for in-hospital cardiac arrest succeed? A qualitative study. Circulation, 138(2), 154–163.
Phrampus, P. E., & O’Donnell, J. M. (2013). Debriefing using a structured and supported approach. In A. I. Levine,
S. DeMaria Jr., A. D. Schwartz, & A. J. Sim (Eds.), The comprehensive textbook of healthcare simulation (pp. 73–84).
New York: Springer Science.
Quest, T. E. (2018). End of life. In R. M. Walls, R. S. Hockberger, & M. Gausche-Hill (Eds.), Rosen’s emergency
medicine: concepts and clinical practice (9th ed., pp. e93–e104). Philadelphia, PA: Elsevier.
Sebat, C., Sinigayan, V., & Albertson, T. (2017). Hospital rapid response systems. Hosp Med Clin, 6(4), 480–491.
Spitzer, C. R., Evans, K., Buehler, J., Ali, N. A., & Besecker, B. Y. (2019). Code blue pit crew model: A novel
approach to in-hospital cardiac arrest resuscitation. Resuscitation, 143, 158–164.
Yeung, J., Scapigliati, A., Hsieh, M., et al. (2020, Jan). Systematic review: Rapid response systems in adults - Consensus
on Science with Treatment Recommendations [Internet]. Brussels, Belgium: International Liaison Committee on
Resuscitation (ILCOR) Education, Implementation and Teams Task Force. Retrieved from ILCOR: Consensus
on Science with Treatment Recommendations (CoSTR): https://costr.ilcor.org/document/rapid-response-
systems-in-adults-systematic-review.
C H A P T E R 3
Patient Assessment and
Resuscitation Therapies

INTRODUCTION
Recognizing when a patient’s condition becomes unstable requires good patient assessment skills
and is essential for improved patient outcomes. This chapter reviews a systematic approach to patient
assessment. It also discusses common therapies performed by members of a rapid response or resus-
citation team in the first few minutes of a respiratory or cardiovascular emergency.

L E A R N I N G OBJECTIVES
After completing this chapter, you should be able to:
1. Describe the purpose and components of a basic life support (BLS) assessment.
2. Differentiate between the purposes and components of the primary and secondary
assessments.
3. Describe the steps needed to perform the head tilt–chin lift and the jaw thrust
maneuvers and relate the mechanism of injury to the opening of the airway.
4. Describe the procedure for suctioning the upper airway, and discuss possible
complications associated with this procedure.
5. Discuss the indications, contraindications, advantages, and disadvantages of
pharyngeal airways, and demonstrate how to correctly size and insert these airway
adjuncts.
6. Give examples of the methods used to deliver positive pressure ventilation.
7. Describe the oxygen liter flow per minute and the estimated inspired oxygen
concentration delivered with a pocket face mask and a bag-mask device (BMD).
8. Describe how to ventilate a patient with a BMD and two rescuers.
9. Recognize the signs of adequate and inadequate bag-mask ventilation (BMV).
10. Differentiate between supraglottic airways (SGAs) and intraglottic airways.
11. Give examples of the methods used to confirm correct endotracheal tube (ETT)
placement.
12. Describe the advantages, disadvantages, oxygen liter flow per minute, and estimated
oxygen percentage delivered with each of the following devices: nasal cannula,
simple face mask, partial nonrebreather mask, and nonrebreather mask.
13. Describe the normal sequence of electrical conduction through the heart.
14. Describe the location, the function, and, where appropriate, the intrinsic rate of the
following structures: the sinoatrial (SA) node, the atrioventricular (AV) bundle, and the
Purkinje fibers.
33
34 CHAPTER 3  Patient Assessment and Resuscitation Therapies

15. Relate the cardiac surfaces or areas represented by the electrocardiogram (ECG)
leads.
16. Explain defibrillation, synchronized cardioversion, and transcutaneous pacing
(TCP), the indications for each procedure, and the steps required to perform
them safely.

LEARNING PL AN
• Read this chapter before class. Take the time to highlight important concepts as you
read.
• Master the following skills:
n Ensure scene safety and the use of personal protective equipment.
n Assign team member roles or perform as a team member in a simulated patient
situation.
n Direct or perform a BLS assessment, primary assessment, and secondary
assessment.
n Demonstrate manual methods for opening the airway.
n Demonstrate the procedure for suctioning the upper airway.
n Demonstrate how to size and insert an oropharyngeal airway (OPA) and a
nasopharyngeal airway (NPA).
n Perform two-rescuer BMV when indicated.
n Demonstrate how to troubleshoot inadequate BMV.
n Demonstrate how to confirm the correct positioning of an ETT.
n Demonstrate the application of ECG monitoring leads.
n Demonstrate how to perform defibrillation, synchronized cardioversion, and TCP
safely.
n Review your performance as a team leader or team member during a postevent
debriefing.
• Develop and use flashcards, flowcharts, and mnemonics to help enhance your
retention of the information presented.
• Complete the quiz at the end of this chapter and review the quiz answers provided.

KEY TERMS
Accessory pathway  An extra bundle of working myocardial tissue that forms a
connection between the atria and ventricles outside the normal conduction
system.
Automated external defibrillation  The placement of pads on a patient’s chest and
interpretation of the patient’s cardiac rhythm by the defibrillator’s computerized
analysis system. Depending on the type of automated external defibrillator (AED) used,
the machine will deliver a shock (if a shockable rhythm is detected) or instruct the
operator to deliver a shock.
Bundle of His  Fibers located in the upper portion of the interventricular septum that
conduct an electrical impulse through the heart.
Capnography  The graphic representation of the exhaled carbon dioxide concentration in
respiratory gases.
Capnometer  A monitoring device that measures and numerically displays the
concentration of exhaled carbon dioxide.
Conduction system  A system of pathways in the heart composed of specialized
electrical (i.e., pacemaker) cells.
Defibrillation  Delivery of an electrical current across the heart muscle over a very brief
period to terminate an abnormal heart rhythm; also called unsynchronized
countershock or asynchronous countershock because the delivery of current has no
relationship to the cardiac cycle.
Electrocardiogram (ECG)  A recording of the heart’s electrical activity from the body
surface that appears on ECG paper as specific waveforms and complexes.
CHAPTER 3  Patient Assessment and Resuscitation Therapies 35

His-Purkinje system  Portion of the conduction system consisting of the bundle of His,
bundle branches, and Purkinje fibers.
Lead  A record (i.e., tracing) of electrical activity between two electrodes.
Manual defibrillation  The placement of adhesive pads on a patient’s chest, interpretation
of the patient’s cardiac rhythm by a trained healthcare professional, and the healthcare
professional’s decision to deliver a shock (if indicated).
Oxygenation  The process of getting oxygen into the body and to its tissues for
metabolism.
Respiration  The exchange of oxygen and carbon dioxide during cellular metabolism.
Symptomatic bradycardia  A term used to describe a patient who experiences signs and
symptoms of hemodynamic compromise related to a slow heart rate.
Synchronized cardioversion  A type of electrical therapy during which a shock is timed or
programmed for delivery during ventricular depolarization (i.e., the QRS complex).
Ventilation  The mechanical movement of gas or air into and out of the lungs.

PART I: PATIENT ASSESSMENT


Regardless of your level of licensure or certification and the environment in which you work, a sys-
tematic approach to patient assessment is essential to identify a patient’s illness or injury, ascertain
its severity, determine the need for emergency care, and evaluate their response to the treatment
provided.

BASIC LIFE SUPPORT ASSESSMENT


A BLS assessment, also called a rapid assessment, can be performed by any healthcare provider
to determine if resuscitation is needed (Box 3.1). After providing initial emergency care, evaluation
of the patient continues with a primary assessment and, if time and conditions permit, a secondary
assessment.
Before approaching the patient, make sure that the scene is safe. Note any hazards or potential
hazards and any visible mechanism of injury or illness. To minimize their risk of infectious disease
exposure, clinicians should presume that any patient with a respiratory complaint is coronavirus disease
2019 (COVID-19) positive until proved otherwise and don full airborne personal protective equip-
ment (PPE) before approaching the patient. Appropriate PPE includes a mask, eye protection, a gown,
and disposable gloves (MEGG). Many organizations have adopted the practice of assuming that all
patients are COVID-19 positive until proved otherwise.
Once you come into view of the person, form a general impression. Forming a general impression
helps you gauge the severity of their condition and quickly determine if a life-threatening problem
exists that requires immediate care. Many agencies are using a “scout” approach when forming a general
impression in which one provider, wearing PPE, forms a general impression of the person while posi-
tioned at least 6 feet from him or her. When the general impression is complete, the scout notifies the
remainder of the team of their initial findings, the PPE necessary to enter the patient area, the level
of medical care required, and if additional personnel or resources are needed. Box 3.2 shows examples
of information that you may obtain when forming a general impression.

BOX 3.1 Basic Life Support Assessment


• Ensure that the scene is safe
• Assess for responsiveness, activate the emergency response system when warranted
• Simultaneously assess for breathing and a pulse
• Provide indicated care
36 CHAPTER 3  Patient Assessment and Resuscitation Therapies

BOX 3.2 Forming a General Impression


• Apparent degree of distress
• Dress, hygiene, grooming (e.g., clothing appropriate to the climate, fits well, hair clean and
brushed)
• Eye contact (e.g., aware of your approach) and facial expression (e.g., sad, angry, fearful,
anxious)
• Odors (e.g., breath, body, urine, feces)
• Position (e.g., sitting, standing, or lying comfortably; tripod position, fetal position, prone,
supine)
• Presence of life-threatening bleeding
• Skin signs (e.g., pink, pale, cyanotic, mottled, flushed, ashen, diaphoretic)
• Sounds (e.g., moaning, stridor, gurgling, wheezing, snoring)
• Work of breathing (e.g., pursed-lip breathing, use of accessory muscles, leaning forward to
inhale, retractions)

If the scene is safe, and after forming a general impression, approach the person and assess respon-
siveness using the AVPU acronym: A 5 Alert, V 5 Responds to verbal stimuli, P 5 Responds to
painful stimuli, and U 5 Unresponsive. Start by asking, “Are you all right?” or “Can you hear me?” If
there is no response, then gently tap or squeeze the person’s shoulder while repeating verbal cues.
If the person is responsive, proceed systematically, starting with the primary assessment and then
the secondary assessment, which are discussed later. If the person is unresponsive, ask someone to ac-
tivate the emergency response system and get a defibrillator while you continue patient care. After
ensuring that the person is positioned face-up on a firm, flat surface, look for signs of normal breathing
while simultaneously checking for a carotid pulse for 5 to 10 seconds. If the person is breathing nor-
mally and is uninjured, place them in a recovery position and perform a primary assessment. If
the person is not breathing normally, but a pulse is present, provide rescue breathing. If opioid overdose
is suspected, give naloxone per your agency’s protocol. If the person is not breathing, or only gasping,
and does not have a pulse (or you are unsure if there is a pulse), start cardiopulmonary resuscitation
(CPR) immediately, beginning with chest compressions. As soon as it arrives, attach an automated
external defibrillator (AED) and follow its prompts. Control bleeding if there is visible external
hemorrhage.

PRIMARY ASSESSMENT
The primary assessment is a rapid hands-on evaluation to detect the presence of life-threatening prob-
lems affecting the patient’s respiratory, cardiac, or neurologic function and to correct them immediately.
If the patient is responsive, continue your assessment using the following sequence: Airway, Breathing,
Circulation, Disability (referring to a brief neurologic exam), and Exposure (Table 3.1). If the patient
is unresponsive, use the following approach: Circulation, Airway, Breathing, Disability, and Exposure.

Airway
If the patient is responsive, ask questions to determine their level of responsiveness and the adequacy
of their airway and breathing. Observe for agitation, confusion, restlessness, or combativeness, which
may be the result of hypoxia. Also observe if the patient can speak in sentences before requiring
a breath or if they experience shortness of breath after speaking only a few words. To minimize
the risk of infectious disease spread, ask a patient who is coughing, sneezing, or expectorating to wear
a mask. If the patient is unresponsive, manual maneuvers, such as a head tilt–chin lift or jaw thrust,
may be needed to open the patient’s airway. Manual airway maneuvers are discussed later in this
chapter.
Assess for sounds of airway compromise, such as snoring, gurgling, or stridor. If the airway is
not clear, clear it with suctioning, patient positioning, or both, as indicated. Decide if the patient
can protect their airway. If needed, insert an OPA or NPA to keep the airway open and consider the
need for insertion of an advanced airway. If the airway is open, move on and assess the patient’s
breathing.
CHAPTER 3  Patient Assessment and Resuscitation Therapies 37

TABLE 3.1 Primary Assessment


Assessment Possible Interventions
Airway
• Is the airway open? • Use head tilt–chin lift or jaw thrust if needed to
• Is the airway clear? open the airway
• Can the patient protect their airway? • Clear airway obstruction, if present
• Suction as needed
• Insert oropharyngeal or nasopharyngeal airway
if required to maintain open airway; consider
advanced airway
Breathing
• Is breathing adequate or inadequate? • Assist ventilations if breathing is inadequate or absent
• Assess lung sounds • Use pulse oximetry to assess the adequacy of
• Evaluate depth, rate, rhythm, and symmetry oxygenation
of movement with each breath • Give supplemental oxygen if indicated
• Measure adequacy of oxygenation • Use capnography to assess ventilation effectiveness,
• Assess ventilation effectiveness if available
• Give naloxone if opioid overdose is suspected
Circulation
• Look for life-threatening bleeding • Control hemorrhage, if present
• Evaluate pulse rate, rhythm, and quality (e.g., • If there is no pulse, begin chest compressions until
fast/slow, regular/irregular, weak/strong) an AED is available
• Assess perfusion (e.g., skin temperature, color, • Attach ECG electrodes and connect the patient
and condition [moisture]) to an ECG monitor; obtain a 12-lead ECG if
• Establish cardiac monitoring appropriate
• Assess the need for electrical therapy (e.g., • Perform defibrillation, cardioversion, or pacing
defibrillation, cardioversion, pacing) as indicated
• Establish vascular access (IV or IO) • Give fluids and medications as indicated
Disability
• Reassess responsiveness • Use AVPU or Glasgow Coma Scale; use the
• Measure blood glucose level, if indicated National Institutes of Health Stroke Scale or
• Assess pupil size, equality, reactivity to light similar tool if stroke is suspected
• Perform ongoing neurologic assessments
Exposure
Expose the patient as necessary Remove clothing as necessary to facilitate examination
Vital Signs
Assess vital signs Attach a pulse oximeter, cardiac monitor, and BP
monitor, if not already done
AED, Automated external defibrillator; AVPU, A 5 Alert, V 5 Responds to Verbal stimuli, P 5 Responds to Painful stimuli,
and U 5 Unresponsive; BP, blood pressure; ECG, electrocardiogram; IO, intraosseous; IV, intravenous.

Breathing
If the patient is breathing, determine if breathing is adequate or inadequate (Table 3.2). The evaluation of
a patient’s breathing should include an assessment of the depth of breathing, ventilation rate, and sym-
metry of movement with each breath. Ventilation (which is often misnamed respiration) is the mechani-
cal movement of air into and out of the lungs. Respiration is the exchange of oxygen and carbon dioxide
during cellular metabolism. During normal, quiet breathing, an adult male moves an average of 500 mL
(5 to 7 mL/kg) of air into and out of the respiratory tract with each breath; this amount is called the tidal
volume. Chest expansion should be adequate with sufficient tidal volume to make the chest rise equally
with no excessive use of accessory muscles during inspiration or expiration. Look for signs of increased
work of breathing, such as pursed-lip breathing, use of accessory muscles, leaning forward to inhale, or
retractions. Auscultate breath sounds to detect decreased ventilation, crackles, wheezes, or rhonchi.
In addition to clinical assessments, use pulse oximetry to assess the adequacy of oxygenation. Give
100% oxygen during respiratory and cardiac arrest. For most patients who are not in respiratory or
38 CHAPTER 3  Patient Assessment and Resuscitation Therapies

TABLE 3.2 Signs of Adequate and Inadequate Breathing


Adequate Breathing Inadequate Breathing
Rate 12 to 20 breaths per minute (adult) Ventilatory rate outside normal range for age
at rest and situation
Rhythm Regular Irregular
Quality • Breath sounds clear and equal • Abnormal breath sounds (stridor, wheezing,
bilaterally crackles, silent chest)
• Chest expansion adequate and • Chest expansion unequal or inadequate
equal with each breath (paradoxical, splinting, asymmetric)
• No excessive use of accessory • Increased effort (work) of breathing (retrac-
muscles during inhalation or tions, accessory muscle use, sweating, tripod
exhalation position, flared nostrils, or pursed lips)
Depth (tidal volume) Adequate Inadequate; unusually deep or shallow
Skin Normal color, warm, dry Pale, cyanotic; cool, clammy

cardiac arrest, target an oxygen saturation of 95% to 98%. Target an oxygen saturation of 90% for acute
coronary syndromes and target an oxygen saturation of 92% to 98% during postcardiac arrest care. If a
nasal cannula is used but the patient has known or suspected COVID-19, consider applying a surgical
mask over the nasal cannula to decrease virus spread.
Use capnography, if available, to assess the effectiveness of the patient’s ventilation. Depending on the
situation, assist the patient’s breathing using noninvasive positive pressure ventilation (NPPV), mouth-
to-mask ventilation, or BMV if ventilatory efforts are inadequate. If the patient requires breathing as-
sistance and COVID-19 is known or suspected, use a BMD with a high-efficiency particulate air
(HEPA) filter and ensure a tight face to mask seal (Edelson et al., 2020).
Give naloxone if an opioid overdose is suspected. Oxygen delivery devices and techniques of positive
pressure ventilation are discussed later in this chapter. If the patient’s breathing is adequate, move on
to assessment of circulation.

Circulation
Look for life-threatening bleeding and control hemorrhage, if present. Quickly assess the patient’s
heart rate, pulse quality (i.e., fast or slow, regular or irregular, weak or strong), and skin temperature,
color, and moisture to assess perfusion. If there is no pulse, begin chest compressions until an AED is
available. Establish cardiac monitoring and obtain a 12-lead ECG if appropriate. Assess the need
for electrical therapy (e.g., defibrillation, cardioversion, pacing) and perform as indicated. Establish
vascular access (intravenous [IV] or intraosseous [IO]). Give fluids and medications as needed.

Disability
Reassess the patient’s level of responsiveness using AVPU or the Glasgow Coma Scale; use the
National Institutes of Health Stroke Scale or similar tool if stroke is suspected. Measure the patient’s
blood glucose level to rule out hypoglycemia, if indicated. Assess pupil size, equality, and reactivity to
light. Perform ongoing neurologic assessments as indicated.

Exposure
Expose pertinent areas of the patient’s body to facilitate examination. Doing so may reveal a medical
identification bracelet or necklace, implanted pacemaker or defibrillator, surgical scars, signs of trauma, or
other significant findings. When you are finished, respect the patient’s privacy and recover exposed areas.

Vital Signs
After managing life threats, measure the patient’s vital signs to establish a baseline from which subse-
quent measurements can be compared. If not already done, attach a pulse oximeter, cardiac monitor,
and blood pressure (BP) monitor.
CHAPTER 3  Patient Assessment and Resuscitation Therapies 39

ACLS Pearl
Pulse oximetry provides important information about oxygenation but does not provide information
about the effectiveness of a patient’s ventilation. Capnography provides information about the efficacy
of ventilation but does not measure oxygenation. Using pulse oximetry and/or capnography are ad-
juncts to—not a replacement of—vigilant patient assessment. You must correlate your assessment
findings with readings from these tools to determine appropriate treatment interventions for the patient.

ACLS Pearl
Repeat the primary assessment:
• With any sudden change in the patient’s condition
• When interventions do not appear to be working
• When vital signs are unstable
• Before any procedures are performed
• When a change in rhythm is observed on the cardiac monitor

SECONDARY ASSESSMENT
A secondary assessment is performed after all life-threatening conditions have been identified and
managed. During this portion of the patient assessment, a focused history is obtained using the
SAMPLE mnemonic (Box 3.3), a focused physical examination is performed, and diagnostic tests
are obtained. The history is often acquired while the physical examination is being performed, and
emergency care is being given. The goals of the secondary assessment are to consider the information
obtained during the primary assessment, as well as the history, physical, and diagnostic data obtained
during the secondary assessment to formulate a differential diagnosis, identify potentially reversible
causes, and implement a treatment plan.
During the secondary assessment, reassess the effectiveness of initial airway maneuvers and inter-
ventions. If needed, insert an advanced airway. If endotracheal intubation is to be performed and
COVID-19 is known or suspected, ask the team member with the highest likelihood of first-pass suc-
cess to perform the procedure. If an advanced airway has been inserted, confirm proper placement
using clinical assessment and waveform capnography. Make sure the tube is adequately secured. Obtain
a chest radiograph to confirm appropriate placement.
Reassess lung sounds, chest rise, and the adequacy of oxygenation and ventilation. If the patient has
a pulse, check its rate and quality often. If not already done, attach ECG electrodes, connect the patient
to a cardiac monitor, and obtain a 12-lead ECG if appropriate. Perform defibrillation, cardioversion,
or pacing as indicated. If not already done, establish vascular access and give medications appropriate
for the cardiac rhythm/clinical situation.
Search for, find, and treat reversible causes of the cardiac arrest, rhythm, or clinical situation. Reas-
sess the effectiveness of the care given thus far and troubleshoot as needed. If the patient is responsive
and complaining of discomfort, begin appropriate pain management if their BP and other vital signs
will tolerate it.
Facilitate family presence for invasive and resuscitative procedures following your organization’s
policies and procedures. In many organizations, the role of the family support person is fulfilled by an
emergency nurse, a member of the pastoral care team, or a social worker. Explain what is being done
for the patient to family members who are present.

BOX 3.3 SAMPLE History


Signs and symptoms
Allergies
Medications
Past medical history
Last oral intake
Events surrounding the illness or injury
40 CHAPTER 3  Patient Assessment and Resuscitation Therapies

PART II: RESUSCITATION THERAPIES


Therapies performed by members of a rapid response or resuscitation team in the first few minutes of
a respiratory or cardiovascular emergency are typically related to airway and breathing management,
ECG monitoring and electrical therapies, and vascular access and medications. The performance of
high-quality CPR, which is vital during a cardiac arrest, was discussed in Chapter 1.

AIRWAY AND BREATHING MANAGEMENT


The possibility of airborne pathogen transmission related to droplet and aerosol formation must be
considered when managing a patient’s airway and breathing. Be sure to use hand hygiene, eye protection
(e.g., face shield or goggles), a mask or other respiratory protective device, gloves, and a gown.

Tools for Assessing Oxygenation and Ventilation


A pulse oximeter, which is commonly called a pulse ox, is a small instrument with a light sensor that
quickly calculates the percentage of hemoglobin that is saturated with oxygen in a pulsating capillary
bed. This calculation is called the saturation of peripheral oxygen (SpO2). The oximeter displays this
value as a percentage and the patient’s pulse rate on its screen. The oximeter’s sensor is typically applied
to a finger, but the forehead, an earlobe, or a toe can also be used with the selection of a sensor that is
appropriate for the chosen site.
Capnography is the graphic representation of the exhaled carbon dioxide (CO2) concentration in
respiratory gases. Capnography provides healthcare professionals with breath-to-breath patient infor-
mation, thereby enabling the early recognition of hypoventilation, apnea, or airway obstruction and
thus preventing hypoxic episodes. A capnometer is a monitoring device that measures and numerically
displays the concentration of exhaled CO2.
Alveolar CO2 and arterial CO2 (PaCO2) values are closely related in patients with normal cardio-
pulmonary function, and they usually range between 35 and 45 millimeters of mercury (mm Hg). In
patients with healthy lung and cardiac function, normal values for end-tidal carbon dioxide (EtCO2)
are typically within the same range but are dependent on adequate ventilation and adequate perfusion.
A change in either factor can increase or decrease the amount of exhaled CO2.
Digital capnometers use infrared technology to analyze exhaled gas and provide a numeric display
of exhaled CO2. Awareness of this value is beneficial as trends in CO2 levels can be monitored, and
the effectiveness of treatment can be determined. A colorimetric capnometer functions through a
pH change that occurs with the breath of a patient. The patient’s breath causes a chemical reaction on
pH-sensitive litmus paper housed in the detector. The capnometer is placed between a tracheal tube or
other advanced airway device and a ventilation device (Fig. 3.1). The presence of CO2, which is evi-
denced by a color change on the colorimetric device, suggests placement of the tube in the trachea. A
colorimetric capnometer is qualitative in that it simply shows the presence of CO2. It does not provide
an actual CO2 reading or indicate the presence of hypercarbia, and it does not enable ongoing monitor-
ing to ensure that the tube remains in the trachea. A lack of CO2 (i.e., no color change) suggests tube
placement in the esophagus, particularly in patients with a perfusing rhythm (i.e., not in cardiac arrest).
Some manufacturers of colorimetric capnometers recommend ventilating the patient at least 6 times
before attempting to use an exhaled CO2 detector to assess tracheal tube placement. The rationale for this
action is to quickly wash out any retained CO2 that is present in the stomach or esophagus as a result of
BMV. Any CO2 that is detected after six positive pressure ventilations can be presumed to be from the lungs.
Colorimetric capnometers are susceptible to inaccurate results because of the age of the paper and
exposure of the paper to the environment. A colorimetric capnometer may not change color if the paper
is contaminated with patient secretions (e.g., vomitus) or acidic drugs (e.g., tracheally administered
epinephrine) (Cantineau et al., 1994). When CO2 is not detected, use an alternative method to confirm
tracheal tube placement, such as direct visualization or the use of an esophageal detector device (EDD).

Manual Airway Maneuvers


The most common cause of partial airway obstruction in an unresponsive patient is the result of a loss
of muscle tone, which causes the tongue to fall back into the pharynx and block airflow. Manual airway
maneuvers are performed to lift the tongue off the back of the throat and open the airway.
CHAPTER 3  Patient Assessment and Resuscitation Therapies 41

Fig. 3.1  ​Colorimetric exhaled carbon dioxide detector. (From Sole, M.L., Klein, D.G., Moseley, M.J. [2017]. Introduction to
critical care nursing, ed 7, St Louis, Elsevier.)

If the unresponsive patient is breathing, snoring sounds are a sign of airway obstruction from the
displacement of the tongue. If the patient is not breathing, airway obstruction from the tongue may go
undetected until positive pressure ventilation is attempted. Ventilating a nonbreathing patient with an
airway obstruction is difficult. If the tongue is the cause of the airway obstruction, repositioning the
patient’s head and jaw may be all that is needed to open the airway.

ACLS Pearl
In the unresponsive patient, a partial or complete airway obstruction can result when the muscles
of the tongue and laryngopharynx relax, thus allowing the tongue and other soft tissues to block
the opening of the laryngopharynx.

Head Tilt–Chin Lift


The head tilt–chin lift is the preferred technique for opening the airway of an unresponsive patient
without a suspected cervical spine injury. Follow these steps to perform a head tilt–chin lift:
1. Place the patient in a supine position.
2. Place one hand on the patient’s forehead and apply downward pressure with your palm to gently tilt
the patient’s head back (Fig. 3.2).

Fig. 3.2  ​Using the head tilt–chin lift maneuver to open the airway. (From Kacmarek, R.M., Stoller, J.K., Heuer, A.J. [2017].
Egan’s fundamentals of respiratory care, ed 11, St. Louis, Elsevier.)
42 CHAPTER 3  Patient Assessment and Resuscitation Therapies

3. Place the tips of the fingers of your other hand under the bony part of the patient’s chin, and gently
lift and pull the jaw forward. Positioning your fingers under the bony part of the patient’s chin is
important because compression of the soft tissue under the patient’s chin can obstruct the airway.
4. If needed, open the patient’s mouth by pulling down on the patient’s lower lip using the thumb of
the same hand used to lift the chin.

Jaw Thrust
A jaw thrust maneuver (also called the modified jaw thrust) is the technique that is recommended for
opening the airway when cervical spine injury is suspected. This maneuver may be performed with or
without an accompanying head tilt. For patients who are unresponsive without any risk of spinal injury,
perform the following technique:
1. With the patient supine, position yourself above the patient’s head or at their side, looking at the
face.
2. Place your fingers on each side of the lower jaw at the angle of the jaw near the bottom of the patient’s
ears (Fig. 3.3).
3. Lift the jaw forward toward the patient’s face and gently open the mouth.
4. Gently tilt the patient’s head while maintaining displacement of the lower jaw.
The jaw thrust maneuver is a difficult technique for one person to manage. In most cases,
one rescuer is needed to displace the patient’s lower jaw forward while a second rescuer ventilates
the patient. Healthcare professionals should use the head tilt–chin lift maneuver to open the
airway if use of the jaw thrust maneuver is unsuccessful. Table 3.3 summarizes manual airway
maneuvers.

Fig. 3.3  ​The jaw thrust maneuver is used to open the airway when cervical spine injury is suspected. (From Kacmarek,
R.M., Stoller, J.K., Heuer, A.J. [2017]. Egan’s fundamentals of respiratory care, ed 11, St. Louis, Elsevier.)

TABLE 3.3 Manual Airway Maneuvers


Considerations Head Tilt-Chin Lift Jaw Thrust Without Neck Extension
Indications • Unresponsive patient with no mechanism • Unresponsive patient with a possible
for cervical spine injury cervical spine injury
Advantages • Simple to perform • No equipment required
• No equipment required • Noninvasive
• Noninvasive
Disadvantages • Does not protect the lower airway from • Difficult to maintain
aspiration • Second rescuer needed for bag-mask
• May cause spinal movement ventilation
• Does not protect the lower airway from
aspiration
• May cause spinal movement
CHAPTER 3  Patient Assessment and Resuscitation Therapies 43

SUCTIONING
Suctioning is performed for the following reasons:
• To remove vomitus, saliva, blood, or foreign material from the patient’s airway
• To maintain patency of an artificial airway (e.g., ETT, tracheostomy tube)
• To improve gas exchange by allowing air to pass through to the lower airway
• To obtain secretions for diagnosis
Rigid suction catheters, also called tonsil tip or Yankauer catheters, are made of hard plastic and
angled to help with the removal of secretions from the mouth and throat (Fig. 3.4). Because of its size,
a rigid suction catheter is not used to suction the nares, except externally. The catheter typically has
several openings at the distal end through which particles may be suctioned.
Flexible suction catheters are also called whistle tip, soft, or French catheters. They are long, narrow,
flexible pieces of plastic that are used to clear blood or mucus from the oropharynx or nasopharynx, an
ETT, or a tracheostomy tube (Fig. 3.5). When suctioning the lower airway, the outer diameter of the
suction catheter should be no more than half the internal diameter of the tracheal or tracheostomy tube
to minimize the risk of atelectasis and hypoxemia when suction is applied (Tiffin et al., 1990).
Insert a suction catheter without applying suction. Apply suction as the catheter is withdrawn and
do not apply suction for more than 10 seconds to avoid hypoxia. After suctioning, reevaluate airway
patency and auscultate lung sounds. Document the amount, color, and consistency of any secretions
that are obtained. Box 3.4 shows possible complications of suctioning.

ACLS Pearl
Stimulation of the larynx by a suction catheter, laryngoscope blade, or tracheal tube can result in
bradycardia, hypotension, and a decreased ventilatory rate because the larynx is innervated with
nerve endings from the vagus nerves. Monitor the patient closely for these effects and discontinue
the treatment that is causing them if they appear.

Tip

Catheter

Thumb-control
valve

Handle

Large openings at tip

Connector for vacuum

Vacuum tubing

Fig. 3.4  ​A rigid suction catheter is used to remove secretions from the mouth and throat. (From Kacmarek, R.M., Stoller, J.K.,
Heuer, A.J. [2017]. Egan’s fundamentals of respiratory care, ed 11, St. Louis, Elsevier.)
44 CHAPTER 3  Patient Assessment and Resuscitation Therapies

Thumb-control valve

Connector for vacuum

Catheter
Vacuum tubing
Tip with a single opening

Patient end
Fig. 3.5  ​A flexible suction catheter is used to remove secretions from the lower airway. (From Kacmarek, R.M., Stoller,
J.K., Heuer, A.J. [2017]. Egan’s fundamentals of respiratory care, ed 11, St. Louis, Elsevier.)

BOX 3.4 Suctioning: Possible Complications


• Bronchospasm • Increased intracranial pressure
• Dysrhythmias • Local swelling
• Hemorrhage • Tracheal infection
• Hypertension or hypotension • Trauma to the tracheal or bronchial mucosa
• Hypoxia or hypoxemia

Pharyngeal Airways
Manual maneuvers facilitate the opening of an airway. Airway adjuncts, such as pharyngeal airways, are
devices that assist in keeping the airway open by keeping the tongue away from the posterior wall of
the pharynx.

Oropharyngeal Airways
An oropharyngeal airway (OPA), also called an oral airway, is a J-shaped plastic device that is used to
create an air passage between the patient’s mouth and the posterior wall of the pharynx. Because OPA
insertion may provoke vomiting and thus increase the risk of aspiration in a patient with an intact gag
reflex, indications for insertion include patients who are unresponsive and have no gag reflex. An OPA
may be used as a bite block after the insertion of a tracheal tube or an orogastric tube.
OPAs are available in a variety of sizes that range from 0 for neonates up to 6 for large adults. The
size of the airway is based on the distance, in millimeters, from the flange to the distal tip.
There are two main OPA designs. The Guedel airway (Flexicare, Irvine, CA) has a tubular configuration
with a single center channel that allows for ventilation and the passage of a suction catheter (Fig. 3.6A).
The Berman airway (Medline, Mundelein, IL) has two airway channels along each side of the device
through which a suction catheter can be passed to remove secretions from the back of the throat
(Fig. 3.6B). When correctly positioned, the flange of the device rests on the patient’s lips or teeth. The
distal tip lies between the base of the tongue and the back of the throat, thereby preventing the tongue
from blocking the airway (Fig. 3.6C). Air passes around and through the device.
Proper OPA size is determined by holding the device against the side of the patient’s face and
selecting an airway that extends from the corner of the mouth to the tip of the earlobe or the angle
of the jaw (Fig. 3.7). To prevent inaccurate measurements for patients who experience facial droop-
ing after a stroke, some experts recommend measuring from the first incisor or from the center of
the lips to the tip of the earlobe or to the angle of the jaw. If an OPA is too long, it may press
the epiglottis against the entrance of the larynx, which may result in a complete airway obstruction
(Fig. 3.8). If the airway is too short, it will not displace the tongue, and it may advance out of the
mouth (Fig. 3.9).
CHAPTER 3  Patient Assessment and Resuscitation Therapies 45

Oropharyngeal tube in place

Body (2)
Flange (1)

Channel (3)
A Flange (1)
Body (2)

Channel (3)

B C
Fig. 3.6  ​(A) Guedel airway. (B) Berman airway. (C) Oropharyngeal airway in place.  (From Kacmarek, R.M., Stoller, J.K.,
Heuer, A.J. [2017]. Egan’s fundamentals of respiratory care, ed 11, St. Louis, Elsevier.)

Fig. 3.7  ​Select an oropharyngeal airway of appropriate size. (From Roberts, J.R., Custalow, C.B., Thomsen, T.W. [2019].
Roberts and Hedges’ clinical procedures in emergency medicine and acute care, ed 7, Philadelphia, Elsevier.)

Fig. 3.8  ​An oropharyngeal airway that is too long may press the epiglottis against the entrance of the larynx, which may
result in a complete airway obstruction. (From McSwain, N., Paturas, J. [2003]. The basic EMT, ed 2, Mosby.)
46 CHAPTER 3  Patient Assessment and Resuscitation Therapies

Fig. 3.9  ​An oropharyngeal airway that is too short will not displace the tongue, and it may advance out of the mouth. (From
McSwain, N., Paturas, J. [2003]. The basic EMT, ed 2, Mosby.)

Fig. 3.10  ​Open the patient’s mouth and insert the oropharyngeal airway with the tip pointing toward the roof of the mouth.
(From Roberts, J.R., Custalow, C.B., Thomsen, T.W. [2019]. Roberts and Hedges’ clinical procedures in emergency medicine
and acute care, ed 7, Philadelphia, Elsevier.)

When inserting an OPA into a patient’s mouth, hold the device at its flange end and insert it with
the tip pointing toward the roof of the mouth (Fig. 3.10). As the distal end nears the back of the throat,
rotate the airway 180 degrees so that it is positioned over the tongue. Alternatively, the airway can be
inserted sideways and turned 90 degrees into position. When the OPA is inserted correctly, the flange
should rest comfortably on the patient’s lips or teeth. Continue to use the head tilt–chin lift or jaw
thrust maneuver to keep the airway open after the OPA is in place. Confirm proper placement of the
OPA by ventilating the patient. If the airway is placed correctly, chest rise should be visible, and breath
sounds should be present on auscultation of the lungs during ventilation. If the patient is not breathing
or if their breathing is inadequate, begin positive pressure ventilation.
Another method of OPA insertion requires the use of a tongue blade to depress the tongue. To use
this method, insert the airway with its tip facing the floor of the patient’s mouth (i.e., curved side
down). Depress the tongue using a tongue blade and gently advance the OPA into place over the
tongue.
If the patient’s gag reflex returns or if they spontaneously attempt to displace the OPA, remove the
airway to minimize the risk of aspiration.

Nasopharyngeal Airways
A nasopharyngeal airway (NPA, also called a nasal airway or nasal trumpet) is a soft, uncuffed tube
that is designed to keep the tongue away from the back of the throat. Indications for the use of a nasal
airway include unresponsive patients or those with an altered level of consciousness who continue to
have an intact gag reflex but who need assistance with maintaining an open airway. Do not use an NPA
for patients who have sustained trauma to the nasal area or when space-occupying lesions or foreign
objects block the nasal passages (Barnes, 2017).
CHAPTER 3  Patient Assessment and Resuscitation Therapies 47

NPAs are available in many sizes varying in length and internal diameter (Fig. 3.11). Proper airway
size is determined by holding the device against the side of the patient’s face and selecting an airway
that extends from the tip of the nose to the angle of the jaw or to the earlobe (Fig. 3.12). An NPA that
is too long may stimulate the gag reflex; one that is too short may not be inserted far enough to keep
the tongue away from the back of the throat.
Before inserting an NPA, lubricate the distal tip of the device liberally with a water-soluble lubri-
cant to minimize resistance and to decrease the irritation of the nasal passage. Hold the nasal airway
at its flange end like a pencil, and slowly insert it into the larger of the patient’s two nares, with the
bevel facing the nasal septum (Fig. 3.13). During insertion, do not force the airway, because it may cut
or scrape the nasal mucosa; this may result in significant bleeding, which increases the risk of aspira-
tion. Bleeding can occur in up to 30% of patients after NPA insertion (Link et al., 2015). If you
encounter resistance, a gentle back-and-forth rotation of the device between your fingers may ease
insertion. If resistance continues, withdraw the nasal airway, reapply lubricant, and attempt insertion
in the patient’s other nostril.

Fig. 3.11  ​Nasopharyngeal airways. (From Harkreader, H., Hogan, M.A., Thobaben, M. [2007]. Fundamentals of nursing:
caring and clinical judgment, ed 3, St Louis, Saunders.)

Fig. 3.12  ​A nasopharyngeal airway of proper size extends from the tip of the patient’s nose to the angle of the jaw or the
earlobe. (From Roberts, J.R., Custalow, C.B., Thomsen, T.W. [2019]. Roberts and Hedges’ clinical procedures in emergency
medicine and acute care, ed 7, Philadelphia, Elsevier.)

Fig. 3.13  ​Nasopharyngeal airway insertion.  (From Roberts, J.R., Custalow, C.B., Thomsen, T.W. [2019]. Roberts and
Hedges’ clinical procedures in emergency medicine and acute care, ed 7. Philadelphia. Elsevier.)
48 CHAPTER 3  Patient Assessment and Resuscitation Therapies

Advance the airway along the floor of the nostril, following the natural curvature of the nasal pas-
sage until the flange is flush with the nostril. If you observe blanching of the nostril after placement of
the adjunct, the diameter of the NPA is too big. Remove the NPA and insert a smaller airway.
The proper placement of the device is confirmed by ventilating the patient. If the nasal airway is correctly
placed, chest rise should be visible, and breath sounds should be present on auscultation of the lungs during
ventilation. If the patient is not breathing or if breathing is inadequate, begin positive pressure ventilation.
Table 3.4 shows the indications, contraindications, advantages, and disadvantages of pharyngeal airways.

Positive Pressure Ventilation


Adequate oxygenation requires an open airway and adequate air exchange. After opening the airway,
determine whether the patient’s breathing is adequate or inadequate. If the patient’s ventilatory efforts
are inadequate, assist their breathing by delivering positive pressure ventilation (i.e., forcing air into the
lungs). Mouth-to-mask ventilation, BMV, and NPPV are examples of methods that may be used to
deliver positive pressure ventilation.

Mouth-to-Mask Ventilation
The device used for mouth-to-mask ventilation is commonly called a pocket mask, pocket face mask,
ventilation face mask, or resuscitation mask. A pocket face mask is a clear, semirigid mask that is sealed

TABLE 3.4 Pharyngeal Airways


Considerations Oropharyngeal Airway Nasopharyngeal Airway
Indications • Helps maintain an open airway in To aid in maintaining an airway when use of
an unresponsive patient with no an OPA is contraindicated or difficult to
gag reflex who is not intubated place, such as when the patient’s jaw is
• Helps maintain an open airway in clenched during a seizure or if oral trauma
an unresponsive patient with no is present
gag reflex who is being ventilated
with a bag-mask or other positive
pressure device
• May be used as a bite block after
insertion of a tracheal tube or
orogastric tube
Contraindications Responsive patient with an intact • Severe craniofacial trauma
gag reflex • Known or suspected basal skull fracture
• Severe coagulopathy
• Patient intolerance
Sizing Corner of the mouth to the tip of the Tip of nose to the angle of the jaw or the
earlobe or the angle of the jaw earlobe
Advantages • Positions the tongue forward and • Provides a patent airway
away from the back of the throat • Tolerated by responsive patients
• Easily placed • Does not require the mouth to be open
Disadvantages • Does not protect the lower airway • Does not protect the lower airway from
from aspiration aspiration
• May produce vomiting if used in • Improper technique may result in severe
a responsive or semiresponsive bleeding; resulting epistaxis may be diffi-
patient with a gag reflex cult to control
• Suctioning through the device is difficult
• Although tolerated by most responsive
and semiresponsive patients, can stimu-
late the gag reflex in sensitive patients,
precipitating vomiting, gagging, or
laryngospasm
Precautions Use of the device does not eliminate Use of the device does not eliminate the
the need for maintaining proper need for maintaining proper head position
head position

OPA, Oropharyngeal airway.


CHAPTER 3  Patient Assessment and Resuscitation Therapies 49

around the patient’s mouth and nose (Fig. 3.14). Masks used for ventilation should be made of trans-
parent material to allow assessment of the patient’s lip color and detection of vomitus, secretions, or
other substances, and they should be equipped with an oxygen inlet and a standard connector that
enables connection to a bag-mask (or other ventilation) device.
When ventilating with a patient using a pocket mask, connect a one-way valve to the ventilation
port on the mask. If an oxygen inlet is present on the mask and oxygen is available, connect oxygen
tubing to the oxygen inlet, and set the flow rate at 10 to 12 L/min. Position yourself at the patient’s
head or side. Positioning yourself directly above the patient’s head allows you to watch the patient’s
chest while delivering ventilations. This position is used if the patient is in respiratory arrest (but
not cardiac arrest) or when performing two-rescuer CPR. If you are by yourself, positioning yourself
at the patient’s side allows you to maintain the same position for both rescue breathing and chest
compressions.
Open the patient’s airway. If needed, clear the patient’s airway of secretions or vomitus. If the pa-
tient is unresponsive and has no gag reflex, insert an OPA. Select a mask of appropriate size and place
it on the patient’s face. A mask of correct size should extend from the bridge of the nose to the groove
between the lower lip and chin; it should not extend beyond the chin. If the mask is not properly po-
sitioned and a tight seal maintained, air is likely to leak from between the mask and the patient’s face,
thereby resulting in the delivery of less tidal volume to the patient. Less tidal volume results in less lung
inflation, which means less oxygenation.
The E-C clamp technique, also called the E-C grip, can be used to create a good face-to-mask seal
and provide effective ventilation (Fig. 3.15). Apply the narrow portion (i.e., apex) of the mask over the
bridge of the patient’s nose and stabilize it in place with your thumbs. Lower the mask over the patient’s
face and mouth. Use your index fingers to stabilize the wide end (i.e., base) of the mask over the groove
between the patient’s lower lip and chin. When properly positioned, your thumb and index finger create
a “C.” Gently push down on the mask to establish a tight mask seal. Position your remaining fingers

Fig. 3.14  ​Pocket mask.  (From Perry, A.G., Potter, P.A., Ostendorf, W.R., Laplante, N. [2018]. Clinical nursing skills and
techniques, ed 9, St Louis, Elsevier.)

Fig. 3.15  ​The E-C clamp technique for mouth-to-mask or bag-mask ventilation.  (From Roberts, J.R., Custalow, C.B.,
Thomsen, T.W. [2019]. Roberts and Hedges’ clinical procedures in emergency medicine and acute care, ed 7, Philadelphia,
Elsevier.)
50 CHAPTER 3  Patient Assessment and Resuscitation Therapies

along the angle of the jaw to form an “E.” Place your little finger at the angle of the jaw. Use the fingers
forming an “E” to lift the bony ridge of the lower jaw upward to the mask. Be careful not to compress
the soft tissues under the jaw; doing so can cause an airway obstruction and make ventilation difficult
(Artime & Hagberg, 2020).
Ventilate the lungs through the one-way valve on the top of the mask. If the patient has experienced
a respiratory or cardiac arrest, ventilate at a rate of 1 breath every 6 seconds, or about 10 breaths per
minute (Panchal et al., 2020). Deliver each breath over 1 second and stop ventilation when you see the
chest gently rise.

ACLS Pearl
Gastric distention is a complication of positive pressure ventilation that can lead to vomiting and
subsequent aspiration. Gastric distention also restricts the movement of the diaphragm, impeding
ventilation, and decreases the effectiveness of CPR if the patient is in cardiac arrest.

Another method used for ventilation is the thenar eminence (TE) technique, also called the
TE grip. When using this method, the TEs of both hands are used to hold the mask in place
(Fig. 3.16). The rescuer’s fingers are positioned under the angle of the patient’s mandible to perform
a jaw lift (Fig. 3.17). Research has shown that the use of the TE technique is easier for inexperi-
enced providers and results in improved ventilation when compared with the E-C clamp technique
(Gerstein et al., 2013). Table 3.5 shows the indications, advantages, and disadvantages of mouth-
to-mask ventilation.

Fig. 3.16  ​The thenar eminences of both hands of the rescuer hold the face mask firmly in place. (From Roberts, J.R.,
Custalow, C.B., Thomsen, T.W. [2019]. Roberts and Hedges’ clinical procedures in emergency medicine and acute care,
ed 7, Philadelphia, Elsevier.)

Fig. 3.17  ​The rescuer’s fingers are positioned under the angle of the patient’s mandible to perform a jaw lift.  (From
Roberts, J.R., Custalow, C.B., Thomsen, T.W. [2019]. Roberts and Hedges’ clinical procedures in emergency medicine and
acute care, ed 7, Philadelphia, Elsevier.)
CHAPTER 3  Patient Assessment and Resuscitation Therapies 51

TABLE 3.5 Mouth-to-Mask Ventilation


Inspired oxygen • Without supplemental oxygen equals about 16% to 17% (exhaled air)
concentration • Mouth-to-mask breathing combined with supplemental oxygen at a minimum flow
rate of 10 L/min equals about 50%
Advantages • Esthetically more acceptable than mouth-to-mouth ventilation
• Easy to teach and learn
• Physical barrier between the rescuer and the patient’s nose, mouth, and secretions
• Reduces (but does not prevent) the risk of exposure to infectious diseases
• Use of a one-way valve at the ventilation port decreases exposure to the patient’s
exhaled air
• If the patient resumes spontaneous breathing, the mask can be used as a simple
face mask to deliver 40% to 60% oxygen by giving supplemental oxygen through
the oxygen inlet on the mask (if so equipped)
• Can deliver a greater tidal volume compared with a BMD
• Rescuer can feel the compliance of the patient’s lungs
Disadvantages • Rescuer fatigue
• Possible gastric distention

BMD, Bag-mask device.

Fig. 3.18  ​Bag-mask devices. (From istock.com.)

Bag-Mask Ventilation
A BMD is a self-inflating bag with a nonrebreathing valve mechanism (Fig. 3.18). A BMD is also
referred to as a bag-mask, bag-valve-mask device or bag-mask resuscitator (when using the mask), or
as a bag-valve device (when the mask is not used [i.e., when ventilating a patient with a tracheal tube
or tracheostomy tube in place]). The BMD should be equipped with a transparent disposable plastic
mask capable of sealing tightly against the face; standard fittings to allow for attachment of the device
to an advanced airway or another ventilation device; and an oxygen-collecting device (i.e., reservoir) to
allow delivery of high concentrations of oxygen.

Oxygen Delivery
When using a BMD, the amount of delivered O2 is dependent on the ventilatory rate, the volume
delivered during each breath, the O2 flow rate into the ventilating bag, the filling time for the reservoir
bag, and the type of reservoir used (Driver & Reardon, 2019a). Delivered tidal volumes vary with bag
type, hand size, and patient body characteristics (Rouse & Frakes, 2010).
When using a BMD without supplemental oxygen, 21% oxygen (i.e., room air) is delivered to the
patient. The BMD should be connected to an oxygen source. To do this, attach one end of a piece of
oxygen connecting tubing to the oxygen inlet on the BMD and the other end to an oxygen regulator.
A BMD that is used with supplemental oxygen set at a flow rate of 10 to 15 L/min delivers about 40%
to 60% oxygen to the patient when a reservoir is not used. Ideally, an oxygen reservoir should be
attached to the bag-mask to deliver a high concentration of oxygen. The reservoir collects a volume of
100% oxygen that is equal to the capacity of the bag. After squeezing the bag, it reexpands and draws
100% oxygen from the reservoir into the bag. A BMD that is used with supplemental oxygen set at a
flow rate of 10 to 15 L/min and with an attached reservoir delivers about 90% to 100% oxygen to the
patient. Box 3.5 shows the advantages and disadvantages of BMV.
52 CHAPTER 3  Patient Assessment and Resuscitation Therapies

BOX 3.5 Bag-Mask Ventilation


Advantages • Initial ventilation technique in apneic patients
• Provides a means for delivery of an oxygen-enriched mixture to the patient
• Can be used with the spontaneously breathing patient, as well as with the nonbreathing patient
• Conveys a sense of compliance of the patient’s lungs to the BMD operator
• Provides a means for immediate ventilatory support
Disadvantages • Requires practice to be used effectively
• Possible delivery of inadequate tidal volumes
• Causes rescuer fatigue
• Ventilating too fast or with excessive force can result in gastric inflation,
increasing aspiration risk
BMD, Bag-mask device.

Ventilating With a Bag-Mask Device


Performing positive pressure ventilation with a BMD can be difficult. Several reasons contribute to this,
but none as much as the inability to create a good seal with the mask while simultaneously generating
an adequate tidal volume by squeezing the bag. In addition to ensuring the delivery of an adequate tidal
volume, maintaining a tight face-to-mask seal when ventilating a patient with known or suspected
COVID-19 is essential to minimize provider exposure to potentially infectious droplets or aerosols.
Securely attach a HEPA filter to any manual or mechanical device in the path of exhaled gas before
delivering any ventilations to a patient with known or suspected COVID-19 (Edelson et al., 2020).
BMV should be a two-rescuer operation. With two people, one is assigned the responsibility of
opening and maintaining the airway while creating a good seal with the mask. That frees a second
person to squeeze the bag. Physical characteristics that can create difficulty in maintaining an adequate
face-to-mask seal include the provider’s gender, height, weight, hand size, and grip strength (Peck et
al., 2018). Patient characteristics that are known to create higher rates of BMV difficulty include the
presence of facial hair, lack of teeth, large neck circumference, obesity, excessive pharyngeal tissue, in-
ability to flex or extend the head, mandibular immobility, and advanced age (Peck et al., 2018).
To ventilate a patient with a BMD, position yourself at the top of the supine patient’s head and
open the patient’s airway. If needed, clear the patient’s airway of secretions. If the patient is unrespon-
sive, insert an OPA. Next, select a bag and mask of appropriate size for the patient. A mask of correct
size should extend from the bridge of the nose to the groove between the lower lip and chin. Connect
the bag to the mask if this has not already been done. Connect the bag to oxygen at a flow rate of
15 L/min and attach a reservoir.
Although single-rescuer BMV is not recommended during CPR (Link et al., 2015), if you find your-
self in this situation, use the E-C clamp technique previously described to lift the jaw upward to the mask
while gently but firmly pressing the mask against the patient’s face with one hand. Squeeze the bag with
the other hand (Fig. 3.19). If a second rescuer is present, the E-C clamp or TE technique can be used.

Fig. 3.19  ​Single-rescuer bag-mask ventilation using the E-C clamp. (From Sole, M.L., Klein, D.G., Moseley, M.J. [2017].
Introduction to critical care nursing, ed 7, St Louis, Elsevier.)
CHAPTER 3  Patient Assessment and Resuscitation Therapies 53

ACLS Pearl
Leaving well-fitting dentures in place can help achieve a good mask seal. If the presence of a
beard makes it challenging to achieve a tight face to mask seal, the use of a water-soluble lubri-
cant liberally applied to the mask area in contact with the beard or placing an adhesive plastic
dressing over the facial hair can enhance making a face-to-mask seal in these patients (Artime &
Hagberg, 2020; Rouse & Frakes, 2010).

If an assistant is available, ask them to squeeze the bag until the patient’s chest rises. At the same
time, lift the patient’s jaw to the mask, press the mask firmly against the patient’s face, and maintain
the patient’s proper head position (see Fig. 3.17). Observe the rise and fall of the patient’s chest with
each ventilation. Deliver a tidal volume of about 500 to 600 mL with each breath, which is enough to
produce gentle chest rise, over 1 second. If the patient has experienced a respiratory or cardiac arrest,
ventilate at a rate of 1 breath every 6 seconds, or about 10 breaths per minute. Periodically listen to
breath sounds to ensure adequate ventilation.
A reliable indicator of adequate ventilation is the rise and fall of the patient’s chest wall with each
ventilation. Improvements in skin color, pulse oximeter and capnography readings, heart rate, and
responsiveness are additional signs that the patient is well ventilated.
During BMV, avoid excessive ventilation (either by rate or volume) and allow adequate time for
exhalation to occur. Also feel for compliance when ventilating the patient’s lungs. Pulmonary compli-
ance refers to the resistance of the patient’s lung tissue to ventilation. The lungs are ordinarily pliable
and expand easily. If the lungs feel stiff or inflexible, lung compliance is said to be poor. Upper airway
obstruction, lower airway obstruction, severe bronchospasm, and tension pneumothorax are examples
of conditions that can cause poor lung compliance and an inability to ventilate. If at any time you sense
poor compliance, reassess the patient to ensure that the airway remains unobstructed and that lung
sounds are clear and equal.

Troubleshooting Bag-Mask Ventilation


Improper BMV may lead to hypoventilation, hypoxemia, acidosis, hypoxic brain injury, cardiac arrest,
and death (Peck et al., 2018). Common problems associated with BMV include inadequate ventilation,
delivering ventilations at an improper rate, and gastric inflation. Inadequate ventilation may be the
result of failing to maintain an open airway with proper positioning, failing to provide an effective mask
seal while simultaneously maintaining an open airway, incomplete bag compression, or a combination
of these factors. During a cardiac arrest, providing BMV at too fast a rate or with too much volume
increases intrathoracic pressure, impedes venous return, decreases cardiac output, decreases coronary
and cerebral artery perfusion pressures, and decreases survival rates (Aufderheide et al., 2004). Squeez-
ing the bag too fast or too forcefully can result in gastric inflation, increasing the risk for aspiration.
If the chest does not rise and fall with BMV, reassess the patient in the following manner:
• Begin by reassessing the patient’s head position. Reposition the airway and try to ventilate again.
• Inadequate tidal volume delivery may be the result of an improper mask seal or incomplete bag
compression. If air is escaping from under the mask, reposition your fingers and the mask, and
reevaluate the effectiveness of bag compression.
• Check for an airway obstruction. Lift the jaw and suction the airway as needed. If the chest still
does not rise, select an alternative method of positive pressure ventilation.

Noninvasive Positive Pressure Ventilation


NPPV, also called noninvasive ventilation (NIV), is the delivery of ventilatory support to a spontaneously
breathing patient without using an invasive artificial airway (e.g., ETT, tracheostomy tube). Ventilatory
support is provided through a portable or standard ventilator using an interface, such as a face mask,
nasal pillows, or a helmet. Patients must be alert, cooperative, and hemodynamically stable with intact
upper airway reflexes (Slutsky & Brochard, 2020). Safe and successful use of NPPV requires effective
communication between the healthcare provider and the patient before and during treatment, continuous
monitoring of the patient’s heart rate and pulse oximetry, and intermittent BP measurements.
NPPV encompasses various modes of positive pressure ventilation, including continuous positive
airway pressure (CPAP) and bilevel positive airway pressure (BiPAP). With noninvasive CPAP, con-
tinuous pressure that is greater than atmospheric pressure is delivered throughout the respiratory cycle.
When BiPAP is administered, two levels of pressure are applied; a higher pressure is used during
54 CHAPTER 3  Patient Assessment and Resuscitation Therapies

BOX 3.6 Noninvasive Positive Pressure Ventilation: Contraindications


Altered mental status Inability to fit mask
Cardiac arrest Inability to protect airway
Complete upper airway obstruction Recent facial, esophageal, or gastric surgery
Excessive secretions Respiratory arrest
Facial trauma or deformity Uncontrolled vomiting
Hemodynamic instability Uncooperative patient
High risk for aspiration

inspiration (i.e., inspiratory positive airway pressure), and a lower pressure is used during expiration
(i.e., expiratory positive airway pressure), thus decreasing the patient’s inspiratory muscle workload.
NPPV has been effectively used to improve dyspnea, avoid or decrease the rates of endotracheal
intubation, reduce mortality rates, and decrease the duration of hospital stays in patients with severe
acute exacerbations of chronic obstructive pulmonary disease (COPD), acute cardiogenic pulmonary
edema, postoperative respiratory failure, and in hypoxic respiratory failure in immunocompromised
patients. Box 3.6 shows contraindications for NPPV.

ACLS Pearl
Because BiPAP is the most common mode used with NPPV, some clinicians use the terms BiPAP
and NPPV synonymously.

Advanced Airways
SGA devices, also called extraglottic airways, are advanced airways that are blindly inserted to allow
rapid oxygenation and ventilation. SGAs are available in a range of sizes and can be placed while CPR
is in progress, thereby minimizing interruptions when performing chest compressions.
Endotracheal intubation is an example of an intraglottic airway procedure in which a tube is placed
directly into the trachea. This procedure requires special training, equipment, and supplies. It may
be performed for a variety of reasons, including for the delivery of anesthesia, to assist a patient’s
breathing with positive pressure ventilation, and to protect the patient’s airway from aspiration.

ACLS Pearl
Advanced airway insertion requires a high degree of skill and knowledge, as well as regular prac-
tice to maintain proficiency. Regular practice, continuing education programs, and an effective
quality management program to monitor skill performance are essential for all healthcare profes-
sionals who perform this skill.

There is currently no supporting evidence that an advanced airway (i.e., SGA or tracheal intubation)
during CPR improves survival or survival with a favorable neurologic or functional outcome after
adult cardiac arrest in any setting when compared with BMV. Current recommendations include the
following (Panchal et al., 2020):
• Either BMV or an advanced airway strategy may be considered during CPR for adult cardiac arrest
in any setting. Experts used the phrase, “advanced airway strategy” because the insertion of an
advanced airway usually begins with a period of BMV. When providing their recommendations, the
experts recognized that the optimal timing and reasons for transitioning to an advanced airway vary
based on the clinical situation.
• If an advanced airway is used, an SGA can be used for adults with out-of-hospital cardiac arrest
(OHCA) in settings with a low tracheal intubation success rate or minimal training opportunities
for ETT placement.
• If an advanced airway is used, either an SGA or tracheal intubation can be used for adults with
OHCA in settings with high tracheal intubation success rates or optimal training opportunities for
ETT placement.
• If an advanced airway is used in the in-hospital setting by expert providers trained in these proce-
dures, either the SGA or ETT can be used.
CHAPTER 3  Patient Assessment and Resuscitation Therapies 55

In cardiac arrest situations, members of the resuscitation team may opt to delay the insertion of an
advanced airway until after several minutes of cardiac arrest management or until there is a return of
spontaneous circulation. If the decision is made to insert an advanced airway during the resuscitation
effort, perform continuous compressions with positive-pressure ventilation without interrupting chest
compressions, delivering 1 breath every 6 seconds (10 breaths per minute) (Panchal et al., 2020). Avoid
delivering an excessive number or volume of ventilations.
In cardiac arrest situations involving a patient with a known or suspected COVID-19, experts
recommend the following (Edelson et al., 2020):
• When possible, close the door to prevent airborne contamination of adjacent indoor space.
• Ask the provider with the highest likelihood of success on the first attempt to perform the procedure.
Consider the use of video laryngoscopy (if available) to reduce intubator exposure to aerosolized
particles.
• Pause chest compressions to intubate.
• Connect the ETT to a ventilator with a HEPA filter, when available. Once on a closed circuit,
minimize disconnections to reduce aerosolization.
• If intubation is delayed, consider manual ventilation with an SGA or BMD with a HEPA filter.

Confirming Endotracheal Tube Placement


Clinical assessments that are initially used to confirm correct placement of an ETT include the
following:
• Visualizing the passage of the ETT between the vocal cords
• Auscultating the presence of bilateral breath sounds
• Confirming the absence of sounds over the epigastrium during ventilation
• Observing adequate chest rise with each ventilation
In addition to clinical assessments, secondary methods used to verify proper ETT positioning
include the following:
• Using continuous waveform capnography (preferred for confirming and monitoring correct ETT
placement in a patient who has adequate tissue perfusion)
• Using a nonwaveform CO2 detector
• Using an EDD
• Obtaining a chest radiograph
• Using ultrasound imaging
For in-hospital cardiac arrests, The Joint Commission requires timely confirmation of correct ETT
positioning within 1 minute of initial placement using capnometry, electronic waveform capnography,
esophageal detection devices, exhaled CO2 colorimetric monitor, or revisualization with direct laryn-
goscopy that the ETT is correctly placed in the trachea, rather than in the esophagus (Institute of
Medicine et al., 2015).
Do not rely exclusively on one method or device to detect and monitor for inadvertent esophageal intubation.

ACLS Pearl
An advanced airway that is misplaced or that becomes dislodged can be fatal. Make it a habit to
recheck the placement of an advanced airway immediately after insertion, after securing the tube,
during intrafacility or interfacility transport, upon destination arrival, and whenever the patient is
moved. Be certain to document the cm position of the tube at the patient’s teeth/lips. Capnography
can be used to alert you to a misplaced or dislodged tube immediately.

Esophageal Detector Devices


EDDs, also called esophageal intubation detectors, are used to help determine whether a tracheal tube
is in the trachea or the esophagus. There are two types of esophageal detectors: syringes and bulbs.
The syringe device is connected to a tracheal tube with the plunger fully inserted into the barrel of
the syringe. If the tube is in the trachea, the plunger can be readily withdrawn from the syringe barrel.
If the tracheal tube is in the esophagus, resistance will be felt when the plunger is withdrawn because
the walls of the esophagus will collapse when negative pressure is applied to the syringe. The EDD
should be checked for air leaks before use. If any connections are loose, the leak may allow the syringe
to be easily withdrawn, thus mimicking the tracheal location of the tube (Driver & Reardon, 2019b).
The bulb device is compressed before it is connected to a tracheal tube, creating a vacuum as the
pressure on the bulb is released. If the tube is in the trachea, the bulb will refill easily when pressure is
56 CHAPTER 3  Patient Assessment and Resuscitation Therapies

released, thereby indicating proper tube placement. If the tracheal tube is in the esophagus, the bulb
will remain collapsed, which indicates improper tube placement. Conditions in which the trachea tends
to collapse can result in misleading findings. Examples of these conditions include morbid obesity, late
pregnancy, status asthmaticus, and the presence of profuse tracheal secretions.

Oxygen Delivery Devices


The fraction of inspired gas that is oxygen is abbreviated as FiO2 and is expressed as a percentage.
Research has shown that routine use of supplemental oxygen in cardiac patients may have untoward
effects, including increased coronary vascular resistance, reduced coronary blood flow, and increased
risk of mortality (Amsterdam et al., 2014). Indications for supplemental oxygen administration include
clinically significant hypoxemia, heart failure, dyspnea, cyanosis, or when other high-risk features of
hypoxemia are present (Amsterdam et al., 2014; O’Gara et al., 2013).

Nasal Cannula
A nasal cannula, which is also called nasal prongs, is a piece of plastic tubing with two soft prongs that
project from the tubing. The prongs are inserted into the patient’s nostrils, and the tubing is then se-
cured to the patient’s face (Fig. 3.20). Oxygen flows from the cannula into the patient’s nasopharynx,
which acts as an anatomic reservoir. Factors that influence the FiO2 delivered by a nasal cannula in-
clude the oxygen flow rate, the patient’s ventilatory rate and tidal volume, and if the patient’s mouth is
open or closed when breathing. Because of these variables, it is not possible to know the precise FiO2
provided by a nasal cannula; however, a general rule of thumb when using this device is that for every
liter-per-minute (L/min) increase in oxygen flow, the effective FiO2 increases by about 4 percentage
points. For example, giving supplemental O2 at 1 L/min by cannula would raise the FiO2 to about 24%,
2 L/min would raise it to 28%, and up to 6 L/min would raise it to 44%. Box 3.7 shows the advantages
and disadvantages of using a nasal cannula.

Right nasal prong

Left nasal prong

Delivery tube Restraining band

Fig. 3.20  ​Low-flow nasal cannula. (From Kacmarek, R.M., Stoller, J.K., Heuer, A.J. [2017]. Egan’s fundamentals of respiratory
care, ed 11, St. Louis, Elsevier.)

BOX 3.7 Low-Flow Nasal Cannula: Advantages and Disadvantages


ADVANTAGES DISADVANTAGES
• Comfortable and well tolerated by most patients • Primarily used for spontaneously breathing
• Does not interfere with patient assessment or patients; sometimes used to preoxygenate
impede patient communication with healthcare patients (including those who are apneic)
personnel before emergent intubation
• Allows for talking and eating • Easily displaced
• No rebreathing of expired air • Nasal passages must be open
• Can be used with mouth breathers • Drying to mucous membranes; may cause
• Useful for patients who are predisposed to sinus pain
carbon dioxide retention • Tubing may cause skin breakdown or irritation
• Can be used for patients who require oxygen • Deviated septum and mouth breathing may
but who cannot tolerate a nonrebreather mask reduce FiO2
FiO2, Fraction of inspired oxygen.
CHAPTER 3  Patient Assessment and Resuscitation Therapies 57

High-flow nasal cannula (HFNC) systems are being used with increasing frequency for some
critically ill patients. Components needed to provide HFNC oxygen include a nasal cannula that can
accommodate high inlet flow, a high-flow oxygen flow meter, and a humidifier. Commercially available
humidified HFNC systems use flow rates of 5 to 40 L/min and deliver a FiO2 of close to 100% (Driver
& Reardon, 2019a).

Simple Face Mask


A simple face mask, which is also called a standard mask, is a plastic reservoir that has been designed
to fit over the nose and mouth of a spontaneously breathing patient. The mask is secured around the
patient’s head using an elastic strap. The internal capacity of the mask produces a reservoir effect. Small
holes on each side of the mask allow for the passage of inspired and expired air. Supplemental oxygen
is delivered through a small-diameter tube connected to the base of the mask (Fig. 3.21).
When using a simple face mask, the oxygen flow rate must be higher than 5 L/min to flush the
buildup of the patient’s exhaled carbon dioxide from the mask. At 5 to 10 L/min, the simple face mask
can deliver an inspired oxygen concentration of approximately 35% to 60%. The patient’s actual
inspired oxygen concentration varies because the amount of air that mixes with supplemental oxygen
is dependent on the patient’s inspiratory flow rate. Advantages and disadvantages of using a simple face
mask are shown in Box 3.8.

Exhalation ports

Oxygen inlet

Fig. 3.21  ​Simple face mask. (From Kacmarek, R.M., Stoller, J.K., Heuer, A.J. [2017]. Egan’s fundamentals of respiratory
care, ed 11, St. Louis, Elsevier.)

BOX 3.8 Simple Face Mask: Advantages and Disadvantages


ADVANTAGES DISADVANTAGES
• Higher oxygen concentration delivered • Can only be used in a spontaneously breathing patient
than by nasal cannula • Not tolerated well by severely dyspneic patients
• Can be uncomfortable
• Difficult to hear the patient speaking when the device is in
place
• Must be removed at meals
• Requires a tight face seal to prevent the leakage of oxygen
• Side holes in the mask permit inhalation of room air
• Oxygen flow rates of more than 10 L/min do not enhance
delivered oxygen concentration
58 CHAPTER 3  Patient Assessment and Resuscitation Therapies

Partial Rebreather Mask


A partial rebreather mask, also called a partial rebreathing mask, is similar to a simple face mask but it
has an attached oxygen-collecting device (i.e., reservoir) at the base of the mask that is filled before
patient use (Fig. 3.22A). When the patient breathes in, 100% oxygen is drawn into the mask from the
reservoir (bag). When the patient breathes out, oxygen enters the bag from the oxygen source, and
some of the patient’s exhaled air enters the bag (i.e., an amount that is approximately equal to the
volume of the patient’s anatomic dead space). The amount of CO2 that is rebreathed is negligible as
long as the oxygen flow keeps the bag from collapsing more than about one-third during inhalation
(Heuer, 2017).
The oxygen concentration of the patient’s exhaled air, in combination with the supply of 100%
oxygen, allows for the use of oxygen flow rates that are lower than those that are necessary for a
nonrebreather mask. Depending on the patient’s breathing pattern, the mask fit, and the oxygen
flowmeter setting, oxygen concentrations of 35% to 60% can be delivered when an oxygen flow rate
is used that prevents the reservoir bag from entirely collapsing on inspiration (i.e., typically 6 to 10 L/
min). Advantages and disadvantages of using a partial rebreather mask are shown in Box 3.9.

Valves

Reservoir bag Reservoir bag

A B
Fig. 3.22  ​(A) Partial rebreather mask. (B) Nonrebreather mask. (From Kacmarek, R.M., Stoller, J.K., Heuer, A.J. [2017].
Egan’s fundamentals of respiratory care, ed 11, St. Louis, Elsevier.)

BOX 3.9 Partial Rebreather Mask: Advantages and Disadvantages


ADVANTAGES DISADVANTAGES
• Higher oxygen concentration • Can only be used in a spontaneously breathing patient
delivered than by nasal cannula • Not tolerated well in severely dyspneic patients
• Can be uncomfortable
• Difficult to hear the patient speaking when the device is in
place
• Must be removed at meals
• Requires a tight face seal to prevent the leakage of oxygen
• May cause skin irritation
• Lacks inspiratory valve; thus exhaled air mixes with
inspired air
CHAPTER 3  Patient Assessment and Resuscitation Therapies 59

ACLS Pearl
When using a partial rebreather or nonrebreather mask, make sure that the bag does not collapse
when the patient inhales. Should the bag collapse, increase the delivered oxygen by 2 L incre-
ments until the bag remains inflated during inhalation. The reservoir bag must remain at least
two-thirds full so that sufficient supplemental oxygen is available for each breath.

Nonrebreather Mask
A nonrebreather mask, also called a nonrebreathing mask, is similar to a partial rebreather mask, but it
does not permit the mixing of the patient’s exhaled air with 100% oxygen. A one-way valve between
the mask and the reservoir bag and a flap over one of the exhalation ports on the side of the mask
prevent the inhalation of room air (Fig. 3.22B). When the patient breathes in, oxygen is drawn into
the mask from the reservoir (i.e., bag) through the one-way valve that separates the bag from the mask.
When the patient breathes out, the exhaled air exits through the open side port on the mask. The one-
way valve prevents the patient’s exhaled air from returning to the reservoir bag (thus the name nonre-
breather) and ensures a supply of 100% oxygen to the patient, with minimal dilution from room air.
A nonrebreather mask is the delivery device of choice when high concentrations of oxygen are
needed for the spontaneously breathing patient. In some areas, it is used to provide passive oxygen-
ation for the first few minutes of an adult witnessed arrest of presumed cardiac etiology while CPR
is ongoing. For spontaneously breathing patients, and depending on the patient’s breathing pattern,
the fit of the mask, and the oxygen flowmeter setting, oxygen concentrations of 60% to 80% can be
delivered when an oxygen flow rate (typically a minimum of 10 L/min) is used that prevents the
reservoir bag from collapsing completely on inspiration (Heuer, 2017). Inflate the reservoir bag with
oxygen before placing the nonrebreather mask on the patient. Advantages and disadvantages of using
a nonrebreather mask are shown in Box 3.10. A summary of oxygen percentages by device is shown
in Table 3.6.

BOX 3.10 Nonrebreather Mask: Advantages and Disadvantages


ADVANTAGES DISADVANTAGES
• Higher oxygen concentration delivered than • Primarily used for spontaneously breathing patients;
by nasal cannula, simple face mask, and used in some areas to provide passive oxygenation
partial rebreather mask for the first few minutes of an adult witnessed
• Inspired oxygen is not mixed with room air arrest of presumed cardiac etiology while CPR is
ongoing
• Not tolerated well in severely dyspneic patients
• Can be uncomfortable
• Difficult to hear the patient speaking when the device
is in place
• Must be removed at meals
• Mask must fit snugly on the patient’s face to prevent
room air from mixing with oxygen inhaled from the
reservoir bag
• May cause skin irritation
CPR, Cardiopulmonary resuscitation.

TABLE 3.6 Oxygen Percentage Delivery by Device


Approximate Inspired
Device Oxygen Concentration Liter Flow (Liters/Minute)
Nasal cannula 24% to 44% 1 to 6
Simple face mask 35% to 60% 5 to 10
Partial rebreather mask 35% to 60% Typically 6 to 10 to prevent bag collapse on
inspiration
Nonrebreather mask 60% to 80% Typically a minimum of 10 to prevent bag collapse
on inspiration
60 CHAPTER 3  Patient Assessment and Resuscitation Therapies

ELECTROCARDIOGRAM MONITORING
A prerequisite to participation in most Advanced Cardiac Life Support (ACLS) courses is the comple-
tion of a basic ECG recognition course. This requirement exists because there simply is no time in an
ACLS course to cover detailed information about rhythm recognition. A basic ECG course teaches you
how to identify cardiac rhythms. An ACLS course quickly reviews cardiac rhythms but focuses on teach-
ing how to recognize serious signs and symptoms related to those rhythms and how to manage them.
The heart usually beats at a very regular rate and rhythm. If this pattern is interrupted, an abnormal
heart rhythm can result. Although arrhythmia technically means “absence of rhythm” and dysrhythmia
means “abnormal heart rhythm,” these terms are used interchangeably by healthcare professionals to
refer to disturbances in cardiac rhythm.

Conduction System
The heart’s pacemaker cells are arranged in a system of interconnected pathways called the conduction
system. The conduction system makes sure that the chambers of the heart contract in a coordinated
fashion.
The normal heartbeat is the result of an electrical impulse (i.e., an action potential) that begins in
the SA node. The SA node usually is the primary pacemaker of the heart because it has the fastest
firing rate of all of the heart’s normal pacemaker sites (Fig. 3.23). The built-in (i.e., intrinsic) rate of
the SA node is 60 to 100 beats per minute (beats/min).
The AV node receives the impulse from the SA node and delays relay of the impulse to the bundle
of His, allowing time for the atria to empty their contents into the ventricles before the onset of ven-
tricular contraction. The bundle of His, also called the common bundle or the AV bundle, connects the
AV node with the bundle branches. When an abnormal electrical pathway bypasses the AV node and
bundle, the abnormal route is called an accessory pathway.
The AV bundle has pacemaker cells that have an intrinsic rate of 40 to 60 beats/min. The AV node
and the AV bundle are called the AV junction. The term His-Purkinje system or His-Purkinje network
refers to the bundle of His, bundle branches, and Purkinje fibers.
The right and left bundle branches receive the impulse from the bundle of His and relay it to the
Purkinje fibers in the ventricular myocardium. The Purkinje fibers have an intrinsic pacemaker ability
of 20 to 40 beats/min.

Sinoatrial
(SA) node Atrial excitation
Excitation of ventricles begins
(initial downward deflection is
a Q wave)
Pulmonary artery

R
Left
atrium
Right
atrium

Internodal
pathways T
Septum P

Left Q S
Atrioventricular ventricle
(AV) node

AV bundle Right
(bundle of His) ventricle

Fig. 3.23  ​Schematic drawing of the conducting system of the heart. An impulse normally is generated in the sinoatrial node.
It travels through the atria to the atrioventricular node, down the bundle of His and Purkinje fibers, and the ventricular myo-
cardium. Recording of the depolarizing and repolarizing currents in the heart with electrodes on the surface of the body
produces characteristic waveforms. (From Banasik, J.L., Copstead, L.C. [2019]. Pathophysiology, ed 6, St Louis, Elsevier.)
CHAPTER 3  Patient Assessment and Resuscitation Therapies 61

Leads
A lead is a record (i.e., tracing) of electrical activity between two electrodes. Each lead records the
average current flow at a specific time in a portion of the heart. A 12-lead ECG provides views of
the heart in both the frontal and horizontal planes and views the surfaces of the left ventricle from
12 different angles. Because the position of the positive electrode on the body determines the area of
the heart that is seen by each lead, accurate placement of the positive electrode is essential.
Six leads view the heart in the frontal plane (Fig. 3.24). Leads I, II, and III are called standard limb leads.
Leads aVR, aVL, and aVF are called augmented limb leads. Six chest (i.e., precordial or “V”) leads view the
heart in the horizontal plane (Fig. 3.25), allowing a view of the front and left side of the heart. The chest leads
are identified as V1, V2, V3, V4, V5, and V6. Table 3.7 summarizes the ECG leads and the heart areas viewed.
Use right chest leads to evaluate the right ventricle (Fig. 3.26). The placement of right chest leads
is identical to the placement of the standard chest leads except that it is done on the right side of
the chest. If time does not permit obtaining all of the right chest leads, the lead of choice is V4R.
Leads V7, V8, and V9 permit viewing of the posterior surface of the heart (Fig. 3.27). All of the leads
are placed on the same horizontal line as V4 to V6. Lead V7 is placed at the posterior axillary line. Lead
V8 is placed at the angle of the scapula (i.e., the posterior scapular line), and lead V9 is placed over the
left border of the spine.

ACLS Pearl
Multiple-lead ECGs are used to help spot infarctions of the right ventricle and the posterior wall
of the left ventricle. The 15-lead ECG uses all of the leads of a standard 12-lead ECG plus leads
V4R, V8, and V9 or a standard 12-lead plus posterior leads V7, V8, and V9. A 16-lead ECG machine
allows recording of a standard 12-lead plus leads V3R, V4R, V5R, and V6R. An 18-lead ECG uses
all of the leads of a standard 12-lead ECG plus leads V4R, V5R, V6R, V7. V8, and V9.

Fig. 3.24  ​View of the standard limb leads and augmented leads. (From Boron, W.F., Boulpaep, E.L. [2017]. Medical physiology,
ed 3, Philadelphia, Elsevier.)

Sternal
angle
Fourth
intercostal
space
V1 V2

V3

V4
V5

V6

Fig. 3.25  ​Chest (i.e., precordial) leads V1 through V6. (From Roberts, J.R., Custalow, C.B., Thomsen, T.W. [2019]. Roberts
and Hedges’ clinical procedures in emergency medicine and acute care, ed 7, Philadelphia, Elsevier.)
62 CHAPTER 3  Patient Assessment and Resuscitation Therapies

TABLE 3.7 Leads and Heart Surfaces Viewed


Lead Positive Electrode Position Heart Area Viewed
I Left arm Lateral
II Left leg Inferior surface
III Left leg Inferior surface
aVR Right arm None
aVL Left arm Lateral surface
aVF Left foot (i.e., leg) Inferior surface
V1 Right side of sternum, fourth intercostal space Interventricular septum
V2 Left side of sternum, fourth intercostal space Interventricular septum
V3 Midway between V2 and V4 Anterior surface
V4 Left midclavicular line, fifth intercostal space Anterior surface
V5 Left anterior axillary line; same level as V4 Lateral surface
V6 Left midaxillary line; same level as V4 Lateral surface

RV2 RV1
RV3

RV6 RV4
RV5

Fig. 3.26  ​Electrode locations for recording a right chest electrocardiogram (ECG). Right chest leads are not part of a
standard 12-lead ECG but are used when a right ventricular infarction is suspected. (From Roberts, J.R., Custalow, C.B.,
Thomsen, T.W. [2019]. Roberts and Hedges’ clinical procedures in emergency medicine and acute care, ed 7, Philadelphia,
Elsevier.)

Tip of
scapula

V7 V8 V9

Fig. 3.27  ​Left posterior chest lead placement. (From Roberts, J.R., Custalow, C.B., Thomsen, T.W. [2019]. Roberts and
Hedges’ clinical procedures in emergency medicine and acute care, ed 7, Philadelphia, Elsevier.)
CHAPTER 3  Patient Assessment and Resuscitation Therapies 63

Electrocardiography Paper
ECG paper is graph paper made up of small and large boxes measured in millimeters (mms). The
smallest boxes are 1 mm wide and 1 mm high (Fig. 3.28). The horizontal axis of the paper corresponds
with time, which is stated in seconds. ECG paper normally records at a constant speed of 25 mm/
second. Thus each horizontal 1 mm box represents 0.04 second (25 mm/s 3 0.04 second 5 1 mm).
Heavy lines appear after every five small boxes on the paper. The heavier lines indicate one large box,
which represents 0.20 second.
The vertical axis of the graph paper represents the voltage or amplitude of the ECG waveforms or deflec-
tions. Voltage is measured in mV. Amplitude is measured in mm. When properly calibrated, a small box is
1 mm high (i.e., 0.1 mV), and a large box, which is equal to five small boxes, is 5 mm high (i.e., 0.5 mV).

Electrocardiogram Components
An ECG waveform (i.e., a deflection) is movement away from the baseline (i.e., isoelectric line) in
either a positive (i.e., upward) or negative (i.e., downward) direction. Waveforms are named alpha-
betically, beginning with P, QRS, and T (Fig. 3.29). A line between waveforms is called a segment.
It is named by the waveform that precedes or follows it. An interval is made up of a waveform and a
segment. ECG components are summarized in Tables 3.8 and 3.9.

1 mm = 0.1 mV
5 mm = 0.20 s
Amplitude (voltage)

5 mm = 0.5 mV

1 mm = 0.04 s

Duration (time)

Fig. 3.28  ​Electrocardiogram strip showing the markings for measuring amplitude and duration of waveforms, using a
standard recording speed of 25 mm/s. (From Banasik, J.L., Copstead, L.C. [2019]. Pathophysiology, ed 6, St Louis, Elsevier.)

QRS
complex

ST
PR segment
segment

T
P

Q
PR interval
S

QT interval

Fig. 3.29  Components of the electrocardiogram recording.  (From Workman, M.L, LaCharity, L. [2016]. Understanding
pharmacology: essentials for medication safety, ed 2, St. Louis, Elsevier.)
64 CHAPTER 3  Patient Assessment and Resuscitation Therapies

TABLE 3.8 Electrocardiogram Components—Waveforms and Complexes


Component Description
P wave • First wave in the cardiac cycle
• Represents atrial depolarization
• Positive in leads I, II, III, aVL, aVF, and V2 through V6
QRS complex • Consists of the Q wave, R wave, and S wave; Q wave is the first downward (i.e.,
negative) deflection following the P wave; R wave is the first positive (i.e., upright)
waveform following the P wave; S wave is the negative waveform following the
R wave
• Represents the spread of the electrical impulse through the ventricles (i.e., ventricular
depolarization)
• In adults, the normal duration is 0.11 second or less
T wave • Represents repolarization of both ventricles
• Direction of the T wave is normally the same as the QRS complex that precedes it
U wave • Small waveform that, when seen, follows the T wave
• Thought to represent late repolarization of the Purkinje fibers

TABLE 3.9 Electrocardiogram Components—Segments and Intervals


Component Description
PR segment • Part of the PR interval, specifically, the horizontal line between the end of the P wave
and the beginning of the QRS complex
• Represents the spread of the electrical impulse from the AV node, through the
AV bundle, the right and left bundle branches, and the Purkinje fibers to activate
ventricular muscle
ST segment • Area between the QRS complex and the T wave
• Represents the early part of repolarization of the right and left ventricles
• The junction where the QRS complex and the ST segment meet is called the J point
TP segment • Area between the end of the T wave and the beginning of the next P wave during
which there is no electrical activity
PR interval • Consists of the P wave plus the PR segment and reflects total supraventricular activity
• In adults, normally measures 0.12 to 0.20 second
QT interval • Period from the beginning of the QRS complex to the end of the T wave
• Represents total ventricular activity; this is the time from ventricular depolarization
(i.e., activation) to repolarization (i.e., recovery)
• Duration varies with age, gender, and heart rate
• Considered short if it is 0.39 second or less and prolonged if it is 0.46 second or
longer in women or 0.45 second or longer in men

AV, Atrioventricular.

ELECTRICAL THERAPIES
Electrical therapies include defibrillation, synchronized cardioversion, and TCP.

Defibrillation
Defibrillation is the delivery of an electrical current across the heart muscle over a very brief period
to terminate an abnormal heart rhythm. Defibrillation is also called unsynchronized countershock or
asynchronous countershock because the delivery of current has no relationship to the cardiac cycle.
Defibrillation is indicated for the cardiac arrest rhythms pulseless ventricular tachycardia (pVT) and
ventricular fibrillation (VF).
Automated external defibrillation refers to the placement of pads on a patient’s chest and the
interpretation of the patient’s cardiac rhythm by the defibrillator’s computerized analysis system.
Manual defibrillation refers to the following: placement of adhesive pads on a patient’s chest,
the interpretation of the patient’s cardiac rhythm by a trained healthcare professional, and
the healthcare professional’s decision to deliver a shock, if indicated. In the hospital setting, it
CHAPTER 3  Patient Assessment and Resuscitation Therapies 65

is recommended that AEDs or manual defibrillators should be readily accessible in any patient
area and that all staff should know the location of this equipment and how to use it (Morrison
et al., 2013).
Defibrillation does not “jump start” the heart. The shock attempts to deliver a uniform electrical
current of sufficient intensity to depolarize myocardial cells (including fibrillating cells) at the same
time, thereby briefly “stunning” the heart and providing an opportunity for the heart’s natural pacemak-
ers to resume normal activity. When the cells repolarize, the pacemaker with the highest degree of
automaticity should assume responsibility for pacing the heart.

Monophasic Versus Biphasic Defibrillation


Different types of defibrillation waveforms exist. Waveforms are classified by whether the current flow
delivered is in one direction, two directions, or multiple directions. When a monophasic waveform is
used, current passes through the heart in one (i.e., mono) direction. Although few monophasic wave-
form defibrillators are manufactured today, many are still in use. With biphasic waveforms, energy is
delivered in two (i.e., bi) phases. The current moves in one direction for a specified period, stops, and
then passes through the heart a second time in the opposite direction during a very short period (i.e.,
milliseconds).
Defibrillators using biphasic waveforms are preferred to monophasic defibrillators for the treatment
of both atrial and ventricular dysrhythmias because of their greater success with dysrhythmia termina-
tion (Panchal et al., 2020). Both escalating (i.e., increasing energy levels) and nonescalating (i.e., no
increase in energy level) biphasic waveform defibrillators are available. When preparing to deliver electri-
cal therapy to a patient, knowledge of the type of device that you are using (i.e., monophasic versus biphasic)
and the manufacturer’s recommended energy levels for the dysrhythmia you are treating is essential.

Reducing Transthoracic Impedance


Transthoracic impedance (resistance) refers to the resistance of the chest wall to the flow of current
at the interface between the patient’s chest wall and the adhesive pads used for electrical therapy. If
transthoracic resistance is high, the amount of current that is delivered to the myocardium can be
compromised, leading to failed shocks. Transthoracic impedance varies significantly among individuals.

Adhesive Pad Size


Adhesive pads are used to deliver current from the defibrillator to the patient. These pads consist of a
flexible metal “paddle,” a layer of conductive gel, and an adhesive ring that holds them in place on the
patient’s chest. The pads are disposable and have multiple functions.

ACLS Pearl
When applying adhesive pads to the patient’s bare chest, press from one edge of the pad across
the entire surface to remove all air and to avoid the development of air pockets.

Use adult pads for patients weighing more than 10 kg (22 lb.) (i.e., generally older than age 1 year)
(de Caen et al., 2015). Carefully follow all manufacturer instructions because the optimum pad sizes
for defibrillation and pacing based on patient age and weight vary by manufacturer.

Adhesive Pad Position


When applying adhesive pads, remove the patient’s clothing and expose the chest. Do not use alcohol,
tincture of benzoin, or antiperspirant when preparing the skin for pad placement. Look at the patient’s
chest for transdermal patches or disks, which may be used to deliver medications, such as nitroglycerin,
nicotine, analgesics, hormones, or antihypertensives. If a medication patch, disk, or ointment is located
at or near the site of pad placement, remove it and wipe the area clean (do not use alcohol or alcohol-
based cleansers) before applying the adhesive pads.
Place adhesive pads on the patient’s bare chest following the manufacturer’s instructions (Fig. 3.30).
The pads are usually labeled according to their intended position on the chest (e.g., sternum/apex,
front/back). The typical pad position that is used during resuscitation is the sternum–apex position,
which is also called the anterolateral or apex–anterior position. This position is often used because the
anterior chest is usually easy to get to, and placement of the pads in this position approximates ECG
electrode positioning in lead II. If the patient is a woman, elevate the left breast and place the apex pad
66 CHAPTER 3  Patient Assessment and Resuscitation Therapies

Fig. 3.30  ​Apply adhesive pads to the patient’s bare chest per the manufacturer’s instructions.  (From Roberts, J.R.,
Custalow, C.B., Thomsen, T.W. [2019]. Roberts and Hedges’ clinical procedures in emergency medicine and acute care,
ed 7, Philadelphia, Elsevier.)

lateral to or underneath the breast. Placing defibrillation pads directly on breast tissue results in higher
transthoracic impedance, thereby reducing current flow.
Another common position used for pad placement is the anterior–posterior position. In this position,
one pad is placed over the patient’s left chest with the upper edge of the pad below the nipple.
The other is placed on the back, just below their left scapula. Alternative positions (e.g., anterior–left
infrascapular, anterior–right infrascapular) may be considered based on individual patient characteristics
(Panchal et al., 2020).

ACLS Pearl
Experts note that current across the myocardium is likely to be maximal when the adhesive pads
used for defibrillation and cardioversion are placed so that the area of the heart that is fibrillating
lies directly between them (i.e., the ventricles in VF, the atria in atrial fibrillation). Therefore the
optimal electrode position may not be the same for ventricular and atrial dysrhythmias (Ristagno
et al., 2019).

Selected Energy
When electrical therapy is used to treat an abnormal heart rhythm, it is essential to select the appropri-
ate energy level (i.e., the right amount of joules). If the energy level selected and the current delivered is
too low, the shock will not eliminate the abnormal rhythm. During adult cardiac arrest, use 360 joules
( J) for all shocks when using a monophasic defibrillator (Link et al., 2015). When using a biphasic
defibrillator, use the energy level recommended by the manufacturer for the initial shock. If you are
using a defibrillator that is capable of delivering escalating energy levels, a higher energy dose for the
second and subsequent shocks may be considered (Panchal et al., 2020). If you do not know what the
recommended energy dose is, consider using the maximum dose available (Panchal et al., 2020).

Defibrillation Procedure
The procedure described next assumes that the patient is an adult and confirmed to be unresponsive,
apneic, and pulseless. It also assumes that the patient’s cardiac rhythm is pVT or VF and that team
members are available to assist with procedures during the resuscitation effort.
Be sure that high-quality CPR is continued as the defibrillator is readied for use (Fig. 3.31). While
CPR continues, instruct a team member to expose the patient’s chest and to remove any transdermal
medication patches or ointment from the patient’s chest, if present. Remove the adhesive pads from
their sealed package. Check the pads for the presence of adequate gel. Attach the pads to the hands-
free defibrillation cable, and then attach the adhesive pads to the patient’s chest in the position recom-
mended by the manufacturer (Fig. 3.32).
Turn the power to the defibrillator on and verify the presence of a shockable rhythm on the moni-
tor (Fig. 3.33). Select an appropriate energy level (Fig. 3.34) and then charge the defibrillator
(Fig. 3.35). All team members, except for the chest compressor, should clear the patient as the machine
charges. When the defibrillator is charged, the chest compressor should immediately clear the patient.
CHAPTER 3  Patient Assessment and Resuscitation Therapies 67

If a shockable rhythm is still present, call “Clear!” Look around you (360 degrees) to be sure that ev-
eryone—including you—is clear of the patient, the bed, and any equipment that is connected to the
patient. Be sure oxygen is not flowing over the patient’s chest.

ACLS Pearl
Remove supplemental oxygen sources from the area of the patient’s bed before defibrillation
attempts are made, and place them at least 3.5 to 4 feet away from the patient’s chest. Examples
of supplemental oxygen sources include masks, nasal cannulae, resuscitation bags, and ventilator
tubing.

Fig. 3.31  ​Continue cardiopulmonary resuscitation (CPR) Fig. 3.32  ​Attach the adhesive pads to the patient’s
while the defibrillator is readied for use. (From Roberts, J.R., chest. (From Roberts, J.R., Custalow, C.B., Thomsen, T.W.
Custalow, C.B., Thomsen, T.W. [2019]. Roberts and Hedges’ [2019]. Roberts and Hedges’ clinical procedures in emer-
clinical procedures in emergency medicine and acute care, ed gency medicine and acute care, ed 7, Philadelphia, Elsevier.)
7, Philadelphia, Elsevier.)

Fig. 3.33  ​Verify the presence of a shockable rhythm on Fig. 3.34  ​Select an appropriate energy level using the
the cardiac monitor.  (From Roberts, J.R., Custalow, C.B., manufacturer’s recommended energy dose.  (From Rob-
Thomsen, T.W. [2019]. Roberts and Hedges’ clinical proce- erts, J.R., Custalow, C.B., Thomsen, T.W. [2019]. Roberts
dures in emergency medicine and acute care, ed 7, Phila- and Hedges’ clinical procedures in emergency medicine
delphia, Elsevier.) and acute care, ed 7, Philadelphia, Elsevier.)

Fig. 3.35  ​Charge the defibrillator and clear everyone Fig. 3.36  ​After ensuring that everyone is clear of the patient,
from the patient.  (From Roberts, J.R., Custalow, C.B., press the “Shock” control to defibrillate. Immediately resume
Thomsen, T.W. [2019]. Roberts and Hedges’ clinical proce- CPR.  (From Roberts, J.R., Custalow, C.B., Thomsen, T.W.
dures in emergency medicine and acute care, ed 7, Phila- [2019]. Roberts and Hedges’ clinical procedures in emergency
delphia, Elsevier.) medicine and acute care, ed 7, Philadelphia, Elsevier.)
68 CHAPTER 3  Patient Assessment and Resuscitation Therapies

Press the “Shock” control to defibrillate the patient (Fig. 3.36). Release the shock control after the
shock has been delivered. Instruct the team to resume CPR.

ACLS Pearl
When defibrillating or cardioverting a patient with a permanent pacemaker or an implantable
cardioverter-defibrillator (ICD), be careful not to place the adhesive pads directly over the pulse
generator (there will be a bulge under the patient’s skin). The anterior–posterior and anterolateral
pad positions are considered acceptable in these patients. Because some of the defibrillation
current flows down the pacemaker leads, a patient who has a permanent pacemaker or ICD
should have the device checked to ensure proper function after defibrillation.

Synchronized Cardioversion
Synchronized cardioversion is a type of electrical therapy during which a shock is timed or pro-
grammed for delivery during ventricular depolarization (i.e., the QRS complex). When the “Sync”
control is pressed, a synchronizing circuit in the machine searches for the QRS complex. The machine
delivers the shock during this portion of the cardiac cycle, reducing the potential for the delivery of
current during ventricular repolarization, which includes the vulnerable period of the T wave (i.e., the
relative refractory period).
Synchronized cardioversion is used to treat rhythms that have a clearly identifiable QRS complex
and a rapid ventricular rate (e.g., some narrow-QRS tachycardias, unstable atrial fibrillation [AFib] or
atrial flutter, monomorphic ventricular tachycardia [VT]) in a patient who has a pulse and signs of
hemodynamic compromise. Synchronized cardioversion is not used to treat disorganized rhythms (e.g.,
polymorphic VT) or those that do not have a clearly identifiable QRS complex (e.g., VF).

Cardioversion Procedure
Before performing synchronized cardioversion, take appropriate standard precautions and obtain a
12-lead ECG. Identify the rhythm on the cardiac monitor and verify that the procedure is indicated.
Print an ECG strip to document the patient’s rhythm and assess the patient for serious signs and
symptoms caused by the tachycardia. Make sure that oxygen delivery devices, a BMD, suction, and
emergency medications are available. Give supplemental oxygen, if indicated, and start an IV. If the
patient is awake, explain the procedure and obtain informed consent. If time and the patient’s clinical
condition permits, administer sedation before performing the procedure (Fig. 3.37).
Place the patient in a supine position and remove clothing from the patient’s upper body. With
gloves, remove transdermal medication patches, bandages, jewelry, and any other materials from the
sites to be used for pad placement; do not attempt to administer shocks through them. Keep monitor-
ing electrodes and wires well away from the area where adhesive pads will be placed. Contact may cause
electrical arcing and patient skin burns during the procedure.
Turn on the power to the defibrillator. Place the adhesive pads on the patient’s bare chest per the
manufacturer’s instructions. Some studies have shown that placement of the pads in an anteroposterior
position is more effective than the anteroapical position in elective cardioversion of AFib; however, most
studies have failed to demonstrate any clear advantage of any specific pad position (Ristagno et al., 2019).
Press the “Sync” control on the defibrillator to select the synchronized mode (Fig. 3.38). Select a
lead with an optimum QRS complex amplitude and no artifact. Make sure the machine is marking or
flagging each QRS complex and that no artifact is present. The sense marker should appear near the
middle of each QRS complex. If sense markers do not appear or are seen in the wrong place (e.g., on
a T wave), adjust the ECG size, or select another lead.
Select the energy level appropriate for the patient’s rhythm on the defibrillator (Fig. 3.39). Turn on
the ECG recorder for a continuous printout. Next, press the “Charge” button on the defibrillator and
recheck the ECG rhythm (Fig. 3.40). If the rhythm is unchanged, call “Clear!” and look around you.
Make sure that everyone is clear of the patient, the bed, and any equipment that is connected to the
patient. Make sure oxygen is not flowing over the patient’s chest to decrease the risk of combustion in
the presence of electrical current. After confirming that the area is clear, depress the “Shock” control
until the energy is delivered (Fig. 3.41).
Reassess the rhythm and the patient (Fig. 3.42). If the tachycardia persists, make sure that the ma-
chine is in “Sync” mode before delivering another shock because many defibrillators default to the
CHAPTER 3  Patient Assessment and Resuscitation Therapies 69

Fig. 3.37  ​Administer sedation if time and the patient’s Fig. 3.38  ​Place adhesive pads in proper position on the
condition permit.  (From Roberts, J.R., Custalow, C.B., patient’s bare chest according to the defibrillator manu-
Thomsen, T.W. [2019]. Roberts and Hedges’ clinical proce- facturer’s instructions. Press the “Sync” control on the
dures in emergency medicine and acute care, ed 7, Phila- defibrillator. Make sure the machine is marking each QRS
delphia, Elsevier.) complex and that no artifact is present.  (From Roberts,
J.R., Custalow, C.B., Thomsen, T.W. [2019]. Roberts and
Hedges’ clinical procedures in emergency medicine and
acute care, ed 7, Philadelphia, Elsevier.)

Fig. 3.39  ​Select the appropriate energy level on the Fig. 3.40  ​Press the “Charge” button the defibrillator and
defibrillator. (From Roberts, J.R., Custalow, C.B., Thomsen, recheck the electrocardiogram rhythm.  (From Roberts,
T.W. [2019]. Roberts and Hedges’ clinical procedures in J.R., Custalow, C.B., Thomsen, T.W. [2019]. Roberts and
emergency medicine and acute care, ed 7, Philadelphia, Hedges’ clinical procedures in emergency medicine and
Elsevier.) acute care, ed 7, Philadelphia, Elsevier.)

Fig. 3.41  ​Call “Clear!” After confirming that the area is Fig. 3.42  ​Reassess the rhythm and the patient.  (From
clear, depress the “Shock” button until the energy is deliv- Roberts, J.R., Custalow, C.B., Thomsen, T.W. [2019]. Rob-
ered.  (From Roberts, J.R., Custalow, C.B., Thomsen, T.W. erts and Hedges’ clinical procedures in emergency medi-
[2019]. Roberts and Hedges’ clinical procedures in emer- cine and acute care, ed 7, Philadelphia, Elsevier.)
gency medicine and acute care, ed 7, Philadelphia, Elsevier.)
70 CHAPTER 3  Patient Assessment and Resuscitation Therapies

unsynchronized mode after cardioversion. If the rhythm changes to VF, quickly repeat the primary
assessment (i.e., assess responsiveness, check for breathing and a pulse). At the same time, ask another
team member to verify that all electrodes and cable connections are secure. If no pulse is present, ensure
that the machine is not in “Sync” mode and defibrillate.

Possible Complications
Possible complications of defibrillation or cardioversion include the following:
• Injury to the operator or other team members if improper technique is used
• Risk of fire from the combination of electrical and oxygen sources
• Myocardial damage or dysfunction
• Embolic episodes
• Dysrhythmias, including asystole, AV block, bradycardia, or VF after cardioversion

Transcutaneous Pacing
TCP is the use of electrical stimulation through pacing pads that are positioned on a patient’s torso to
stimulate the contraction of the heart. TCP is also called temporary external pacing or noninvasive
pacing.
TCP requires attaching two pacing electrodes to the skin surface of the patient’s outer chest wall.
Although TCP is a type of electrical therapy, the current delivered is considerably less than that used
for cardioversion or defibrillation. The stimulating current selected for TCP is measured in milliam-
peres (mA). The power delivered during each pacing impulse is less than 1/1000 of that delivered
during defibrillation (Bessman, 2019). The range of output current of a transcutaneous pacemaker var-
ies depending on the manufacturer. Because TCP is painful in conscious patients, sedation, analgesia,
or both may be needed to minimize the patient’s discomfort associated with this procedure.

Indications
The term symptomatic bradycardia is used to describe a patient who experiences signs and symptoms
of hemodynamic compromise (e.g., syncope or presyncope, transient dizziness or lightheadedness,
heart failure symptoms, confusional states resulting from inadequate cerebral perfusion) related to a
slow heart rate (Kusumoto et al., 2018). TCP is indicated for symptomatic bradycardias unresponsive
to atropine therapy or when atropine is not immediately available or indicated. It may also be used as
a bridge until transvenous pacing can be accomplished or until the cause of the bradycardia is reversed
(as in cases of drug overdose or hyperkalemia). Some clinicians prophylactically apply pacing electrodes
to all critically ill patients with bradycardia to facilitate immediate TCP should decompensation occur
(Bessman, 2019). Whether or not TCP is effective, the patient should be prepared for transvenous
pacing, and expert consultation sought.

Procedure
Take appropriate standard precautions and verify that the procedure is indicated. Place the patient on
oxygen, if indicated. Assess the patient’s vital signs and establish IV access. Because continuous moni-
toring of the patient’s ECG is essential throughout the procedure, apply ECG electrodes. Position the
ECG electrodes as far away as possible from where the pacing pads are to be applied to minimize
distortion of the ECG signal by the pacing current (Boehm, 2007; Del Monte, 2006). Identify the
rhythm on the cardiac monitor. Record a rhythm strip and verify the presence of a paceable rhythm.
To improve electrode adherence and maximize the delivery of energy through the chest wall, pre-
pare the skin on the patient’s chest (and back if the anterior–posterior pad position is to be used) by
washing with a nonemollient soap and water (Spotts, 2017). When preparing the skin, avoid the use
of flammable liquids (e.g., alcohol, benzoin) because of the increased potential for burns (Spotts, 2017).
Remove any transdermal medication patches that may be present and wipe away any residue.
Apply adhesive pacing pads to the patient according to the manufacturer’s recommendations
(Fig. 3.43). Do not place the pads over open cuts, sores, drains, dressings, or over an implanted pace-
maker or defibrillator. Avoid placing the pacing pads over bone (e.g., sternum, spine, scapula) because
this increases the level of energy needed to achieve capture, increases patient discomfort, and increases
the possibility of noncapture (Spotts, 2017). When using the anterior–posterior position for pad place-
ment, place the anterior electrode between the xiphoid process and the left nipple, which corresponds
CHAPTER 3  Patient Assessment and Resuscitation Therapies 71

with the V2 to V3 ECG electrode position (Boehm, 2007; Del Monte, 2006). Ensure that the upper
edge of the electrode is below the nipple. If the patient is female, place the electrode beneath the breast
(Bessman, 2019). Place the posterior electrode beneath the left scapula and lateral to the spine at the
level of the heart. Some clinicians recommend placing the posterior pad first to prevent buckling of the
anterior electrode when rolling the patient to the side (Boehm, 2007). When using the anterior–lateral
position for pad placement, which is also called the sternum–apex position, the lateral (i.e., apex) pad
is placed lateral to the left nipple in the left midaxillary line, which corresponds with the V6 ECG
electrode position. The anterior electrode is placed to the right of the sternum and below the clavicle.
Do not reverse placement of the pacing pads; doing so can result in the need for more current to
achieve capture, which can result in increased patient discomfort (Del Monte, 2006).
Next, connect the pacing cable to the pacemaker and the adhesive pads on the patient. Turn on the
power to the pacemaker. Set the pacing rate to the desired number of paced pulses per minute (ppm)
(Fig. 3.44). In general, a rate that is between 60 and 90 ppm can maintain an adequate BP and cerebral
perfusion in an adult (Del Monte, 2006).
After setting the rate, start the pacemaker (Fig. 3.45). Slowly increase the stimulating current
(i.e., output or mA) until pacer spikes are visible before each QRS complex (i.e., electrical capture)
(Fig. 3.46). This control is usually labeled “Current,” “Pacer output,” or “mA.” Electrical capture occurs
when a pacing stimulus leads to ventricular depolarization, which is evidenced on the cardiac monitor
as a QRS complex after each pacing marker. Although the amount of current necessary to achieve
capture varies among individuals, it does not appear to correlate with body surface area or patient

Fig. 3.43  ​Apply adhesive pacing pads to the patient Fig. 3.44  ​Turn the pacemaker on and set the pacing
according to the manufacturer’s recommendations. rate to the desired number of ppm.  (From Roberts, J.R.,
(From Roberts, J.R., Custalow, C.B., Thomsen, T.W. [2019]. Custalow, C.B., Thomsen, T.W. [2019]. Roberts and
Roberts and Hedges’ clinical procedures in emergency Hedges’ clinical procedures in emergency medicine and
medicine and acute care, ed 7, Philadelphia, Elsevier.) acute care, ed 7, Philadelphia, Elsevier.)

Fig. 3.45  ​After setting the rate, start the pacemaker and Fig. 3.46  ​After achieving electrical capture, assess for
slowly increase the current output until electrical capture mechanical capture by palpating for a pulse. (From Rob-
is achieved. (From Roberts, J.R., Custalow, C.B., Thomsen, erts, J.R., Custalow, C.B., Thomsen, T.W. [2019]. Roberts
T.W. [2019]. Roberts and Hedges’ clinical procedures in and Hedges’ clinical procedures in emergency medicine
emergency medicine and acute care, ed 7, Philadelphia, and acute care, ed 7, Philadelphia, Elsevier.)
Elsevier.)
72 CHAPTER 3  Patient Assessment and Resuscitation Therapies

weight (Boehm, 2007; Del Monte, 2006). Electrical capture usually is seen in the form of a wide QRS
and a broad T wave on the ECG. The captured QRS complex may appear in a positive or negative
direction (Del Monte, 2006). For some patients, electrical capture is less obvious; it may only be seen
as a change in the shape of the QRS.

ACLS Pearl
During TCP, the muscle twitching that occurs with skeletal muscle contraction is not an indicator
of electrical or mechanical capture (Boehm, 2007).

Next, assess mechanical capture. Mechanical capture refers to the contraction of the myocardium and
occurs when pacing produces a response that can be measured, such as a palpable pulse. To minimize
confusion between the presence of an actual pulse and skeletal muscle contractions caused by the pace-
maker, assess mechanical capture by assessing the patient’s femoral pulse, right brachial pulse, or right
radial pulse. The patient’s palpated pulse rate should be at least equal to that of the pacer. Additional
methods used to confirm pacemaker capture include the use of a bedside echocardiogram or verifying
that the patient’s pulse oximetry waveform is consistent with that assessed with both electrical and
mechanical pacemaker capture. After achieving capture, continue pacing at an output level slightly
higher (e.g., about 2 mA) than the threshold of consistent electrical capture.
Assess the patient’s mental status, oxygen saturation, skin color, BP, and other vital signs. Closely
monitor the patient and assess the skin under the pacing electrodes for irritation after the first
30 minutes of pacing and periodically after that (Boehm, 2007). Documentation should include the
following (Boehm, 2007; Del Monte, 2006):
• The date and time pacing was initiated (including baseline and pacing rhythm strips)
• The pacing rate selected
• The current required to obtain capture
• The patient’s response with capture (i.e., mental status, BP, oxygen saturation)
• Medications administered during the procedure
• The date, time, and reason pacing was terminated, if applicable

Limitations
The main limitation of TCP is patient discomfort. The discomfort is proportional to the intensity of
skeletal muscle contraction and the direct electrical stimulation of cutaneous nerves. Patients have
described the sensations associated with skeletal muscle contractions as tapping, twitching, or thudding
(Boehm, 2007; Del Monte, 2006). Sensations associated with cutaneous nerve stimulation have been
described as tingling, stinging, pinching, or burning (Boehm, 2007; Del Monte, 2006). When using the
anterior–posterior position for pacing pad placement, discomfort may be reduced in some patients by
moving the anterior electrode from its V2 to V3 position more laterally to a V6 position, recognizing
that pacing will be temporarily discontinued during the period in which the pacing pad is moved
(Boehm, 2007; Del Monte, 2006).
Another possible limitation of TCP is the use of incompatible pacing electrodes. For example, TCP
electrodes used in the out-of-hospital setting may be incompatible with those used in the emergency
department. Similarly, TCP electrodes/connectors used in the emergency department may be incom-
patible with those used in other areas of the hospital (Bessman, 2019).
Capture can be challenging to achieve, or it may be inconsistent for some patients. Increased
stimulating current may be required for patients with increased chest wall muscle mass, COPD, pleu-
ral effusions, dilated cardiomyopathy, hypoxia, or metabolic acidosis because of the extremely high
current thresholds required. Possible complications of TCP appear in Box 3.11.

BOX 3.11 Transcutaneous Pacing: Possible Complications


• Interference with sensing from patient agitation or muscle contractions
• Discomfort as a result of the electrical stimulation of the skin and muscles
• Failure to recognize that the pacemaker is not capturing
• Failure to recognize the presence of underlying treatable ventricular fibrillation
• Tissue damage, including third-degree burns, with improper or prolonged external pacing
• When pacing is prolonged, pacing threshold changes, thereby leading to capture failure
CHAPTER 3  Patient Assessment and Resuscitation Therapies 73

Alternative Therapies
Therapies such as cough CPR, the precordial thump, and fist pacing are temporizing measures that are
sometimes used in specific situations with select patients when definitive therapy is not immediately
available.
Cough CPR is repeated deep breaths followed by forceful and repetitive coughing. At the onset of the
abnormal rhythm (and before consciousness is lost), the patient is instructed to cough hard and keep
coughing every few seconds. This technique has been successfully used in the cardiac catheterization
laboratory. Current resuscitation guidelines note that cough CPR may be considered as a temporizing
measure for the witnessed, monitored onset of a hemodynamically significant tachydysrhythmia or bra-
dydysrhythmia before a loss of consciousness without delaying definitive therapy (Panchal et al., 2020).
A precordial thump is a forceful blow delivered to the adult sternum to terminate dysrhythmias,
such as ventricular tachycardia (VT). The thump is delivered sharply to the patient’s midsternum using
the ulnar side of a clenched fist from a height of about 8 inches (20 cm). Possible complications as-
sociated with a precordial thump include rate acceleration of VT, conversion of VT into VF, develop-
ment of complete AV block, and asystole. Although this technique is not recommended for routine
use during cardiac arrest, current resuscitation guidelines indicate that the precordial thump may be
considered at the onset of a rescuer-witnessed, monitored, unstable ventricular tachydysrhythmia if a
defibrillator is not immediately ready for use and the thump can be performed without delaying CPR
or shock delivery (Panchal et al., 2020).
Fist pacing, also called percussion pacing, is the delivery of repetitive, rhythmic thumps to the cen-
ter of the chest. This technique is typically used as a means of providing mechanical pacing to terminate
a hemodynamically unstable bradydysrhythmia until a transcutaneous or transvenous pacemaker is
available. Current resuscitation guidelines say that fist pacing may be considered as a temporizing
measure in exceptional circumstances, such as witnessed, monitored in-hospital arrest (e.g., cardiac
catheterization laboratory) for bradyasystole before a loss of consciousness and if performed without
delaying definitive therapy (Panchal et al., 2020).

VASCULAR ACCESS
During circulatory collapse or cardiac arrest, the preferred vascular access site is the largest, most
accessible vein that does not require the interruption of resuscitation efforts. If vascular access has not
been established before the arrest, attempt peripheral IV access—preferably the antecubital or external
jugular vein—for medication administration (Granfeldt et al., 2020). Normal saline is the preferred IV
fluid because it expands intravascular volume better than dextrose.
If you cannot quickly achieve peripheral IV access, attempt IO access before considering the place-
ment of a central line. Research shows that IO placement for resuscitation is significantly faster than
central line placement, with a lower failure rate (Deitch, 2019). To improve flow rates during an IO
infusion, the use of a pressure bag or infusion pump may be necessary.

MEDICATION ADMINISTRATION
During cardiac arrest, give IV/IO drugs rapidly by bolus injection. Follow each drug with a 20 mL
bolus of IV fluid to aid delivery of the drug(s) to the central circulation.
If you are unable to achieve vascular access during a cardiac arrest, you can use the tracheal route to
administer selected medications (e.g., epinephrine, lidocaine, naloxone). The tracheal route is not pre-
ferred because it is associated with lower blood concentrations than the same dose given intravascularly,
and lower rates of return of spontaneous circulation and survival (Panchal et al., 2020). The ET dose
should be at least equal to the IV/IO dose of the same medication when given for the same indication.
Most studies show that higher doses (e.g., 2 to 2.5 times the intravascular dose) are often needed when
giving drugs tracheally. The optimal tracheal dose of most drugs is unknown. In adults, the total recom-
mended volume to instill is 10 mL. Deliver several rapid ventilations with a BMD after administering
the drug and diluent to enhance drug delivery into the lungs.
Common medications used when managing cardiovascular emergencies and mentioned throughout
this book appear in Table 3.10.
74 CHAPTER 3  Patient Assessment and Resuscitation Therapies

TABLE 3.10 Common Medications Used in Cardiovascular Emergencies


Medication Action, Indications, Adult Dosage, and Considerations
Adenosine Action • Naturally present throughout the body
(Adenocard) • Rapidly metabolized in the blood vessels; half-life is less than
10 seconds’
• Briefly and rapidly slows impulse formation in the SA node and
conduction time through the AV node
Indications • Most stable narrow-QRS regular tachycardias
• Unstable narrow-QRS regular tachycardia while making
preparations for synchronized cardioversion
• Stable, regular, monomorphic wide-QRS tachycardia
(diagnostic aid)
Adult dosage Initial dose is 6 mg rapid IV push over 1 to 3 seconds. If no re-
sponse within 1 to 2 minutes, give 12 mg rapid IV push. May re-
peat 12 mg dose once in 1 to 2 minutes. Follow each adenosine
dose immediately with a 20 mL normal saline flush.
Considerations • Constant ECG monitoring is essential
• Do not administer for hemodynamically unstable, irregularly
irregular, or polymorphic wide-QRS tachycardia (Panchal et al.,
2020)
• Adverse effects (e.g., facial flushing, dyspnea, chest pressure)
and patient reports of a metallic taste and feeling of impending
doom are common but transient and usually resolve within
1 to 2 minutes
• Reduce the dose by one-half in patients on dipyridamole (Persan-
tine), carbamazepine (Tegretol), those with transplanted hearts,
or if given via a central venous line
• Larger doses may be required for patients with a significant
blood level of theophylline, caffeine, or theobromine
Amiodarone Action • Blocks sodium, potassium, and calcium channels and also possesses
(Cordarone) alpha- and beta-adrenergic-blocking properties
• Directly depresses the automaticity of the SA and AV nodes
• Slows conduction through the AV node and in the accessory
pathway of patients with WPW preexcitation pattern
• Coronary and peripheral vasodilator
Indications • pVT/VF unresponsive to CPR, defibrillation, and epinephrine
• Stable narrow-QRS tachycardias if the rhythm persists
despite vagal maneuvers or adenosine or the tachycardia
is recurrent
• To control the ventricular rate in AFib with a rapid ventricular
response without preexcitation
• Stable wide-complex tachycardia
• Stable monomorphic VT
• PMVT with normal QT interval
Adult dosage • pVT/VF: Initial bolus—300 mg IV/IO; can be followed by 1 dose
of 150 mg if needed
• Other indications: Loading dose—150 mg IV over 10 minutes.
May repeat every 10 minutes if needed. After conversion,
follow with an infusion of 10 to 50 mg/h over 24 hours
(Panchal et al., 2020). Total dose over 24 hours should not
exceed 2.2 g.
Considerations • Adverse effects include hypotension and bradycardia, including
AV block. Slow the infusion rate or discontinue if seen.
• Prolongs the PR, QRS, and QT intervals, and has an additive
effect with other medications that prolong the QT interval
• Avoid in patients with preexcited AFib or atrial flutter
CHAPTER 3  Patient Assessment and Resuscitation Therapies 75

TABLE 3.10 Common Medications Used in Cardiovascular Emergencies—cont’d


Medication Action, Indications, Adult Dosage, and Considerations
Atropine sulfate Action • Increases heart rate and AV conduction velocity by blocking the
effects of the vagus nerve on the SA and AV nodes
• Relaxes bronchial smooth muscle, dilates pupils, decreases se-
cretion from salivary glands, sweat glands, bronchial glands, and
acid-secreting cells of the stomach
• Decreases motility of the gastrointestinal tract
Indications First-line drug for symptomatic bradycardia (e.g., sinus bradycardia,
sinus arrest, AV block at the level of the AV node)
Adult dosage 0.5 to 1 mg IV every 3 to 5 minutes to a total dose of 3 mg
(Kusumoto et al., 2018)
Considerations • Second-degree AV block type II and third-degree AV blocks are un-
likely to respond to atropine. In these situations, an IV infusion of a
beta-adrenergic drug (e.g., dopamine, epinephrine) or TCP is pre-
ferred while preparing for transvenous pacing (Link et al., 2015)
• Do not push slowly or in smaller than recommended doses; it
may cause paradoxical slowing of the heart rate
• May result in tachycardia, palpitations, and ventricular ectopy
• Use with caution in acute coronary syndromes; excessive increases in
heart rate may further worsen ischemia or increase the size of infarction
• Transplanted hearts do not usually respond to atropine because
they lack vagal nerve innervation
Beta blockers (e.g., Action • Slow heart rate and AV node conduction
atenolol, esmo- • Reduce blood pressure
lol, metoprolol, • Decrease myocardial contractility
propranolol) • Decrease myocardial oxygen consumption
Indications • Stable narrow-QRS tachycardias if the rhythm persists despite
vagal maneuvers or adenosine or if the tachycardia is recurrent
• For ventricular rate control in AFib and atrial flutter if no signs of
heart failure or preexcitation
• Specific forms of PMVT (e.g., ischemic PMVT, congenital long-QT
syndrome PMVT, catecholaminergic PMVT)
Considerations • Evidence is inadequate regarding the routine use of beta blockers
early (within the first hour) after ROSC (Panchal et al., 2018)
• Avoid in patients with wide-QRS tachycardia, preexcited AFib
or atrial flutter, and patients with reactive airway disease or
decompensated heart failure
• Some beta blockers should be used with caution in patients with
impaired renal or liver function
• Adverse effects include hypotension, bradycardia, and the
precipitation of heart failure
Calcium blockers Action Nondihydropyridine calcium blockers (e.g., diltiazem, verapamil)
decrease heart rate and myocardial contractility, slow conduction
through the AV node, and have some peripheral arterial dilatory
effects
Indications • Stable narrow-QRS tachycardia if the rhythm persists despite
vagal maneuvers or adenosine or if the tachycardia is recurrent
• To control the ventricular rate in patients with AFib or atrial flutter
without preexcitation
Adult dosage • Diltiazem: Initial dose is 15 to 20 mg (0.25 mg/kg) IV over 2 minutes.
If needed, follow in 15 minutes with 20 to 25 mg (0.35 mg/kg) IV
over 2 minutes.
• Verapamil: Initial dose is 2.5- to 5-mg slow IV push over 2 minutes.
Give over 3 to 4 minutes in older adults or when BP is within the
lower range of normal. May repeat with 5 to 10 mg in 15 to 30
minutes (if no response and BP remains normal or elevated) to a to-
tal dose of 20 mg.
Continued
76 CHAPTER 3  Patient Assessment and Resuscitation Therapies

TABLE 3.10 Common Medications Used in Cardiovascular Emergencies—cont’d


Medication Action, Indications, Adult Dosage, and Considerations
Considerations • Can worsen hypotension and should not be given to patients with
a systolic BP of less than 90 mm Hg. Use with caution in patients
with mild to moderate hypotension. Closely monitor BP, heart
rate, and the ECG.
• Avoid in patients with wide-QRS tachycardias unless the dys-
rhythmia is known to be of supraventricular origin and not being
conducted by an accessory pathway (Panchal et al., 2020)
• Avoid in patients with impaired ventricular function, heart failure,
cardiomyopathy, and acute coronary syndromes (Panchal et al., 2020)
• Avoid in patients with preexcited AFib/atrial flutter
• Contraindicated in second- or third-degree AV block
• Do not give IV calcium blockers and IV beta blockers together
or in close proximity (within a few hours); doing so may cause
severe hypotension
Dopamine Action • Naturally occurring immediate precursor of norepinephrine in the
(Intropin, body
Dopastat) • Increases heart rate and contractility
Indications • Temporizing measure in the management of symptomatic brady-
cardia that has not responded to atropine, or for which atropine
is inappropriate, while waiting for a pacemaker
• Hypotension that occurs after ROSC
Adult dosage Continuous IV infusion of 5 to 20 mcg/kg/minute; start at 5 mcg/kg/
minute and increase by 5 mcg/kg/minute every 2 minutes
(Kusumoto et al., 2018); titrate infusion rate according to BP
and other clinical responses
Considerations • Carefully monitor BP and ECG
• Administer using an infusion pump
• Correct hypovolemia before beginning dopamine therapy for the
treatment of hypotension and shock
• Extravasation into surrounding tissue may cause necrosis and
sloughing
• Gradually taper this drug before discontinuing the infusion
Epinephrine Action Increases heart rate, force of contraction, BP, and myocardial
(Adrenalin) oxygen consumption, and causes vasoconstriction
Indications • Cardiac arrest: VF, pVT, asystole, PEA
• Symptomatic bradycardia
• Management of hypotension during postcardiac arrest care
Adult dosage • Cardiac arrest: IV/IO: 1 mg (10 mL) of 1:10,000 solution IV push, follow
with 20 mL fluid flush; may repeat 1 mg dose every 3 to 5 minutes
• Symptomatic bradycardia or hypotension: Continuous IV infusion
at 2 to 10 mcg/minute titrated to hemodynamic response
• For hypotension postcardiac arrest after fluid bolus if tolerated:
continuous IV/IO infusion of 0.1 to 0.5 mcg/kg/minute
Considerations • Available in different concentrations and different medication con-
tainers. Read the label carefully before giving to ensure that you are
giving the right dose and using the right concentration of the drug
• Give as soon as feasible in cardiac arrest associated with a
nonshockable rhythm (Panchal et al., 2020)
• In trials, epinephrine is given after the third shock in cardiac ar-
rest associated with a shockable rhythm; however, the American
Heart Association supports giving epinephrine after the second
shock (i.e., when initial attempts with CPR and defibrillation have
proved unsuccessful (Panchal et al., 2020)
• Increases myocardial oxygen demand; may cause postresuscitation
myocardial dysfunction and ventricular dysrhythmias
• Administer an epinephrine infusion via an infusion pump
• Check IV site frequently for signs of tissue sloughing
• Should not be administered in the same IV line as alkaline
solutions: this inactivates epinephrine
CHAPTER 3  Patient Assessment and Resuscitation Therapies 77

TABLE 3.10 Common Medications Used in Cardiovascular Emergencies—cont’d


Medication Action, Indications, Adult Dosage, and Considerations
Isoproterenol Action Nonselective beta-adrenergic agonist that increases heart rate and
(Isuprel) contractility, enhancing the function of the SA and AV nodes,
without causing vasoconstriction
Indications • Temporizing measure in the management of symptomatic brady-
cardia that has not responded to atropine, or for which atropine
is inappropriate, while waiting for a pacemaker
• Torsades de pointes unresponsive to magnesium sulfate
Adult dosage • Symptomatic bradycardia: 20 to 60 mcg IV bolus followed by
doses of 10 to 20 mcg IV, or an infusion of 1 to 20 mcg/minute
based on heart rate response (Kusumoto et al., 2018)
• Torsades de pointes: titrate to increase heart rate until dysrhythmia
is suppressed
Considerations • Because it increases myocardial oxygen demand, avoid use in
patients with coronary ischemia
• Carefully monitor the ECG, BP, and drip rate; also monitor for the
development of ischemic chest discomfort
Lidocaine Action Shortens the effective refractory period and duration of the action
(Xylocaine) potential in the His-Purkinje system without adversely affecting
the automaticity of the SA node
Indications • Stable monomorphic VT
• Consider as an alternative to amiodarone for pVT/VF that is
unresponsive to defibrillation (Panchal et al., 2020)
Adult dosage • 1 to 1.5 mg/kg IV/IO bolus; consider repeat dose (0.5 to 0.75 mg/
kg) at 5 to 10 minute intervals
• Cumulative IV/IO bolus dose should not exceed 3 mg/kg
• Maintenance infusion: 1 to 4 mg/minute
Considerations • Evidence is inadequate regarding the routine use of lidocaine
after cardiac arrest or early (within the first hour) after ROSC
• Prophylactic administration may be considered in certain circum-
stances (e.g., during EMS transport) when treatment of recurrent
pVT/VF may be challenging (Panchal et al., 2018)
Magnesium sulfate Action • Essential for the activity of many enzyme systems
• Plays an essential role in regulating the flow of sodium, potassium,
and calcium across cell membranes
• Vasodilator
Indications • May be considered for PMVT with a prolonged QT interval (Panchal
et al., 2018)
• For cardiotoxicity and cardiac arrest from severe hypomagnese-
mia, in addition to standard ACLS care (Panchal et al., 2020)
Adult dosage • If pulseless, give 1 to 2 g or 2 to 4 mL of a 50% solution diluted
in 10 mL D5W or normal saline IV/IO push over 5 to 20 minutes
• If pulse present, give 1 to 2 g IV diluted in 50 to 100 mL D5W or
normal saline over 5 to 60 minutes
Considerations • Should not be routinely used during cardiac arrest (Panchal et al.,
2020)
• Use with caution in patients receiving digitalis, patients with
impaired renal function, and patients with preexisting heart
blocks
• Calcium is the antidote for magnesium toxicity
Naloxone (Narcan) Action Displaces opioids at opioid-occupied receptor sites in the CNS;
reverses opioid-induced sleep or sedation, increasing ventilatory
rate
Indications • Suspected acute opioid overdose
• Complete or partial reversal of opioid-induced respiratory
depression
Adult dosage • IV/IM/IO/subcut: 0.04 to 0.4 mg every 2 to 3 minutes as
needed
• IN: 2 mg every 3 to 5 minutes as needed
Continued
78 CHAPTER 3  Patient Assessment and Resuscitation Therapies

TABLE 3.10 Common Medications Used in Cardiovascular Emergencies—cont’d


Medication Action, Indications, Adult Dosage, and Considerations
Considerations • Ineffective if respiratory depression is caused by hypnotics, seda-
tives, or other nonnarcotic CNS depressants
• Effects of opioids are often longer than those of naloxone; thus
respiratory depression may return when naloxone has worn off.
Monitor the patient closely. Repeated small doses or an infusion
of naloxone can be beneficial (Panchal et al., 2020)
• Higher doses may be needed to reverse the effects of propoxy-
phene, pentazocine, methadone, and fentanyl
Norepinephrine Action Causes vasoconstriction, increases heart rate, and increases
(Levophed) myocardial oxygen requirements
Indications Management of hypotension during postcardiac arrest care
Adult dosage For hypotension postcardiac arrest after fluid bolus if tolerated:
continuous IV/IO infusion of 0.1 to 0.5 mcg/kg/minute
Considerations • Extravasation into surrounding tissue may cause necrosis
and sloughing; it should be administered via an infusion
pump into a central vein or a large peripheral vein to reduce
the risk of necrosis of the overlying skin from prolonged
vasoconstriction
• Monitor BP every 2 to 3 minutes until stabilized, then every 5 min-
utes. Continuously monitor the patient’s ECG
• Taper off gradually when discontinuing the infusion
Procainamide Action • Blocks sodium channels, delaying repolarization and increasing
(Pronestyl) the action potential duration
• Suppresses ectopy in atrial and ventricular tissue
• Prolongs the PR and QT intervals
Indications • To control the ventricular rate in the patient with preexcited AFib
• Stable wide-complex tachycardia
• Stable monomorphic VT with a normal QT interval
Adult dosage • 20 to 50 mg/minute IV (or 100 mg every 5 minutes) until the dys-
rhythmia resolves, hypotension ensues, the QRS prolongs by more
than 50% of original width, or a total cumulative dose of 17 mg/
kg is administered
• Maintenance infusion of 1 to 4 mg/minute
Considerations • During administration, carefully monitor the patient’s ECG and
BP. Watch the ECG closely for increasing PR and QT intervals,
widening of the QRS complex, heart block, and/or onset of
TdP. If the BP falls 15 mm Hg or more, temporarily discontinue
administration.
• Reduce the maintenance infusion rate in patients with impaired
or reduced renal function
• Avoid use in patients with QT prolongation or heart failure
Sotalol (Betapace) Action Slows heart rate, decreases AV nodal conduction, decreases
myocardial contractility, and lengthens the refractory period in
cardiac tissue
Indications • Stable wide-complex tachycardia
• Stable monomorphic VT
Adult dosage (100 mg) 1.5 mg/kg IV over 5 minutes
Considerations Use with caution in patients with bronchospastic disease
• Monitor carefully for bronchospasm, bradycardia, hypotension,
and new dysrhythmias, including TdP
• Closely monitor the QT interval every 2 to 4 hours after each
dose; if the QT interval lengthens to 0.5 second or greater,
reduce the dose or discontinue the drug (Page et al., 2016)
• Avoid in patients with a prolonged QT interval, those taking other
QT-prolonging drugs, those with uncontrolled heart failure or
renal impairment, and those with hypokalemia
CHAPTER 3  Patient Assessment and Resuscitation Therapies 79

TABLE 3.10 Common Medications Used in Cardiovascular Emergencies—cont’d


Medication Action, Indications, Adult Dosage, and Considerations
Vasopressin Action Causes constriction of peripheral, coronary, and renal vessels
Indications Cardiac arrest
Considerations • Can increase peripheral vascular resistance and cause cardiac
ischemia
• Tissue necrosis if extravasation occurs
• Vasopressin, alone or in combination with epinephrine, may be
considered during an adult cardiac arrest but offers no advantage
as a substitute for epinephrine alone (Panchal et al., 2020)

ACLS, Advanced cardiac life support; AFib, atrial fibrillation; AT, atrial tachycardia; AV, atrioventricular; BP, blood pressure;
CNS, central nervous system; CPR, cardiopulmonary resuscitation; D5W, dextrose 5% in water; ECG, electrocardiogram;
EMS, emergency medical services; IM, intramuscular; IN, intranasal; IO, intraosseous; IV, intravenous; MI, myocardial infarc-
tion; PEA, pulseless electrical activity; PMVT, polymorphic ventricular tachycardia; pVT, pulseless ventricular tachycardia;
ROSC, return of spontaneous circulation; SA, sinoatrial; SBP, systolic blood pressure; subcut, subcutaneous; SVT, supraven-
tricular tachycardia; TCP, transcutaneous pacing; TdP, torsades de pointes; VF, ventricular fibrillation; VT, ventricular
tachycardia; WPW, Wolff-Parkinson-White.
80 CHAPTER 3  Patient Assessment and Resuscitation Therapies

STOP AND REVIEW


Identify one or more choices that best complete the statement or answer the question.

_ ____ 1. The purpose of the primary assessment is to:


a. Perform a detailed head-to-toe physical examination.
b. Determine the number of personnel needed to assist in the patient’s care.
c. Focus on the patient’s chief complaint/reason for seeking medical assistance.
d. Detect the presence of life-threatening problems that require rapid intervention.

_____ 2. Upon finding an unresponsive adult patient, you called for help and asked that
someone get an automated external defibrillator (AED) or manual defibrillator.
Your next action should be to:
a. Begin chest compressions.
b. Reposition the patient’s head.
c. Open the airway and begin rescue breathing.
d. Simultaneously look for breathing and feel for a pulse.

_____ 3. During cardiac arrest, IV medications given via a peripheral IV should be given
rapidly by bolus injection and followed with a __ bolus of IV fluid.
a. 5 mL
b. 10 mL
c. 20 mL
d. 50 to 100 mL

_ ____ 4. For which of the following rhythms is defibrillation indicated? Select all that apply.
a. VF
b. pVT
c. Pulseless electrical activity (PEA)
d. Asystole

_ ____ 5. Endotracheal intubation:


a. Is contraindicated in unresponsive patients.
b. Eliminates the risk of aspiration of gastric contents.
c. Should be preceded by efforts to ventilate by another method.
d. When attempted, should be performed in less than 60 seconds.

_ ____ 6. From the subsequent list, select the choices that are possible complications of TCP.
a. Emboli
b. Flail chest
c. Pneumothorax
d. Tissue damage
e. Failure to recognize VF

_ ____ 7. Select the leads that view the lateral surface of the left ventricle.
a. I
b. II
c. III
d. aVL
e. aVF
f. V4
g. V5
h. V6
CHAPTER 3  Patient Assessment and Resuscitation Therapies 81

_____ 8. Team members are ventilating a patient with a BMD. Supplemental oxygen is
connected to the bag and set at a flow rate of 10 to 15 L/min. A reservoir is not
in use. These rescuers can successfully deliver about __ oxygen to the patient.
a. 16%
b. 21%
c. 40% to 60%
d. 90% to 100%

_ ____ 9. Which of the following statements is true about a nasopharyngeal airway (NPA)?
a. An NPA can be placed in either nostril to help maintain an open airway.
b. The distal tip of a properly positioned NPA rests in the patient’s trachea.
c. Use an NPA only for unresponsive patients who do not have a gag reflex.
d. A correctly sized NPA extends from the corner of the patient’s mouth to the
tip of the ear lobe.

_____ 10. A patient is in cardiac arrest and CPR is in progress. Two attempts to establish
peripheral IV access have been unsuccessful. To administer medications to this
patient, your best course of action in this situation is to:
a. Proceed with the insertion of a central line.
b. Continue attempts to establish peripheral IV access.
c. Intubate the patient and administer drugs via the tracheal tube.
d. Establish vascular access by means of an intraosseous infusion.

_____ 11. Which of the following statements are true about synchronized cardioversion?
Select all that apply.
a. Synchronized cardioversion is the electrical therapy of choice for symptomatic
bradycardias.
b. During cardioversion, a shock is delivered during the QRS complex.
c. Synchronized cardioversion is not used to treat disorganized rhythms or those
that do not have a clearly identifiable QRS complex.
d. To be effective, high-quality CPR must precede electrical therapy with synchronized
cardioversion.

_ ____ 12. Which of the following will deliver the highest oxygen concentration?
a. A nasal cannula with an oxygen flow rate of 4 L/min
b. A pocket mask with an oxygen flow rate of 10 L/min
c. A simple face mask with an oxygen flow rate of 8 L/min
d. A nonrebreather mask with an oxygen flow rate of 10 L/min

_ ____ 13. Which of the following are true regarding TCP?


a. Do not place pacing pads over the spine or sternum.
b. Assess electrical capture by palpating the patient’s femoral pulse.
c. Sedation, analgesia, or both may be needed to minimize patient discomfort.
d. The stimulating current delivered during each pacing impulse is about the same
as that delivered during defibrillation.

_____ 14. Which of the following memory aids may be used when evaluating a patient’s level
of responsiveness?
a. CAB
b. AVPU
c. ABCDE
d. OPQRST
82 CHAPTER 3  Patient Assessment and Resuscitation Therapies

_____ 15. If no head or neck trauma is suspected, which of the following techniques should
healthcare professionals use to open the airway?
a. Jaw thrust
b. Tongue–jaw lift
c. Head tilt–chin lift
d. Head tilt–neck lift

_____ 16. A 60-year-old woman has experienced a cardiopulmonary arrest. A healthcare


professional trained in endotracheal intubation has intubated the patient. Which
of the following is the preferred initial method for confirming and monitoring the
correct placement of an ETT?
a. Obtaining a chest radiograph
b. Using an esophageal detector device
c. Using continuous waveform capnography
d. Observing condensation in the ETT during BMV

_____ 17. A 19-year-old man is unresponsive and not breathing. A slow, weak pulse is present.
Your best course of action will be to:
a. Begin chest compressions.
b. Insert an advanced airway.
c. Insert an OPA and begin BMV.
d. Administer oxygen by simple face mask.

_ ____ 18. An OPA:


a. May result in an airway obstruction if improperly inserted.
b. Is usually well tolerated in the responsive or semiresponsive patient.
c. Should be lubricated with a petroleum-based lubricant before insertion.
d. May inadvertently enter the cranial vault if used in a patient with a craniofacial
injury.

_____ 19. A 75-year-old man has experienced a cardiac arrest and high-quality CPR is in
progress. The cardiac monitor shows VF. You have a monophasic defibrillator
available to you. You should:
a. Defibrillate using 360 J.
b. Perform synchronized cardioversion.
c. Defibrillate using the machine’s lowest energy setting.
d. Continue CPR and await the arrival of a biphasic defibrillator.

_____ 20. Which of the following are recommended when performing defibrillation? Select
all that apply.
a. Check for a pulse immediately after defibrillation to determine next steps.
b. Visually check and ensure that everyone is clear of the patient before shock
delivery.
c. Remove transdermal medication patches or ointment from the patient’s chest
before the procedure.
d. All team members, except for the chest compressor, should clear the patient as
the machine charges.

_ ____ 21. Which of the following leads view the heart in the frontal plane?
a. I, II, and III
b. aVR, aVL, and aVF
c. V1, V2, and V3
d. V4, V5, and V6
CHAPTER 3  Patient Assessment and Resuscitation Therapies 83

_ ____ 22. Which of the following reflects the correct order for performing a BLS assessment?
a. Ensure scene safety, assess for breathing and a pulse, assess responsiveness,
activate the emergency response system, and provide indicated care
b. Ensure scene safety, activate the emergency response system, check for a pulse,
assess responsiveness, assess for breathing, and provide indicated care
c. Ensure scene safety, check for a pulse, activate the emergency response system,
assess responsiveness, assess for breathing, and provide indicated care
d. Ensure scene safety, assess responsiveness and activate the emergency response sys-
tem when warranted, assess for breathing and a pulse, and provide indicated care

STOP AND REVIEW  ANSWERS


1. D. The primary assessment is a rapid hands-on evaluation to detect the presence of life-threatening prob-
lems affecting the patient’s respiratory, cardiac, or neurologic function and to correct them immediately.

2. D. If you find an unresponsive patient, call for help and ask someone to get an AED or manual
defibrillator. Look at the chest for movement while simultaneously feeling for a carotid pulse for up
to 10 seconds. Gasping, if present, is abnormal breathing and should not be interpreted as a sign of
effective breathing. If the patient has no pulse, begin chest compressions. If the patient is breathing
normally, continue monitoring until additional help arrives. If the patient is not breathing normally
but a pulse is present, provide rescue breathing and recheck for a pulse about every 2 minutes.

3. C. In cardiac arrest, drugs that are given from a peripheral vein require 1 to 2 minutes to reach the
central circulation. Following each drug with a 20 mL bolus of IV fluid can help speed the delivery
of the drug to the central circulation.

4. A, B. Defibrillation is indicated in the management of pVT and VF. It is not indicated in the man-
agement of PEA. Remember that defibrillation is performed to depolarize the myocardial cells at
one time and provide an opportunity for one of the heart’s natural pacemakers to take over. In PEA,
an organized rhythm is present on the monitor. Thus pacemaker activity is already present, but there
is inadequate cardiac output and no pulse. PEA is not shocked because a shock could disrupt the
organized rhythm and cause chaos (i.e., VF). Defibrillation is not indicated in asystole.

5. C. Endotracheal intubation should be preceded by attempts to ventilate by another method. This


procedure is indicated in situations where the patient is unable to protect their airway. Endotracheal
intubation reduces (but does not eliminate) the risk of aspiration of gastric contents and, when at-
tempted, should be performed in less than 30 seconds.

6. D, E. Tissue damage, including third-degree burns, can occur with improper or prolonged external
pacing. Failure to recognize the presence of underlying treatable ventricular fibrillation is another
possible complication of TCP.

7. A, D, G, H. Leads I, aVL, V5, and V6 view the lateral surface of the left ventricle.

8. C. A BMD that is used without supplemental oxygen will deliver 21% oxygen (i.e., room air, not
expired air) to the patient. A BMD that is used with supplemental oxygen set at a flow rate of
10 to 15 L/min delivers about 40% to 60% oxygen to the patient when a reservoir is not used.
A BMD that is used with supplemental oxygen set at a flow rate of 10 to 15 L/min and with an
attached reservoir delivers approximately 90% to 100% oxygen to the patient.

9. A. An NPA can be used in an unresponsive patient and may be useful in semiresponsive patients
who have a gag reflex. It can be placed in either nostril to help maintain an open airway. To select
an NPA of the proper size, hold the device against the side of the patient’s face. Select an airway
that extends from the tip of the patient’s nose to the angle of the jaw or to the earlobe. When an
NPA of the proper size is correctly positioned, the tip rests in the back of the throat.
84 CHAPTER 3  Patient Assessment and Resuscitation Therapies

10. D. When peripheral IV cannulation is unsuccessful or is taking too long, an IO infusion is an


alternative method of gaining access to the vascular system. It should be considered before consid-
ering the placement of a central line. To improve flow rates during an IO infusion, the use of a
pressure bag or infusion pump may be necessary. If IV or IO access cannot be achieved to give
drugs during a cardiac arrest, the tracheal route can be used to give selected medications; however,
intravascular drug administration provides more predictable drug delivery and pharmacologic
effect.

11. B, C. Synchronized cardioversion is the timed delivery of a shock during the QRS complex. It is
indicated in the management of a patient with a pulse who is exhibiting serious signs and symp-
toms related to a tachycardia. It is used to treat rhythms that have a clearly identifiable QRS
complex and a rapid ventricular rate (such as some narrow-QRS tachycardias and monomorphic
VT). Synchronized cardioversion is not used to treat disorganized rhythms (e.g., polymorphic VT)
or those that do not have a clearly identifiable QRS complex (e.g., VF). Because cardioversion is
not used for patients who have no pulse, the performance of high-quality CPR is irrelevant.

12. D. Of the oxygen delivery devices listed, a nonrebreather mask with an oxygen flow rate of
10 L/min will deliver the highest oxygen concentration.

13. A, C. Avoid placing pacing pads over bone (e.g., sternum, spine, scapula) because this increases the
level of energy needed to achieve capture, increases patient discomfort, and increases the possibil-
ity of noncapture. Electrical capture occurs when a pacing stimulus leads to ventricular depolariza-
tion and is seen in the form of a wide QRS and a broad T wave on the ECG. Mechanical capture
refers to the contraction of the myocardium and occurs when pacing produces a response that can
be measured, such as a palpable pulse. TCP may be useful for symptomatic bradycardias when the
slow heart rate causes the patient’s signs and symptoms. The stimulating current delivered during
each pacing impulse is less than 1/1000 of that delivered during defibrillation. Because TCP is
painful in conscious patients, sedation, analgesia, or both may be needed to minimize the patient’s
discomfort associated with this procedure.

14. B. The AVPU acronym is used to assess a patient’s level of responsiveness quickly. AVPU—Alert,
responds to Verbal stimuli, responds to Painful stimuli, Unresponsive. ABCDE is an acronym that
reflects the components of the primary survey. Use the OPQRST acronym when evaluating a
patient’s complaint of pain. CAB is an acronym that emphasizes the importance of performing
chest compressions first, followed by opening the airway and assessing breathing, in victims of
cardiac arrest.

15. C. The head tilt–chin lift is the preferred technique for opening the airway of an unresponsive
patient without a suspected cervical spine injury. If trauma is suspected, use the jaw thrust maneu-
ver. Healthcare professionals should use the head tilt–chin lift maneuver to open the airway if use
of the jaw thrust maneuver is unsuccessful.

16. C. In addition to clinical assessments, the use of continuous waveform capnography is preferred
as the initial method for confirming and monitoring the correct placement of an ETT.

17. C. The patient has experienced a respiratory arrest. Your best course of action will be to insert an
OPA and begin positive pressure ventilation with a BMD. Chest compressions are not indicated
because the patient has a pulse. Although the insertion of an advanced airway is appropriate, it is
typically preceded by another form of ventilation (such as BMV), while preparations are made to
insert the airway. The use of a simple face mask is inappropriate because it can only be used in a
spontaneously breathing patient.

18. A. An OPA should only be used in unresponsive patients who have no cough or gag reflex because
it may stimulate vomiting or laryngospasm in responsive or semiresponsive patients. A petroleum-
based lubricant should never be used because it may cause tissue inflammation. A nasopharyngeal
airway (not an OPA) may inadvertently enter the cranial vault if inserted into the nose of a patient
who has sustained a craniofacial injury.
CHAPTER 3  Patient Assessment and Resuscitation Therapies 85

19. A. During adult cardiac arrest, use 360 J for all shocks when using a monophasic defibrillator.
When using a biphasic defibrillator, use the energy level recommended by the manufacturer for
the initial shock. If you do not know what the recommended energy level is, consider defibrillation
at the maximal dose. The second and subsequent energy doses should be equivalent, and higher
doses may be considered. Synchronized cardioversion is used to treat rhythms that have a clearly
identifiable QRS complex and a rapid ventricular rate in a patient who has a pulse.

20. B, C, D. Be sure that high-performance CPR is continued as the defibrillator is readied for use.
While CPR continues, instruct a team member to expose the patient’s chest and to remove any
transdermal medication patches or ointment from the patient’s chest, if present. All team mem-
bers, except for the chest compressor, should clear the patient as the machine charges. When the
defibrillator is charged, the chest compressor should immediately clear the patient. Call “Clear!”
Look around you (360 degrees) to be sure that everyone—including you—is clear of the patient,
the bed, and any equipment that is connected to the patient. Be sure oxygen is not flowing over
the patient’s chest. Press the “Shock” control to defibrillate the patient. Release the “Shock” control
after the energy dose has been delivered. Instruct the team to resume CPR.

21. A, B. Frontal plane leads view the heart from the front of the body as if it were flat. Directions in
the frontal plane are superior, inferior, right, and left. Six leads view the heart in the frontal plane.
Leads I, II, and III are called standard limb leads. Leads aVR, aVL, and aVF are called augmented
limb leads. Six chest (precordial or “V”) leads view the heart in the horizontal plane. The chest
leads are identified as V1, V2, V3, V4, V5, and V6.

22. D. When performing a BLS assessment, ensure that the scene is safe, assess the victim for respon-
siveness and activate the emergency response system when warranted, simultaneously assess for
breathing and a pulse, and then provide indicated care.

REFERENCES
Amsterdam, E. A., Wenger, N. K., Brindis, R. G., et al. (2014). 2014 AHA/ACC guideline for the management
of patients with non-ST-elevation acute coronary syndromes. J Am Coll Cardiol, 64(24), e139–e228.
Artime, C. A., & Hagberg, C. A. (2020). Airway management in the adult. In M. A. Gropper, R. D.
Miller, N. H. Cohen, L. I. Eriksson, L. A. Fleisher, K. Leslie, & J. P. Wiener-Kronish (Eds.), Miller’s anesthesia
(9th ed., pp. 1373–1412). Philadelphia, PA: Elsevier.
Aufderheide, T. P., Sigurdsson, G., Pirrallo, R. G., et al. (2004). Hyperventilation-induced hypotension during
cardiopulmonary resuscitation. Circulation, 109(16), 1960–1965.
Barnes, T. A. (2017). Emergency cardiovascular life support. In R. M. Kacmarek, J. K. Stoller, & A. J. Heuer (Eds.),
Egan’s fundamentals of respiratory care (11th ed., pp. 790–819). St. Louis, MO: Elsevier.
Bessman, E. S. (2019). Emergency cardiac pacing. In J. R. Roberts, C. B. Custalow, & T. W. Thomsen (Eds.),
Roberts and Hedges’ clinical procedures in emergency medicine and acute care (7th ed., pp. 288–308). Philadelphia,
PA: Elsevier.
Boehm, J. (2007, Jul). Tried and true: noninvasive transthoracic pacing. Retrieved from Zoll Code Communications:
http://www.zoll.com/CodeCommunicationsNewsletter/CCNLPacing/CCNLPacing.htm
Cantineau, J. P., Merckx, P., Lambert, Y., Sorkine, M., Bertrand, C., & Duvaldestin, P. (1994). Effect of epineph-
rine on end-tidal carbon dioxide pressure during prehospital cardiopulmonary resuscitation. Am J Emerg Med,
12(3), 267–270.
de Caen, A. R., Berg, M. D., Chameides, L., et al. (2015). American Heart Association Guidelines for CPR & ECC.
Retrieved from American Heart Association. In Web-based integrated guidelines for cardiopulmonary resus-
citation and emergency cardiovascular care – Part 12: Pediatric advanced life support: Eccguidelines.heart.org
Deitch, K. (2019). Intraosseous infusion. In J. R. Roberts, C. B. Custalow, & T. W. Thomsen (Eds.), Roberts and
Hedges’ clinical procedures in emergency medicine and acute care (7th ed., pp. 461–475). Philadelphia, PA: Elsevier.
Del Monte, L. (2006). Noninvasive pacing: what you should know. Redmond, WA: Medtronic Emergency Response
Systems.
Driver, B. E., & Reardon, R. F. (2019a). Basic airway management and decision making. In J. R. Roberts, C. B.
Custalow, & T. W. Thomsen (Eds.), Roberts and Hedges’ clinical procedures in emergency medicine and acute care
(7th ed., pp. 39–61). Philadelphia, PA: Elsevier.
86 CHAPTER 3  Patient Assessment and Resuscitation Therapies

Driver, B. E., & Reardon, R. F. (2019b). Tracheal intubation. In J. R. Roberts, C. B. Custalow, & T. W. Thomsen
(Eds.), Roberts and Hedges’ clinical procedures in emergency medicine and acute care (11th ed., pp. 62–110). Philadelphia,
PA: Elsevier.
Edelson, D. P., Sasson, C., Chan, P. S., et al. (2020). Interim guidance for basic and advanced life support in adults,
children, and neonates with suspected or confirmed COVID-19. Circulation, e20201405 [online ahead of
print].
Gerstein, N. S., Carey, M. C., Braude, D. A., et al. (2013). Efficacy of facemask ventilation techniques in novice
providers. J Clin Anesth, 25(3), 193–197.
Granfeldt, A., Avis, S. R., Lind, P. C., et al. (2020). Intravenous vs. intraosseous administration of drugs during cardiac
arrest: A systematic review. Retrieved from Consensus on Science with Treatment Recommendations [Internet].
Brussels, Belgium: International Liaison Committee on Resuscitation (ILCOR) Advanced Life Support Task
Force: https://costr.ilcor.org/document/iv-vs-io-administration-of-drugs-during-cardiac-arrest-systematic-review.
Heuer, A. J. (2017). Oxygen therapy. In R. M. Kacmarek, J. K. Stoller, & A. J. Heuer (Eds.), Egan’s fundamentals of
respiratory care (11th ed., pp. 905–926). St. Louis, MO: Elsevier.
Institute of Medicine, Graham, R., McCoy, M. A., & Schultz, A. M. (Eds.). (2015). Strategies to improve cardiac
arrest survival: A time to act. Washington, DC: National Academies Press.
Kusumoto, F. M., Schoenfeld, M. H., Barrett, C., et al. (2018). 2018 ACC/AHA/HRS guideline on the evaluation
and management of patients with bradycardia and cardiac conduction delay. Circulation, 140(8), e382–e482.
Link, M. S., Berkow, L. C., Kudenchuk, P. J., et al. (2015). American Heart Association Guidelines for CPR & ECC.
Retrieved from American Heart Association. In Web-based integrated guidelines for cardiopulmonary resus-
citation and emergency cardiovascular care – Part 7: Adult advanced cardiovascular life support: Eccguidelines.
heart.org
Morrison, L. J., Neumar, R. W., Zimmerman, J. L., et al. (2013). Strategies for improving survival after in-hospital
cardiac arrest in the United States: 2013 consensus recommendations: A consensus statement from the
American Heart Associations. Circulation, 127(14), 1538–1563.
O’Gara, P. T., Kushner, F. G., Ascheim, D. D., et al. (2013). 2013 ACCF/AHA guideline for the management of
ST-elevation myocardial infarction. J Am Coll Cardiol, 61(4), e78–e140.
Panchal, A. R., Bartos, J. A., Cabañas, J. G., et al. (2020). Part 3: Adult basic and advanced life support: 2020
American Heart Association guidelines for cardiopulmonary resuscitation and emergency cardiovascular care.
Circulation, 142(16 suppl 2), S366–S468.
Panchal, A. R., Berg, K. M., Kudenchuk, P. J., et al. (2018). 2018 American Heart Association focused update on
advanced cardiovascular life support use of antiarrhythmic drugs during and immediately after cardiac arrest.
Circulation, 138(23), e740–e749.
Peck, J., Viswanath, O., & Rosen, G. (2018). Mask ventilation grip: A life-saving innovation. Ochsner J, 18(2),
112–113.
Ristagno, G., Mancini, M. B., Avis, S., et al. (2019). Pad size, orientation and placement for cardiac arrest. Retrieved
from Consensus on Science with Treatment Recommendations [Internet]. Brussels, Belgium: International
Liaison Committee on Resuscitation (ILCOR) Advanced Life Support Task Force: https://costr.ilcor.org/
document/pad-size-orientation-and-placement-scoping-review.
Rouse, M., Frakes, M. (2010). Airway management. In R. S. Holleran (Ed.), ASTNA patient transport: Principles
and practice (4th ed., pp. 181–233). St. Louis, MO: Mosby.
Slutsky, A. S., & Brochard, L. (2020). Mechanical ventilation. In L. Goldman, & A. I. Schafer (Eds.), Goldman-
Cecil medicine (26th ed., pp. 635–641). Philadelphia, PA: Elsevier.
Spotts, V. (2017). Temporary transcutaneous (external) pacing. In D. L. Wiegand (Ed.), AACN procedure manual
for high acuity, progressive, and critical care (7th ed., pp. 399–406). St. Louis, MO: Elsevier.
Tiffin, N. H., Keim, M. R., & Frewen, T. C. (1990). The effects of variations in flow through an insufflating cath-
eter and endotracheal tube and suction catheter size on test lung pressures. Respir Care, 35(9), 889–897.
CHAPTER 4
The Patient With Respiratory
Compromise

INTRODUCTION
Respiratory complaints are common in patients of all ages. This chapter reviews respiratory distress,
respiratory failure, and respiratory arrest; implementing a treatment plan based on the severity of a
patient’s respiratory compromise; and the initial management of a patient who experiences a respiratory
arrest.

L E A R N I N G OBJECTIVES
After completing this chapter, you should be able to:
1. Describe the typical presentation of a patient with difficulty breathing.
2. Identify the signs of increased work of breathing.
3. Summarize the initial emergency care for a patient who has experienced a respiratory
arrest.

L E A R N I N G PLAN
• Read this chapter before class. Take the time to highlight important concepts as you
read.
• Master the following skills:
• Ensure scene safety and the use of personal protective equipment.
• Assign team member roles or perform as a team member in a simulated patient
situation.
• Direct or perform a basic life support (BLS) assessment, primary assessment, and
secondary assessment.
• Obtain vital signs; establish vascular access; attach a pulse oximeter, blood
pressure (BP) and cardiac monitor; give supplemental oxygen (O2) if indicated, and
order a 12-lead electrocardiogram (ECG).
• Demonstrate manual methods for opening the airway.
• Demonstrate the procedure for suctioning the upper airway.
• Demonstrate how to size and insert an oropharyngeal airway (OPA) and a
nasopharyngeal airway (NPA).
• Perform two-rescuer bag-mask ventilation (BMV) when indicated.
• Demonstrate how to troubleshoot inadequate BMV.
87
88 CHAPTER 4  The Patient With Respiratory Compromise

• Review your performance as a team leader or team member during a postevent


debriefing.
• Develop and use flashcards, flowcharts, and mnemonics to help enhance your
retention of the information presented.
• Complete the quiz at the end of this chapter and review the quiz answers provided.
• Read the case study at the end of this chapter and answer each question that follows
it. Compare your answers with the answers provided at the end of the case study.

RESPIRATORY COMPROMISE
Respiratory distress, respiratory failure, and respiratory arrest reflect increasing levels of severity of
respiratory compromise (Fig. 4.1).

Respiratory Distress
Respiratory distress is characterized by the presence of an increased ventilatory rate and work of
breathing. Signs of respiratory distress reflect an attempt to compensate for hypoxia. An increase in
the ventilatory rate is an early sign of respiratory distress. Initially, the patient can speak in sentences
before pausing for a breath, but as their distress worsens, they are only able to speak using phrases.
Additional signs are shown in Box 4.1. Uncorrected respiratory distress may lead to respiratory
failure.

Fig. 4.1  ​Respiratory complaints are common in patients of all ages. (From iStock.)

BOX 4.1 Signs of Respiratory Distress


• Mental status changes (e.g., anxiety, • Retractions
restlessness, irritability, decreased • Stridor
ability to concentrate) • Tachycardia or bradycardia
• Nasal flaring • Tachypnea
• Pallor, mottling • Wheezing
• Pursed-lip breathing • Use of accessory muscles of breathing
CHAPTER 4  The Patient With Respiratory Compromise 89

BOX 4.2 Signs of Respiratory Failure


• Accessory muscle use
• Changes in mental status (e.g., agitation, irritability, confusion, lethargy)
• End-tidal carbon dioxide (EtCO2) level greater than 50 mm Hg
• Inadequate ventilatory effort
• Nasal flaring
• Oxygen saturation less than 90%
• Pallor, mottling, or cyanosis despite oxygen therapy
• Retractions
• Stridor
• Tachypnea with periods of bradypnea; slowing to bradypnea/agonal breathing

Respiratory Failure
Acute respiratory failure develops when the respiratory system is unable to maintain adequate oxygen-
ation, adequate carbon dioxide elimination, or both. Hypoxemic respiratory failure refers to respiratory
failure associated with failure to oxygenate. It is characterized by a partial pressure of oxygen (PaO2) of
less than 60 mm Hg while breathing room air. Examples of conditions that may result in hypoxemic
respiratory failure include chronic obstructive pulmonary disease (COPD), asthma, pulmonary emboli,
pulmonary edema, inhalation of toxic gases, and viral, bacterial, or fungal pulmonary infections.
Hypercarbic respiratory failure, also known as ventilatory failure, is characterized by a partial pressure
of carbon dioxide (PaCO2) of more than 50 mm Hg, usually with a pH of less than 7.35. Examples of
possible causes of hypercarbic respiratory failure include the following:
• Impaired respiratory control resulting from drug overdoses, brainstem lesions, stroke, or multiple
sclerosis
• Neurologic disease resulting from spinal cord trauma, Guillain-Barré syndrome, or myasthenia
gravis
• Increased work of breathing resulting from COPD, asthma, morbid obesity, pneumothorax, or upper
airway obstruction
• Increased carbon dioxide production caused by agitation, fever, shivering, or occupational
exposure
A patient may have both forms of respiratory failure, which is a combined failure of oxygenation
and ventilation.
Box 4.2 lists possible signs of respiratory failure. The patient with respiratory failure typically talks
in words; they are unable to speak using sentences or phrases before needing to pause and take a breath.
Although tachycardia is often seen with early respiratory failure, the patient may become bradycardic
with impending respiratory arrest.

Respiratory Arrest
With respiratory arrest, the patient is unresponsive and is either not breathing or is not breathing
effectively (i.e., only gasping, agonal breathing), but a pulse is present. An untreated respiratory arrest
will result in a cardiac arrest. Possible causes of respiratory arrest include the following:
• Upper or lower airway obstruction
• Decreased ventilatory effort resulting from central nervous system disorders, drug overdoses, or
metabolic disorders
• Respiratory muscle weakness caused by neuromuscular disorders (e.g., myasthenia gravis, Guillain-
Barré syndrome)

PATIENT ASSESSMENT
As you approach the patient with a respiratory complaint, be sure to use appropriate personal protective
equipment (PPE). Remember that the possibility of airborne pathogen transmission related to droplet
and aerosol formation must be considered when caring for these patients. In addition to wearing
90 CHAPTER 4  The Patient With Respiratory Compromise

protective eye equipment, a mask and face shield, and a gown, providers at some institutions caring for
known or suspected coronavirus-19 (COVID-19) patients routinely double glove and replace the outer
glove before coming in contact with a different patient. Hand sanitizer is applied to the inner glove
before applying a new second glove.
Form a general impression to determine if the patient is sick (i.e., unstable) or not sick (i.e., stable)
and to determine the urgency of further assessment and care. Red flags that suggest impending dete-
rioration include noisy breathing, breathing faster or more slowly than normal, flaring nostrils, the use
of accessory muscles, or the presence of pallor, mottling, or cyanosis. Noting the patient’s position can
help determine the severity of the patient’s respiratory problem. For example, a patient who is sitting
upright with their elbows braced on a table or with their hands on their knees and elbows out while
leaning forward is said to be tripoding or assuming a tripod position. If abnormal findings are present,
activate the rapid response team (if applicable), proceed immediately to the primary assessment, and
begin emergency care. If the patient’s condition does not appear to be urgent, work at a reasonable pace,
and proceed systematically with your assessment.
As you begin your assessment, ask a team member to obtain the patient’s vital signs and attach a
pulse oximeter and cardiac monitor. If the patient is responsive, ask questions to determine level of
responsiveness and the adequacy of the airway and breathing. Observe for agitation, confusion, rest-
lessness, or combativeness, which may be the result of hypoxia. Also observe if the patient can speak
in sentences before requiring a breath or if they experience shortness of breath after speaking only a
few words. Depending on the severity of the patient’s condition, it may be necessary to ask questions
of a family member. Attempt to determine when the patient’s symptoms began, how long they have
been present, precipitating factors, and relief measures, such as position and use of over-the-counter
or prescribed medications. Ask if the patient has a cough, hemoptysis, hoarseness or changes in voice,
fever, or chills, and if they have traveled recently. If the patient has a cough, ask if it is productive or
nonproductive; if it is harsh, dry, or hacking; and about the color, amount, and consistency of any
mucus expectorated.
During your assessment, make a note of the patient’s physical characteristics. For example, a barrel
chest may be present in the patient who has COPD. Assess the patient’s rate, rhythm, and depth of
breathing, and symmetry of movement with each breath for 1 full minute. Listen for stridor, audible
wheezes, and voice changes. Look for signs of increased work of breathing and auscultate breath and
heart sounds. Assess the patient’s heart rate, pulse quality, and skin temperature, color, and moisture.
Expose the patient for further examination as necessary. If opioid overdose is suspected, be alert for
clues, such as track marks along the length of veins suggesting IV opioid injection or marks or scars
from “skin popping” that may represent sites of subcutaneous opioid injection. Look for and remove
medication patches (e.g., fentanyl) if present.
If oxygenation is inadequate, provide supplemental oxygen. Give 100% oxygen during respiratory
and cardiac arrest. For most patients who are not in respiratory or cardiac arrest, target an oxygen
saturation of 95% to 98%. Target an oxygen saturation of 90% for acute coronary syndromes, and tar-
get an oxygen saturation of 92% to 98% during postcardiac arrest care.
If ventilatory efforts are inadequate and depending on the situation, assist the patient’s breathing
using noninvasive positive pressure ventilation (NPPV), mouth-to-mask ventilation, or BMV.

THERAPEUTIC INTERVENTIONS
The goal of management for the patient with respiratory compromise is to ensure an open airway and
provide the necessary support to ensure adequate oxygenation and ventilation. Identification of the
underlying cause of the patient’s respiratory compromise is essential. Initial diagnostic evaluations often
include a 12-lead ECG, chest radiograph, and laboratory studies (e.g., arterial or venous blood gases,
complete blood count with differential, metabolic panel). Additional studies that may be ordered include
spirometry, peak flow rate, sputum culture, chest computed tomography, bedside echocardiography,
D-dimer assay, toxicology screen, and blood alcohol level.
Because the causes of respiratory distress are many, possible therapeutic interventions include allowing
the patient to assume a position of comfort, the administration of supplemental oxygen if indicated, es-
tablishing vascular access, and pharmacologic therapy (e.g., bronchodilators, steroids, diuretics) (Fig. 4.2).
Reassess the patient often and be prepared to assist ventilations.
CHAPTER 4  The Patient With Respiratory Compromise 91

Fig. 4.2  ​Depending on its cause, therapeutic interventions for respiratory distress may include the administration of
nebulized medications. (From iStock.)

Depending on its cause and severity, possible therapeutic interventions for respiratory failure
include suctioning, administration of supplemental oxygen, establishing vascular access, NPPV, BMV,
mechanical ventilation, and treatment of specific contributing or causative factors.
Therapeutic interventions for respiratory arrest (discussed in Chapter 3) include the following:
• Manual maneuvers to open the airway
• Removal of a foreign body if present
• Insertion of an OPA or NPA
• Suctioning
• BMV with supplemental oxygen
• Possible insertion of an advanced airway by an appropriately trained clinician
• Establishing vascular access
• Treatment of specific contributing or causative factors
When ventilating with a bag-mask device (BMD), be sure to observe the rise and fall of the patient’s
chest with each ventilation. Ventilate the adult patient at a rate of 1 breath every 6 seconds, or about 10
breaths per minute (Panchal et al., 2020). Establish capnography to assess the adequacy of ventilation.
Closely monitor the patient’s oxygen saturation, capnography waveform, and heart rate, and periodically
listen to breath sounds to ensure adequate ventilation. If an opioid overdose is suspected, give naloxone
as soon as it is available. Reassess the patient’s pulse about every 2 minutes. If your assessment reveals
the patient no longer has a pulse, begin cardiopulmonary resuscitation immediately.

ACLS Pearl
The minimum patient observation time after naloxone administration is 1 to 2 hours; however,
overdoses of long-acting opioids or sustained-release opioid preparations require more extended
observation periods, generally at least 4 to 6 hours. Monitor for opioid withdrawal in dependent
patients.

The prone position is sometimes used in the management of patients with adult respiratory distress
syndrome, including COVID-19, to improve ventilation (Ghelichkhani & Esmaeili, 2020). If a patient
with known or suspected COVID-19 is in a prone position, does not have an advanced airway in place,
and experiences a cardiopulmonary arrest, experts recommend attempting to position the patient
supine for continued resuscitation (Edelson et al., 2020).
If a patient with known or suspected COVID-19 does have an advanced airway in place and is in a
prone position at the time of a cardiopulmonary arrest, the patient should be left in a prone position
unless rescuers can position the patient supine without risking aerosolization and equipment discon-
nections (Edelson et al., 2020). If the patient remains in a prone position during a cardiopulmonary
arrest, consider placing defibrillator pads in the anterior-posterior position and provide chest compres-
sions with the rescuer’s hands positioned over the T7/T10 vertebral bodies (Edelson et al., 2020).
92 CHAPTER 4  The Patient With Respiratory Compromise

STOP AND REVIEW


Identify one or more choices that best complete the statement or answer the question.

____ 1. You and a coworker arrive to find a 78-year-old woman unresponsive in bed. She is
not breathing but does have a pulse. You have a pocket face mask on hand that is
equipped with an oxygen inlet. After quickly connecting oxygen tubing to the inlet
on the mask, you should set the oxygen flow rate at:
a. 1 to 2 L/min.
b. 4 to 6 L/min.
c. 8 to 10 L/min.
d. 10 to 12 L/min.

_ ___ 2. Which of the following statements is true about BMV?


a. A BMD is only used with nonbreathing patients.
b. A mask of correct size should extend just beyond the chin.
c. Lift the patient’s mandible to the mask when applying the mask to the face.
d. A BMD that is used without supplemental oxygen will deliver about 40% to
60% oxygen to the patient.

____ 3. Which of the following are signs of adequate ventilation when delivering ventilations
with a BMD?
a. The presence of gurgling sounds during ventilation
b. Improvements in color, responsiveness, and heart rate
c. The rise and fall of the patient’s chest wall with each ventilation
d. The collapse of the oxygen reservoir on the BMD with each ventilation
e. The BMD becomes progressively more difficult to compress with each ventilation

_ ___ 4. Which of the following devices may be used to deliver positive pressure ventilation?
a. Nasal cannula
b. Pocket face mask
c. Simple face mask
d. Partial rebreather mask

_ ___ 5. Which of the following are common problems associated with BMV?
a. Ventilating at too fast a rate
b. Incomplete bag compression
c. Inability to achieve an effective mask seal
d. Using excessive force during bag compression

_ ___ 6. Which of the following are contraindications for NPPV?


a. Nausea
b. Respiratory arrest
c. Shortness of breath
d. Uncooperative patient
CHAPTER 4  The Patient With Respiratory Compromise 93

CASE STUDY 4.1


Identify one or more choices that best complete the statement or answer the question.

You are a member of your facility’s Rapid Response System (RRS). Your team has been summoned
to a medical floor where the bedside nurse states that the patient, a 65-year-old woman who has pneu-
monia, has been experiencing increasing shortness of breath since yesterday. The nurse activated the
RRS because the patient has become increasingly lethargic, and her oxygen saturation is now 88%
despite the administration of supplemental oxygen at 2 L/min by nasal cannula. An IV is in place.
Emergency equipment, including a biphasic manual defibrillator with transcutaneous pacing (TCP)
capability, is available to you. As you work through this case study, remember that there may be alterna-
tive actions that are perfectly acceptable, yet not discussed here.

____ 1. As you approach the patient, you observe that she is supine in her hospital bed.
Her eyes are closed, there is no visible chest rise, her lips are blue, and her skin is
pale. Select the correct statements about this patient’s general impression findings.
a. Appearance is normal.
b. Appearance is abnormal.
c. Breathing is normal.
d. Breathing is abnormal.
e. Circulation is normal.
f. Circulation is abnormal.

_ ___ 2. You quickly determine that the patient is unresponsive. Your next action should be to:
a. Insert an OPA.
b. Begin chest compressions.
c. Begin BMV.
d. Simultaneously assess for breathing and a pulse.

____ 3. You find that the patient is not breathing, but a slow, weak carotid pulse is present.
You should now:
a. Attach a cardiac monitor.
b. Insert an OPA and begin chest compressions.
c. Attach an automated external defibrillator (AED) to the patient and follow the
machine’s prompts.
d. Open the patient’s airway and assess for sounds of airway compromise.

____ 4. The patient’s airway is clear. You have asked the airway team member to insert an
OPA. The correct size OPA should extend from:
a. The tip of the nose to the earlobe.
b. The tip of the nose to the angle of the jaw.
c. The corner of the mouth to the tip of the nose.
d. The corner of the mouth to the angle of the jaw.

_ ___ 5. An OPA:
a. Protects the lower airway from aspiration.
b. Eliminates the need for manual head positioning to maintain an open airway.
c. May be used in responsive or unresponsive patients with an intact gag reflex.
d. Should only be used in unresponsive patients who do not have an intact gag reflex.

____ 6. An OPA has been inserted. The patient is still not breathing. You ask two team
members to begin positive pressure ventilation with a BMD. Select the correct
statements about ventilating with a BMD.
a. Deliver each breath over 2 seconds.
b. Stop ventilation when gentle chest rise is observed.
c. Ventilate the patient at a rate of 10 breaths per minute.
d. Ensure there is a tight seal between the patient’s face and the mask.
94 CHAPTER 4  The Patient With Respiratory Compromise

____ 7. The patient’s chest does not rise despite attempts to ventilate the patient with a
BMD. What is the first thing you should do to remedy this problem?
a. Select a smaller mask.
b. Reassess the patient’s head position.
c. Ask an experienced team member to intubate the patient.
d. Ask another team member to assume responsibility for ventilating the patient.

____ 8. Equal chest rise is now present with BMV. Breath sounds reveal clear upper
lobes and diminished sounds in the lower lobes bilaterally. The patient’s BP is
108/74 millimeters of mercury (mm Hg). She has been placed on the cardiac
monitor, which reveals a sinus bradycardia at 50 beats per minute. What should
be done now?
a. Ask a team member to make preparations for TCP.
b. Ask a qualified team member to intubate the patient.
c. Remove the nasal cannula and apply a simple face mask.
d. Instruct a team member to administer atropine 0.5 mg IV.

____ 9. The patient has been intubated. The airway team member states that she visualized
passing the endotracheal tube (ETT) between the vocal cords and has inflated the
tube cuff. While manually holding the ETT in place and before securing the tube,
methods to confirm proper positioning of the tube include:
a. Using an esophageal detector device (EDD)
b. Observing condensation in the ETT
c. Auscultating the presence of bilateral breath sounds
d. Confirming the presence of gurgling sounds during auscultation of the
epigastrium

____ 10. After confirming correct positioning, the ETT is secured with an ETT holder.
Although the patient’s color is improving, she remains unresponsive, and spontane-
ous breathing is absent. The cardiac monitor reveals a sinus rhythm at 86 beats
per minute. Her BP is 118/80 mm Hg. Ventilations are being monitored with
capnography. Which of the following interventions are appropriate at this time?
a. Apply a pulse oximeter.
b. Repeat the primary assessment.
c. Order a 12-lead ECG, chest radiograph, and laboratory studies.
d. Reassess breathing and the presence of a pulse every 15 minutes.

STOP AND REVIEW  ANSWERS


1. D. If not already attached, connect a one-way valve to the ventilation port on the pocket face
mask and connect oxygen tubing to the oxygen inlet on the mask. Set the oxygen flow rate at 10 to
12 L/min.

2. C. A BMD can be used to assist ventilation in a spontaneously breathing patient, as well as with
the nonbreathing patient. A mask of correct size should extend from the bridge of the nose to the
groove between the lower lip and chin; it should not extend beyond the chin. Lift the patient’s
mandible to the mask when applying the mask to the face rather than pushing the mask down on
the face. A BMD that is used without supplemental oxygen will deliver 21% oxygen (i.e., room air)
to the patient. A BMD that is used with supplemental oxygen set at a flow rate of 10 to 15 L/min
delivers about 40% to 60% oxygen to the patient when a reservoir is not used. A BMD that is used
with supplemental oxygen set at a flow rate of 10 to 15 L/min and with an attached reservoir deliv-
ers approximately 90% to 100% oxygen to the patient.
CHAPTER 4  The Patient With Respiratory Compromise 95

3. B, C. A reliable indicator of ventilation adequacy is the rise and fall of the patient’s chest wall with
each ventilation. Other indicators that the patient is well ventilated is an improvement of the pa-
tient’s condition, as evidenced by improvements in color, pulse oximeter and capnography readings,
heart rate, and responsiveness. Gurgling sounds are abnormal and indicate the need for suctioning.
If the oxygen reservoir on the bag-mask device collapses with each ventilation, it may indicate that
the oxygen flow is too low, or the ventilation rate is too rapid. If the bag-mask device becomes
progressively more difficult to squeeze when ventilating a patient, assess the need to suction, ensure
that the airway is open, suspect that there may be excessive air in the stomach (anticipate vomiting),
and suspect a possible pneumothorax.

4. B. NPPV, mouth-to-mask ventilation, and BMV are examples of methods that may be used to
deliver positive pressure ventilation. The remaining devices listed (nasal cannula, simple face mask,
and partial rebreather mask) do not deliver a tidal volume; they are oxygen delivery devices and
require a spontaneously breathing patient.

5. A, B, C, D. Common problems associated with BMV include inadequate ventilation, delivering


ventilations at an improper rate, and gastric inflation. Inadequate ventilation may be the result of
failing to maintain an open airway with proper positioning, failing to achieve an effective mask seal
while simultaneously maintaining an open airway, incomplete bag compression, or a combination of
these factors. During a cardiac arrest, providing BMV at too fast a rate or with too much volume
increases intrathoracic pressure, impedes venous return, decreases cardiac output, and decreases
coronary and cerebral artery perfusion pressures. Squeezing the bag too fast or too forcefully can
result in gastric inflation, increasing the risk for aspiration.

6. B, D. Contraindications for NPPV include the following:


Altered mental status Inability to fit mask
Cardiac arrest Inability to protect airway
Complete upper airway obstruction Recent facial, esophageal, or gastric surgery
Excessive secretions Respiratory arrest
Facial trauma or deformity Uncontrolled vomiting
Hemodynamic instability Uncooperative patient
High risk for aspiration

C A S E S T U D Y 4 . 1 A NSWERS
1. B, D, F. The general impression findings are abnormal (Appearance: abnormal; Breathing: abnormal;
Circulation: abnormal skin color).

2. D. After determining that the patient is unresponsive, your next action should be to simultaneously
assess for breathing and a pulse for 5 to 10 seconds.

3. D. If the patient had no pulse, you would direct your team to start chest compressions and attach
an AED to the patient. In this situation, chest compressions are not indicated because a pulse is
present. Your next step should be to open the patient’s airway. Because there is no evidence of
trauma, open the patient’s airway using a head tilt–chin lift. If there were factors that suggested
trauma in this situation, you would open the airway with a jaw thrust maneuver. Assess for sounds
of airway compromise, such as snoring, gurgling, or stridor. If the airway is not clear, clear it with
suctioning, patient positioning, or both, as indicated. Look in the mouth for blood, broken teeth or
loose dentures, gastric contents, and foreign objects.

4. D. Proper OPA size is determined by holding the device against the side of the patient’s face and
selecting an airway that extends from the corner of the mouth to the tip of the earlobe or to the
angle of the jaw. To prevent inaccurate measurements for patients who experience facial drooping
after a stroke, some experts recommend measuring from the first incisor or from the center of the
lips to the tip of the earlobe or to the angle of the jaw.
96 CHAPTER 4  The Patient With Respiratory Compromise

5. D. An OPA should only be used in unresponsive patients who do not have an intact gag reflex.
Use in a responsive or semiresponsive patient may stimulate vomiting or laryngospasm. An OPA
does not protect the lower airway from aspiration. Insertion of an OPA does not eliminate the
need for using the head tilt–chin lift or jaw thrust maneuver to keep the airway open after the
OPA is in place.

6. B, C, D. Begin positive pressure ventilation with a BMD connected to 100% oxygen. Ideally, two
team members should be assigned this task. Ask one team member to open and maintain the
airway while creating a good seal with the mask. Ask the other team member to squeeze the bag.
Observe the rise and fall of the patient’s chest with each ventilation. Deliver each breath over
1 second and stop ventilation when gentle chest rise is observed. Ventilate the adult patient at a rate
of 1 breath every 6 seconds, or about 10 breaths per minute. Ask a team member to assess baseline
breath sounds while the patient is being ventilated. Look for improvement of the patient’s condition
as evidenced by improvements in color, pulse oximeter readings, heart rate, and responsiveness.

7. B. If the chest does not rise and fall with BMV, your first action should be to reposition the
patient’s head and try to ventilate again.

8. B. Ask a qualified team member to intubate the patient while you perform a focused physical ex-
amination. Resist the temptation to treat the patient’s bradycardia with atropine. The most likely
cause of the patient’s bradycardia is hypoxia. Make sure the patient is adequately oxygenated and
ventilated before administering atropine or performing TCP. Although removing the nasal can-
nula is appropriate, replacing it with a simple face mask is not because the use of a simple face mask
requires a spontaneously breathing patient.

9. A, C. While holding the ETT in place, auscultate over the epigastrium (should be silent) and then
in the midaxillary and anterior chest line on the right and left sides of the patient’s chest while
the patient is being ventilated. Observe the patient’s chest for adequate chest rise with ventilation.
In addition to clinical assessments, secondary methods that may be used to verify proper ETT
positioning include the use of continuous waveform capnography (preferred for confirming
and monitoring correct ETT placement in a patient who has adequate tissue perfusion), use of a
nonwaveform CO2 detector, use of an EDD, obtaining a chest radiograph, or use of ultrasound
imaging. Fogging or vapor condensation on the inside of the tracheal tube is not a reliable indica-
tor of proper tube position. After proper positioning is confirmed, secure the tube in place.

10. A, B, C. Repeat the primary assessment and obtain another set of vital signs. Apply a pulse oxim-
eter and continue monitoring the effectiveness of ventilation using capnography. Adjust ventilation
as needed to maintain EtCO2 values between 35 mm Hg and 45 mm Hg. Order laboratory stud-
ies, a 12-lead ECG and chest radiograph, and review the results to attempt to determine possible
causes of the patient’s respiratory arrest. Reassess breathing and the presence of a pulse about every
2 minutes. Transfer the patient for continued monitoring and care. Request a team debriefing after
the patient’s transfer is complete.

REFERENCES
Edelson, D. P., Sasson, C., Chan, P. S., et al. (2020). Interim guidance for basic and advanced life support in adults,
children, and neonates with suspected or confirmed COVID-19. Circulation, e20201405 [online ahead of
print].
Ghelichkhani, P., & Esmaeili, M. (2020). Prone position in management of COVID-19 patients; a commentary.
Arch Acad Emerg Med, 8(1), e48.
Panchal, A. R., Bartos, J. A., Cabañas, J. G., et al. (2020). Part 3: Adult basic and advanced life support: 2020
American Heart Association guidelines for cardiopulmonary resuscitation and emergency cardiovascular care.
Circulation, 142(16 suppl 2), S366–S468.
CHAPTER 5
Bradycardias

INTRODUCTION
Recall from Chapter 3 that the term symptomatic bradycardia is used to describe a patient who
experiences signs and symptoms of hemodynamic compromise related to a slow heart rate. The
bradycardia algorithm is a treatment guideline to use when providing care to patients who are expe-
riencing symptomatic bradycardia. This chapter reviews electrocardiogram (ECG) recognition and
the initial management of symptomatic patients with sinus bradycardia, junctional and ventricular
escape rhythms, first-degree atrioventricular (AV) block, second-degree AV blocks types I and II, and
third-degree AV block.

L E A R N I N G OBJECTIVES
After completing this chapter, you should be able to:
1. Differentiate among the following ECG rhythms: sinus bradycardia, junctional rhythm,
ventricular escape rhythm, and AV blocks: first-degree, second-degree type I,
second-degree type II, 2:1 AV block, and third-degree AV block.
2. Given a patient situation, identify a patient who is experiencing a bradycardia as
asymptomatic, stable but symptomatic, symptomatic and unstable, or pulseless.
3. Given a patient situation, describe the initial emergency care for a patient experiencing
symptomatic bradycardia.

L E A R N I N G PLAN
• Read this chapter before class.
• Master identification of the following rhythms: sinus bradycardia, junctional rhythm,
ventricular escape rhythm, and AV blocks: first-degree, second-degree type I,
second-degree type II, 2:1 AV block, and third-degree AV block.
• Master the following medications: oxygen (O2), atropine, dopamine, and epinephrine.
• Master the following skills: primary and secondary assessment, supplemental O2
delivery devices, attachment and use of ECG monitoring leads, intravenous (IV) access,
IV medication administration, and operation of a transcutaneous pacemaker.
• Master the following skills:
n Ensure scene safety and the use of personal protective equipment.
n Assign team member roles or perform as a team member in a simulated patient
situation.
n Direct or perform a basic life support (BLS) assessment, primary assessment, and
secondary assessment.
97
98 CHAPTER 5  Bradycardias

Obtain vital signs; establish vascular access; attach a pulse oximeter, blood
n

pressure (BP) and cardiac monitor; give supplemental O2 if indicated; and order
a 12-lead ECG.
n Quickly recognize if a patient is asymptomatic, stable but symptomatic,
symptomatic and unstable, or pulseless.
n Demonstrate familiarity with the bradycardia algorithm.
n Demonstrate an understanding of the actions, indications, dosages, adverse
effects, and contraindications for the medications used in the treatment of
symptomatic bradycardia.
n Administer medications and perform transcutaneous pacing (TCP) when indicated.
n Consider reperfusion therapy if the patient’s signs and symptoms are consistent
with an acute coronary syndrome (ACS), and there are no contraindications.
n Consider the possible reversible causes of a cardiac emergency.
n Verbalize when it is best to seek expert consultation.
n Review your performance as a team leader or team member during a postevent
debriefing.
• Complete the chapter quiz and review the quiz answers provided.
• Read the case studies at the end of this chapter and compare your answers with the
answers provided.

KEY TERMS
Absolute bradycardia  A heart rate of less than 60 beats per minute (beats/min).
Relative bradycardia  A term that refers to a situation in which a patient’s heart rate may
be more than 60 beats/min but, physiologically, the patient needs a more rapid rate (as
in hypovolemia) and is unable to increase their heart rate because of sinoatrial (SA)
node disease, beta blockers, or other medications.

OVERVIEW
Cardiac output 5 stroke volume 3 heart rate. Therefore a decrease in either stroke volume or heart
rate may result in a decrease in cardiac output. In adults and adolescents, a bradycardia exists when
the ventricular rate is less than 60 beats/min. Many patients tolerate a heart rate of 50 to 60 beats/
min but become symptomatic when the rate drops below 50 beats/min. An absolute bradycardia is
a heart rate of less than 60 beats/min. When a patient has a relative bradycardia, their heart rate may
be more than 60 beats/min but, physiologically, the patient needs a more rapid rate (as in hypovole-
mia) and is unable to increase their heart rate because of SA node disease, beta blockers, or other
medications.

RHYTHM REVIEW
Sinus Bradycardia
Sinus bradycardia occurs when the SA node fires at a rate that is slower than 60 beats/min. When seen
in heart transplant recipients, a bradycardia is sometimes defined as a heart rate that is persistently
slower than 70 or 80 beats/min (Kusumoto et al., 2018). The term severe sinus bradycardia is some-
times used to describe sinus bradycardia with a rate of less than 40 beats/min. ECG characteristics of
sinus bradycardia include the following (Fig. 5.1):
Rhythm: R to R and P to P intervals are regular
Rate: Less than 60 beats/min
P waves: Positive (i.e., upright) in lead II; one precedes each QRS complex; P waves look alike
PR interval: 0.12 to 0.20 second and constant from beat to beat
QRS duration: 0.11 second or less unless abnormally conducted
CHAPTER 5  Bradycardias 99

Fig. 5.1  ​Sinus bradycardia with ST segment depression.

Sinus bradycardia is often seen in well-trained athletes at rest or other individuals during sleep.
Other possible causes of sinus bradycardia include the following:
• Advanced age • Medications (e.g., amiodarone, beta
• Carotid sinus stimulation blockers, calcium blockers, digoxin)
• Eye surgery • Myocardial infarction (MI)
• Hypothermia • Obstructive sleep apnea
• Hypothyroidism • Post heart transplant
• Increased intracranial pressure • SA node disease
• Inflammatory conditions (e.g., pericarditis, • Vagal stimulation
myocarditis) • Vomiting
• Intracranial tumors

Junctional Escape Rhythm


When the AV junction assumes responsibility for pacing the heart, it typically does so at a rate of
40 to 60 beats/min. Because a junctional rhythm starts from above the ventricles and follows the heart’s
normal conduction pathway, the QRS complex is narrow. Characteristics of a junctional escape rhythm
include the following (Fig. 5.2):
Rhythm: Very regular
Rate: 40 to 60 beats/min
P waves: May occur before, during, or after the QRS; if visible, the P wave is inverted in leads II, III,
and aVF
PR interval: If a P wave occurs before the QRS, the PR interval is usually 0.12 second or less; if no
P wave occurs before the QRS, there is no PR interval
QRS duration: 0.11 second or less unless abnormally conducted
Possible causes of junctional rhythm include the following:
• Disease of the SA node
• Cardiac surgery immediately preceding the junctional rhythm
• Inflammatory conditions (e.g., Lyme disease, rheumatic heart disease)
• Medications (e.g., beta blockers, calcium blockers, digoxin toxicity)
• MI
The patient may be asymptomatic with a junctional escape rhythm, or they may experience
signs and symptoms that may be associated with the slow heart rate and decreased cardiac
output.

Fig. 5.2  ​Junctional escape rhythm with ST segment elevation. (From Aehlert, B. [2004]. ECG study cards, St. Louis, Mosby.)
100 CHAPTER 5  Bradycardias

Fig. 5.3  ​Ventricular escape rhythm. (From Aehlert, B. [2006]. ECGs made easy, ed 3, St. Louis, Mosby.)

Ventricular Escape Rhythm


A ventricular escape rhythm, which is also called an idioventricular rhythm, occurs at a rate of 20 to
40 beats/min. When the ventricular rate slows to a rate of less than 20 beats/min, some clinicians
refer to the rhythm as an agonal rhythm or dying heart. The QRS complexes are wide because
the impulses begin in the ventricles, bypassing the normal conduction pathway. The patient with a
ventricular escape rhythm may experience serious signs and symptoms because of the slow ventricular
rate. Possible causes of this dysrhythmia include digitalis toxicity, metabolic imbalances, MI, and
SA or AV node disease. ECG characteristics of a ventricular escape rhythm include the following
(Fig. 5.3):
Rhythm: Ventricular rhythm is essentially regular
Rate: Ventricular rate is 20 to 40 beats/min
P waves: Usually absent or, with retrograde conduction to the atria; may appear after the QRS (usually
upright in the ST segment or T wave)
PR interval: None
QRS duration: 0.12 second or greater; the T wave is frequently in the opposite direction of the QRS
complex

Atrioventricular Blocks
An AV block is a delay or block in the transmission of an impulse from the atria to the ventricles that
is caused by an anatomic or functional impairment in the conduction system (Issa et al., 2019). An AV
block can be transient or permanent.
AV blocks are classified into: (1) first-degree AV block, (2) second-degree AV block, and (3) third-
degree AV block. With first-degree AV block, impulses from the SA node to the ventricles are delayed;
they are not blocked. With second-degree AV blocks, there is an intermittent disturbance in the con-
duction of impulses between the atria and the ventricles. With third-degree AV block, there is a com-
plete block in the conduction of impulses between the atria and the ventricles.

First-Degree Atrioventricular Block


Despite its name, a first-degree AV block is associated with a delay, not a block, in impulse conduction
that results in a constant PR interval of more than 0.20 second in duration. A severe first-degree AV
block exists when the PR interval is longer than 0.30 second (Kusumoto et al., 2018). The patient with
a first-degree AV block is often asymptomatic. Characteristics of first-degree AV block include the
following (Fig. 5.4):
Rhythm: Regular
Rate: Usually within normal range, but depends on underlying rhythm
P waves: A QRS complex follows every positive (i.e., upright) P wave
PR interval: Fixed duration of more than 0.20 second
QRS duration: Usually 0.11 second or less unless abnormally conducted
First-degree AV block may be a normal finding in individuals with no history of cardiac disease. It
may also occur because of acute MI, degenerative fibrosis and sclerosis of the conduction system, effects
of medications, ischemia or injury to the AV node or AV bundle, or myocarditis.
CHAPTER 5  Bradycardias 101

Fig. 5.4  ​Sinus rhythm with a first-degree atrioventricular block, ST-segment depression. (From Aehlert, B. [2004]. ECG
study cards, St. Louis, Mosby.)

Second-Degree Atrioventricular Block Type I


Second-degree AV block type I is also known as type I block, Mobitz I, or Wenckebach. It is associated
with a cyclic pattern that consists of conducted P waves (i.e., each P wave is followed by a QRS) and
then the sudden failure of a P wave that is not conducted (i.e., the P wave is not accompanied by a
QRS) to the ventricles. The P wave that is not conducted ends a group of beats. The cycle then begins
again. The repetition of this cyclic pattern is called grouped beating. The characteristics of second-de-
gree AV block type I can be summarized as follows (Fig. 5.5):
Rhythm: Ventricular irregular; atrial regular; grouped beating may be present
Rate: Atrial rate is greater than the ventricular rate
P waves: Normal in size and shape; some P waves are not followed by a QRS complex
PR interval: Progressive prolongation of the PR interval (although lengthening may be very slight)
until a P wave appears without a QRS complex; the PR interval after a nonconducted P wave is
shorter than the interval preceding the nonconducted beat
QRS duration: Usually 0.11 second or less; complexes are periodically dropped
This type of AV block can result from blockage of the right coronary artery, as in a right ventricular
or inferior MI. Second-degree AV block type I can also occur in athletes and during sleep in patients
without evidence of heart disease. Other possible causes include aortic valve disease, medications (e.g.,
beta blockers, calcium blockers, digoxin), mitral valve prolapse, and rheumatic heart disease. The patient
with second-degree AV block type I is usually asymptomatic because the ventricular rate often remains
nearly normal, and cardiac output is not significantly affected.

Fig. 5.5  ​Second-degree atrioventricular block type I. (From Aehlert, B. [2004]. ECG study cards, St. Louis, Mosby.)

Second-Degree Atrioventricular Block Type II


Second-degree AV block type II is also called type II block or Mobitz II AV block (Fig. 5.6).
The site of the block in type II block is almost always below the AV node, occurring most often
in the bundle branches (Issa et al., 2019). Although second-degree AV block type II is less
common than type I, type II is more serious and has a relatively high risk of progression to
third-degree AV block ( John, 2018). Characteristics of second-degree AV block type II include the
following:
Rhythm: Ventricular irregular; atrial regular
Rate: Atrial rate is greater than the ventricular rate; ventricular rate is often slow
P waves: Normal in size and shape; some P waves are not followed by a QRS complex
102 CHAPTER 5  Bradycardias

Lead II

Fig. 5.6  ​Second-degree atrioventricular block type II. (From Aehlert, B. [2004]. ECG study cards, St. Louis, Mosby.)

PR interval: Within normal limits or prolonged but constant for the conducted beats; the PR intervals
before and after a blocked P wave are constant
QRS duration: Within normal limits if the block occurs above or within the bundle of His; greater than
0.11 second if the block occurs below the bundle of His; complexes are periodically absent after P
waves
Second-degree AV block type II may occur because of an anterior MI, acute myocarditis, aortic
valve disease, cardiomyopathy, fibrosis of the conduction system, or rheumatic heart disease. If the
ventricular rate is within normal limits, the patient may be asymptomatic. More commonly, the ven-
tricular rate is significantly slowed, and serious signs and symptoms result because of the slow rate and
decreased cardiac output. Closely monitor the patient because this dysrhythmia may abruptly progress
to third-degree AV block.

2:1 Atrioventricular Block


With second-degree 2:1 AV block, there is one conducted P wave followed by a blocked P wave;
thus, two P waves occur for every one QRS complex (i.e., 2:1 conduction). Because there are
no two PQRST cycles in a row from which to compare PR intervals, 2:1 AV block cannot be
conclusively classified as type I or type II. To determine the type of block with certainty,
close ECG monitoring of the patient is necessary until the conduction ratio of P waves to
QRS complexes changes to 3:2, 4:3, and so on, which would enable PR interval comparison. If
the QRS complex is narrow (i.e., it measures 0.11 second or less), the block is likely to be a form
of second-degree AV block type I. A 2:1 AV block associated with a wide QRS complex (i.e., more
than 0.11 second) is usually a type II block. Characteristics of 2:1 AV block can be summarized
as follows (Fig. 5.7):
Rhythm: Ventricular regular; atrial regular
Rate: Atrial rate is twice the ventricular rate
P waves: Normal in size and shape; every other P wave is not followed by a QRS complex
PR interval: Constant
QRS duration: May be narrow or wide; complexes are absent after every other P wave

Fig. 5.7  ​A 2:1 atrioventricular block with narrow-QRS complexes. (From Aehlert, B. [2018]. ECGs made easy, ed 6,
St. Louis, Mosby.)

Third-Degree Atrioventricular Block


With third-degree AV block, there is a complete block in the conduction of impulses between the atria
and the ventricles. The site of the block may occur at the level of the AV node, the bundle of His, or
CHAPTER 5  Bradycardias 103

Fig. 5.8  ​Third-degree atrioventricular block with ST segment depression and inverted T waves. (From Aehlert, B. [2004].
ECG study cards, St. Louis, Mosby.)

distal to the bundle of His. A secondary pacemaker (either junctional or ventricular) stimulates
the ventricles; therefore the QRS may be narrow or wide, depending on the location of the escape
pacemaker and the condition of the intraventricular conduction system. Wide-QRS escape rhythms
associated with third-degree AV block are often unstable, with a risk of syncope and sudden death
( John, 2018). ECG characteristics of third-degree AV block include the following (Fig. 5.8):
Rhythm: Ventricular regular; atrial regular; no relationship between the atrial and ventricular rhythms
(i.e., AV dissociation is present)
Rate: The ventricular rate is determined by the origin of the escape pacemaker; the atrial rate is greater
than (and independent of ) the ventricular rate
P waves: Normal in size and shape; some P waves are not followed by a QRS complex
PR interval: None: the atria and the ventricles beat independently of each other; thus there is no true
PR interval
QRS duration: Narrow or wide, depending on the location of the escape pacemaker and the condition
of the intraventricular conduction system
Causes of third-degree AV block include MI, the effects of medications, disease of the conduction
system, and congenital heart disease. The patient’s signs and symptoms will depend on the origin
of the escape pacemaker (i.e., junctional versus ventricular) and the patient’s response to a slower
ventricular rate. When third-degree AV block occurs during an acute anterior MI, it generally occurs
abruptly during the first 24 hours after MI, is often associated with a ventricular rate less than
40 beats/min and a wide QRS complex, and has an increased risk of ventricular asystole (Issa et al.,
2019). Most patients with third-degree AV block have an indication for permanent pacemaker
placement.

PATIENT ASSESSMENT
If a patient presents with a bradycardia, ask a team member to attach a pulse oximeter and cardiac
monitor, and obtain the patient’s baseline vital signs. At the same time, perform a primary and second-
ary assessment and evaluate how the patient is tolerating the rhythm. Box 5.1 shows examples of pos-
sible signs and symptoms associated with symptomatic bradycardia. If the patient is experiencing
symptoms, try to determine their frequency, timing, duration, severity, longevity, circumstances,

BOX 5.1 Symptomatic Bradycardia: Possible Signs and Symptoms


• Acute altered mental status • Ongoing ischemic chest discomfort
• Acute heart failure • Pulmonary congestion
• Diaphoresis • Shortness of breath
• Dizziness • Signs of shock
• Fatigue • Syncope or presyncope
• Hypotension • Weak pulses
• Lightheadedness • Weakness
104 CHAPTER 5  Bradycardias

triggers, and alleviating factors (Kusumoto et al., 2018). Because medications can cause or worsen a
bradycardia, be sure to ask about any prescribed and over-the-counter medications, including herbal
supplements, the patient may be taking and any recent medication changes.

THERAPEUTIC INTERVENTIONS
The bradycardia algorithm is shown in Fig. 5.9. Based on your assessment findings, determine if the
patient who is experiencing a bradycardia is asymptomatic, stable but symptomatic, symptomatic and
unstable, or pulseless. If the patient is not breathing and has no pulse (i.e., is in cardiac arrest) despite

• Obtain H&P, evaluate ECG, assess vital signs, and assess patient stability
• Determine if heart rate is appropriate for patient’s clinical condition

Search for and treat reversible causes

• Maintain an open airway, give oxygen if indicated, assist breathing as needed


• Continue monitoring cardiac rhythm, oxygen saturation, and vital signs
• Establish vascular access, obtain a 12-lead ECG (if available), and attempt to identify
and treat the underlying cause of the dysrhythmia using the Hs and Ts while providing care

Is the bradycardia persistent and causing moderate or severe signs and symptoms?

Yes – give atropine No – continue monitoring

If symptoms persist despite atropine:


• Transcutaneous pacing
or
• Dopamine
or
• Epinephrine
or
• Isoproterenol

• Consider expert consult


• Consider transvenous pacing

Assessment Medications

• Heart rates below 50 beats/min and accompanied by • Atropine IV/IO push: 0.5 to 1 mg every 3 to 5 min to a
moderate to severe symptoms typically require maximum dose of 3 mg (avoid giving atropine to
treatment patients after heart transplant; aminophylline may be
• Examples of moderate to severe signs and symptoms ordered as an alternative to atropine for these
include acutely altered mental status, acute heart patients)
failure, hypotension, ongoing ischemic chest • Dopamine IV/IO infusion: 5 to 20 mcg/kg/min; start at
discomfort, pulmonary congestion, shortness of 5 mcg/kg/min and increase by 5 mcg/kg/min every 2
breath, and signs of shock min; titrate to BP and other clinical responses
• Epinephrine IV/IO infusion: 2 to 10 mcg/min titrated
to hemodynamic response
• Isoproterenol: 20 to 60 mcg IV bolus followed by doses
of 10 to 20 mcg IV, or an infusion of 1 to 20 mcg/min
based on heart rate response

BP = Blood pressure; ECG = electrocardiogram; H&P = history and physical examination; IO = intraosseous; IV = intravenous
(Sources: Kusumoto, et al., 2018; Panchal, 2020.)

Fig. 5.9  Adult bradycardia with a pulse algorithm.


CHAPTER 5  Bradycardias 105

BOX 5.2 Hs and Ts


Hypovolemia Tamponade, cardiac
Hypoxia Tension pneumothorax
Hypothermia Thrombosis: lungs (i.e., massive pulmonary embolism)
Hypokalemia/Hyperkalemia Thrombosis: heart (i.e., acute coronary syndromes)
Hydrogen ion (acidosis) Tablets/toxins: drug overdose

the appearance of organized electrical activity on the cardiac monitor, pulseless electrical activity
(PEA) exists. PEA is discussed in Chapter 7.
If the patient is asymptomatic, no treatment is necessary, but monitoring and follow-up may be
warranted depending on the dysrhythmia. If the patient is symptomatic but stable (i.e., perfusion is
adequate), monitor their cardiac rhythm and vital signs, and regularly reassess for changes. When a
patient has frequent but intermittent symptoms, continuous ambulatory ECG monitoring may be
ordered in nonemergent situations to help determine if a dysrhythmia is a precipitating cause.
If the patient is symptomatic and unstable because of the slow rate, assess their oxygen saturation
level, and determine whether signs of increased work of breathing are present. Give supplemental
oxygen if oxygenation is inadequate and assist breathing if ventilation is inadequate. Establish intra-
venous (IV) access, obtain a 12-lead ECG if doing so will not delay patient treatment, and get a
cardiology consult. Attempt to identify and treat the underlying cause of the patient’s dysrhythmia
using the Hs and Ts memory aid (Box 5.2). If the patient has an altered mental status, obtain a bed-
side glucose. Depending on the patient’s history and presentation, possible laboratory studies include
a thyroid level, digoxin level, metabolic panel, toxicology screen, and/or serum cardiac markers if an
ACS is suspected.
Atropine is the drug of choice for symptomatic bradycardia if the dysrhythmia persists despite ad-
equate oxygenation and ventilation. TCP or a dopamine or epinephrine IV infusion are alternative
therapies (see Chapter 3). Atropine is often effective at improving the patient’s heart rate and signs
and symptoms associated with their bradycardia. However, because the effects of atropine are not al-
ways sustained, frequent reassessment is essential. Some clinicians find it prudent to apply pacing
electrodes to enable immediate TCP and to have an IV infusion of dopamine or epinephrine prepared
in advance should decompensation occur. If symptomatic bradycardia causes shock and vascular access
is not yet available, initiate TCP while continuing attempts to establish vascular access (Panchal et al.,
2020).
Atropine is most likely to be effective when symptomatic bradycardia is associated with a narrow-
QRS complex. It is unlikely to be effective with wide-QRS AV blocks because the site of the block is
usually below the AV node. In situations such as this, pacing or the administration of beta-adrenergic
medications are preferable (Link et al., 2015). The choice of transcutaneous versus temporary transve-
nous pacing varies by institution and equipment availability. If TCP is available, it should be readied
for immediate use, should the patient’s condition deteriorate.
Isoproterenol is another alternative that is given intravenously. Because it can cause ventricular
irritability, isoproterenol is not routinely used in the treatment of symptomatic bradycardia; however, it
is used for the heart transplant patient experiencing symptomatic bradycardia. Atropine is typically
ineffective in patients with a transplanted heart because the vagus nerve is not reattached during the
transplant procedure.

ACLS Pearl
Although calcium administration is not part of the symptomatic bradycardia algorithm, IV calcium
is useful in the treatment of many types of bradydysrhythmias, especially those that occur be-
cause of an overdose of a calcium blocker (e.g., verapamil, diltiazem) or because of hyperkalemia.
Possible therapies that may be used during a beta blocker or calcium blocker overdose include
an IV infusion of glucagon and high-dose insulin therapy.
106 CHAPTER 5  Bradycardias

STOP AND REVIEW


Identify one or more choices that best complete the statement or answer the question.

_ ____ 1. Which of the following are features of third-degree AV block?


a. Constant PR intervals
b. More P waves than QRSs
c. Irregular ventricular rhythm
d. P waves that occur regularly

_____ 2. How would you differentiate a junctional escape rhythm at 40 beats/min from a
ventricular escape rhythm at the same rate?
a. It is impossible to differentiate a junctional escape rhythm from a ventricular
escape rhythm.
b. The junctional escape rhythm will have a narrow QRS complex; the ventricular
escape rhythm will have a wide QRS complex.
c. The rate (40 beats/min) would indicate a junctional escape rhythm, not a ven-
tricular escape rhythm.
d. The junctional escape rhythm will have a wide QRS complex; the ventricular
escape rhythm will have a narrow QRS complex.

_ ____ 3. Which of the following are features of 2:1 AV block?


a. Constant PR interval
b. Atrial rate is twice the ventricular rate
c. P waves may occur before, during, or after the QRS
d. Atrial rhythm is regular, but ventricular rhythm is irregular

_____ 4. Which of the following dysrhythmias has the greatest potential for sudden,
third-degree AV block?
a. Junctional rhythm
b. Sinus bradycardia
c. First-degree AV block
d. Second-degree AV block type II

_____ 5. Depending on the severity of the patient’s signs and symptoms, management
of symptomatic bradycardia may require which of the following interventions?
a. Defibrillation
b. Atropine IV bolus
c. Adenosine IV bolus
d. Epinephrine IV infusion
e. Dopamine IV infusion
f. Synchronized cardioversion

_ ____ 6. Which of the following are ECG characteristics of a ventricular escape rhythm?
a. Wide QRS complexes
b. Atrial rate 40 to 100 beats/min
c. QRS duration of 0.11 second or less
d. Essentially regular ventricular rhythm
e. Constant PR interval of 0.12 to 0.20 second
f. Rapid ventricular rhythm with no pattern or regularity
CHAPTER 5  Bradycardias 107

CASE STUDY 5.1


Identify one or more choices that best complete the statement or answer the question.

You and your team have been summoned to care for a 75-year-old woman who is complaining of
feeling lightheaded, occasionally nauseated, and unusually tired for the past 4 days. Emergency equip-
ment, including a biphasic manual defibrillator with TCP capability, is available. As you work through
this case study, remember that there may be alternative actions that are perfectly acceptable, yet not
discussed here.

_____ 1. The patient is lying supine on a stretcher and is aware of your approach. Her
breathing is not labored, and her skin is pink. Select the correct statements
about this patient’s general impression findings.
a. Appearance is normal.
b. Appearance is abnormal.
c. Breathing is normal.
d. Breathing is abnormal.
e. Circulation is normal.
f. Circulation is abnormal.

_____ 2. While a team member performs a primary and secondary assessment, which of the
following actions should occur at the same time?
a. Obtain the patient’s vital signs.
b. Attach a pulse oximeter and cardiac monitor.
c. Ask a qualified team member to prepare to insert an advanced airway.
d. Take a moment to familiarize yourself with the pacing functions on the defibrillator.

_ ____ 3. Which of the following are correct regarding symptomatic bradycardia?


a. The patient’s heart rate is less than 60 beats/min.
b. The patient is experiencing serious signs and symptoms.
c. The patient’s symptoms are related to the bradycardia.
d. The dysrhythmia responsible for the bradycardia is an AV block.

_____ 4. The patient’s BP is 88/54 millimeters of mercury (mm Hg), her ventilatory rate
is 16 breaths/min, and her heart rate is 40 beats/min. Breath sounds are clear bi-
laterally, and her skin is warm, pink, and moist. The patient’s oxygen saturation
level (SpO2) on room air is 96%. How would you categorize this
patient’s condition based on this information?
a. Asymptomatic
b. Stable but symptomatic
c. Symptomatic and unstable
d. Pulseless

_____ 5. Your assessment reveals the patient is alert and oriented to person, place, time, and
event. She reports feeling lightheaded, occasionally nauseated, and unusually tired
for the past 4 days. The patient is allergic to penicillin. She has a history of hyper-
lipidemia and gastroesophageal reflux disease, for which she takes simvastatin and
omeprazole daily. The patient denies chest pain and shortness of breath and reports
no recent illnesses or injuries. She has been placed on the cardiac monitor, which
reveals the following rhythm (Fig. 5.10). This rhythm is:
a. 2:1 AV block.
b. Sinus bradycardia.
c. Third-degree AV block.
d. Ventricular escape rhythm.
e. Second-degree AV block type I.
f. Second-degree AV block type II.
108 CHAPTER 5  Bradycardias

Fig. 5.10  ​Case Study 5.1, question 5. (From Aehlert, B. [2004]. ECG study cards, St. Louis, Mosby.)

_ ____ 6. Which of the following interventions would be appropriate at this time?


a. Obtain a 12-lead ECG.
b. Start an IV of normal saline.
c. Obtain a portable chest radiograph.
d. Instruct a team member to make preparations for synchronized cardioversion.

_____ 7. As the actions you requested are being carried out, you instruct a team member
to administer atropine IV to this patient. Which of the following statements are
correct?
a. The initial recommended dosage of atropine is 0.5 to 1 mg IV.
b. Atropine is the drug of choice for most symptomatic bradycardias.
c. Each dose of atropine should be immediately followed by a 20-mL saline flush.
d. Atropine may be repeated every 3 to 5 minutes as needed to a maximum dose
of 3 mg.

_____ 8. Two doses of atropine have been administered according to your instructions and
increased the patient’s ventricular rate and improved her symptoms. If the use of
atropine had been ineffective, or if the patient’s symptoms return, which of the
following actions would be reasonable to consider?
a. Make preparations for TCP.
b. Prepare a dopamine infusion.
c. Prepare an epinephrine infusion.
d. Watchful waiting would be the most prudent course of action.

_____ 9. The patient’s BP is now 118/66 mm Hg and her ventilatory rate is 16 breaths/min.
Her skin is warm, pink, and dry. The cardiac monitor reveals a sinus rhythm at
85 beats/min. The patient reports that she is feeling better. Next steps should in-
clude which of the following?
a. Request a team debriefing.
b. Prepare to discharge the patient home.
c. Continue to monitor the patient’s ECG and vital signs.
d. Arrange for the patient’s immediate transfer to the intensive care unit.
e. Review the results of the patient’s 12-lead ECG and laboratory studies to
determine the cause of the bradycardia.
CHAPTER 5  Bradycardias 109

CASE STUDY 5.2


Identify one or more choices that best complete the statement or answer the question.

A 61-year-old man presents with dizziness. You have a sufficient number of advanced life support
personnel available to assist you and carry out your instructions. Emergency equipment, including a
biphasic manual defibrillator with TCP capability, is available. As you work through this case study,
remember that there may be alternative actions that are perfectly acceptable, yet not discussed here.

_____ 1. The patient is lying supine on a stretcher and is aware of your approach. His breathing
is not labored, and his skin is pale. Select the correct statements about this patient’s
general impression findings.
a. Appearance is normal.
b. Appearance is abnormal.
c. Breathing is normal.
d. Breathing is abnormal.
e. Circulation is normal.
f. Circulation is abnormal.

_ ____ 2. Which of the following actions should you take at this time?
a. Attach a pulse oximeter.
b. Attach a cardiac monitor.
c. Obtain the patient’s vital signs.
d. Perform a primary and secondary assessment.

_____ 3. The patient’s blood pressure is 72/44 mm Hg, his ventilatory rate is 18 breaths/min,
and his heart rate is about 30 beats/min. Breath sounds reveal wheezes in the right
upper lobe, and his skin is cool, pale, and moist. The patient’s SpO2 on room air is 92%.
Based on the information provided, how would you categorize this patient’s condition?
a. Asymptomatic
b. Stable but symptomatic
c. Symptomatic and unstable
d. Pulseless

_____ 4. Your assessment reveals the patient is alert and oriented to person, place, time, and
event. He reports that he suddenly felt dizzy during a meeting with a coworker.
The patient has no known allergies. He has a history of chronic obstructive pulmo-
nary disease, for which he occasionally uses a Combivent Respimat inhaler, and
hypertension, for which he takes lisinopril daily. The patient denies chest pain and
shortness of breath and reports no recent illnesses or injuries. He has been placed
on the cardiac monitor (Fig. 5.11). The rhythm shown is:
a. 2:1 AV block.
b. Sinus arrhythmia.
c. Third-degree AV block.
d. Junctional escape rhythm.
e. Second-degree AV block type I.
f. Second-degree AV block type II.

Fig. 5.11  Case Study 5.2, question 4. ​(From Aehlert, B. [2004]. ECG study cards, St. Louis, Mosby.)
110 CHAPTER 5  Bradycardias

_ ____ 5. Which of the following interventions would be appropriate at this time?


a. Prepare for TCP.
b. Obtain a 12-lead ECG.
c. Start an IV of normal saline.
d. Obtain a portable chest radiograph.

_____ 6. As the actions you requested are being carried out, a team member asks if you want
to administer atropine IV to this patient. Which of the following statements are
correct?
a. Atropine is contraindicated in this situation.
b. Atropine may be considered but is unlikely to be effective.
c. Pacing or the use of beta-adrenergic agents is preferable in this situation.
d. Atropine may be considered and is likely to be effective based on the patient’s
cardiac rhythm.

_____ 7. You elect to begin TCP. Pacing pads have been applied to the patient, and the
procedure has been explained to him. At what rate (i.e., paced pulses per minute
[ppm]) should the pacemaker be set?
a. Between 40 and 60 ppm
b. Between 50 and 70 ppm
c. Between 60 and 90 ppm
d. Between 100 and 120 ppm

_____ 8. The cardiac monitor reveals the following rhythm (Fig. 5.12). Which of the following
statements are true?
a. The rhythm shown is evidence of mechanical capture.
b. The rhythm shown is a ventricular paced rhythm with 100% capture.
c. Continue pacing at an output level slightly higher than that used to achieve
capture.
d. Confirm that the patient’s pulse oximetry waveform is consistent with palpable
pulses and the rhythm shown here.

_____ 9. The patient’s blood pressure is now 104/60 mm Hg and his ventilatory rate is
16 breaths/min. His skin is warm, pink, and dry. What would you like to
do next?
a. Closely monitor the patient’s ECG and vital signs.
b. Administer medications to ensure patient comfort.
c. Periodically assess the patient’s skin at pacing electrode sites.
d. Review the results of the diagnostics performed to determine the cause of the
bradycardia.

Fig. 5.12  Case Study 5.2, question 8. ​(From Aehlert, B. [2004]. ECG study cards, St. Louis, Mosby.)
CHAPTER 5  Bradycardias 111

STOP AND REVIEW  ANSWERS


1. B, D. In a third-degree AV block, there are more P waves than QRSs, P waves occur regularly, there is
a regular ventricular rhythm, there is no pattern to PR intervals, and the QRS may be narrow or wide.

2. B. Although junctional and ventricular rhythms are ectopic pacemaker sites, their rhythms can
generally be differentiated by the width of their QRS complexes. The junctional escape rhythm will
have a narrow QRS complex; the ventricular escape rhythm will have a wide QRS complex.

3. A, B. Second-degree 2:1 AV block is characterized by P waves that are normal in size and shape,
but every other P wave is not followed by a QRS. The atrial rate is twice the ventricular rate. The
PR interval for the conducted beats is constant.

4. D. Second-degree AV block type II, often associated with anterior MI, has an increased risk of
mortality because of its relatively high risk of progression to advanced or third-degree AV block.

5. B, D, E. Atropine, administered IV, is the drug of choice for symptomatic bradycardia. TCP or a
dopamine, epinephrine, or isoproterenol IV infusion may be tried if atropine is ineffective. Defibril-
lation, synchronized cardioversion, and adenosine are not indicated in the treatment of bradycardias.

6. A, D. A ventricular escape rhythm exists when three or more ventricular beats occur in a row at
a rate of 20 to 40 beats/min. The ventricular rhythm is essentially regular and the QRS complexes
seen with this rhythm are wide because the impulses begin in the ventricles, bypassing the normal
conduction pathway. P waves are usually absent; therefore there is no PR interval.

C A S E S T U D Y 5 . 1 A NSWERS
1. A, C, E. The general impression findings are normal. Appearance is normal, breathing is normal,
and circulation is normal.

2. A, B. Ask a team member to attach a pulse oximeter and cardiac monitor and obtain the patient’s
baseline vital signs while you perform (or direct a team member to perform) a primary and second-
ary assessment. Based on the information provided so far, preparations for the insertion of an
advanced airway are premature at this time. As the team leader, you should take a “stand back”
approach so that you can observe and assess all of the actions of your team. Ideally, you should not
assume responsibility for operating equipment, such as the pacing functions of a defibrillator,
yourself. Furthermore, it is simply unacceptable that you would not know how to use a piece
of equipment before being faced with a situation in which it is needed. At this point, we do
not yet know if the pacer will be required, but if it is, ask a qualified team member to operate the
equipment.

3. A, B, C. In adults, a bradycardia is a heart rate less than 60 beats/min. The term symptomatic
bradycardia is used to describe a patient who experiences signs and symptoms of hemodynamic
compromise related to a slow heart rate. Although an AV block may be the cause of symptomatic
bradycardia, it is not the only dysrhythmia that can cause symptoms (e.g., sinus bradycardia, junc-
tional escape rhythm, ventricular escape rhythm).

4. B. Because of this patient’s hypotension and skin condition, she should be considered stable but
symptomatic.

5. A. The rhythm is 2:1 AV block.


112 CHAPTER 5  Bradycardias

6. A, B, C. Continue to monitor the patient’s oxygen saturation and vital signs. Ask the IV team
member to start an IV of normal saline. Obtain a 12-lead ECG, a cardiology consult, laboratory
studies, and a portable chest radiograph. Synchronized cardioversion is used in the management of
unstable patients who have a tachycardia with a pulse; it is not indicated in the management of a
bradycardia.

7. A, B, D. Atropine is the drug of choice for most symptomatic bradycardias. It is a reasonable choice
in this situation because the patient’s dysrhythmia is an AV block with a narrow-QRS complex,
which implies that the block is likely at the level of the AV node. Atropine is unlikely to be effective
when the site of an AV block is below the AV node, which is usually associated with a wide-QRS.
The recommended dosage of atropine is 0.5 to 1 mg IV, repeated every 3 to 5 minutes as needed,
to a maximum dose of 3 mg. Each dose of adenosine (not atropine) should be immediately followed
by a 20 mL saline flush. Adenosine is used in the management of symptomatic patients with a
tachycardia; it is not used in the treatment of bradycardias.

8. A, B, C. Although atropine is often effective at improving the patient’s heart rate and signs and
symptoms associated with their bradycardia, the effects of atropine are not always sustained—there-
fore frequent reassessment is essential. Atropine is often used as a bridge to longer-lasting interven-
tions, such as TCP or the IV administration of dopamine or epinephrine infusions. With this in
mind, some clinicians find it prudent to apply pacing electrodes to enable immediate TCP and to
have an IV infusion of dopamine or epinephrine prepared in advance should decompensation occur.

9. A, C, E. Continue to monitor the patient’s ECG and vital signs closely. Review your history and
physical examination findings and the results of the patient’s 12-lead ECG and laboratory studies
to try to determine the cause of the patient’s bradycardia. Arrange for the patient’s transfer for
continued care, which is more likely to be to a telemetry unit rather than to the intensive care unit.
Request a team debriefing after the transfer of patient care is complete.

C A S E S T U DY 5.2 ANSWERS
1. A, C, F. The general impression findings are abnormal. Appearance is normal, breathing is normal,
but circulation is abnormal (abnormal skin color).

2. A, B, C, D. Ask a team member to attach a pulse oximeter and cardiac monitor and obtain the
patient’s baseline vital signs while you perform a primary and secondary assessment.

3. C. This patient should be considered symptomatic and unstable because of his marked hypotension
and skin condition.

4. C. The dysrhythmia is a third-degree AV block.

5. A, B, C, D. Continue to monitor the patient’s oxygen saturation and vital signs. Ask the IV team
member to start an IV of normal saline. Order a 12-lead ECG, a cardiology consult, laboratory
studies, and a portable chest radiograph. Instruct the defibrillation team member to prepare for
TCP.

6. B, C. Atropine is unlikely to be effective when the site of an AV block is below the AV node; pacing
or the use of beta-adrenergic agents is preferable.

7. C. In general, a rate that is between 60 and 90 ppm will maintain an adequate blood pressure and
cerebral perfusion in an adult. After the rate has been regulated, start the pacemaker. Titrate the
stimulating current (i.e., output or mA) slowly but steadily until pacer spikes are visible before each
QRS complex.
CHAPTER 5  Bradycardias 113

8. B, C, D. The rhythm shown is a ventricular paced rhythm with 100% capture and indicates electri-
cal capture. Mechanical capture occurs when pacing produces a response that can be measured,
such as a palpable pulse. After capture is achieved, continue pacing at an output level slightly higher
(e.g., about 2 mA) than the threshold of consistent electrical capture. For example, if the monitor
revealed 100% capture when you reached 80 mA and mechanical capture is confirmed at that set-
ting, your final current setting would be about 82 mA. Verify that the patient’s pulse oximetry
waveform is consistent with that assessed with both electrical and mechanical pacemaker capture.

9. A, B, C, D. Continue to monitor the patient’s ECG and vital signs closely. Because TCP is uncom-
fortable, administer medications as ordered to ensure patient comfort. Assess the skin under the
pacing electrodes for irritation after the first 30 minutes of pacing and periodically after that. Re-
view the results of the patient’s 12-lead ECG and laboratory studies to try to determine the cause
of the patient’s bradycardia. Arrange for patient transfer for continued care and request a team
debriefing after the transfer of patient care is complete.

REFERENCES
Issa, Z. F., Miller, J. M., & Zipes, D. P. (2019). Atrioventricular conduction abnormalities. In Z. F. Issa, J. M. Miller,
& D. P. Zipes (Eds.), Clinical arrhythmology and electrophysiology: A companion to Braunwald’s heart disease
(3rd ed., pp. 255–285). Philadelphia, PA: Elsevier.
John, R. M. (2018). Atrioventricular block. In D. P. Zipes, J. Jalife, & W. G. Stevenson (Eds.), Cardiac electro-
physiology: From cell to bedside (7th ed., pp. 1003–1010). Philadelphia, PA: Elsevier.
Kusumoto, F. M., Schoenfeld, M. H., Barrett, C., et al. (2018). 2018 ACC/AHA/HRS guideline on the evaluation
and management of patients with bradycardia and cardiac conduction delay. Circulation, 140(8), e382–e482.
Link, M. S., Berkow, L. C., Kudenchuk, P. J., et al. (2015). American Heart Association Guidelines for CPR & ECC.
Retrieved from American Heart Association. In Web-based integrated guidelines for cardiopulmonary resus-
citation and emergency cardiovascular care – Part 7: Adult advanced cardiovascular life support: Eccguidelines.
heart.org
Panchal, A. R., Bartos, J. A., Cabañas, J. G., et al. (2020). Part 3: Adult basic and advanced life support: 2020
American Heart Association guidelines for cardiopulmonary resuscitation and emergency cardiovascular care.
Circulation, 142(16 suppl 2), S366–S468.
This page intentionally left blank
CHAPTER 6
Tachycardias

INTRODUCTION
The tachycardia algorithm is a treatment guideline that is used when providing care to patients who
have a tachycardia with a pulse. You must be able to recognize if a patient is asymptomatic, stable but
symptomatic, symptomatic and unstable, or pulseless. Care of the pulseless patient with a tachycardia
is provided using the cardiac arrest algorithm, which is discussed in Chapter 7. This chapter reviews
the electrocardiogram (ECG) characteristics of narrow-QRS, wide-QRS, and irregular tachycardias,
and the initial emergency care for a patient experiencing a tachycardia.

L E A R N I N G OBJECTIVES
After completing this chapter, you should be able to:
1. Differentiate among the following ECG rhythms: narrow-QRS tachycardias (sinus
tachycardia, atrial tachycardia [AT], atrioventricular [AV] nodal reentrant tachycardia
[AVNRT], AV reentrant tachycardia [AVRT]), wide-QRS tachycardias (monomorphic
ventricular tachycardia [VT]), and irregular tachycardias (multifocal atrial tachycardia
[MAT], atrial fibrillation [AFib], atrial flutter, and polymorphic VT [PMVT]).
2. Given a patient situation, identify a patient who is experiencing a tachycardia as
asymptomatic, stable but symptomatic, symptomatic and unstable, or pulseless.
3. Given a patient situation, describe the initial emergency care for a patient experiencing
a narrow-QRS tachycardia, wide-QRS tachycardia, or irregular tachycardia.

L E A R N I N G PLAN
• Read this chapter before class. Remember to highlight critical concepts as you read.
• Develop and use flashcards, flowcharts, and mnemonics to help enhance your
retention of the information presented. Flashcards can be particularly helpful with
rhythm recognition.
• Master identification of the following rhythms: sinus tachycardia, AT, AVNRT, AVRT,
monomorphic VT, MAT, AFib, atrial flutter, and PMVT.
• Master the following medications: oxygen (O2), adenosine, amiodarone, beta blockers,
diltiazem, magnesium sulfate, procainamide, sotalol, and verapamil.
• Master the following skills:
n Ensure scene safety and the use of personal protective equipment.
n Assign team member roles or perform as a team member in a simulated patient
situation.

115
116 CHAPTER 6  Tachycardias

Direct or perform a basic life support (BLS) assessment, primary assessment, and
n

secondary assessment.
n Obtain vital signs; establish vascular access; attach a pulse oximeter and blood
pressure (BP) and cardiac monitor; give supplemental O2 if indicated; and order a
12-lead ECG.
n Quickly identify an ECG rhythm, determining whether the QRS is narrow or wide
(and if it is wide, if the QRS is monomorphic or polymorphic), regular or irregular.
n Quickly recognize if a patient is asymptomatic, stable but symptomatic,
symptomatic and unstable, or pulseless.
n Demonstrate familiarity with the tachycardia algorithm.
n Demonstrate an understanding of what vagal maneuvers are and when they are
indicated.
n Demonstrate an understanding of the actions, indications, dosages, adverse
effects, and contraindications for the medications used in the treatment of a
narrow-QRS or wide-QRS tachycardia.
n Deliver the correct type of energy (synchronized cardioversion versus defibrillation)
and the proper energy level for the tachycardia if electrical therapy is indicated.
n Demonstrate the safe operation of a defibrillator if electrical therapy is indicated.
n Recognize the need to change from synchronized cardioversion to defibrillation if the
rhythm changes to pulseless ventricular tachycardia (pVT) or ventricular fibrillation (VF).
n Consider the possible reversible causes of a cardiac emergency.
n Verbalize when it is best to seek expert consultation.
n Review your performance as a team leader or team member during a postevent
debriefing.
• Complete the chapter quiz and review the quiz answers provided.
• Read the case studies at the end of this chapter and compare your answers with the
answers provided.

KEY TERMS
Supraventricular arrhythmias  Rhythms that begin in the sinoatrial (SA) node, the atrial
tissue, or the AV junction.

OVERVIEW
The signs and symptoms that are experienced by a patient with a tachycardia depend on the ventricu-
lar rate, how long the tachycardia lasts, the patient’s general health, and the presence of underlying
heart disease. The faster the heart rate, the more likely the patient is to have signs and symptoms resulting
from the rapid rate.

NARROW-QRS TACHYCARDIAS
Supraventricular arrhythmias include rhythms that begin in the SA node, the atrial tissue, or the AV
junction (i.e., they begin above the bifurcation of the bundle of His).

Sinus Tachycardia
If the SA node fires at a rate faster than normal for the patient’s age, the rhythm is called sinus tachy-
cardia. In adults, sinus tachycardia is faster than 100 beats per minute (beats/min) with a maximum
rate of about 220 beats/min, minus the patient’s age in years. ECG characteristics of sinus tachycardia
include the following (Fig. 6.1):
Rhythm: R-R and P-P intervals are regular
Rate: Faster than 100 beats/min with the maximum rate about 220 beats/min, minus the patient’s age
in years
CHAPTER 6  Tachycardias 117

Fig. 6.1  ​Sinus tachycardia. (From Aehlert, B. [2004]. ECG study cards, St. Louis, Mosby.)

BOX 6.1 Causes of Sinus Tachycardia


• Acute myocardial infarction • Medications (e.g., epinephrine, atropine,
• Caffeine-containing beverages dopamine)
• Dehydration, hypovolemia • Nicotine
• Exercise • Pain
• Fear and anxiety • Pregnancy
• Fever • Pulmonary embolism
• Heart failure • Shock
• Hyperthyroidism • Stimulant drugs (e.g., cocaine, amphet-
• Hypoxia amines, “ecstasy”)
• Infection • Sympathetic stimulation
• Inflammation

P waves: Positive (upright) in lead II; one precedes each QRS complex; P waves look alike
PR interval: 0.12 to 0.20 second and constant from beat to beat
QRS duration: 0.11 second or less unless abnormally conducted
Sinus tachycardia is a normal response to the body’s demand for increased cardiac output, which
results from many conditions (Box 6.1). The patient is often aware of an increase in heart rate. Some
patients complain of palpitations, a racing heart, or a feeling of pounding in their chests. Sinus tachy-
cardia in a patient who is having an acute myocardial infarction (MI) may be an early warning signal
for heart failure, cardiogenic shock, and more serious dysrhythmias. Treatment for sinus tachycardia is
directed at correcting the underlying cause (e.g., fever, pain, dehydration, anxiety). Sinus tachycardia in
a patient experiencing an acute MI may be treated with medications to slow the heart rate and decrease
myocardial oxygen demand (e.g., beta blockers), provided there are no signs of heart failure or other
contraindications.

ACLS Pearl
Some dysrhythmias with very rapid ventricular rates (RVRs, i.e., faster than 150 beats/min) require
the delivery of antiarrhythmics or a shock to slow or stop the rhythm. However, it is essential to
remember that giving antiarrhythmics or shocking a sinus tachycardia is inappropriate; instead,
treat the cause of the tachycardia.

Supraventricular Tachycardias
Supraventricular tachycardias (SVTs) involve tissue within the bundle of His or above and are associ-
ated with ventricular rates faster than 100 beats/min at rest (Page et al., 2016). Examples of SVTs are
shown in Fig. 6.2.

ACLS Pearl
Some SVTs need the AV node to sustain the rhythm, and some do not. For example, AVNRT and
AVRT require the AV node as part of the reentry circuit to continue the tachycardia. Other SVTs
use the AV node only to conduct the rhythm to the ventricles. For example, AT, atrial flutter, and
AFib arise from a site (or sites) within the atria; they do not need the AV node to sustain the rhythm.
118 CHAPTER 6  Tachycardias

Irregular Regular

AFib Aflutter AVNRT

SN SN
AVN

AVN

His-Purkinje system His-Purkinje system


MAT AVRT-orthodromic AVRT-antidromtic AT

Fig. 6.2  ​Basic mechanisms of supraventricular tachycardia. Typical atrial flutter (Aflutter) is a reentrant circuit around the
tricuspid valve in the right atrium. Atrioventricular nodal reentrant tachycardia (AVNRT) is reentry within the atrioventricular
node (AVN) and tissue around the node. Orthodromic AVNRT is a reentry circuit that traverses down the AVN and up a bypass
tract, leading to a narrow QRS. In antidromic atrioventricular reentrant tachycardia (AVRT), conduction is first down the
bypass tract and then up the AVN, leading to a wide QRS complex. Atrial tachycardia (AT ) is an ectopic focus of atrial activ-
ity at a faster rate than the sinoatrial node (SN). Atrial fibrillation (AFib) is several simultaneous wavelets in the atrium with
variable conduction through the AVN. Multifocal atrial tachycardia (MAT) involves at least three distinct ectopic atrial sites.
(From Link, M.S. [2012]. Clinical practice: Evaluation and initial treatment of supraventricular tachycardia. N Engl J Med,
367 [15], 1438–1448.)

Atrial Tachycardia
AT consists of a series of regular rapid beats from an irritable site in the atria at a rate faster than
100 beats/min (Ellenbogen & Koneru, 2018). Although the P waves preceding each QRS complex
appear upright, they tend to look different from those seen when the impulse is initiated from the SA
node. ECG characteristics of AT include the following:
Rhythm: Regular
Rate: 101 to 250 beats/min
P waves: One P wave precedes each QRS complex in lead II; these P waves differ in shape from sinus
P waves; an isoelectric baseline is usually present between P waves; if the atrial rhythm originates
in the low portion of the atrium, P waves will be negative in lead II; with rapid rates, it may be
challenging to distinguish P waves from T waves
PR interval: May be shorter or longer than normal; may be difficult to measure because P waves may
be hidden in the T waves of preceding beats
QRS duration: 0.11 second or less unless abnormally conducted
Paroxysmal supraventricular tachycardia (PSVT) is a term used to describe a rapid, regular SVT
that starts and ends suddenly. These features are characteristic of AVNRT or AVRT, and, less fre-
quently, AT (Page et al., 2016) (Fig. 6.3). PSVT may last for minutes, hours, or days. If the onset or
end of PSVT is not observed on the ECG, the dysrhythmia is simply called SVT.
Focal AT is a type of atrial tachycardia that begins in a small area (i.e., focus) within the atria. Its atrial
rate is often between 100 and 250 beats/min (Page et al., 2016). A patient with focal AT often presents with
PSVT. Automatic AT, which is also called ectopic AT, is another type of focal AT in which a small cluster
of cells with altered automaticity fire. Vagal maneuvers do not usually stop the tachycardia, but they may
slow the ventricular rate. Multifocal AT is discussed later in this chapter with irregular tachycardias.
CHAPTER 6  Tachycardias 119

Atrial tachycardia Sinus rhythm

P' P

Fig. 6.3  ​Atrial tachycardia (AT) (a type of supraventricular tachycardia) that ends spontaneously with the abrupt resump-
tion of sinus rhythm. AT that starts or ends suddenly is called paroxysmal supraventricular tachycardia. The P9 waves of
the tachycardia (rate: about 150 beats/min) are superimposed on the preceding T waves. (From Goldberger, A.L., Goldberger,
Z.D. [2018]. Goldberger’s clinical electrocardiography, ed 9, Philadelphia, Elsevier.)

A rhythm that lasts from three beats up to 30 seconds is a nonsustained rhythm. A sustained rhythm
is one that lasts more than 30 seconds. Focal AT can be sustained or nonsustained. If episodes of AT
are short, the patient may be asymptomatic. Nonsustained focal AT typically does not require treat-
ment (Page et al., 2016).

Reentrant Tachycardias
Reentry is the spread of an impulse through tissue already stimulated by that same impulse; an electri-
cal impulse is delayed, blocked, or both, in one or more areas of the conduction system while the
impulse is conducted normally through the rest of the conduction system. The result is a delayed
electrical impulse entering cardiac cells that have just been depolarized by the normally conducted
impulse. Reentry is a common mechanism for AVNRT and AVRT. With AVNRT, the electrical
circuit or loop (i.e., the reentrant circuit) exists within the AV node. With AVRT, a reentry circuit is
formed that often involves four components—the atrium, the normal AV node, the ventricle, and an
accessory pathway, also called a bypass tract (Saksena et al., 2012).

Atrioventricular Nodal Reentrant Tachycardia


AVNRT is the most common SVT (Page et al., 2016). It results from a reentry circuit that uses two
separate pathways leading into the AV node (Peterson, 2018). One pathway conducts impulses rapidly
but has a long refractory period (i.e., slow recovery time). The other pathway conducts impulses slowly
but has a short refractory period (i.e., fast recovery time). The pathways join into a final common path-
way before impulses exit the AV node and continue to the bundle of His.
Typical AVNRT is the most common type of AVNRT and is usually caused by single or multiple
premature atrial complexes (PACs). A PAC, finding the fast pathway refractory, conducts the impulse
to the ventricles using the slow pathway (Mani & Pavri, 2014). If the fast pathway has recovered, the
impulse can conduct rapidly up the fast pathway and depolarize the atria. The impulse can then reen-
ter the slow pathway to conduct again. If this pattern continues, AVNRT is initiated, resulting in a very
rapid and regular ventricular rhythm ranging from 150 to 250 beats/min. A less common type of
AVNRT is called atypical AVNRT. It is often initiated by a premature complex that conducts down
the fast pathway and returns using the slow pathway.
ECG characteristics of AVNRT include the following (Fig. 6.4):
Rhythm: Ventricular rhythm is usually very regular
Rate: 150 to 250 beats/min

Fig. 6.4  ​Atrioventricular nodal reentrant tachycardia.


120 CHAPTER 6  Tachycardias

P waves: P waves are often hidden in the QRS complex; if the ventricles are stimulated first and then
the atria, a negative (i.e., inverted) P wave will appear after the QRS in leads II, III, and aVF; when
the atria are depolarized after the ventricles, the P wave typically distorts the end of the QRS
complex
PR interval: P waves are not seen before the QRS complex; therefore, the PR interval is not measurable
QRS duration: 0.11 second or less unless abnormally conducted
Because AVNRT may be short-lived or sustained, treatment depends on the duration of the tachy-
cardia and severity of the patient’s signs and symptoms. Signs and symptoms may include palpitations,
chest discomfort, dizziness, difficulty breathing, a pounding sensation in the neck, lightheadedness,
anxiety, weakness, or syncope.

Atrioventricular Reentrant Tachycardia


AVRT is caused by the presence of an accessory pathway that serves as a conduit for impulses that
originate from the SA node and allows rapid conduction, bypassing the AV node either on its way to
the ventricles or on its return to the atria, resulting in a reentrant circuit (Mottram & Svenson, 2011)
(Fig. 6.5). Ventricular preexcitation occurs when a supraventricular impulse travels through an acces-
sory pathway and excites the ventricles earlier than would be expected if the impulse traveled only by
way of the normal AV conduction system. The number of atrial impulses reaching the ventricles may
approach 300 to 350 beats/min, which significantly increases the risk of development of VF.

Normal Sinus Rhythm

SA

AV

A
LEAD II
WPW: Sinus Rhythm WPW: Atrioventricular
Reentrant
Tachycardia
SA (AVRT)
SA
BT BT
AV AV

LEAD II LEAD II
B C
Delta Wave P Wave
Fig. 6.5  ​Normal and abnormal conduction pathways. (A) Conduction during sinus rhythm in the normal heart spreads from
the sinoatrial (SA) node to the atrioventricular (AV) node and then down the bundle branches. The jagged line indicates
physiologic slowing of conduction in the AV node. (B) With Wolff-Parkinson-White (WPW) pattern, an abnormal accessory
conduction pathway called a bypass tract (BT) connects the atria and ventricles. With WPW, during sinus rhythm, the elec-
trical impulse is conducted quickly down the bypass tract, preexciting the ventricles before the impulse arrives via the AV
node. Consequently, the PR interval is short and the QRS complex is wide, with slurring at its onset (delta wave). (C) WPW
predisposes patients to develop an atrioventricular reentrant tachycardia (AVRT), in which a premature atrial beat may
spread down the normal pathway to the ventricles, travel back up the bypass tract, and recirculate down the AV node again.
This reentrant loop can repeat itself over and over, resulting in a tachycardia. Notice the normal QRS complex and often
negative P wave in lead II during this type of tachycardia. (From Goldberger, A.L., Goldberger, Z.,D. [2018]. Goldberger’s clinical
electrocardiography, ed 9, Philadelphia, Elsevier.)
CHAPTER 6  Tachycardias 121

Fig. 6.6  ​This rhythm strip shows an example of intermittent preexcitation. The first three beats show preexcitation. This is
followed by abrupt normalization of the QRS complex in the next two beats. The preexcitation pattern returns for the final
three beats. (From Zipes, D.P., Jalife, J. [2000]. Cardiac electrophysiology: From cell to bedside, ed 3, Philadelphia, Saunders.)

The most common form of preexcitation is the Wolff-Parkinson-White (WPW) pattern, which
includes a triad of findings that consist of the following: (1) a short PR interval, (2) a delta wave, and
(3) a wide QRS complex. A delta wave is the initial slurred deflection at the beginning of the QRS
complex and represents the relatively slow ventricular depolarization over the accessory pathway (Mark
et al., 2009). A patient is said to have WPW syndrome when a WPW preexcitation pattern is present
on the ECG, and a tachydysrhythmia occurs that is related to the accessory pathway (Berger et al.,
2016). ECG characteristics of the WPW pattern include the following (Fig. 6.6):
Rhythm: Regular, unless associated with AFib
Rate: Usually 60 to 100 beats/min, if the underlying rhythm is sinus in origin
P waves: Normal and positive in lead II unless WPW is associated with AFib
PR interval: 0.12 second or less if P waves are observed because the impulse travels very quickly across
the accessory pathway, bypassing the usual delay in the AV node
QRS duration: Usually more than 0.12 second; slurred upstroke of the QRS complex (i.e., delta wave)
may be seen in one or more leads
Although some people with AVRT never have symptoms, common signs and symptoms associated
with AVRT and an RVR include anxiety, chest discomfort, dizziness, lightheadedness, palpitations
(common), shortness of breath during exercise, signs of shock, and weakness. Consultation with a
cardiologist is recommended when caring for a patient with AVRT.

WIDE-QRS TACHYCARDIAS
The QRS duration of a wide-QRS tachycardia is 0.12 second or more. Most wide-complex tachycar-
dias are VTs. Some wide-complex tachycardias are SVT with a bundle branch block (BBB) or aberrant
conduction. Still others are ventricular-paced rhythms or a tachycardia with AV conduction associated
with or mediated by an accessory pathway (i.e., preexcited tachycardia). It is best to seek expert con-
sultation when treating a patient who has a wide-complex tachycardia.

Ventricular Tachycardia
VT exists when three or more sequential premature ventricular complexes (PVCs) occur at a rate faster
than 100 beats/min. VT may occur as a short run that lasts less than 30 seconds and spontaneously
ends (i.e., nonsustained VT) (Fig. 6.7). Sustained VT persists for more than 30 seconds and may require

Fig. 6.7  ​Nonsustained ventricular tachycardia.  (From Crawford, M.V., Spence, M.I. [1994]. Commonsense approach to
coronary care, rev ed 6, St Louis, Mosby.)
122 CHAPTER 6  Tachycardias

Fig. 6.8  ​Monomorphic ventricular tachycardia. (From Aehlert, B. [2004]. ECG study cards, St. Louis, Mosby.)

therapeutic interventions to terminate the rhythm. VT may occur with or without pulses, and the
patient may be stable or unstable with this rhythm.
VT, like PVCs, may originate from an ectopic focus in either ventricle. When the QRS complexes
of VT are of the same shape and amplitude, the rhythm is called monomorphic VT. Characteristics of
monomorphic VT include the following (Fig. 6.8):
Rhythm: Ventricular rhythm essentially regular
Rate: 101 to 250 (121 to 250 per some cardiologists) beats/min
P waves: Usually not seen; if present, they have no set relationship with the QRS complexes that appear
between them at a rate different from that of the VT
PR interval: None
QRS duration: 0.12 second or greater; often difficult to differentiate between the QRS and the T wave
When the QRS complexes of VT vary in shape and amplitude from beat to beat, the rhythm is
called polymorphic VT. In PMVT, the QRS complexes appear to twist from upright to negative or
negative to upright and back. PMVT is discussed later in this chapter with irregular tachycardias.
The patient who has sustained monomorphic VT may be stable for long periods. However, when
the ventricular rate is very fast, or when myocardial ischemia is present, monomorphic VT can dete-
riorate to PMVT or VF. Syncope or near-syncope may occur because of an abrupt onset of VT. The
patient’s only warning symptom may be a brief period of lightheadedness.
VT may occur in a patient who has an implantable cardioverter-defibrillator (ICD) in place. If the
rate of the VT is below the programmed tachycardia detection rate, the ICD will not treat it. It is
essential to identify this situation and consult with an expert immediately. VT may be terminated
with the use of the programmer that corresponds with the implanted device.

ACLS Pearl
An SVT with an intraventricular conduction delay may be difficult to distinguish from VT. Keep in
mind that VT is considered a potentially life-threatening dysrhythmia. If you are unsure whether a
regular, wide-QRS tachycardia is VT or SVT with an intraventricular conduction delay, treat the
rhythm as VT until proven otherwise. Obtaining a 12-lead ECG may help differentiate VT from SVT,
but do not delay treatment if the patient is symptomatic.

IRREGULAR TACHYCARDIAS
The severity of signs and symptoms associated with an irregular tachycardia varies depending on the
ventricular rate, how long the rhythm has been present, and the patient’s cardiovascular status. The
patient may be asymptomatic and not require treatment or may experience serious signs and symptoms.
It is best to seek expert consultation when treating a patient who has an irregular tachycardia.

Multifocal Atrial Tachycardia


Multifocal AT (MAT) is an automatic tachycardia that is the result of the random and chaotic firing
of multiple ectopic sites in the atria. ECG characteristics of MAT include the following (Fig. 6.9):
Rhythm: Ventricular rhythm is always irregular as the pacemaker site shifts from the SA node to
ectopic atrial locations or the AV junction
CHAPTER 6  Tachycardias 123

3
III 1 2

Fig. 6.9  ​Multifocal atrial tachycardia. Note the rapidly occurring P waves showing variable shapes and PR intervals. This
fast, irregular rhythm may be mistaken for atrial fibrillation. Arrows with numbers (1–3) above show a segment with multiple
consecutive different P waves.  (From Goldberger, A.L., Goldberger, Z.D. [2018]. Goldberger’s clinical electrocardiography,
ed 9, Philadelphia, Elsevier.)

Rate: Ventricular rate faster than 100 beats/min


P waves: Size, shape, and direction may change from beat to beat; at least three different P-wave
configurations (seen in the same lead) are required for a diagnosis of MAT
PR interval: Varies as the pacemaker site shifts from the SA node to ectopic atrial locations or the AV
junction
QRS duration: 0.11 second or less unless abnormally conducted
MAT is most often seen in older adults with advanced pulmonary disease. Because MAT can be
challenging to treat, it is best to consult a cardiologist before starting treatment.

Atrial Flutter
Atrial flutter is a macroreentrant AT in which an irritable site within the atria fires regularly at a very
rapid rate. Typical atrial flutter involves a reentry circuit around the tricuspid valve in the right atrium.
Characteristics of atrial flutter include the following (Fig. 6.10):
Rhythm: Atrial regular; ventricular regular or irregular depending on AV conduction and blockade
Rate: The atrial rate generally ranges from 240 to 300 beats/min; the ventricular rate varies and is de-
termined by AV blockade; the ventricular rate will usually not exceed 180 beats/min as a result of
the intrinsic conduction rate of the AV junction
P waves: No identifiable P waves; saw-toothed “flutter” waves are present
PR interval: Not measurable
QRS duration: 0.11 second or less but may be widened if flutter waves are buried in the QRS complex
or if abnormally conducted
The severity of signs and symptoms associated with atrial flutter vary, depending on the ventricular
rate, how long the rhythm has been present, and the patient’s cardiovascular status. It is best to consult
a cardiologist when considering treatment options.

Atrial Fibrillation
AFib is the most common cardiac dysrhythmia in the United States. It occurs because of altered
automaticity in one or several rapidly firing sites in the atria or reentry involving one or more circuits
in the atria. ECG characteristics of AFib include the following (Fig. 6.11):
Rhythm: Ventricular rhythm usually irregularly irregular
Rate: Atrial rate usually 400 to 600 beats/min; ventricular rate variable

Fig. 6.10  ​Atrial flutter with ST-segment depression. (From Aehlert, B. [2018]. ECGs made easy, ed 6, St. Louis, Elsevier.)
124 CHAPTER 6  Tachycardias

II

MCLI

Fig. 6.11  ​Atrial fibrillation. (From Aehlert, B. [2004]. ECG study cards, St. Louis, Mosby.)

II

Fig. 6.12  ​Atrial fibrillation with a rapid ventricular response. (From Aehlert, B. [2004]. ECG study cards, St. Louis, Mosby.)

P waves: No identifiable P waves; fibrillatory waves present; erratic, wavy baseline


PR interval: Not measurable
QRS duration: 0.11 second or less unless abnormally conducted
Atrial flutter or AFib that has an RVR is described as uncontrolled (Fig. 6.12). Atrial flutter or
AFib with a rapid ventricular response is commonly called Aflutter with RVR or AFib with RVR.
About 25% of patients who experience AFib are asymptomatic (Morady & Zipes, 2019). When
symptoms are present, the severity of the signs and symptoms associated with AFib vary with the
ventricular rate (either too fast or too slow), the loss of effective atrial contraction, sympathetic activa-
tion, and the beat-to-beat variability in ventricular filling ( January et al., 2014). It is best to consult a
cardiologist when considering specific therapies.

Polymorphic Ventricular Tachycardia


With PMVT, the QRS complexes vary in shape and amplitude from beat to beat and appear to twist
from upright to negative or negative to upright and back, resembling a spindle. The ECG characteris-
tics of PMVT include the following (Fig. 6.13):
Rhythm: Ventricular rhythm may be regular or irregular
Rate: Ventricular rate 150 to 300 beats/min; typically 200 to 250 beats/min
P waves: None
PR interval: None

Fig. 6.13  ​When the QRS complexes of ventricular tachycardia (VT) vary in shape and amplitude, the rhythm is termed
polymorphic VT. (From Aehlert, B. [2004]. ECG study cards, St. Louis, Mosby.)
CHAPTER 6  Tachycardias 125

QRS duration: 0.12 second or more; there is a gradual alteration in the amplitude and direction of the
QRS complexes; a typical cycle consists of 5 to 20 QRS complexes
Several types of PMVT and their possible causes have been identified. PMVT that occurs in the
presence of a long QT interval (generally, 0.50 second or more) is called torsades de pointes (TdP). A
long QT interval may be congenital, acquired (typically caused by antiarrhythmic drug use or hypoka-
lemia, which are typically associated with bradycardia), or idiopathic (neither familial nor with an
identifiable acquired cause). PMVT that occurs in the presence of a normal QT interval is simply re-
ferred to as polymorphic VT or normal-QT PMVT.
The signs and symptoms associated with PMVT are usually related to the decreased cardiac output
that occurs because of the fast ventricular rate. Signs of shock are often present. The patient may expe-
rience a syncopal episode or seizures. The rhythm may occasionally terminate spontaneously and recur
after several seconds or minutes, or it may deteriorate to VF. The patient with sustained PMVT is
rarely hemodynamically stable.

PATIENT ASSESSMENT
If a patient presents with a tachycardia, perform a primary and secondary assessment, and evaluate how
the patient is tolerating the rhythm. Ask a team member to attach a pulse oximeter and cardiac monitor,
and obtain the patient’s vital signs. In addition to obtaining a SAMPLE (signs and symptoms, allergies,
medications, past history, last oral intake, events prior) history, ask the patient who is experiencing a
tachycardia about the number of episodes, their frequency and duration, the patient’s symptoms during
episodes, and possible triggers. Ask the patient what they have tried to relieve symptoms. Often, patients
who have frequent episodes of SVT learn how to perform vagal maneuvers and know when to seek
medical attention. Ask about any recent illnesses, the presence of comorbid conditions, current medica-
tions, and recent changes in medications, diet, or activity.
Signs and symptoms that may be associated with RVRs may include the following:
• Chest pain or pressure • Nervousness, anxiety
• Dizziness • Palpitations (common)
• Dyspnea • Signs of shock
• Lightheadedness • Syncope
• Nausea • Weakness

ACLS Pearl
In a patient with coronary artery disease, any tachycardia can cause problems. The heart’s de-
mand for oxygen increases as the heart rate increases. As the heart rate increases, there is less
time for the ventricles to fill and less blood for the ventricles to pump out with each contraction,
which can lead to decreased cardiac output. Because the coronary arteries fill when the ventricles
are at rest, rapid heart rates decrease the time available for coronary artery filling. Chest discom-
fort can result if the supplies of blood and oxygen to the heart are inadequate.

THERAPEUTIC INTERVENTIONS
When a patient presents with signs and symptoms related to a tachycardia, ask yourself the following
questions. The answers to these questions can help guide your treatment decisions.
• Is the patient asymptomatic, symptomatic but stable, symptomatic and unstable, or pulseless?
• Is the QRS wide or narrow? If it is wide, is it monomorphic or polymorphic?
• Is the ventricular rhythm regular or irregular?
Give supplemental oxygen if oxygenation is inadequate and assist breathing if ventilation is inade-
quate. Establish vascular access and obtain a 12-lead ECG, if available. It is also important to obtain a
12-lead when performing any intervention to terminate the tachycardia (e.g., vagal maneuvers, medica-
tion administration, cardioversion). Attempt to identify and treat the underlying cause of the patient’s
dysrhythmia using the Hs and Ts while providing patient care.
If the patient is asymptomatic, no treatment is necessary, but monitoring and follow-up may
be warranted. For example, the patient’s provider may order ambulatory monitoring, exercise stress
testing, or both to capture and identify the dysrhythmia and detect possible precipitating causes.
126 CHAPTER 6  Tachycardias

If the patient is stable but symptomatic, and the symptoms are the result of the rapid heart rate,
medications are usually administered to slow or terminate the dysrhythmia. If the tachycardia produces
serious signs and symptoms, typically with heart rates of 150 beats/min or more, the patient is consid-
ered unstable. Unstable patients who have a pulse and serious signs and symptoms caused by the
tachycardia should receive immediate synchronized cardioversion. Synchronized cardioversion was
discussed in Chapter 3. The tachycardia algorithm is shown in Fig. 6.14.

• Obtain H&P, evaluate ECG, assess vital signs, and assess patient stability
• Determine if heart rate is appropriate for patient’s clinical condition

Search for and treat reversible causes

• Maintain an open airway, give oxygen if indicated, assist breathing as needed


• Continue monitoring cardiac rhythm, oxygen saturation, and vital signs
• Establish vascular access, obtain a 12-lead ECG (if available), and attempt to identify and treat the underlying
cause of the dysrhythmia using the Hs and Ts while providing care

Key questions

• Is the patient asymptomatic, symptomatic but stable, symptomatic and unstable, or pulseless? (If pulseless,
go to cardiac arrest algorithm)
• Is the QRS wide or narrow? If it is wide, is it monomorphic or polymorphic?
• Is the ventricular rhythm regular or irregular?

Is the patient asymptomatic, stable, or unstable?

Asymptomatic Symptomatic but stable Symptomatic and unstable

Continue Narrow-QRS Wide-QRS • Perform synchronized


monitoring • Establish vascular access • Establish vascular cardioversion
• Obtain 12-lead ECG (do access • Consider sedation
not delay therapy) • Obtain 12-lead ECG (do • Consider adenosine if
• Vagal maneuvers not delay therapy) regular narrow-QRS
• Give adenosine (if • Consider adenosine if tachycardia
regular) regular and
• Beta-blocker or calcium monomorphic
blocker • Consider
• Consider expert consult antiarrhythmic infusion
• Consider expert consult

Assessment Medications Medications (cont’d) Synchronized cardioversion


• Heart rates above 150 • Adenosine rapid IV/IO • Procainamide • Use energy doses
beats/min and push: 6 mg first dose, 12 maintenance infusion: 1 to recommended by the
accompanied by moderate mg second dose after 1 to 4 mg/min; avoid if defibrillator manufacturer
to severe symptoms 2 min; follow each dose prolonged QT or heart • Typical initial biphasic
typically require with normal saline flush failure energy doses:
treatment Antiarrhythmics (for stable • Amiodarone IV infusion: • Narrow-regular QRS: 50 to
• Examples of moderate to wide-QRS tachycardia) 150 mg over 10 min. May 100 J
severe signs and • Procainamide IV infusion: repeat every 10 min if • Narrow-irregular QRS: 120
symptoms include acutely 20 to 50 mg/min IV (or rhythm persists. After to 200 J monophasic
altered mental status, 100 mg every 5 min) until conversion, follow with an • Wide-regular QRS: 100 J
acute heart failure, the dysrhythmia resolves, infusion of 10 to 50 mg/hr • Wide-irregular QRS: do
hypotension, ongoing hypotension ensues, the over 24 hours. NOT sync, use
ischemic chest discomfort, QRS prolongs by more • Sotalol IV infusion: 100 mg defibrillation dose
pulmonary congestion, than 50% of original (1.5 mg/kg) IV over 5 min;
shortness of breath, and width, or a total avoid if prolonged QT
signs of shock cumulative dose of 17
mg/kg is administered
ECG = Electrocardiogram; H&P = history and physical examination; IO = intraosseous; IV = intravenous; J = joules.
(Sources: Link et al., 2015; Panchal et al., 2020.)

Fig. 6.14  Adult tachycardia with a pulse algorithm.


CHAPTER 6  Tachycardias 127

ACLS Pearl
The management of patients who present with a tachycardia is often involved. As an advanced
cardiac life support provider, you need to recognize when to seek expert advice concerning
rhythm interpretation, medications, or patient-management decisions.

Narrow-QRS Tachycardias
Treatment options for symptomatic patients with a regular narrow-QRS tachycardia include vagal
maneuvers, pharmacologic therapy, and synchronized cardioversion.
If the patient is stable but symptomatic, attempt a vagal maneuver if there are no contraindications.
Before performing a vagal maneuver, ensure that oxygen, suction, a defibrillator with pacing capability,
and antiarrhythmic medications are at the bedside. Continuously monitor and record the patient’s
ECG when performing a vagal maneuver. A 12-lead ECG recording is desirable.
Vagal maneuvers are methods used to slow impulse conduction through the AV node and, subse-
quently, the ventricular rate in dysrhythmias that require the AV node to sustain the rhythm. Common
vagal maneuvers include the following:
• Carotid sinus massage (CSM), which is also called carotid sinus pressure. This procedure is per-
formed with the patient’s neck extended. The carotid pulse is palpated and then steady pressure is
applied to the right or left carotid sinus for 5 to 10 seconds (Fig. 6.15). Some clinicians select the
side that is used first based on the nondominant hemisphere of the patient’s brain (Ceylan et al.,
2019). If CSM is unsuccessful, it can be repeated, usually on the opposite side, after 1 to 2 minutes.
Avoid CSM in older adults and in patients who have a history of stroke, known carotid artery
stenosis, or a carotid artery bruit on auscultation. Simultaneous, bilateral carotid pressure is not
recommended.
• The Valsalva maneuver is the forced expiration of air against a closed glottis for 15 seconds before
resuming normal breathing. Examples include bearing down as though to have a bowel movement,
taking a deep breath and then having the patient put their thumb in their mouth with closed lips
and attempting to exhale without expelling any air, asking the patient to blow through an occluded
straw, or asking the patient to blow into a 10 mL syringe with enough force to move the plunger.
Valsalva maneuvers are contraindicated in patients with recent MI, aortic stenosis, carotid artery
stenosis, glaucoma, or retinopathy.
Most providers limit the number of vagal maneuver attempts to two before moving on to other
interventions (Sohinki & Obel, 2014). Performing a Valsalva maneuver or CSM with postural
modifications, such as supine positioning and passive leg raising during the procedure, may increase
the probability of successful rhythm conversion (Appelboam et al., 2015; Minczak & Laub, 2019)
(Fig. 6.16).
Although AT will rarely stop with vagal maneuvers, they are used to try to identify the mechanism
of the SVT (i.e., automatic, triggered activity, reentry) (Page et al., 2016). Vagal maneuvers will not

External
carotid artery
Angle of
Internal mandible
cartoid
artery

Carotid
sinus
Thyroid
cartilage

Sternocleidomastoid
muscle

Fig. 6.15  ​Location of the carotid sinus. (From Roberts, J.R., Custalow, C.B., Thomsen, T.W. [2019]. Roberts and Hedges’
clinical procedures in emergency medicine and acute care, ed 7, Philadelphia, Elsevier.)
128 CHAPTER 6  Tachycardias

Fig. 6.16  ​Passively raising the patient’s legs during the Valsalva maneuver or carotid sinus massage is recommended and
may increase the probability of successful rhythm conversion. (From Roberts, J.R., Custalow, C.B., Thomsen, T.W. [2019].
Roberts and Hedges’ clinical procedures in emergency medicine and acute care, ed 7, Philadelphia, Elsevier.)

usually convert AFib or atrial flutter because the reentry circuit is located in the atria, not the AV node;
however, these maneuvers may help to identify the rhythm by momentarily slowing AV conduction
and revealing underlying flutter or fibrillatory waves before the ventricular response returns to its
former rate.
Pharmacologic therapy is indicated if vagal maneuvers fail or if they are contraindicated. Adenosine
(see Chapter 3) is the initial drug of choice for regular narrow-QRS tachycardias. If adenosine fails to
convert the rhythm, or if the dysrhythmia recurs, longer-acting medications that slow AV node con-
duction, such as calcium blockers (e.g., verapamil, diltiazem) or beta blockers (see Chapter 3) may be
used to slow the ventricular rate or terminate the dysrhythmia. Be sure to continuously monitor the
patient’s ECG during the administration of these medications.
If the patient is symptomatic and the tachycardia is causing persistent signs of hemodynamic com-
promise, sedation should be administered (if time and conditions permit), and synchronized cardiover-
sion should be performed using the energy settings recommended by the defibrillator manufacturer. In
clinical practice, healthcare practitioners sometimes give a trial of adenosine while preparations are
made for cardioversion.

ACLS Pearl
Medications such as adenosine, diltiazem, and verapamil should be avoided for preexcited AFib
or atrial flutter that conducts to the ventricles using both the AV node and an accessory pathway
(Link et al., 2015). These medications are contraindicated because they slow or block conduction
across the AV node. Still, they may speed up conduction through the accessory pathway, thereby
resulting in a further increase in the ventricular rate. In these patients, antiarrhythmics, such as
amiodarone, procainamide, or sotalol can be considered.

Wide-QRS Tachycardias
If the patient is stable, the QRS is wide, the rhythm is regular, and the QRS complexes are of simi-
lar shape (i.e., monomorphic), consider administering adenosine to try to identify the origin of the
tachycardia while continuously monitoring the patient’s ECG (Link et al., 2015). With few excep-
tions, adenosine will generally have no effect if the rhythm is VT. If the wide-QRS rhythm is actu-
ally SVT with aberrancy, adenosine administration will usually result in a transient slowing of the
ventricular rate or conversion to sinus rhythm. Verapamil is contraindicated for wide-QRS tachycar-
dias unless the dysrhythmia is known to be of supraventricular origin and not being conducted by
CHAPTER 6  Tachycardias 129

an accessory pathway (Panchal et al., 2020). For the pharmacologic termination of a stable wide-
QRS tachycardia that is most likely VT, procainamide, amiodarone, or sotalol (see Chapter 3) can
be used. These medications are considered first-line antiarrhythmics for monomorphic VT, and they
have complex mechanisms of action. They are used for both atrial and ventricular dysrhythmias.
Procainamide should be avoided if the patient has a prolonged QT interval or signs of heart failure.
Sotalol should also be avoided if the patient has a prolonged QT interval. Although lidocaine is a
ventricular antiarrhythmic, it is considered a second-line antiarrhythmic for the management of
monomorphic VT because it is reportedly less effective for the termination of VT than the first-line
agents. If the decision is made to administer procainamide, amiodarone, or sotalol, consult with an
expert before another drug is administered (Link et al., 2015). If the diagnosis of SVT cannot be
proved or cannot be made easily, then treat the patient as if VT is present.
If the patient is symptomatic and unstable, administer sedation if time and conditions permit, and
perform synchronized cardioversion using the energy settings recommended by the defibrillator
manufacturer.

Irregular Tachycardias
The treatment of MAT is directed at the underlying cause (e.g., hypoxia, acidosis, electrolyte dis-
turbances). Because MAT does not involve reentry through the AV node, it is unlikely that vagal
maneuvers or giving adenosine will terminate the rhythm. Metoprolol is useful for rate control, but
it should be avoided in patients with impaired left ventricular function or bronchospastic pulmo-
nary disease; in such cases, amiodarone may be preferred. MAT is unresponsive to cardioversion
(Link et al., 2015).
With AFib and atrial flutter, ineffective atrial contraction increases the patient’s risk of atrial throm-
bus formation, leading to stroke, peripheral thromboembolism, or both ( January et al., 2014). The
CHA2DS2-VASc (congestive heart failure, hypertension, age 75 years or older, diabetes mellitus, prior
stroke or transient ischemic attack or thromboembolism, vascular disease, age 65 to 74 years, sex cat-
egory) score is recommended to assess the risk of ischemic stroke for adults with nonvalvular AFib or
atrial flutter ( January et al., 2019).
The two primary treatment strategies used to control symptoms associated with atrial flutter or
AFib are rate control and rhythm control. With a rate control strategy, the patient remains in atrial
flutter or AFib, but the ventricular rate is controlled (i.e., slowed) to decrease acute symptoms, reduce
signs of ischemia, and reduce or prevent signs of heart failure from developing. Rate control is
achieved using medications that prolong the refractory period of the AV node (e.g., beta blockers or
calcium blockers) or catheter ablation. These medications must be used with caution in patients with
hypotension or heart failure. Beta blockers are preferred in patients with cardiomyopathy or ischemic
heart disease, and following surgical procedures; verapamil and diltiazem are preferred in patients
with reactive airway disease (Issa et al., 2019). IV amiodarone can be useful for rate control in
critically ill patients without preexcitation, but it is less effective than nondihydropyridine calcium
blockers ( January et al., 2014).
Rhythm control (i.e., termination of AFib or atrial flutter and restoring sinus rhythm) is achieved
using a combination of approaches including pharmacologic or electric cardioversion and catheter
ablation. Because pharmacologic or electric cardioversion carries a risk of thromboembolism, antico-
agulation is recommended before attempting the conversion of AFib or atrial flutter to sinus rhythm
when the duration of the dysrhythmia exceeds 48 hours or when the duration of the dysrhythmia is
unknown ( January et al., 2019). Shorter durations of AFib do not exclude the possibility of thrombo-
embolism (Link et al., 2015).
If the patient with AFib or atrial flutter is symptomatic and unstable, administer sedation if time and
conditions permit and perform synchronized cardioversion. Some studies have shown anteroposterior
electrode placement superior to anterolateral placement. If cardioversion is attempted using one manner
of electrode placement and fails, experts recommend using the alternative placement before attempting
another shock ( January et al., 2014). Use the energy settings recommended by the defibrillator manu-
facturer. Anticoagulation should be started as soon as possible and continued for at least 4 weeks after
cardioversion unless contraindicated ( January et al., 2019). If the patient has risk factors for stroke in
association with AFib, most clinicians administer anticoagulation during the cardioversion and for the
subsequent 4 weeks (Zimetbaum, 2020).
130 CHAPTER 6  Tachycardias

It is best to seek expert consultation when treating the patient with PMVT because of the diverse
mechanisms of PMVT, for which there may or may not be clues as to its specific cause at the time of
the patient’s presentation. Treatment options vary and can be contradictory. For example, a medication
that may be appropriate for the treatment of TdP may be contraindicated when treating another form
of PMVT. In general, if the patient is symptomatic because of the tachycardia, treat ischemia (if it is
present) and correct electrolyte abnormalities. If the QT interval is prolonged, the cause of the long
QT should be determined and corrected, if possible. Discontinue any medications that the patient may
be taking that prolong the QT interval. In general, IV magnesium is the initial treatment for the stable
patient with PMVT associated with a prolonged QT interval (i.e., TdP). Beta blockers may be useful
for certain forms of PMVT (e.g., ischemic PMVT, congenital long-QT syndrome PMVT, catechol-
aminergic PMVT). Amiodarone and beta blockers may be effective for PMVT with a normal QT
interval. PMVT that is associated with Brugada syndrome may be responsive to isoproterenol (Link
et al., 2015). Adenosine should not be given for PMVT because it may cause degeneration of the
dysrhythmia to VF (Panchal et al., 2020).
Because the QRS complexes of PMVT are disorganized (i.e., they differ in amplitude and direc-
tion), synchronized cardioversion is generally not possible when managing an unstable patient with
this rhythm. Therefore if the patient with PMVT is unstable or has no pulse, proceed with defibrilla-
tion as for VF (Panchal et al., 2020).
CHAPTER 6  Tachycardias 131

STOP AND REVIEW


Identify one or more choices that best complete the statement or answer the question.

_ ___ 1. Which of the following are examples of irregular tachycardias?


a. MAT
b. AFib
c. PMVT
d. Atrial flutter
e. Junctional rhythm
f. Sinus tachycardia

____ 2. A 62-year-old man is complaining of palpitations that came on suddenly after


walking up a short flight of stairs. His symptoms have been present for about 20
minutes. He denies chest pain and is not short of breath. His skin is warm and dry;
breath sounds are clear. Blood pressure is 144/88 millimeters of mercury (mm Hg),
pulse 186, ventilations 18 breaths/min. The cardiac monitor reveals sustained
monomorphic VT. An IV has been established. Which of the following medications
may be considered in this situation?
a. Sotalol
b. Atropine
c. Morphine
d. Dopamine
e. Furosemide
f. Amiodarone
g. Nitroglycerin
h. Procainamide

_ ___ 3. Diltiazem may be used:


a. To control the ventricular rate in AFib and atrial flutter.
b. As a diagnostic tool in the treatment of preexcited AFib.
c. To increase the ventricular rate in second- or third-degree AV block.
d. To convert wide-QRS tachycardia of uncertain origin to a sinus rhythm.

____ 4. A 56-year-old man is complaining of a rapid pounding sensation in his neck that
came on suddenly 20 minutes ago while eating lunch. Although he is clearly anx-
ious, he denies chest pain or shortness of breath. His blood pressure is 134/82 mm
Hg, pulse 180, ventilations 18 breaths/min. His oxygen saturation level (SpO2) is
97% on room air. The cardiac monitor shows a regular narrow-QRS tachycardia
without visible P waves. Which of the following actions should be taken at this
time?
a. Establish IV access.
b. Attempt a vagal maneuver.
c. Administer amiodarone IV bolus.
d. Perform synchronized cardioversion.

_ ___ 5. Which of the following are endpoints for procainamide administration?


a. The dysrhythmia is suppressed.
b. The patient becomes hypotensive.
c. A total cumulative dose of 3 mg/kg is administered.
d. The QRS widens by more than 50% of its original width.
132 CHAPTER 6  Tachycardias

____ 6. When administering amiodarone for dysrhythmias other than a shockable cardiac
arrest rhythm, the correct initial IV dosage is:
a. 1 to 1.5 mg/kg
b. 150 mg IV over 10 minutes
c. 0.5 mg every 3 to 5 minutes
d. 1 to 2 g diluted in 50 to 100 mL D5W over 15 minutes

____ 7. A 29-year-old man presents with acute altered mental status. His blood pressure is
50/P, ventilations 8 breaths/min. His SpO2 on room air is 92% and his heart rate is
250 beats/min. His skin is pale, cool, and clammy. The cardiac monitor reveals
PMVT. Your best course of action in this situation will be to:
a. Establish IV access.
b. Give adenosine rapid IV push.
c. Give epinephrine 1 mg rapid IV push.
d. Give diltiazem IV push over 2 minutes.
e. Consider sedation and defibrillate immediately.
f. Perform immediate synchronized cardioversion.
g. Assist ventilations and give supplemental oxygen.

____ 8. Which of the following may be used in the management of a symptomatic but
stable patient with a regular wide-QRS tachycardia?
a. Sotalol
b. Verapamil
c. Amiodarone
d. Procainamide

____ 9. With which type of tachycardia does the impulse begin above the ventricles but
travels via a pathway other than the AV node and bundle of His?
a. AT
b. AVRT
c. AVNRT
d. Sinus tachycardia

____ 10. A 68-year-old man is complaining of chest pain. His blood pressure is 50/32 mm
Hg, his pulse is 230 beats/min, and his ventilatory rate is 6 breaths/min. The cardiac
monitor reveals a regular, narrow-QRS tachycardia. Your best course of action will be
to:
a. Defibrillate immediately.
b. Prepare a dopamine infusion.
c. Begin transcutaneous pacing.
d. Sedate and perform synchronized cardioversion.

____ 11. Which of the following may be effective in the management of PMVT with a
normal QT interval?
a. Adenosine
b. Magnesium
c. Amiodarone
d. Beta blockers

_ ___ 12. Which of the following reflects the correct initial dosage of adenosine?
a. 6 mg IV bolus over 1 to 2 minutes
b. 3 mg rapid IV bolus over 1 to 3 seconds followed by a 20 mL saline flush
c. 6 mg rapid IV bolus over 1 to 3 seconds followed by a 20 mL saline flush
d. 12 mg rapid IV bolus over 1 to 3 seconds followed by a 20 mL saline flush
CHAPTER 6  Tachycardias 133

_ ___ 13. Select the correct statements regarding vagal maneuvers.


a. CSM should be avoided in older patients.
b. Application of a cold stimulus to the face is often effective in adults.
c. An ECG monitor should be used when a vagal maneuver is performed.
d. When CSM is performed, steady pressure is applied to the carotid sinus for
20 to 30 seconds.
e. Passive leg raising when performing a vagal maneuver may increase the probability
of successful rhythm conversion.

____ 14. A 72-year-old man reports feeling lightheaded. His blood pressure is 110/64 mm
Hg, his pulse is 190 beats/min, and his ventilatory rate is 16 breaths/min. The
patient denies chest pain. Breath sounds are clear. The cardiac monitor reveals
monomorphic VT. Recommended treatment in this situation includes which of the
following?
a. Establish vascular access.
b. Give adenosine 6 mg rapidly IV.
c. Give epinephrine 1 mg slowly IV.
d. Administer sublingual nitroglycerin.
e. Give procainamide 20 to 50 mg/min IV.
f. Begin cardiopulmonary resuscitation (CPR) and defibrillate immediately.
134 CHAPTER 6  Tachycardias

C A S E S T U DY 6.1
Identify one or more choices that best complete the statement or answer the question.

You and your team have been summoned to care for a 72-year-old man complaining of palpitations
and chest heaviness. Emergency equipment, including a biphasic manual defibrillator, is available to
you. As you work through this case study, remember that there may be alternative actions that are
perfectly acceptable, yet not discussed here.

____ 1. As you approach the patient, you observe that he is sitting upright on a stretcher.
He appears anxious, his breathing is not labored, and his skin is pink. The patient
speaks hurriedly, telling you that his heart is “racing and feels like it is going to
pound out of my chest.” Select the correct statements about this patient’s general
impression findings.
a. Appearance is normal.
b. Appearance is abnormal.
c. Breathing is normal.
d. Breathing is abnormal.
e. Circulation is normal.
f. Circulation is abnormal.

____ 2. You have asked a team member to perform a primary and secondary assessment.
Which of the following actions should occur at the same time?
a. Obtain the patient’s vital signs.
b. Attach a pulse oximeter and cardiac monitor.
c. Instruct a team member to establish IV access.
d. Ask the patient to bear down as if having a bowel movement.

____ 3. The patient’s BP is 142/90 mm Hg, his heart rate is 214 beats/min, and his
ventilatory rate is 18 breaths/min. Breath sounds are clear and equal. The patient’s
SpO2 on room air is 96%. His skin is warm, pink, and dry. The patient has been
placed on the cardiac monitor (Fig. 6.17). What is the rhythm shown on the
monitor?
a. AFib
b. Sinus tachycardia
c. Monomorphic VT
d. Narrow-QRS tachycardia

II

Fig. 6.17  Case Study 6.1, question 3. (From Aehlert, B. [2004]. ECG study cards, St. Louis, Mosby.)
CHAPTER 6  Tachycardias 135

____ 4. The following information has been obtained from the patient:
Signs/Symptoms: Palpitations and chest “heaviness” began 1 hour ago when the
patient began feeling as if everything was spinning around him and felt a heavi-
ness in his chest at the same time; says this has happened once before but only
lasted a minute or two; rates his chest discomfort 1/10
Allergies: None
Medications: Lisinopril, hydrochlorothiazide
Past history: Hypertension
Last oral intake: Lunch 1 hour ago
Events prior: Patient was walking from his kitchen to his living room when his
symptoms began
The physical examination reveals no abnormalities. An IV has been started in
the right antecubital vein. Based on the information provided, how would you
categorize this patient’s condition?
a. Asymptomatic
b. Stable but symptomatic
c. Symptomatic and unstable
d. Pulseless

_ ___ 5. Next steps should include which of the following?


a. Obtain a 12-lead ECG.
b. Attempt a vagal maneuver.
c. Obtain a cardiology consult.
d. Obtain laboratory studies as ordered.
e. Administer procainamide 20 to 50 mg/min IV.
f. Administer amiodarone 150 mg IV over 10 minutes.
g. Attempt to identify and treat the underlying cause of the dysrhythmia.

____ 6. The patient’s condition and cardiac rhythm remain unchanged despite your efforts.
Which of the following actions should be taken at this time?
a. Give adenosine 6 mg rapid IV bolus.
b. Give diltiazem 15 to 20 mg IV over 2 minutes.
c. Give verapamil 2.5 to 5 mg IV over 2 minutes.
d. Prepare to perform synchronized cardioversion.

____ 7. After administering the initial dose of the ordered medication, a team member
tells you that the patient’s BP is now 74/52 mm Hg. A weak pulse is present, and
the patient’s skin is cool, pale, and clammy. The rhythm on the monitor remains
unchanged. What actions should be taken at this time?
a. Administer sedation.
b. Prepare to defibrillate.
c. Begin high-quality CPR.
d. Administer epinephrine 1 mg IV bolus.
e. Prepare to perform synchronized cardioversion.

_ ___ 8. What initial biphasic energy setting is typically used in this situation?
a. 25 J to 50 J
b. 50 J to 100 J
c. 120 J to 200 J
d. 200 J to 360 J
136 CHAPTER 6  Tachycardias

____ 9. When synchronized cardioversion is indicated for a patient with a regular narrow-
QRS tachycardia, which of the following should be observed to ensure that the
procedure is performed safely?
a. Administer sedation if time and the patient’s condition permit.
b. Place adhesive pads on the patient’s bare chest.
c. Press the “Sync” control on the defibrillator.
d. Ensure the machine is marking each QRS complex.
e. Charge the machine to 360 joules.
f. Ensure that everyone is clear of the patient before delivering a shock.
g. Depress the “Shock” control until the shock is delivered.

____ 10. Sedation was administered and a shock was delivered as instructed. The cardiac
monitor reveals this rhythm (Fig. 6.18). What is the rhythm?
a. AFib
b. Atrial flutter
c. Sinus rhythm
d. Junctional rhythm

II

Fig. 6.18  Case Study 6.1, question 10. (From Aehlert, B. [2004]. ECG study cards, St. Louis, Mosby.)

____ 11. The patient is awake and alert. Strong carotid and radial pulses are present. His
ventilatory rate is 14 breaths/min. Breath sounds are clear and equal. The patient’s
BP is 108/88 mm Hg, and his SpO2 is 98% on room air. Which of the following
should be done now?
a. Request a team debriefing.
b. Closely monitor the patient’s vital signs.
c. Arrange for patient transfer for continued care.
d. Repeat the primary and secondary assessments.
e. Review the results of the patient’s 12-lead ECG and laboratory studies.
CHAPTER 6  Tachycardias 137

CASE STUDY 6.2


Identify one or more choices that best complete the statement or answer the question.

You and your team have been summoned to care for a 61-year-old man with dizziness and difficulty
breathing. Emergency equipment, including a biphasic manual defibrillator, is available to you. As you
work through this case study, remember that there may be alternative actions that are perfectly accept-
able, yet not discussed here.

____ 1. The patient is sitting upright on a stretcher and is aware of your approach. His
breathing is slightly labored, and his skin is pale. Select the correct statements
about this patient’s general impression findings.
a. Appearance is normal.
b. Appearance is abnorm.al
c. Breathing is normal.
d. Breathing is abnormal.
e. Circulation is normal.
f. Circulation is abnormal.

____ 2. As you begin your assessment, you find that the patient is alert and oriented
to person, place, time, and event. He reports several episodes of dizziness since
5:30 am today and says that he can feel his heart beating faster than normal during
these episodes. The patient is allergic to codeine. He has a history of high choles-
terol and hypertension, for which he takes Lipitor and lisinopril. As you continue
your assessment, you should direct your team to carry out which of the following
actions?
a. Establish IV access.
b. Obtain the patient’s vital signs.
c. Obtain a portable chest radiograph.
d. Attach a pulse oximeter and cardiac monitor.
e. Order laboratory studies and obtain a 12-lead ECG.

____ 3. The patient’s vital signs are as follows: Temperature 97.8° F (36.6° C);
BP 63/40 mm Hg; heart rate 150 beats/min; and ventilatory rate 20 breaths/
min. Breath sounds are clear and equal, and his skin is cool, pale, and dry.
The patient’s SpO2 on room air is 88% and he has been placed on the cardiac
monitor (Fig. 6.19). This rhythm is:
a. AVRT
b. PMVT
c. AVNRT
d. Monomorphic VT

Fig. 6.19  Case Study 6.2, question 3. (From Aehlert, B. [2004]. ECG study cards, St. Louis, Mosby.)
138 CHAPTER 6  Tachycardias

____ 4. Based on the information provided, how would you categorize this patient’s
condition?
a. Asymptomatic
b. Stable but symptomatic
c. Symptomatic and unstable
d. Pulseless

____ 5. An IV has been started in the left antecubital vein and supplemental oxygen is
being administered. Next steps should include which of the following?
a. Request a cardiology consult.
b. Apply adhesive pads to the patient’s chest.
c. Ask the patient to perform a vagal maneuver.
d. Ask a team member to prepare a dopamine infusion.

_ ___ 6. Which of the following actions should be taken at this time?


a. Administer sedation.
b. Defibrillate immediately.
c. Attempt a vagal maneuver.
d. Direct your team to begin CPR.
e. Perform synchronized cardioversion.
f. Administer procainamide 20 to 50 mg/min IV.
g. Administer amiodarone 150 mg IV over 10 minutes.

____ 7. After performing the actions you requested, a team member alerts you to a change
on the cardiac monitor (Fig. 6.20). The rhythm shown is:
a. AFib
b. PMVT
c. Complete AV block
d. Sinus rhythm with a run of monomorphic VT

Fig. 6.20  Case Study 6.2, question 7. (From Aehlert, B. [2004]. ECG study cards, St. Louis, Mosby.)

_ ___ 8. Which of the following actions should be taken at this time?


a. Repeat the primary assessment.
b. Obtain the patient’s vital signs.
c. Review the patient’s 12-lead ECG.
d. Request and review the patient’s laboratory results.

____ 9. The patient’s vital signs are as follows: BP 132/84 mm Hg; heart rate 100 beats/
min; and ventilatory rate 16 breaths/min. Breath sounds are clear and equal, and
his skin is warm, pink, and dry. The patient’s SpO2 is 98% on supplemental oxygen.
Which of the following actions is reasonable at this time?
a. Administer adenosine 6 mg rapid IV push.
b. Administer lidocaine 1 to 1.5 mg/kg IV push.
c. Administer procainamide 20 to 50 mg/min IV.
d. Arrange for patient transfer for continued care.
e. Administer magnesium 1 to 2 g IV over 15 minutes.
CHAPTER 6  Tachycardias 139

STOP AND REVIEW  ANSWERS


1. A, B, C, D. Examples of irregular tachycardias include AFib, atrial flutter, MAT, and PMVT. Sinus
tachycardia is typically regular. A junctional rhythm is not a tachycardia.

2. A, F, H. From the information provided, the patient appears to be clinically stable at this time. Procain-
amide would be appropriate to consider in this situation. Acceptable alternatives include amiodarone and
sotalol. Dopamine increases the force of myocardial contraction, heart rate, and blood pressure. Because
this patient is not hypotensive and he has a rapid heart rate, dopamine is not indicated. Nitroglycerin is
a vasodilator. The patient has no complaint of chest pain and shows no signs of heart failure, so nitroglyc-
erin is not indicated. Furosemide (Lasix) is also not indicated because there are no signs of pulmonary
congestion. Atropine is not indicated because the patient has a tachycardia, not a bradycardia.

3. A. Diltiazem is a calcium blocker that may be used to control the ventricular rate in stable narrow-
QRS tachycardias or AFib or atrial flutter with a rapid ventricular response if the rhythm persists
despite vagal maneuvers or adenosine in patients with an adequate blood pressure. Calcium block-
ers should be avoided in patients with wide-QRS tachycardia unless it is known with certainty to
be supraventricular in origin (may precipitate VF). Calcium blockers should be avoided in patients
with preexcited AFib/atrial flutter and are contraindicated in second- or third-degree AV block.

4. A, B. The patient appears stable but symptomatic because of the rapid rate. Treatment usually
includes oxygen (if indicated), IV access, and vagal maneuvers. Vagal maneuvers are performed to
slow conduction through the AV node and try to stop the rhythm, assuming there are no contra-
indications to performing them. If vagal maneuvers fail, adenosine is the drug of choice, except for
patients with severe asthma.

5. A, B, D. Endpoints for procainamide administration include the onset of hypotension, suppression


of the dysrhythmia, widening of the QRS by more than 50% of its original width, and administra-
tion of a total cumulative dose of 17 mg/kg.

6. B. When administering amiodarone for dysrhythmias other than a shockable cardiac arrest
rhythm, the correct initial IV dosage is 150 mg IV over 10 minutes. This dose may be repeated if
needed. After conversion, follow with a continuous IV infusion. The total dose over 24 hours
should not exceed 2.2 g.

7. A, E, G. The patient is symptomatic and unstable (acute altered mental status, hypotension). You need
to work quickly. Consider sedation and defibrillate immediately. Ask a team member to assist ventila-
tions and give supplemental oxygen. Although synchronized cardioversion is an appropriate treatment
for unstable patients with a tachycardia and a pulse, it is used for tachycardias that have a relatively
uniform QRS amplitude. Because the amplitude and direction of the QRS complexes of PMVT vary,
defibrillation should be used instead. Epinephrine and diltiazem are not indicated, and adenosine
should not be given for PMVT because it may cause degeneration of the dysrhythmia to VF.

8. A, C, D. Procainamide, amiodarone, or sotalol can be considered in the management of hemody-


namically stable patients with a regular wide-QRS tachycardia. Adenosine can also be adminis-
tered to try to identify the origin of the tachycardia. Verapamil is contraindicated for wide-QRS
tachycardias unless the dysrhythmia is known to be of supraventricular origin and not being
conducted by an accessory pathway.

9. B. The AV node is normally the only electrical connection between the atria and ventricles. Preex-
citation is a term used to describe rhythms that originate from above the ventricles but in which
the impulse travels via a pathway other than the AV node and bundle of His. Thus the supraven-
tricular impulse excites the ventricles earlier than would be expected if the impulse traveled by way
of the normal conduction system. Patients with preexcitation syndromes are prone to AVRT.
When the AV junction is bypassed by an abnormal pathway, the abnormal route is called an
accessory pathway. An accessory pathway is an extra bundle of working myocardial tissue that
forms a connection between the atria and ventricles outside the normal conduction system.
140 CHAPTER 6  Tachycardias

10. D. The patient is symptomatic and unstable as evidenced by his chest discomfort and hypotension.
You should administer sedation and perform synchronized cardioversion using the energy levels
recommended by the defibrillator manufacturer. Transcutaneous pacing and CPR are not indi-
cated. Defibrillation is warranted for pulseless VT, VF, and unstable, sustained PMVT.

11. C, D. Amiodarone and beta blockers may be effective for PMVT with a normal QT interval.
Magnesium is the initial treatment for the stable patient with PMVT associated with a prolonged
QT interval. Adenosine should not be given for PMVT because it may cause degeneration of the
dysrhythmia to VF.

12. C. The initial dose of adenosine is 6 mg rapid IV push over 1 to 3 seconds. If there is no response
within 1 to 2 minutes, give 12 mg rapid IV push. The 12-mg dose may be repeated once in 1 to
2 minutes. Follow each adenosine dose immediately with a 20-mL normal saline flush. Reduce
the dose of adenosine by one-half in patients on dipyridamole (Persantine) or carbamazepine
(Tegretol), in those with transplanted hearts, or if given via a central venous line.

13. A, C, E. When using vagal maneuvers, make sure oxygen, suction, a defibrillator, and emergency
medications are available before attempting the procedure. Continuous monitoring of the patient’s
ECG is essential, and a 12-lead ECG recording is desirable. CSM should be avoided in older
adults and in patients who have a history of stroke, known carotid artery stenosis, or a carotid
artery bruit on auscultation. Application of a cold stimulus to the face is often effective in infants,
but seldom effective in adults. When carotid massage is performed, steady pressure is applied to
the carotid sinus for 5 to 10 seconds. Performing a Valsalva maneuver or CSM with postural
modifications, such as supine positioning and passive leg raising during the procedure, may
increase the probability of successful rhythm conversion.

14. A, E. This patient is symptomatic but stable. Because he has a pulse, CPR, defibrillation,
and epinephrine are not indicated. The patient denies chest pain, so nitroglycerin is not indi-
cated. Procainamide, amiodarone, or sotalol can be considered for a stable patient in mono-
morphic VT.

C A S E S T U DY 6.1 ANSWERS
1. A, C, E. The general impression findings are normal. Appearance is normal, breathing is normal,
and circulation is normal.

2. A, B, C. Ask your team members to attach a pulse oximeter and cardiac monitor and obtain the
patient’s baseline vital signs. At this point, we do not yet know what the patient’s cardiac rhythm
is, so it is premature to ask the patient to perform a vagal maneuver. Also there is no information
to indicate if contraindications for performing a vagal maneuver have been ruled out (e.g., presence
of a carotid bruit, known carotid artery stenosis, history of stroke, recent MI, glaucoma, or
retinopathy).

3. D. The monitor shows a narrow-QRS tachycardia with ST-segment depression.

4. B. Based on the patient’s history and physical findings, the patient is stable but symptomatic at this
time.

5. A, B, C, D, G. Because the patient is stable but symptomatic, it is reasonable to obtain a 12-lead


ECG and a cardiology consult, and attempt a vagal maneuver. Obtaining laboratory studies (e.g.,
metabolic panel, toxicology screen) can help identify and treat possible causes of the dysrhythmia.
Although procainamide and amiodarone are antiarrhythmics that are used to treat both atrial and
ventricular dysrhythmias, these medications appear in the tachycardia algorithm in the management
of stable wide-QRS tachycardias.
CHAPTER 6  Tachycardias 141

6. A. Because the patient is stable but a narrow-QRS tachycardia persists despite attempting a vagal
maneuver, adenosine should be given next. Diltiazem or verapamil, which are longer-acting antiar-
rhythmics than adenosine, are reasonable choices if the patient’s rhythm and condition remain
unchanged after giving adenosine, or if the dysrhythmia recurs. Synchronized cardioversion is not
indicated because the patient is stable at this time.

7. A, E. The patient’s change in mental status and blood pressure indicates that he is now symptom-
atic and unstable. Electrical therapy with synchronized cardioversion is warranted. CPR is not
indicated because the patient still has a pulse. Ensure that the code cart, including intubation
equipment, suction, and resuscitation medications, is within arm’s reach. Ask the IV team member
to sedate the patient while the defibrillation team member applies adhesive pads to the patient’s
bare chest. The administration of an epinephrine IV bolus is contraindicated because the patient
has a pulse and his cardiac rhythm is a tachycardia.

8. B. When electrical therapy is indicated, it is best to use the energy settings recommended by the
defibrillator manufacturer. The typical initial biphasic energy setting is 50 J to 100 J.

9. A, B, C, D, F, G. Administer sedation if time and the patient’s condition permit. Place adhesive
pads on the patient’s bare chest according to the defibrillator manufacturer’s instructions. Press the
“Sync” control on the defibrillator. Make sure the machine is marking each QRS complex and that
no artifact is present. Select an energy level using the manufacturer’s recommended energy dose
for the dysrhythmia. Press the “Charge” button the defibrillator and recheck the ECG rhythm. If
the rhythm is unchanged, call “Clear!” and make sure that everyone is clear of the patient, the bed,
and any equipment that is connected to the patient. Make sure oxygen is not flowing over the
patient’s chest to decrease the risk of combustion in the presence of electrical current. After con-
firming that the area is clear, depress the “Shock” control. A slight delay may occur while the
machine detects the next QRS complex. Release the “Shock” control after the shock has been
delivered.

10. C. The monitor shows a sinus rhythm.

11. A, B, C, D, E. Repeat the primary and secondary assessments. Assess the sites where the adhesive
pads were positioned for skin burns and ask the patient if he is experiencing any discomfort. Be-
cause the patient received sedation and because dysrhythmias can recur, monitor the patient’s vital
signs every 5 minutes for the next 30 minutes and closely after that. Review the results of the
patient’s 12-lead ECG and laboratory studies to try to determine the cause of the patient’s tachy-
cardia. Arrange for the patient’s transfer for continued care and request a team debriefing after the
transfer of patient care is complete.

C A S E S T U D Y 6 . 2 A NSWERS
1. A, D, F. The general impression findings are abnormal (Appearance: normal; Breathing: abnormal;
Circulation: abnormal skin color).

2. A, B, C, D, E. All of the actions listed are appropriate. Ask a team member to attach a pulse oxim-
eter and ECG monitor and obtain the patient’s baseline vital signs while you continue your assess-
ment. In addition, order laboratory studies (e.g., complete blood count, coagulation studies, metabolic
panel, lipid profile, serum cardiac markers) and obtain a 12-lead ECG and portable chest radiograph.
Ask a team member to establish IV access.

3. D. The monitor shows monomorphic VT.

4. C. Based on the patient’s history and physical findings (slightly labored breathing; hypotension;
cool, pale skin), the patient is symptomatic and unstable.
142 CHAPTER 6  Tachycardias

5. A, B. Because the patient is symptomatic and unstable, ask the defibrillation team member to apply
adhesive pads to the patient’s bare chest and prepare to shock the patient. Ensure that the code cart,
including intubation equipment, suction, and resuscitation medications, is within arm’s reach. While
preparing to deliver a shock, ask the IV team member to sedate the patient. Vagal maneuvers are
attempted when a hemodynamically stable patient has a regular narrow-QRS tachycardia. Because
the patient’s hypotension is likely a result of his tachycardia, attempting to stop the dysrhythmia
should improve his BP; therefore a dopamine infusion is unnecessary at this time.

6. A, E. Because the patient is symptomatic and unstable, it is imperative that you and your team act
quickly. Administer sedation and then direct a team member to perform synchronized cardiover-
sion. Use the energy settings recommended by the defibrillator manufacturer (typically 100 J). Vagal
maneuvers, CPR, and defibrillation are not indicated in this situation. Although both procainamide
and amiodarone may be used to treat stable wide-QRS tachycardias, the patient is currently unstable;
therefore, electrical therapy is warranted at this time.

7. D. The monitor shows a sinus rhythm at 100 beats/min with a run of monomorphic VT at 136
beats/min.

8. A, B, C, D. Repeating the primary assessment, including reassessing vital signs, is warranted after
performing a therapeutic intervention. Because the patient received sedation, monitor the patient’s
vital signs every 5 minutes for the next 30 minutes and closely after that. Review the patient’s
12-lead ECG and laboratory results to attempt to identify the underlying cause of the dysrhythmia.

9. C, D. The patient is now hemodynamically stable, but he will require ongoing monitoring. It is
reasonable to administer procainamide because the run of VT observed on the monitor indicates
ongoing myocardial irritability. Alternative medications include amiodarone or sotalol. Adenosine,
lidocaine, and magnesium are not indicated. Arrange for the patient’s transfer for continued care
and request a team debriefing after the transfer of patient care is complete.

REFERENCES
Appelboam, A., Reuben, A., Mann, C., et al. (2015). Postural modification to the standard Valsalva manoeuvre for
emergency treatment of supraventricular tachycardias (REVERT): A randomised controlled trial. Lancet,
386(10005), 1747–1753.
Berger, M. G., Rubenstein, J. C., & Roth, J. A. (2016). Cardiac arrhythmias. In I. J. Benjamin, R. C. Griggs, E. J.
Wing, & J. G. Fitz (Eds.), Andreoli & Carpenter’s Cecil essentials of medicine (9th ed., pp. 110–135). Philadelphia,
PA: Saunders.
Ceylan, E., Ozpolat, C., Onur, O., Akoglu, H., & Denizbasi, A. (2019). Initial and sustained response effects of
3 vagal maneuvers in supraventricular tachycardia: A randomized, clinical trial. J Emerg Med, 57(3), 299–305.
Ellenbogen, K. A., & Koneru, J. N. (2018). Atrial tachycardia. In D. P. Zipes, J. Jalife, & W. G. Stevenson (Eds.),
Cardiac electrophysiology: From cell to bedside (7th ed., pp. 681–699). Philadelphia, PA: Elsevier.
Issa, Z. F., Miller, J. M., & Zipes, D. P. (2019). Typical atrial flutter. In Clinical arrhythmology and electrophysiology
(3rd ed., pp. 339–374). Philadelphia, PA: Elsevier.
January, C. T., Wann, L. S., Alpert, J. S., et al. (2014). 2014 AHA/ACC/HRS guideline for the management of
patients with atrial fibrillation: A report of the American College of Cardiology/American Heart Association
Task Force on Practice Guidelines and the Heart Rhythm Society. J Am Coll Cardiol, 64(21), e1–e76.
January, C. T., Wann, L. S., Calkins, H., et al. (2019). 2019 AHA/ACC/HRS focused update of the 2014 AHA/
ACC/HRS guideline for the management of patients with atrial fibrillation. J Am Coll Cardiol, 74(1), 104–132.
Link, M. S., Berkow, L. C., Kudenchuk, P. J., et al. (2015). American Heart Association Guidelines for CPR & ECC.
Retrieved from American Heart Association. In Web-based integrated guidelines for cardiopulmonary resus-
citation and emergency cardiovascular care – Part 7: Adult advanced cardiovascular life support: Eccguidelines.
heart.org.
Mani, B. C., & Pavri, B. B. (2014). Dual atrioventricular nodal pathways physiology: A review of relevant anatomy,
electrophysiology, and electrocardiographic manifestations. Indian Pacing Electrophysiol J, 14(1), 12–25.
Mark, D. G., Brady, W. J., & Pines, J. M. (2009). Preexcitation syndromes: Diagnostic consideration in the ED.
Am J Emerg Med, 27(7), 878–888.
CHAPTER 6  Tachycardias 143

Minczak, B. M., Laub, G. W. (2019). Techniques for supraventricular tachycardias. In J. R. Robert, C. B. Custalow,
& T. W. Thomsen (Eds.), Roberts and Hedges’ clinical procedures in emergency medicine and acute care (7th ed.,
pp. 221–237). Philadelphia, PA: Elsevier.
Morady, F., & Zipes, D. P. (2019). Atrial fibrillation: Clinical features, mechanisms, and management. In D. P.
Zipes, P. Libby, R. O. Bonow, D. L. Mann, G. F. Tomaselli, & E. Braunwald (Eds.), Braunwald’s heart disease:
A textbook of cardiovascular medicine (11th ed., pp. 730–752). Philadelphia, PA: Elsevier.
Mottram, A. R., Svenson, J. E. (2011). Rhythm disturbances. Emerg Med Clin North Am, 29(4), 729–746.
Page, R. L., Joglar, J. A., Caldwell, M. A., et al. (2016). 2015 ACC/AHA/HRS guideline for the management of
adult patients with supraventricular tachycardia. Circulation, 133(14), e506–e574.
Panchal, A. R., Bartos, J. A., Cabañas, J. G., et al. (2020). Part 3: Adult basic and advanced life support: 2020
American Heart Association guidelines for cardiopulmonary resuscitation and emergency cardiovascular care.
Circulation, 142(16 suppl 2), S366–S468.
Peterson, K. (2018). Advanced dysrhythmias. In V. S. Good, & P. L. Kirkwood (Eds.), Advanced critical care nursing
(2nd ed., pp. 12–33). St. Louis, MO: Elsevier.
Saksena, S., Bharati, S., Lindsay, B. D., & Levy, S. (2012). Paroxysmal supraventricular tachycardia and pre-exci-
tation syndromes. In S. Saksena, & A. J. Camm (Eds.), Electrophysiological disorders of the heart (2nd ed., pp.
531–558). Philadelphia, PA: Saunders.
Sohinki, D., & Obel, O. A. (2014). Current trends in supraventricular tachycardia management. Ochsner J, 14(4),
586–595.
Zimetbaum, P. (2020). Supraventricular cardiac arrhythmias. In L. Goldman, & A. I. Schafer (Eds.), Goldman-Cecil
medicine (26th ed., pp. 331–343). Philadelphia, PA: Elsevier.
This page intentionally left blank
C H A P T E R 7
Cardiac Arrest Rhythms

INTRODUCTION
Evaluation of your ability to manage a patient who is experiencing a cardiac arrest and your ability
to manage the team who will assist you in providing patient care is part of the Advanced Cardiac
Life Support (ACLS) course. This chapter discusses the cardiac arrest rhythms and their initial
management.

L E A R N I N G OBJECTIVES
After completing this chapter, you should be able to:
1. Identify four cardiac rhythms that are associated with cardiac arrest.
2. Differentiate between shockable and nonshockable cardiac arrest rhythms.
3. Identify the initial cardiac rhythms that are typically recorded in out-of-hospital cardiac
arrest (OHCA) and in-hospital cardiac arrest (IHCA).
4. Given a patient situation, describe the electrocardiogram (ECG) characteristics and
initial emergency care for cardiac arrest rhythms, including mechanical, pharmacologic
(i.e., indications, contraindications, doses, and route of administration of appropriate
medications), and electrical therapy, where applicable.
5. Discuss immediate postcardiac arrest care upon return of spontaneous circulation (ROSC).

L E A R N I N G PLAN
• Read this chapter before class.
• Master identification of the following rhythms: ventricular fibrillation (VF), monomorphic
ventricular tachycardia (VT), polymorphic ventricular tachycardia (PMVT), asystole, and
pulseless electrical activity (PEA).
• Master the following medications: oxygen (O2), epinephrine, amiodarone, and lidocaine.
• Master the following skills:
n Ensure scene safety and the use of personal protective equipment (PPE).
n Assign team member roles or perform as a team member in a simulated patient
situation.
n Direct or perform a basic life support (BLS) assessment, primary assessment, and
secondary assessment.
n Quickly recognize cardiopulmonary arrest.
n Demonstrate familiarity with the cardiac arrest algorithm.
n Ensure the performance of high-quality cardiopulmonary resuscitation (CPR) when
indicated. 1
145
146 CHAPTER 7  Cardiac Arrest Rhythms

Demonstrate the safe operation of a manual defibrillator and an automated external


n

defibrillator (AED) if electrical therapy is indicated.


n Demonstrate an understanding of the actions, indications, dosages, adverse
effects, and contraindications for the medications used in the treatment of cardiac
arrest.
n Consider the possible reversible causes of a cardiac emergency.
n Direct the performance of appropriate airway management throughout a
resuscitation effort.
n Recognize the ROSC and direct the performance of immediate postcardiac arrest
care.
n Review your performance as a team leader or team member during a postevent
debriefing.
• Develop and use flashcards, flowcharts, and mnemonics to help enhance your
retention of the information presented.
• Read the case studies at the end of this chapter and compare your answers with the
answers provided.

CARDIAC ARREST RHYTHMS


The initial rhythms that may be observed in a cardiac arrest include the following:
1. Pulseless VT (pVT), in which the ECG displays a wide, regular QRS complex at a rate faster than
100 beats per minute (beats/min), but ventricular contraction is absent or ineffective
2. VF, in which irregular, chaotic deflections that vary in shape and height are observed on the ECG,
but there is no coordinated ventricular contraction
3. Asystole, in which no cardiac electrical activity is present and therefore no mechanical response
(i.e., ventricular contraction)
4. PEA, in which electrical activity is visible on the ECG, but the electrical signal fails to initiate a
mechanical response; therefore central pulses are absent
Cardiac arrest rhythms are shockable or nonshockable. A shockable rhythm is one in which the
heart’s abnormal electrical activity may potentially be stopped with defibrillation. pVT and VF are
shockable rhythms. A nonshockable rhythm is one in which electrical activity is present, but there is
no palpable pulse (as in PEA), or electrical activity is absent (as in asystole). When a patient presents
in a shockable rhythm (i.e., pVT, VF), the probability of survival after prompt intervention is about
25% to 30% compared with about 10% to 15% for PEA and less than 5% for asystole (Myerburg,
2020).

Ventricular Tachycardia
VT exists when three or more ventricular complexes occur in immediate succession at a rate faster than
100 beats/min. VT may occur with or without pulses, and the patient may be stable or unstable with
this rhythm (Fig 7.1). The ECG characteristics of VT are discussed in Chapter 6.

Fig. 7.1  ​Monomorphic ventricular tachycardia. (From Aehlert, B. [2006]. ECGs made easy, ed 3, St. Louis, Mosby.)
CHAPTER 7  Cardiac Arrest Rhythms 147

Ventricular Fibrillation
VF is a chaotic rhythm that begins in the ventricles. With VF, there is no organized depolarization of
the ventricles. The ventricular muscle quivers, and as a result, there is no effective myocardial contrac-
tion and no pulse. The resulting rhythm looks chaotic with deflections that vary in shape and ampli-
tude; no normal-looking waveforms are visible. The ECG characteristics of VF can be summarized as
follows:
Rhythm: Rapid and chaotic with no pattern or regularity
Rate: Cannot be determined because there are no discernible waves or complexes to measure
P waves: Not discernible
PR interval: Not discernible
QRS duration: Not discernible
VF with waves that are 3 or more millimeters (mm) high is called “coarse” VF. The amplitude of VF
waveforms decreases over time as myocardial blood flow and energy metabolism diminish (Li & Tang,
2012). VF with low amplitude waves (i.e., less than 3 mm) is called “fine” VF. Fig. 7.2 illustrates a
comparison of ventricular dysrhythmias.

Asystole
Asystole, which is also called ventricular asystole, is a total absence of ventricular electrical activity
(Fig. 7.3). Characteristics of asystole include the following:
Rhythm: Ventricular not discernible; atrial may be discernible
Rate: Ventricular not discernible but atrial activity may be observed (i.e., “P-wave” asystole)
P waves: Usually not discernible
PR interval: Not measurable
QRS duration: Absent
Because there is no electrical activity, there is no mechanical response (i.e., there is no ventricular
contraction, no pulse, and no cardiac output). Some atrial electrical activity may be evident. If atrial

C
1 sec
Fig. 7.2  ​Ventricular tachydysrhythmias. (A) Rhythm strip showing monomorphic ventricular tachycardia (VT). (B) Example
of polymorphic VT. (C) Example of ventricular fibrillation. All tracings are from lead V1. (From Goldman, L., Ausiello, D.A., Arend,
W., et al. [2007]. Cecil medicine, ed 23, Philadelphia, Saunders.)

Fig. 7.3  ​Asystole. (From Aehlert, B. [2004]. ECG study cards, St. Louis, Mosby.)
148 CHAPTER 7  Cardiac Arrest Rhythms

Fig. 7.4  ​“P-wave” asystole. (From Aehlert, B. [2004]. ECG study cards, St. Louis, Mosby.)

electrical activity is present, the rhythm is called “P wave” asystole or ventricular standstill (Fig. 7.4).
Asystole may occur temporarily following the termination of a tachycardia with medications, defibril-
lation, or synchronized cardioversion.

Pulseless Electrical Activity


PEA, formerly called electromechanical dissociation, exists when organized electrical activity (other
than VT) is observed on the cardiac monitor, but the patient is unresponsive and not breathing, and a
pulse cannot be felt (Fig. 7.5). Pseudo-PEA is evidence of cardiac activity without a detectable pulse.
Stated another way, with pseudo-PEA, mechanical contractions are present but are too weak to pro-
duce a palpable pulse in response to electrical depolarization. It is estimated that pseudo-PEA consti-
tutes 42% to 86% of the total PEA population (Rabjohns et al., 2019; Salen et al., 2005).
The use of point-of-care ultrasound (POCUS) can help differentiate between true PEA and
pseudo-PEA. The use of an arterial line during cardiac arrest, high end-tidal carbon dioxide (EtCO2)
readings in intubated patients, and Doppler ultrasound are examples of additional methods that can be
used to detect pseudo-PEA. Although pseudo-PEA has a more favorable prognosis than true PEA
(Rabjohns et al., 2019), the chance for survival for patients with PEA is poor unless the underlying
cause can be rapidly identified and appropriately managed.

ACLS Pearl
Palpating a pulse is often inaccurate, even when performed by experienced healthcare profes-
sionals. It can be particularly challenging to palpate a pulse in patients who are morbidly obese;
those experiencing cardiac tamponade, severe hypovolemia and/or profound shock; and those
who have severe peripheral vascular disease. Because the presence or absence of a pulse is used
to guide patient management decisions, the use of alternative methods, such as POCUS, are be-
ing used with increasing frequency to confirm the presence or absence of a central pulse. With
training, POCUS can be used to detect the presence or absence of a carotid or femoral artery
pulse within the recommended pulse check time of 10 seconds or less (Simard et al., 2019). The
initial use of manual pulse checks is necessary during cardiac arrest while POCUS equipment
is moved to the bedside, and the machine is powered up and prepared for scanning (Simard
et al., 2019).

Fig. 7.5  ​The rhythm shown is a sinus tachycardia; however, if no pulse is associated with the rhythm, the clinical situation
is termed pulseless electrical activity. (From Aehlert, B. [2006]. ECGs made easy, ed 3, St. Louis, Mosby.)
CHAPTER 7  Cardiac Arrest Rhythms 149

OUT-OF-HOSPITAL CARDIAC ARREST


In the United States, most adult OHCAs occur in a home or residence (69.5%), in a public setting
(18.8%), or a nursing home (11.7%) (Benjamin et al., 2019) (Fig. 7.6). An emergency medical service
(EMS) provider witnesses 12% of OHCAs, a layperson observes 37%, and the patient’s collapse is
unwitnessed in 51% of OHCAs (Benjamin et al., 2019). An AED is not applied before the arrival of
EMS personnel in 71.1% of cases (CARES Summary Report, 2017).
When an OHCA occurs, patients who are in sustained VT at the time of initial contact have the
best result. In contrast, those who present with a bradydysrhythmia or asystole at initial contact have
the worst prognosis (Myerburg & Goldberger, 2019). When the initial rhythm recorded is VF, the
patient’s outcome is intermediate between the results associated with sustained VT and those of a
bradydysrhythmia and asystole (Myerburg & Goldberger, 2019). The initial arrest rhythms recorded
for adult nontraumatic OHCA patients who received treatment by EMS personnel in 2017 appear in
Box 7.1. Survival rates for adult nontraumatic OHCAs in 2017 are shown in Table 7.1. Note that the
analysis excludes patients for whom there was missing hospital outcome data.

Fig. 7.6  ​Out-of-hospital cardiac arrest. (From istock.com.)

BOX 7.1 Nontraumatic Out-of-Hospital Cardiac Arrest: First Arrest Rhythm


(2017 data)
Asystole: 50.2%
Idioventricular rhythm/PEA: 21.2%
pVT/VF/Unknown shockable rhythm: 18.4%
Unknown nonshockable rhythm: 10.2%
PEA, Pulseless electrical activity; pVT, pulseless ventricular tachycardia; VF, ventricular fibrillation
(From CARES 2017 Non-Traumatic National Summary Report.)

TABLE 7.1 Adult Nontraumatic Out-of-Hospital Cardiac Arrest Survival Rates


(2017 data)a
Survival to Survival to Survival to Discharge
First Arrest Sustained Hospital Hospital With CPC Score
Rhythm Total (%) ROSC (%) Admission (%) Discharge (%) 1 or 2 (%)b
Shockable (e.g., 18.7 50.1 48.6 29.1 25.7
pVT or VF)
Nonshockable 81.3 27.9 23.5   6.1   4.4
(e.g., asystole
or PEA)
a
Analysis excludes patients with missing hospital outcome data
b
Incomplete data for 82 patients
CPC, Cerebral Performance Category; pVT, pulseless ventricular tachycardia; PEA, pulseless electrical activity; ROSC, return
of spontaneous circulation; VF, ventricular fibrillation.
(From CARES 2017 Non-Traumatic National Survival Report.)
150 CHAPTER 7  Cardiac Arrest Rhythms

IN-HOSPITAL CARDIAC ARREST


In the United States, most adult IHCAs (53.5%) occur in the intensive care unit (ICU), operating
room, or emergency department, with the remainder occurring in a noncritical care area (Benjamin
et al., 2019). Patients who experience an IHCA tend to be older and have more severe illnesses and
secondary comorbidities than those who experience OHCA (Institute of Medicine et al., 2015). The
most common causes of IHCA include cardiac dysrhythmias, acute respiratory insufficiency, and
hypotension (Morrison et al., 2013). Research shows that abnormal vital signs occur during the 4
hours preceding IHCA in 59.4% of patients, with at least one severely abnormal vital sign occurring
in 13.4% (Benjamin et al., 2019).
The initial rhythm associated with adult IHCA is usually PEA (52.3%), followed by asystole
(23.6%), and then VF or pVT (16.1%) (Benjamin et al., 2018). The initial rhythm is unknown in 8%
of cases. Adult IHCA survivors generally have a better outcome, with 25.6% surviving to discharge
compared with 10.4% of OHCA adults surviving to hospital discharge (Benjamin et al., 2019).

ACLS Pearl
It is helpful when a patient has an advance directive, and the healthcare team and facility are
aware of its existence. The patient, their healthcare agent or guardian, and those close to them
must ensure that everyone who might need a copy of the directive has a copy. When caring for
a patient with known or suspected coronavirus disease 2019 (COVID-19), experts recommend
addressing advance directives and goals of care on hospital arrival and with any significant
change in clinical status, such as a physiologic change that requires an increase in the level of
care (Edelson et al., 2020).

THE CARDIAC ARREST ALGORITHM


After ensuring the use of appropriate PPE, the most important priorities during a cardiac arrest are
the performance of high-quality CPR and defibrillation if a shockable rhythm is present. Obtaining
vascular access, giving medications, and inserting an advanced airway is of secondary importance.
Throughout the resuscitation effort, aggressively search for and treat possible reversible causes of the
arrest using the “Hs and Ts” (Table 7.2). The cardiac arrest algorithm appears in Fig. 7.7.
The rhythm present on the cardiac monitor will guide the sequence of procedures that need to be
done next. For example, if the patient is in cardiac arrest and the cardiac monitor shows no electrical
activity, asystole is present. If the monitor shows an organized rhythm despite no palpable central pulse,
PEA is present. Defibrillation is not indicated for asystole or PEA. If the monitor shows VF or pVT,
defibrillation is indicated.

ACLS Pearl
Although not always available, information related to the arrest should be sought, including:
• When and where did the arrest occur?
• Was the arrest witnessed?
• Was CPR performed? If yes, how long was the patient down before CPR was started?
• What was the patient’s initial cardiac rhythm? If pVT/VF, when was the first shock delivered?
• Are there any special circumstances to consider, such as drug overdose, pregnancy, hypo-
thermia, trauma, or do-not-attempt-resuscitation orders?
• What treatment has been given?
• What information is available regarding the patient’s past medical history?

Shockable Rhythms
If the cardiac monitor reveals pVT/VF, ensure that high-quality CPR continues while the defibrillator
is readied for use. The team member responsible for vascular access and medications should prepare
the medications that will likely be needed based on the rhythm on the monitor. Coordinate ventilations
with chest compressions until an advanced airway is placed and its position confirmed. If a monophasic
CHAPTER 7  Cardiac Arrest Rhythms 151

TABLE 7.2 The Hs and Ts: Possible Reversible Causes of Cardiac Arrest
Possible Cause Findings Possible Intervention(s)
Hypoxia • Altered mental status, cyanosis, blood gas Effective oxygenation and ventila-
results, airway obstruction tion, aggressive search for the
• Slow heart rate because of hypoxia cause
• History, flat neck veins, dry mucous membranes, Volume replacement, find source
decreased urine output (e.g., bleeding) and manage
• Narrow QRS complex, rapid rate
Hypothermia • History of cold exposure, core body temperature Rewarming guided by core
• Initial tachycardia, then progressive bradycardia; temperature
J or Osborne waves
Hypokalemia • Prolonged loop or thiazide diuretic therapy; ad- K1 infusion
ministration of K1 deficient parenteral fluids;
severe GI fluid losses from gastric suctioning
or lavage; prolonged vomiting or diarrhea, or
laxative abuse without K1 replacement
• ST segment depression; broad, flat T waves;
appearance of U waves
Hyperkalemia • History of shock, renal failure, or sepsis; severe cell Calcium and/or bicarbonate IV,
damage such as from burns, trauma, crush injuries IV insulin with glucose (alone
• Presence of an arteriovenous fistula or dialysis or in combination), nebulized
catheter or inhaled beta-agonists,
• Peaked (tented) T waves; widening of QRS; hemodialysis
flattened or absent P waves
Hydrogen ion • History of renal failure, toxic ingestion, Treatment directed at underlying
(acidosis) ketoacidosis, shock cause; sodium bicarbonate
• Blood gas results in specific circumstances
Tamponade • History (trauma, renal failure, thoracic Pericardiocentesis
(cardiac) malignancy)
• Neck vein distention
• Narrow QRS complex, rapid rate (impending
tamponade)—deteriorating to sudden
bradycardia as a terminal event
Tension • History (trauma, rib fracture, asthma, severe Needle decompression or
pneumothorax COPD) thoracostomy
• Unequal breath sounds, neck vein distention,
tracheal deviation, difficult to ventilate patient,
hyperresonance to percussion on the affected
side
• Narrow QRS complex, slow rate (because of hypoxia)
Thrombosis • History (cancer, deep vein thrombosis) Possible fibrinolysis, surgical
(pulmonary • Neck vein distention, deep vein thrombosis embolectomy, mechanical
embolism) in lower extremities embolectomy
• Narrow QRS complex
Thrombosis • History, ECG, cardiac biomarkers Possible emergent coronary
(acute coronary • Q waves, ST segment changes, T wave angiography
syndromes) inversion
Toxins
Beta blocker Slow rate, prolonged PR interval, possible AV Glucagon IV, high-dose insulin
block with glucose, calcium, ECMO
Calcium blocker Slow rate, prolonged PR interval, possible AV Calcium IV, high-dose insulin with
block glucose, glucagon IV, ECMO
Digoxin Slow rate, prolonged PR interval, shortened QT Fab antibodies
interval, T wave inversion or flattening
Tricyclic Widening of QRS, prolonged QT interval, ST Sodium bicarbonate IV, ECMO
antidepressants segment changes
AV, Atrioventricular; COPD, chronic obstructive pulmonary disease; ECG, electrocardiogram; ECMO, extracorporeal
membrane oxygenation; GI, gastrointestinal; IV, intravenous.
152 CHAPTER 7  Cardiac Arrest Rhythms

1 Begin high-quality CPR

• Give naloxone if possible opioid overdose, per protocol


• Give 100% oxygen when available
• Attach monitor/defibrillator when available

If known/suspected COVID-19:

• Wear personal protective equipment and limit personnel


• Consider appropriateness of resuscitation
• Ventilate using a bag-mask device with a filter and tight face-to-mask seal or perform continuous compressions while using
a face mask
• Limited aerosolization when giving oxygen
• Prepare to intubate (use provider with highest likelihood of success on first attempt); consider video laryngoscopy

Assess ECG rhythm – Shockable? Yes, go to 2; No, go to 9

2 Shockable rhythm (VF/pVT) 9 Nonshockable rhythm (Asystole/PEA)

3 Shock (defibrillate) x 1 10 Perform CPR (2 min)


• Establish vascular access
4 Perform CPR (2 min) • Give epinephrine as soon as possible
• Establish vascular access • Consider advanced airway
• Consider mechanical compression device if known
or suspected COVID-19
Assess ECG rhythm – Shockable? Assess ECG rhythm – Shockable?
Yes, go to 5; No, go to 12 Yes, go to 5 or 7; No, go to 11
5 Shock (defibrillate) x 1 11

6
Perform CPR (2 min) Perform CPR (2 min)
• Give epinephrine
• Consider advanced airway Search for and treat reversible causes
• Consider mechanical compression device if known
or suspected COVID-19

Assess ECG rhythm – Shockable? Assess ECG rhythm – Shockable?


Yes, go to 7; No, go to 12 Yes, go to 5 or 7; No, go to 12
7 Shock (defibrillate) x 1
8 Perform CPR (2 min)
• Consider amiodarone or lidocaine
• Search for and treat reversible causes

12 Return of Spontaneous Circulation (ROSC)


• If no evidence of ROSC, go to 10 or 11
• If ROSC occurs, begin postcardiac arrest care

High-quality CPR CPR (cont’d) Medications ROSC


• Maximize chest compression fraction • Use waveform capnography to • Epinephrine IV/IO bolus: 1 • Palpable pulse and blood
by minimizing interruptions confirm and monitor advanced mg every 3 to 5 min (consider pressure
• Rate: 100 to 120 compressions/min airway placement; attempt to giving every 4 min so • Sustained increase in
• Depth: At least 2 in (5 cm) improve CPR quality if PETCO2 administration coincides with PETCO2 (typically 40 mm Hg
• Ratio: 30 compressions to 2 breaths if is less than 10 mm Hg or every other rhythm check) and higher)
no advanced airway; if advanced decreasing • Amiodarone IV/IO bolus: 300 Known or suspected COVID-19
airway in place, give 1 breath every 6 Defibrillation Energy mg first dose; 150 mg (after steps 3 or 9)
sec (10 breaths/min) with continuous • Monophasic: 360 J all shocks second dose • Prioritize intubation; pause
compressions; avoid excessive • Biphasic: Per manufacturer’s • Lidocaine IV/IO bolus: 1 to compressions during procedure
ventilation recommendations; if unknown, 1.5 mg/kg first dose; 0.5 to • If delayed intubation, consider
• Avoid leaning on the chest between use maximum energy dose 0.75 mg/kg second dose supraglottic airway or BMD with
compressions to allow full chest recoil available. Second and filter and tight face-to-mask seal
• Rotate compressors every 2 min subsequent doses should be • Connect to ventilator with HEPA
(sooner if fatigued) equivalent and higher doses filter when possible
may be considered.
BMD = Bag-mask device; CPR = cardiopulmonary resuscitation; ECG = electrocardiogram; HEPA = high-efficiency particulate air; IO = intraosseous; IV = intravenous; J = joules;
PEA = pulseless electrical activity; pVT = pulseless ventricular tachycardia; VF = ventricular fibrillation.
(Sources: Edelson et al., 2020; Panchal et al., 2020; Panchal et al., 2018.)

Fig. 7.7  Adult cardiac arrest algorithm.


CHAPTER 7  Cardiac Arrest Rhythms 153

Fig. 7.8  ​Remember to clear everyone from the patient before the delivery of each shock. (From istock.com).

defibrillator is used for shockable cardiac arrest rhythms, the recommended energy dose is 360 joules ( J)
for all shocks (Panchal et al., 2020). Follow the manufacturer’s recommendations if a biphasic defibril-
lator is used, such as an initial energy dose of 120 to 200 J. The second and subsequent energy doses
should be equivalent, and higher doses may be considered (Panchal et al., 2020).
After the first defibrillation, resume CPR immediately and establish vascular access. Recheck the
rhythm after five cycles of CPR (about 2 minutes). Pauses in chest compressions for rhythm checks
should not exceed 10 seconds. If a shockable rhythm is present, continue CPR while the defibrillator
is charging, ensure oxygen is not flowing toward the patient’s chest, check to be sure that everyone is
clear, and then defibrillate (Fig. 7.8). Resume CPR immediately.

ACLS Pearl
To minimize interruptions in chest compressions during a cardiac arrest, the team leader should
communicate clearly with team members about impending pauses in compressions. Doing so
enables multiple rescuers to anticipate and then use the same brief pause to achieve multiple
tasks (Meaney et al., 2013).

If a shockable rhythm persists after delivering the second shock, give a vasopressor (i.e., epineph-
rine) without interrupting CPR. Consider the placement of an advanced airway. If an advanced airway
is inserted, initiate capnography to determine the adequacy of CPR and monitor for ROSC. Consider
the use of a mechanical compression device if COVID-19 is known or suspected. After giving
epinephrine and five cycles of CPR (about 2 minutes), recheck the rhythm. If the monitor shows
that a shockable rhythm is present, clear everyone from the patient, defibrillate, and immediately
resume CPR.

ACLS Pearl
The administration of epinephrine is recommended every 3 to 5 minutes intravenous (IV)/intraos-
seous (IO) throughout a cardiac arrest (Panchal et al., 2020). Consider giving it every 4 minutes so
that its timing coincides with every other rhythm check.
154 CHAPTER 7  Cardiac Arrest Rhythms

If a shockable rhythm persists after delivering a third shock, consider giving an antiarrhythmic while
continuing CPR. Use the Hs and Ts to help identify possible reversible causes of the arrest or factors
that may be complicating the resuscitation effort. After five cycles of CPR (about 2 minutes), recheck
the rhythm. Continue the pattern of 2 minutes of CPR, performing a rhythm check, delivering a shock
(assuming a shockable rhythm persists), resuming CPR, and administering medications until ROSC
is achieved, a nonshockable rhythm is observed, or resuscitation efforts are terminated.
If defibrillation of pVT/VF results in an organized rhythm on the monitor, perform a pulse check.
Resume CPR if you are not sure if a pulse is present. If the patient has a pulse, the algorithm changes
because of the rhythm change, as well as the presence of a pulse. If the organized rhythm on the
monitor does not produce a pulse, PEA exists, and treatment continues using the cardiac arrest algo-
rithm; however, the treatment sequence changes from the shockable rhythm segment of the algorithm
to the nonshockable rhythm segment. If the organized rhythm on the monitor does produce a pulse,
supportive measures must be taken to maintain the perfusing rhythm. These supportive measures
encompass postresuscitation support or postcardiac arrest care (discussed later). If defibrillation of
pVT results in VF (or vice versa), there is no change in the algorithm because pVT and VF are treated
in the same way. If defibrillation successfully terminated pVT/VF, but the rhythm recurs, begin
defibrillation at the last energy level used that resulted in successful defibrillation.

Medications
Medications that may be used in the treatment of pVT/VF include epinephrine and, possibly,
amiodarone or lidocaine.

Vasopressors
Epinephrine is a vasopressor that stimulates both alpha- and beta-adrenergic receptors. A vasopressor
is administered during cardiac arrest to increase the perfusion pressure of: (1) the myocardium, for an
increased chance of ROSC, and (2) the brain, for an increased chance of neurologically intact survival
(Sunde & Steen, 2012). Epinephrine 1 mg should be given by the IV or IO route in adult cardiac arrest.
Higher doses of epinephrine are not recommended (Panchal et al., 2019a). Because the effects of epi-
nephrine do not last long, it should be repeated every 3 to 5 minutes as long as the patient is in cardiac
arrest.
Experts note that the optimal timing for the administration of epinephrine in patients with shock-
able rhythms is unknown. Recent studies that evaluated the administration of epinephrine used pro-
tocols in which epinephrine was given after the third shock. The American Heart Association (AHA)
recommends giving epinephrine after delivery of a second shock if pVT/VF persists. Their rationale is
that the myocardium is likely to be ischemic by this time, so even if the second shock terminates
pVT/VF, the epinephrine and high-quality CPR may improve coronary perfusion pressure (CPP) and
myocardial perfusion and may potentially enable the heart to resume and maintain a spontaneous,
perfusing rhythm. On the other hand, if the second shock did not eliminate the pVT/VF, the epineph-
rine and high-quality CPR may help improve CPP and increase the likelihood that the third shock
will be successful (Panchal et al., 2019b).
Vasopressin is a nonadrenergic vasopressor that causes peripheral, coronary, and renal vasoconstric-
tion. It was removed from the 2015 resuscitation guidelines because research showed there was no
significant benefit for its use compared with, or in addition to, epinephrine. Current resuscitation
guidelines indicate that vasopressin may be considered in adult cardiac arrest, but its administration
offers no advantage as a substitute for epinephrine (Panchal et al., 2020). Similarly, vasopressin, in
combination with epinephrine, may be considered during an adult cardiac arrest but offers no advan-
tage as a substitute for epinephrine alone (Panchal et al., 2020).

Antiarrhythmic Medications
Antiarrhythmics may be given in refractory pVT/VF to aid in successful defibrillation and to reduce
the risk of recurrent dysrhythmias (Panchal et al., 2018); however, there is no evidence to determine
if an antiarrhythmic is best given before or after epinephrine, or even when the antiarrhythmic
should be administered (Panchal et al., 2019b). The resuscitation guidelines recognize that experi-
enced clinicians may opt to tailor the sequence of drug delivery to individual patient needs (Panchal
et al., 2019b).
Amiodarone is an antiarrhythmic that blocks sodium channels, inhibits sympathetic stimulation,
and blocks potassium channels, as well as calcium channels. Amiodarone may be considered for
CHAPTER 7  Cardiac Arrest Rhythms 155

pVT/VF that is unresponsive to CPR, defibrillation, and vasopressor therapy. Lidocaine is a class 1B
antiarrhythmic that inhibits the influx of sodium through the fast channels of the myocardial cell
membrane and decreases conduction in ischemic cardiac tissue without adversely affecting normal
conduction. Lidocaine may be considered as an alternative to amiodarone for pVT/VF that is unre-
sponsive to CPR, defibrillation, and vasopressor therapy. Amiodarone or lidocaine may be particularly
useful for patients with witnessed arrest (Panchal et al., 2018).
Magnesium is an electrolyte that acts as a vasodilator and plays a vital role in regulating the flow of
sodium, potassium, and calcium across cell membranes. Although magnesium may be considered for
PMVT with a prolonged QT interval (i.e., torsades de pointes), it is not recommended for routine use
in adult cardiac arrest (Panchal et al., 2020).

Nonshockable Rhythms
If a rhythm check reveals a nonshockable rhythm, continue high-quality CPR. While CPR continues,
establish vascular access. Give epinephrine every 3 to 5 minutes as soon as feasible; exceptions may
exist when a clear reversible cause can be rapidly identified and treated (Panchal et al., 2019a). Con-
sider the placement of an advanced airway and the use of capnography after its placement. Because
hypoxemia is a possible reversible cause of cardiac arrest, advanced airway placement is theoretically
more important during a cardiac arrest associated with PEA or asystole than with pVT/VF and may
be necessary to achieve adequate oxygenation or ventilation (Link et al., 2015). Consider the use of a
mechanical compression device if COVID-19 is known or suspected.
Reassess the patient’s cardiac rhythm. Perform a pulse check if an organized rhythm is present. If a
pulse is present, begin postcardiac arrest care. If a nonshockable rhythm persists, resume high-quality
CPR. Search for and treat reversible causes of the arrest or factors that may be complicating the resus-
citation effort during every 2 minutes of CPR (Link et al., 2015).

ACLS Pearl
POCUS can be useful in identifying potentially treatable causes of cardiac arrest and guiding
patient management decisions. For example, POCUS can be used to recognize cardiac tampon-
ade and pneumothorax, to detect the presence of tumors or clots, to assess myocardial contrac-
tility during CPR, and to assess ventricular volume.

Continue CPR for 2 minutes before performing another rhythm check. Remember to switch chest
compressors every 2 minutes, or sooner if warranted, to avoid rescuer fatigue. If there is no response to
appropriately performed interventions after a reasonable period, consider termination of efforts after
consultation with the members of the resuscitation team. Examples of factors to consider when decid-
ing to terminate in-hospital resuscitative efforts include the following:
• The time from patient collapse to CPR.
• The patient’s initial cardiac rhythm at the time of the arrest.
• The time from collapse to the first defibrillation attempt (if a shockable rhythm was present).
• The existence of special circumstances (e.g., traumatic injury, asthma, pregnancy, poisoning,
hypothermia, submersion injury, electrical/lightning injury).
• The presence of comorbid disease.
• The duration of the resuscitation effort.
• The presence or absence of sonographically detected cardiac activity.
• The patient’s response to resuscitative measures, including physiologic parameters, such as quantita-
tive waveform capnography, arterial relaxation diastolic pressure, arterial pressure monitoring, and
central venous oxygen saturation (Link et al., 2015).

ACLS Pearl
Monitor the quality of compressions during resuscitation efforts using continuous EtCO2 monitor-
ing. Failure to achieve an EtCO2 of greater than 10 mm Hg immediately after advanced airway
insertion and after 20 minutes of CPR is associated with extremely poor chances for ROSC and
survival. This finding, in combination with other factors, may be considered when deciding when
to terminate resuscitation (Link et al., 2015).
156 CHAPTER 7  Cardiac Arrest Rhythms

Refractory Cardiac Arrest


Current literature lacks consensus concerning the definition of refractory cardiac arrest. Some experts
define refractory cardiac arrest as a lack of ROSC after 30 minutes of appropriate CPR in the absence
of hypothermia (Shanmugasundaram & Lotun, 2018) or the absence of a ROSC despite the provision
of appropriate CPR for more than 30 minutes (Debaty et al., 2017). Others define pVT/VF as refrac-
tory if the rhythm persists, and no ROSC occurs after conventional CPR for more than 10 minutes
(Siao et al., 2015). Still others define refractory pVT/VF as that which remains either recurrent or
incessant after more than three direct current shocks or after antiarrhythmic medications have been
given (Yannopoulos et al., 2019). Regardless of the definition used, refractory cardiac arrest is associ-
ated with high mortality rates.
New treatment paradigms are emerging for selected patients resuscitated from OHCA with favor-
able survival rates. Examples include the use of extracorporeal cardiopulmonary resuscitation (ECPR)
(discussed later) and providing emergent access to the cardiac catheterization laboratory (CCL) for
coronary angiography and possible percutaneous coronary intervention (PCI).

Return of Spontaneous Circulation


If resuscitative efforts result in ROSC, repeat the primary assessment, ask a team member to obtain the
patient’s vital signs, and begin postcardiac arrest care. Focus the efforts of the code team on the following:
• Repeating the primary and secondary assessments
• Anticipating changes in the patient’s condition (and preventing deterioration)
• Stabilizing vital signs
• Securing tubes and lines
• Troubleshooting any problem areas
• Preparing the patient for transport or transfer
• Accurately documenting the events that took place during the resuscitation effort
• Drawing blood for laboratory tests and treating the patient as needed based on results

ACLS Pearl
If ROSC does not occur after resuscitation efforts in the field for a patient with known or sus-
pected COVID-19, experts recommend considering not transferring the patient to the hospital
because of the low likelihood of patient survival balanced against the risk of infectious disease
exposure to prehospital and hospital providers (Edelson et al., 2020).

Patient Transfer
The resuscitation team’s responsibility to the patient continues until patient care is transferred to a
healthcare team with equal or greater expertise. Transfer the patient with oxygen, ECG monitoring,
and resuscitation equipment and ensure that trained personnel accompany the patient. When transfer-
ring care, provide information that is well organized, concise, and complete. Using Situation, Back-
ground, Assessment, and Recommendation (SBAR) (see Chapter 2) or a similar handover format can
help ensure that vital information is relayed to those assuming patient care. Also make sure that the
family has been updated regarding events.

SPECIAL RESUSCITATION SITUATIONS


Some situations require BLS or advanced life support modifications during resuscitative efforts.
Cardiac arrest in patients with known or suspected opioid overdose, known or suspected COVID-19,
and cardiac arrest in pregnancy are briefly discussed subsequently.

Known or Suspected Opioid Overdose


In the United States, opioid overdose primarily affects people between the ages of 25 and 65 years
(Panchal et al., 2020). Researchers suggest that promoting the coprescription of naloxone with opioids
may provide greater access to naloxone for those who are at risk of opioid overdose (Sohn et al., 2020).
CHAPTER 7  Cardiac Arrest Rhythms 157

Initial care for an unresponsive patient who is not breathing normally or is only gasping, who has a
palpable pulse, and who is suspected of having an opioid overdose includes opening the airway and
providing rescue breathing using a barrier or bag-mask device (BMD). In addition to these maneuvers,
it is reasonable for appropriately trained lay rescuers and BLS providers to administer naloxone
(Panchal et al., 2020). Do not wait to assess the patient’s response to naloxone before activating the
emergency response system.
When a patient has experienced a cardiac arrest and opioid overdose is suspected, naloxone can be
given after CPR is begun, in addition to standard resuscitative measures, if doing so does not delay the
delivery of high-quality CPR (Panchal et al., 2020).

Known or Suspected COVID-19


As discussed in previous chapters, many organizations have adopted the practice of assuming that
all patients are COVID-19 positive until proved otherwise. The use of appropriate PPE is essential
to minimize rescuer exposure. Important considerations include the following (Edelson et al.,
2020):
• Limit personnel in the room to those necessary for the resuscitation effort.
• Consider the appropriateness of resuscitation.
• Ventilate using a BMD with a filter and tight face-to-mask seal or perform continuous compressions
while using a face mask.
• Limit aerosolization when giving oxygen.
• Close the door, when possible, to prevent airborne contamination of adjacent indoor space.
• Prepare to intubate (use the provider with the highest likelihood of success on the first attempt);
consider video laryngoscopy.
• If the patient is intubated at the time of the arrest, consider leaving them on a mechanical ventilator
with a high-efficiency particulate air (HEPA) filter to maintain a closed circuit and reduce aerosol-
ization. Adjust the ventilator settings to allow for asynchronous ventilation. If ROSC is achieved,
set the ventilator settings as appropriate to the patient’s clinical condition.
• Attempt to position the patient supine for resuscitation if they are initially in a prone position at
the time of the arrest and do not have an advanced airway in place.
• If the patient is in a prone position at the time of the arrest and does have an advanced airway in
place, consider placing defibrillator pads in the anterior-posterior position and provide CPR with
the patient remaining prone with the rescuer’s hands in the standard position over the T7/T10
vertebral bodies.

Cardiac Arrest and Pregnancy


It is estimated that one in 12,000 hospital admissions for delivery in the United States results in a
maternal cardiac arrest (Panchal et al., 2020). Within the last few decades, increasing maternal age and
the presence of comorbid diseases has increased a woman’s risk of obstetric complications, the likeli-
hood of cardiopulmonary arrest, and the need for resuscitation (Lavecchia & Abenhaim, 2015). Com-
mon causes of maternal cardiac arrest include hemorrhage, heart failure, amniotic fluid embolism,
sepsis, aspiration pneumonitis, venous thromboembolism, preeclampsia/eclampsia, and anesthesia
complications (Panchal et al., 2020). The risk of acute decompensation in critically ill pregnant patients
with COVID-19 may be increased because of the cardiopulmonary physiologic changes of pregnancy
(Edelson et al., 2020).
Priorities during a maternal cardiac arrest include performing high-quality CPR and relieving aor-
tocaval compression using left lateral uterine displacement (Panchal et al., 2020). Current resuscitation
guidelines recommend that local resources for a perimortem cesarean delivery (PMCD) be gathered
as soon as cardiac arrest in a woman in the second half of pregnancy is recognized (Panchal et al.,
2020). Additional recommendations regarding cardiac arrest in pregnancy include the following
( Jeejeebhoy et al., 2015, Panchal et al., 2020):
Oxygenation and ventilation
• Perform BMV with 100% oxygen flowing to the bag at a rate of at least 15 L/min.
• Continuous capnography, in addition to clinical assessment, should be used, if available, to assess
the correct placement of an endotracheal tube (ETT), to monitor CPR quality, to optimize chest
compressions, and to detect ROSC.
158 CHAPTER 7  Cardiac Arrest Rhythms

CPR
• Perform high-quality CPR; minimize interruptions in compressions.
• Provide continuous left lateral uterine displacement.
Defibrillation
• The same currently recommended defibrillation protocol should be used in the pregnant patient as
in the nonpregnant patient. Anterolateral pad placement is recommended; the lateral pad should be
placed under the breast tissue. Resume compressions immediately after defibrillation.
Vascular access and medications
• Establish IV access above the diaphragm to ensure that the gravid uterus does not obstruct the IV
administered therapy.
• Medication doses do not require modification to adapt to the physiologic changes of pregnancy.
Other considerations
• Search for and treat precipitating causes of the arrest.
• Fetal monitoring should not be performed during resuscitation because of potential interference
with maternal resuscitation.
• During cardiac arrest, if the pregnant patient (with a fundus height at or above the umbilicus)
has not achieved ROSC with usual resuscitation measures plus manual uterine displacement, it is
advisable to prepare to evacuate the uterus while resuscitation continues.
• PMCD should be strongly considered for every mother in whom ROSC has not been achieved
after about 5 minutes of resuscitative efforts. For patients with suspected or confirmed COVID-19,
experts recommend that perimortem delivery preparations begin early during the resuscitation
to permit the assembly of obstetric and neonatal teams with PPE, even if ROSC is achieved
and perimortem delivery is not required (Edelson et al., 2020). Factors to consider regarding
the decision to perform a cesarean delivery include the availability of appropriately trained
personnel, gestational age, etiology of the arrest, and available equipment and resources (Lavonas
et al., 2015).
• Targeted temperature management, with continuous fetal monitoring for bradycardia, is recom-
mended for pregnant women who remain comatose after resuscitation from cardiac arrest (Panchal
et al., 2020).

ACLS Pearl
Experts say that hypoxia should always be considered as a cause of maternal cardiac arrest be-
cause oxygen reserves are lower and the metabolic demands are higher in the pregnant patient
compared with the nonpregnant patient (Jeejeebhoy et al., 2015).

POSTCARDIAC ARREST CARE


After ROSC, postcardiac care is administered to reduce the risks of postcardiac arrest syndrome, which
includes brain injury, cardiac damage and dysfunction, systemic ischemia, and reperfusion injuries that
result from prolonged oxygen deprivation and restoration of circulation, and ongoing injury caused by
the precipitating cause of the arrest (Donnino et al., 2015; Institute of Medicine et al., 2015). Care of
the patient during this period has a strong effect on patient morbidity and mortality (Boutsikaris
& Winters, 2012). Best practices include a multidisciplinary team approach that includes personnel
from cardiology, interventional cardiology, cardiac electrophysiology, intensive care, and neurology
(Morrison et al., 2013).
While providing care, use the Hs and Ts to search for and treat potentially reversible causes of the
arrest and consider special circumstances, such as anaphylaxis, asthma, cardiac injury, cardiomyopathy,
choking, drowning, dysrhythmias, electrolyte disturbances, heart failure, hypothermia, myocarditis,
opioid overdose, poisoning, pulmonary disease, pulmonary embolism, sepsis, stroke, terminal cancer,
and trauma. Postcardiac arrest syndrome components appear in Table 7.3, and the postcardiac arrest
algorithm appears in Fig. 7.9.

Oxygenation and Ventilation


Immediately after a ROSC, repeat the primary assessment, and then perform a thorough physical
examination and assess vital signs.
CHAPTER 7  Cardiac Arrest Rhythms 159

TABLE 7.3 Components of Postcardiac Arrest Syndrome


Component Clinical Manifestations Possible Interventions
Hypoxic brain injury Coma TTM
Seizures Seizure control
Myoclonus
Varying degrees of neurocognitive dysfunction
(ranging from memory deficits to a persistent
vegetative state)
Stroke
Brain death
Myocardial dysfunction Circulatory collapse Coronary reperfusion
Dysrhythmias Hemodynamic support
Hypotension Mechanical support
(e.g., LVAD, intraaortic
balloon pump)
Systemic ischemia/ Circulatory collapse Hemodynamic support
reperfusion response Hypotension Temperature control
Hypovolemia
Multiorgan failure
Persistent precipitating Cause-specific (e.g., Hs and Ts, anaphylaxis, Disease-specific
cause asthma, cardiac injury, cardiomyopathy, choking, interventions
drowning, dysrhythmias, electrolyte disturbances,
heart failure, hypothermia, myocarditis, opioid
overdose, poisoning, pulmonary disease, pulmo-
nary embolism, sepsis, stroke, terminal cancer,
and trauma)

LVAD, Left ventricular assist device; TTM, targeted temperature management.

Reassess the effectiveness of initial airway maneuvers and interventions. If tolerated, elevate
the head of the bed 30 degrees to reduce the incidence of cerebral edema, aspiration, and ventilatory-
associated pneumonia. Apply a pulse oximeter and assess oxygen saturation. The highest available oxy-
gen concentration may be used during the period immediately after ROSC to avoid hypoxia until the
arterial oxyhemoglobin saturation or the partial pressure of arterial oxygen can be measured (Panchal
et al., 2020). When resources are available to reliably measure peripheral blood oxygen saturation,
titrating the fraction of inspired oxygen (FiO2) to target an oxygen saturation level (SpO2) of 92% to
98% is reasonable in patients who remain comatose after ROSC (Panchal et al., 2020).
Consider early placement of an ETT, if not already done. Assess and monitor the position of the
ETT and the effectiveness of ventilations with waveform capnography or capnometry. Mechanical
ventilation may be necessary for absent or inadequate spontaneous breathing. Begin ventilations at a
rate of 10 breaths/min and adjust as needed to achieve normal partial pressure of carbon dioxide
(PaCO2) levels (i.e., 35 to 45 mm Hg) (Panchal et al., 2020), unless patient factors prompt more indi-
vidualized treatment. Obtain a chest radiograph to confirm advanced airway placement and identify
potential breathing complications of resuscitation, such as pneumothorax, rib fractures, sternal frac-
tures, pneumonitis, pneumonia, or pulmonary edema (Callaway et al., 2015).

Cardiovascular Care
Heart rate and blood pressure are variable immediately after ROSC. Establish IV access with normal
saline or lactated Ringer’s solution if not already done. If IO access was used during the arrest, establish
an IV line to replace it when time permits. Insert a nasogastric tube and urinary catheter to monitor
intake and output.
Dysrhythmias that occur during the postcardiac arrest period should be treated in the same way as
that for a patient who has not had a cardiac arrest. The prophylactic administration of antiarrhythmic
medications refers to the giving of antiarrhythmics to prevent or reduce the risk of dysrhythmias. At
this time, there is insufficient evidence to support or refute the routine use of lidocaine within the first
hour after ROSC (Panchal et al., 2018). However, in the absence of contraindications, the prophylactic
160 CHAPTER 7  Cardiac Arrest Rhythms

Return of spontaneous circulation (ROSC)

Optimize oxygenation and ventilation

• Consider early endotracheal tube placement and waveform capnography or capnometry to


confirm and monitor tube position and the effectiveness of ventilations
• Avoid excessive ventilation; begin at 10 breaths/min (1 breath every 6 sec)
• Titrate FiO2 to achieve SpO2 of 92% to 98% and PaCO2 of 35 to 45 mm Hg

Treat hypotension

• Give IV/IO crystalloid solution (about 1 to 2 L normal saline or lactated Ringer’s solution)
and/or vasopressor or inotrope infusion as needed to achieve systolic BP greater than
90 mm Hg or mean arterial pressure greater than 65 mm Hg
o Norepinephrine IV infusion: 0.1 to 0.5 mcg/kg/min
o Epinephrine IV infusion: 2 to 10 mcg/min
o Dopamine IV infusion: 5 to 20 mcg/kg/min

Obtain 12-lead ECG as soon as possible

• Consider for emergent cardiac intervention if:


o ECG evidence of STEMI
o Unstable cardiogenic shock
o Mechanical circulatory support required

Does the patient follow commands?

• If yes, transfer for advanced critical care


• If no:
o Begin TTM as soon as possible if no contraindications
o Obtain brain computed tomography scan
o Perform continuous or intermittent EEG monitoring
o Initiate other critical care therapies as warranted (e.g., core temperature monitoring,
monitoring and maintaining normal respiratory parameters, optimizing mechanical
ventilation to minimize lung injury, EEG and glucose monitoring)
o If applicable, transfer the patient for comprehensive post–cardiac arrest care

Search for reversible cause TTM


• Hs and Ts plus consider special circumstances • TTM between 32° C and 36° C for at least 24
such as anaphylaxis, asthma, cardiac injury, hours after achieving the target temperature is
cardiomyopathy, drowning, dysrhythmias, recommended for adult patients who lack a
electrolyte disturbances, heart failure, meaningful response to verbal commands after
hypothermia, myocarditis, opioid overdose, ROSC from OHCA with any initial rhythm and
poisoning, pulmonary disease, pulmonary for IHCA with an initial shockable rhythm.
embolism, and sepsis • Routine prehospital cooling immediately after
ROSC with rapid infusion of cold intravascular
fluids is not recommended
BP = Blood pressure; ECG = electrocardiogram; EEG = electroencephalogram; IHCA = in-hospital cardiac arrest;
IO = intraosseous; IV = intravenous; OHCA = out-of-hospital cardiac arrest; STEMI = ST elevation myocardial infarction;
TTM = targeted temperature management.
(Source: Panchal et al., 2020.)
Fig. 7.9  Adult immediate postcardiac arrest care algorithm.

use of lidocaine may be considered in specific circumstances (such as during EMS transport) when
treatment of recurrent pVT/VF might prove challenging (Panchal et al., 2018). At present, there is
insufficient evidence to support the routine use of a beta blocker after cardiac arrest; however, the ini-
tiation or continuation of an oral or IV beta blocker may be considered early after hospitalization from
cardiac arrest because of pVT/VF (Panchal et al., 2018).
CHAPTER 7  Cardiac Arrest Rhythms 161

Current guidelines consider it reasonable to avoid and to immediately correct hypotension (i.e., sys-
tolic blood pressure less than 90 mm Hg, mean arterial pressure less than 65 mm Hg) during postcardiac
arrest care (Panchal et al., 2020). Administration of IV/IO fluid boluses, about 1 to 2 liters of normal
saline or lactated Ringer’s solution, may be necessary to restore intravascular volume, and the administra-
tion of vasoactive medications may be necessary to maintain perfusion if hypotension persists (Callaway
et al., 2015). For example, chronotropic agents may be needed to improve heart rate, inotropic agents
may be necessary to enhance myocardial contractility, vasoconstrictive medications may be needed to
increase arterial pressure, or vasodilators may be necessary to reduce afterload (Callaway et al., 2015).

Extracorporeal Cardiopulmonary Resuscitation


Extracorporeal membrane oxygenation (ECMO) is a therapy that uses a mechanical device to tem-
porarily support heart or lung function by pumping and oxygenating a patient’s blood outside the
body. There are two types of ECMO. With venous to venous (VV) ECMO, one or more catheters
are connected to one or more veins. VV ECMO is a type of “lung bypass” that provides gas exchange
but no mechanical circulatory support (Stoecklein et al., 2017). Examples of situations in which VV
ECMO is used include pneumonia, pulmonary contusion, status asthmaticus, aspiration or inhalation
injury, and drowning (Stoecklein et al., 2017), and as a bridge to lung transplantation. Venous to arte-
rial (VA), or “heart-lung” ECMO, provides gas exchange and mechanical circulatory support by
pumping oxygenated blood under pressure into the arterial system (Stoecklein et al., 2017). Examples
of situations in which VA ECMO is used include cardiac arrest, cardiogenic shock, poisoning and
drug overdose, pulmonary embolism, hypothermia, massive pulmonary hemorrhage, and sepsis
(Stoecklein et al., 2017).
Because ECMO is a method of circulating blood during cardiac arrest, it is sometimes referred to
as extracorporeal life support (ECLS) or extracorporeal cardiopulmonary resuscitation (ECPR)
(Stoecklein et al., 2017). By maintaining organ perfusion, ECPR gives patient support through the
following (Bharmal et al., 2019; Holmberg et al., 2018):
• Providing time for recuperation from the underlying cause of the arrest
• Serving as a bridge to cardiac transplantation or implantation of a left ventricular assist device when
damage to the heart is irreparable
• Allowing time for diagnostics and definitive therapy for corrective interventions, such as coronary
revascularization in cases of ischemia and/or treatment of potentially reversible causes of the cardiac
arrest.
Current resuscitation guidelines note that there is insufficient evidence to recommend the routine
use of ECPR for patients with cardiac arrest. ECPR may be considered for select cardiac arrest patients
for whom the suspected cause of the cardiac arrest is potentially reversible during a limited period of
mechanical cardiorespiratory support (Panchal et al., 2020).
Implementation of an ECPR program is resource-intensive, requiring specialized equipment and a
well-organized and specially trained multidisciplinary team. ECPR is time-critical and should be ini-
tiated within 60 minutes of the onset of cardiac arrest (Hutin et al., 2018). Complications of ECPR
include bleeding (with the cannulation site being the most common source), limb ischemia, vascular
injury, infection, kidney failure, and stroke.

Emergent Coronary Angiography


After ROSC, all patients should receive continuous ECG monitoring, and a 12-lead ECG should
be obtained as soon as possible to determine whether acute ST segment elevation (STE) is present
(Panchal et al., 2020). It is estimated that 26% to 31% of all postarrest patients have ECG findings
consistent with ST elevation myocardial infarction (STEMI) (Madder & Reynolds, 2018). Among
OHCA patients resuscitated from pVT/VF who had STE on their postarrest ECG, the prevalence of
coronary artery disease (CAD) is 70% to 85%, and more than 90% of these patients had successful PCI
(Yannopoulos et al., 2019).
Experts observe that early access to the cardiac catheterization laboratory (CCL) has the potential
to improve outcomes in OHCA patients and serves three roles in these patients (Yannopoulos et al.,
2019):
1. It provides a more informed diagnosis of CAD or its absence, which can better guide future therapy
even in the absence of a PCI.
2. It allows immediate PCI, which improves hemodynamics and prognosis comparable to patients
with acute coronary syndromes.
162 CHAPTER 7  Cardiac Arrest Rhythms

3. It provides access to circulatory assist devices that can be used to support and stabilize patients
beyond standard medical therapy.
Emergent coronary angiography is recommended for all patients with a presumed cardiac cause of
arrest and with STE (Panchal et al., 2020). It is considered reasonable for electrically or hemody-
namically unstable patients who are comatose after OHCA of suspected cardiac origin but without
STE (Panchal et al., 2020).
Experts have proposed a treatment algorithm to identify appropriate care for comatose survivors of
cardiac arrest and to identify patients who are unlikely to receive substantial benefit from an early
invasive approach. Clinical features that suggest that the patient is less likely to benefit from coronary
intervention include the following (Rab et al., 2015):
• Unwitnessed arrest
• Initial nonshockable cardiac arrest rhythm
• No immediate bystander CPR
• Prolonged resuscitation effort (more than 30 minutes without ROSC)
• Evidence of unresponsive hypoperfusion and microcirculatory failure (pH less than 7.2, lactate
level higher than 7 mmol/L)
• Age greater than 85 years
• Significant comorbid conditions (e.g., end-stage renal disease on dialysis, advanced dementia,
chronic ventilator dependence, respiratory failure, severe frailty and disability)
• Noncardiac cause of the cardiac arrest (e.g., drugs, drowning, choking, acute stroke, respiratory
failure, terminal cancer, trauma)

Neurologic Care
To limit tissue injury after cardiac arrest, targeted temperature management (TTM) between 32 C
and 36 C for at least 24 hours after achieving the target temperature is recommended for adult pa-
tients who lack a meaningful response to verbal commands after ROSC from OHCA with any initial
rhythm, and for IHCA with an initial shockable rhythm (Panchal et al., 2020). The routine prehospital
cooling of patients immediately after ROSC with rapid infusion of cold intravascular fluids is not
recommended (Panchal et al., 2020).
Clinical manifestations of postcardiac arrest brain injury include coma, seizures, myoclonus, various
degrees of neurocognitive dysfunction (ranging from memory deficits to a persistent vegetative state),
and brain death (Callaway et al., 2015). Because seizures after a cardiac arrest may be caused by, as well
as worsen, postcardiac arrest brain injury, an electroencephalogram should be promptly performed and
interpreted and then should be monitored frequently or continuously in comatose survivors of cardiac
arrest (Panchal et al., 2020). Treat postarrest seizures, if present. Common sedatives, such as valproate,
levetiracetam, fosphenytoin, propofol, and midazolam may be effective in suppressing seizure activity
(Panchal et al., 2020).

Other Considerations
Closely monitor the patient’s body temperature. Because fever can impair brain recovery by creating
an imbalance between oxygen supply and demand and it is associated with worsened neurologic out-
come after cardiac arrest, the treatment or prevention of fever is considered a reasonable approach after
ROSC (Panchal et al., 2020).
The optimum adult blood glucose target range in the postcardiac arrest period is unknown. The
resuscitation guidelines recommend using the approach used for the general critically ill population,
which is using insulin therapy when needed to maintain a blood glucose of 150 to 180 mg/dL (Panchal
et al., 2020).
The routine use of steroids for patients with shock after ROSC is of uncertain value; however, be-
cause research has shown improved survival in patients treated with steroids for septic shock, clinicians
should consider the co-occurrence of sepsis and cardiac arrest (Panchal et al., 2020).
CHAPTER 7  Cardiac Arrest Rhythms 163

STOP AND REVIEW


Identify one or more choices that best complete the statement or answer the question.

_ ___ 1. During a cardiac arrest, ventilations should be delivered at a rate of:


a. 4 to 5 breaths/min.
b. 8 breaths/min.
c. 10 breaths/min.
d. 15 breaths/min.

____ 2. In addition to the delivery of high-quality CPR, epinephrine should be administered


as soon as possible for which of the following?
a. VF
b. pVT
c. PEA
d. Asystole

____ 3. A 49-year-old man is found unresponsive, not breathing, and pulseless. The cardiac
monitor reveals monomorphic VT. The two most important actions in the manage-
ment of this patient are:
a. CPR.
b. Defibrillation.
c. Vascular access.
d. Synchronized cardioversion.
e. Immediate insertion of an advanced airway.
f. Administration of resuscitation medications.

____ 4. When a monophasic defibrillator is used for shockable cardiac arrest rhythms, the
initial recommended energy dose is:
a. 120 to 150 J.
b. 120 to 200 J.
c. 200 J.
d. 360 J.

_ ___ 5. Select the correct statements about lidocaine dosing during cardiac arrest.
a. The maximum IV/IO bolus dose is 3 mg/kg.
b. The initial dose is 1 to 1.5 mg/kg IV/IO push.
c. Lidocaine is given as a continuous infusion of 10 to 20 mcg/kg/min.
d. Repeat doses of 0.5 to 0.75 mg/kg IV/IO push may be given at 5 to 10 minute
intervals.

____ 6. A 75-year-old man is on the telemetry floor recovering from an inferior wall myo-
cardial infarction. The nursing staff arrives in the patient’s room in response to an
alarm from his cardiac monitor, which reveals a sinus bradycardia at 40 beats/min.
The patient is unresponsive, pulseless, and apneic. An IV is in place. Which of the
following should be performed now?
a. Begin CPR.
b. Give atropine IV.
c. Give epinephrine IV.
d. Ventilate with a BMD.
e. Defibrillate immediately.
f. Insert an advanced airway.
g. Begin transcutaneous pacing.
164 CHAPTER 7  Cardiac Arrest Rhythms

____ 7. An 88-year-old woman has experienced a cardiac arrest, and multiple attempts to
establish vascular access have been unsuccessful. An ETT is in place. Which of
the following cardiac arrest medications can be given via the tracheal route if
necessary?
a. Atropine
b. Naloxone
c. Lidocaine
d. Epinephrine

____ 8. When caring for a patient in cardiac arrest with a shockable rhythm, the AHA
recommends giving the first dose of epinephrine:
a. Before delivering the first shock.
b. Immediately after delivery of the first shock.
c. After delivery of a second shock, assuming a shockable rhythm persists.
d. After delivery of a third shock, assuming a shockable rhythm persists.

_ ___ 9. Persistent hypotension after ROSC should initially be managed by:


a. Hyperventilating the patient.
b. Considering treatable causes.
c. Giving an antiarrhythmic infusion.
d. Administering a bolus of normal saline or lactated Ringer’s solution.

_ ___ 10. Administration of which of the following may be necessary after ROSC?
a. Inotropic medications to reduce afterload
b. IV/IO fluid boluses to restore intravascular volume
c. Vasoconstrictive medications to increase arterial pressure
d. Chronotropic medications to enhance myocardial contractility
CHAPTER 7  Cardiac Arrest Rhythms 165

CASE STUDY 7.1


Identify one or more choices that best complete the statement or answer the question.

You and your EMS crew are called for a 54-year-old woman who was found unresponsive on the
floor of her home by a neighbor. Compression-only CPR is in progress when you arrive. There are no
visible signs of trauma. Emergency equipment, including a biphasic manual defibrillator with transcu-
taneous pacing (TCP), is available to you. As you work through this case study, remember that there
may be alternative actions that are perfectly acceptable, yet not discussed here.

____ 1. As you approach the patient, you observe that she is supine on the floor. Her eyes
are closed, her lips are blue, and her skin is pale. You see no signs of breathing.
Select the correct statements about this patient’s general impression findings.
a. Appearance is normal.
b. Appearance is abnormal.
c. Breathing is normal.
d. Breathing is abnormal.
e. Circulation is normal.
f. Circulation is abnormal.

____ 2. Your assessment reveals that the patient is unresponsive, apneic, and pulseless.
Which of the following actions should occur at the same time?
a. Obtain the patient’s vital signs.
b. Instruct your team to begin CPR.
c. Instruct a team member to insert an advanced airway.
d. Instruct a team member to attach adhesive pads to the patient.

____ 3. How will you ensure the performance of high-quality chest compressions throughout
this resuscitation effort?
a. Minimize interruptions in compressions.
b. Ventilate at a rate of 12 to 15 breaths/min.
c. Allow full chest recoil after each compression.
d. Ensure chest compressions are performed of a rate of 80 to 100/min.

____ 4. Adhesive pads have been placed on the patient’s chest and the monitor/defibrillator
screen shows pVT at a rate of 180 beats/min. After clearing the patient, she is defi-
brillated. What should be done next?
a. Pause for a 10-second pulse check.
b. Pause for a 10-second rhythm check.
c. Ask a team member to establish vascular access.
d. Resume CPR without pausing for a pulse or rhythm check.

____ 5. While chest compressions continue, which of the following should be performed
by the team member responsible for managing the patient’s airway?
a. Insert an oropharyngeal airway.
b. Use a head tilt–chin lift to open the airway.
c. Assess the patient’s baseline breath sounds.
d. Ventilate with a BMD connected to 100% oxygen.

____ 6. After performing CPR for 2 minutes, a rhythm check shows VF on the monitor,
and the patient is defibrillated. Two attempts at IV access have proved unsuccessful.
Which of the following should be performed at this time?
a. Attempt IO access.
b. Continue high-quality CPR.
c. Consider advanced airway insertion.
d. Give an antiarrhythmic as soon as vascular access is achieved.
166 CHAPTER 7  Cardiac Arrest Rhythms

____ 7. IO insertion is successful, and epinephrine is administered. An ETT is inserted, its


proper position confirmed, and the tube is secured in place. Waveform capnogra-
phy has been initiated. After 2 minutes of CPR, a rhythm check reveals a sinus
tachycardia at a rate of 130 beats/min. Which of the following actions should be
taken at this time?
a. Check for a pulse.
b. Attach a pulse oximeter.
c. Obtain the patient’s vital signs.
d. Repeat the primary assessment.

____ 8. A carotid pulse is present. The patient is breathing shallowly about 8 times per
minute but remains unresponsive. What should be done now?
a. Obtain a 12-lead ECG.
b. Prepare the patient for transport.
c. Assist the patient’s breathing with a BMD.
d. Stop BMV and apply a nonrebreather mask.
e. Attempt to locate someone who can provide additional patient information.

____ 9. As the patient is transferred to the ambulance, her heart rate is strong and regular,
but she remains unresponsive. Her blood pressure is 98/60 mm Hg, and the cardiac
monitor shows sinus tachycardia at 118 beats/min. Ventilations are being
assisted with a BMD. A 12-lead ECG reveals STE in leads V2, V3, and V4. The
following information has been obtained:
Signs/Symptoms: Found unresponsive by a neighbor
Allergies: Unknown
Medications: Augmentin, alendronate (Fosamax)
Past history: Osteoporosis, sinus infection 4 days ago
Last oral intake: Unknown
Events prior: Found unresponsive on the kitchen floor by a neighbor who had last
spoken to the patient about 25 minutes prior
Considering the events associated with this resuscitation effort, which of the
following suggest that this patient may benefit from emergent coronary angiography?
a. Unwitnessed arrest
b. Initial shockable rhythm
c. No bystander CPR
d. Less than 30 minutes to ROSC
e. Age younger than 85 years
f. Significant comorbid conditions

_ ___ 10. Which of the following actions should be performed at this time?
a. Begin transport to the nearest Cardiac Arrest Center.
b. Monitor the patient’s vital signs and ECG every 5 minutes.
c. Titrate oxygen delivery to achieve an SpO2 of 92% to 98%.
d. Begin cooling the patient with rapid infusion of cold IV fluids.
e. Arrange for a team debriefing after the patient’s transfer of care is complete.
f. Titrate ventilations to maintain the patient’s EtCO2 with a range of 35 to
45 mm Hg.
CHAPTER 7  Cardiac Arrest Rhythms 167

CASE STUDY 7.2


Identify one or more choices that best complete the statement or answer the question.

You and your team are summoned to the bedside of a 75-year-old woman who was admitted an
hour ago for shortness of breath. The nurse at the bedside says that she found the patient unresponsive
upon entering the room minutes ago. An IV is in place. Emergency equipment, including a biphasic
manual defibrillator with TCP capability, is available. As you work through this case study, remember
that there may be alternative actions that are perfectly acceptable, yet not discussed here.

____ 1. You observe that the patient is supine on a stretcher and estimate her weight to be
about 100 kg. Her eyes are closed, and her skin is blue from the chest up. You do
not observe any rise and fall of her chest. Select the correct statements about this
patient’s general impression findings.
a. Appearance is normal.
b. Appearance is abnormal.
c. Breathing is normal.
d. Breathing is abnormal.
e. Circulation is normal.
f. Circulation is abnormal.

____ 2. Your assessment reveals that the patient is unresponsive, apneic, and pulseless.
Which of the following actions should occur at this time?
a. Ask your team to begin CPR.
b. Attach the patient to a cardiac monitor.
c. Instruct a team member to administer epinephrine IV.
d. Ask a team member to prepare to perform TCP.

____ 3. While CPR continues, the bedside nurse says that the patient was admitted for
shortness of breath that began at home last evening and worsened this morning.
The patient and her husband arrived home yesterday after a 14-hour drive follow-
ing an out of state visit with family. This morning she told her husband she had
left-sided chest pain when taking a deep breath. A subsequent call to her primary
care physician resulted in her hospital admission. Her vital signs 1 hour ago were
as follows: Temperature 99 F (37.2 C), BP 118/90 mm Hg, ventilatory rate
24 breaths/min, heart rate 98 beats/min, and SpO2 95% (room air). Breath sounds
were clear. The cardiac monitor reveals a sinus tachycardia at
120 beats/min. Which of the following actions should be taken at this time?
a. Request a cardiology consult.
b. Request bedside sonography.
c. Ask a team member to administer epinephrine IV.
d. Ask a team member to administer amiodarone IV.
e. Ask a qualified team member to insert an advanced airway.

____ 4. A team member has intubated the patient. Initial assessments to confirm the correct
placement of the ETT should include which of the following?
a. Obtaining a chest radiograph
b. Observing adequate chest rise with each ventilation
c. Auscultating the presence of bilateral breath sounds
d. Visualizing the passage of the ETT between the vocal cords
e. Confirming the absence of sounds over the epigastrium during ventilation
168 CHAPTER 7  Cardiac Arrest Rhythms

____ 5. Continuous waveform capnography is being used following ETT insertion.


The patient’s EtCO2 value is presently 12 mm Hg. Considering the information
available so far, which of the following are the most likely possible cause(s) of the
patient’s cardiac arrest?
a. Hypoxia
b. Hypothermia
c. Hyperkalemia
d. Toxic ingestion
e. Cardiac tamponade
f. Pulmonary embolism

____ 6. CPR is ongoing, and epinephrine is being administered IV every 3 minutes. After
10 minutes, a team member calls out that the patient’s EtCO2 value has changed
from 12 to 38 mm Hg. Her cardiac rhythm is unchanged (i.e., sinus tachycardia).
Which of the following actions are appropriate at this time?
a. Perform a pulse check.
b. Prepare a lidocaine infusion.
c. Administer amiodarone IV push.
d. Prepare to perform synchronized cardioversion.

_ ___ 7. A pulse is present. Actions to be taken at this time include which of the following?
a. Perform a primary assessment and obtain the patient’s vital signs.
b. Obtain a 12-lead ECG.
c. Obtain blood samples for laboratory testing.
d. Consider targeted temperature management.

____ 8. The patient’s vital signs are as follows: Temperature 99 F (37.2 C), BP 100/70 mm Hg,
pulse 120 beats/min, SpO2 90%. Shallow, spontaneous respirations are present at 8
breaths/min. You should:
a. Continue assisting the patient’s breathing with a BMD.
b. Prepare a dopamine infusion to optimize the patient’s blood pressure.
c. Arrange for the patient’s transfer to the critical care unit for ongoing care.
d. Arrange for a team debriefing after the patient’s transfer of care is complete.

STOP AND REVIEW  ANSWERS


1. C. Ventilate the adult patient who has experienced a respiratory or cardiac arrest at a rate of 1 breath
every 6 seconds, or about 10 breaths/min.

2. C, D. Current guidelines state that epinephrine should be given as soon as feasible after the onset
of cardiac arrest associated with an initial nonshockable rhythm (i.e., PEA or asystole).

3. A, B. CPR and defibrillation are the most critical interventions for the patient in cardiac arrest
associated with pVT or VF. Insertion of advanced airways and administration of resuscitation
medications are of secondary importance. Although synchronized cardioversion may be used in the
treatment of an unstable patient in monomorphic VT with a pulse, it is not indicated for pVT.

4. D. When a monophasic defibrillator is used for shockable cardiac arrest rhythms, the recommended
energy dose is 360 J for all shocks.

5. A, B, D. During cardiac arrest, lidocaine is given IV push. The initial dose of lidocaine is 1 to
1.5 mg/kg IV/IO. Repeat doses of 0.5 to 0.75 mg/kg IV/IO push may be given at 5 to 10 minute
intervals, to a maximum IV/IO bolus dose of 3 mg/kg. Upon ROSC, consider a continuous infusion
of 1 to 4 mg/min.
CHAPTER 7  Cardiac Arrest Rhythms 169

6. A, C, D. Although an organized rhythm is present on the monitor, the patient has no pulse. This
clinical situation is PEA. You should begin CPR immediately, ventilate the patient with a BMD,
and give epinephrine 1 mg IV. Transcutaneous pacing, defibrillation, and atropine administration
are not indicated for PEA.

7. B, C, D. If vascular access cannot be achieved to give drugs during a cardiac arrest, the tracheal
route can be used to administer selected medications (e.g., epinephrine, lidocaine, naloxone). The
tracheal route is not preferred because intravascular drug administration provides more predictable
drug delivery and pharmacologic effect, and multiple studies have shown that giving drugs trache-
ally results in lower blood concentrations than the same dose given intravascularly. Although atro-
pine can also be administered via an ETT, it is no longer recommended for use during an adult
cardiac arrest.

8. C. The AHA recommends giving epinephrine after delivery of a second shock if pVT/VF persists.

9. B, D. Persistent hypotension (i.e., a systolic BP less than 90 mm Hg) after ROSC should initially
be managed by administering 1 to 2 L of normal saline or lactated Ringer’s solution IV/IO bolus,
followed by giving a vasopressor infusion (i.e., epinephrine, dopamine, or norepinephrine) if
needed, and considering treatable causes.

10. B, C. After ROSC, administration of IV/IO crystalloid fluid boluses may be necessary to restore
intravascular volume, and the administration of vasoactive medications may be necessary to main-
tain perfusion if hypotension persists. For example, chronotropic agents may be needed to improve
heart rate, inotropic agents may be necessary to enhance myocardial contractility, vasoconstrictive
medications may be needed to increase arterial pressure, or vasodilators may be necessary to reduce
afterload.

C A S E S T U D Y 7 . 1 A NSWERS
1. B, D, F. The general impression findings are abnormal. Appearance is abnormal, breathing is abnormal,
and circulation is abnormal.

2. B, D. Ask your team to begin CPR and attach the monitor/defibrillator to the patient. Because the
patient is apneic and pulseless, obtaining her vital signs is unnecessary. At this point, the patient can
be ventilated using a BMD rather than asking a team member to insert an advanced airway.

3. A, C. High-quality chest compressions require compressing the chest at an adequate rate (i.e., 100 to
120/min) and depth (at least 2 inches), maximizing the chest compression fraction (i.e., at least 60%)
by minimizing pauses in chest compressions, allowing full chest recoil after each compression, and
avoiding excessive ventilation. The chest compressor and another (predesignated) team member
should rotate positions when chest compressions are interrupted (e.g., while performing a rhythm
check, when a shock is being delivered) to avoid tiring. Ventilate the patient at a rate of 1 breath
every 6 seconds, or about 10 breaths/min with just enough volume to see the chest rise gently.

4. C, D. After the shock is delivered, instruct the team to resume CPR without pausing for a pulse or
rhythm check. Ask a team member to establish vascular access.

5. A, C, D. Although there are no visible signs of trauma, the airway team member should open the
patient’s airway using a jaw thrust because the patient was found on the floor, and you cannot rule
out trauma because of a possible fall injury. After looking in the mouth for blood, broken teeth or
loose dentures, gastric contents, and foreign objects and suctioning the upper airway if necessary,
the airway team member should size and insert an oropharyngeal airway and then begin positive
pressure ventilation with a BMD connected to 100% oxygen. Assess the patient’s baseline breath
sounds while the patient is being ventilated.
170 CHAPTER 7  Cardiac Arrest Rhythms

6. A, B, C. Continue high-quality CPR. If peripheral IV access cannot be quickly achieved, attempt


IO access. If vascular access is achieved, give a vasopressor (i.e., epinephrine). It is too early to give
an antiarrhythmic. Consider the placement of an advanced airway while CPR continues. If an
advanced airway is inserted, initiate capnography to determine the adequacy of CPR and monitor
for ROSC. If an ETT is inserted and its proper position confirmed, epinephrine could be given
using the tracheal route if vascular access had not yet been obtained. The tracheal route is not
preferred because intravascular drug administration provides more predictable drug delivery and
pharmacologic effect.

7. A, B, C, D. Check for a pulse and repeat the primary assessment. If a pulse is present, obtain
the patient’s vital signs. Ask a team member to attach a pulse oximeter and ECG monitor, if not
already done.

8. A, B, C, E. Ask the airway team member to continue to assist the patient’s breathing with a BMD
connected to O2 at a flow rate of 10 to 15 L/min. Obtain a 12-lead ECG. While preparing the
patient for transport, find out if there is someone available who can provide additional information
about the patient so that factors that may have caused the arrest can be identified and treated.

9. B, D, E. Emergent coronary angiography is recommended for all patients with a presumed cardiac
cause of arrest and with STE. It is considered reasonable for electrically or hemodynamically un-
stable patients who are comatose after OHCA of suspected cardiac origin but without STE. Of
the resuscitation effort features listed, those suggesting that the patient may benefit from emergent
coronary angiography include an initial shockable rhythm, less than 30 min to ROSC, and age
younger than 85 years.

10. A, B, C, E, F. In transit to the nearest Cardiac Arrest Center (if consistent with your local protocol),
monitor the patient’s vital signs and ECG every 5 minutes. Titrate oxygenation and ventilations to
maintain the patient’s SpO2 between 92% and 98% and EtCO2 between 35 and 45 mm Hg. The
routine prehospital cooling of patients after ROSC with rapid infusion of cold IV fluids is not
recommended. Arrange for a team debriefing after the patient’s transfer of care is complete.

C A S E S T U DY 7.2 ANSWERS
1. B, D, F. The general impression findings are abnormal. Appearance is abnormal, breathing is abnormal,
and circulation is abnormal.

2. A, B. Ask your team to begin high-quality CPR. Ask another team member to attach the patient
to a cardiac monitor. At the same time, ask the bedside nurse about the reason for the patient’s
admission, any pertinent assessment findings, and the results of any diagnostic tests that may have
been completed. Although an IV is in place, it is premature to administer epinephrine until you
know if the patient’s cardiac rhythm is shockable or nonshockable. If it is a shockable rhythm,
defibrillation takes priority over epinephrine administration. TCP is not appropriate at this time
because: (1) the patient’s cardiac rhythm is not known, and (2) TCP is not used in the management
of cardiac arrest rhythms.

3. A, B, C, E. At this point, sinus tachycardia is visible on the monitor, but the patient has no palpable
pulse. The clinical situation is PEA. It is appropriate to request a cardiology consult and to request
bedside sonography to differentiate between true PEA and pseudo-PEA. While CPR continues,
additional actions that should be taken at this time include administering IV epinephrine and ask-
ing a qualified team member to insert an advanced airway. Amiodarone, which is an antiarrhythmic,
is not used in the management of nonshockable rhythms.

4. B, C, D, E. Clinical assessments, such as visualizing the passage of the ETT between the vocal
cords, auscultating the presence of bilateral breath sounds, confirming the absence of sounds over
the epigastrium during ventilation, and observing adequate chest rise with each ventilation are
CHAPTER 7  Cardiac Arrest Rhythms 171

initially used to confirm correct placement of an ETT. Secondary methods that may be used include
the use of continuous waveform capnography (preferred for confirming and monitoring correct
ETT placement in a patient who has adequate tissue perfusion), use of a nonwaveform CO2 detec-
tor, use of an esophageal detector device, obtaining a chest radiograph, and use of ultrasound imag-
ing. After proper ETT position is confirmed, ask the airway team member to note the centimeter
markings on the tube and then secure the tube in place.

5. A, F. Of the causes listed, the most likely are hypoxia and pulmonary embolism.

6. A. Because an increase in the patient’s EtCO2 value (i.e., from 12 to 38 mm Hg) suggests ROSC,
perform a pulse check. None of the other options listed are appropriate at this time.

7. A, B, C, D. Perform (or direct a team member to perform) a primary assessment and obtain the
patient’s vital signs. Also obtain a 12-lead ECG and blood samples for laboratory testing. Consider
targeted temperature management.

8. A, C, D. Monitor the patient’s vital signs and ECG every 5 minutes. Continue assisting the
patient’s breathing with a BMD until she can be connected to a ventilator. Titrate oxygenation
and ventilations to maintain the patient’s SpO2 between 92% and 98% and EtCO2 between 35 and
45 mm Hg. Arrange for the patient’s transfer to the critical care unit for ongoing care. Arrange for
a team debriefing after the patient’s transfer of care is complete.

REFERENCES
Benjamin, E. J., Muntner, P., Alonso, A., et al. (2019). Heart disease and stroke statistics-2019 update: A report
from the American Heart Association. Circulation, 139(10), e56–e528.
Benjamin, E. J., Virani, S. S., Callaway, C. W., et al. (2018). Heart disease and stroke statistics—2018 Update: A
report from the American Heart Association. Circulation, 137(12), e67–e492.
Bharmal, M. I., Venturini, J. M., Chua, R. F., et al. (2019). Cost-utility of extracorporeal cardiopulmonary
resuscitation in patients with cardiac arrest. Resuscitation, 136, 126–130.
Boutsikaris, D., & Winters, M. E. (2012). Postresuscitation care. Emerg Med Clin North Am, 30(1), 123–140.
Callaway, C. W., Donnino, M. W., Fink, E. L., et al. (2015). American Heart Association Guidelines for CPR & ECC.
Retrieved from American Heart Association. In Web-based integrated guidelines for cardiopulmonary resus-
citation and emergency cardiovascular care – Part 8: Post–cardiac arrest care: Eccguidelines.heart.org.
Cardiac Arrest Registry to Enhance Survival (CARES). (2017). 2017 CARES Non-Traumatic National Survival
Report. Retrieved from https://mycares.net/sitepages/uploads/2018/2017%20Non-Traumatic%20National%20
Survival%20Report.pdf.
Cardiac Arrest Registry to Enhance Survival (CARES). (2017). 2017 CARES Summary Report: Demographic
and survival characteristics of OHCA. Retrieved from https://mycares.net/sitepages/uploads/2018/2017%20
Non-Traumatic%20National%20Summary%20Report.pdf.
Debaty, G., Babaz, V., Durand, M., et al. (2017). Prognostic factors for extracorporeal cardiopulmonary resuscitation
recipients following out-of-hospital refractory cardiac arrest. A systematic review and meta-analysis. Resuscita-
tion, 112, 1–10.
Donnino, M. W., Andersen, L. W., Berg, K. M., et al. (2015). Temperature management after cardiac arrest: An
advisory statement. Circulation, 132(25), 2448–2456.
Edelson, D. P., Sasson, C., Chan, P. S., et al. (2020). Interim guidance for basic and advanced life support in adults,
children, and neonates with suspected or confirmed COVID-19. Circulation, e20201405 [online ahead of
print].
Holmberg, M. J., Geri, G., Wiberg, S., et al. (2018). Extracorporeal cardiopulmonary resuscitation for cardiac
arrest: A systematic review. Resuscitation, 131, 91–100.
Hutin, A., Abu-Habsa, M., Burns, B., et al. (2018). Early ECPR for out-of-hospital cardiac arrest: Best practice
in 2018. Resuscitation, 130, 44–48.
Institute of Medicine, Graham, R., McCoy, M. A., & Schultz, A. M. (Eds.). (2015). Strategies to improve cardiac
arrest survival: A time to act. Washington, DC: National Academies Press.
Jeejeebhoy, F. M., Zelop, C. M., & Lipman, S., et al. (2015). Cardiac arrest in pregnancy: A scientific statement
from the American Heart Association. Circulation, 132(18), 1747–1773.
Lavecchia, M., & Abenhaim, H. A. (2015). Cardiopulmonary resuscitation of pregnant women in the emergency
department. Resuscitation, 91, 104–107.
172 CHAPTER 7  Cardiac Arrest Rhythms

Lavonas, E. J., Drennan, I. R., Gabrielli, A., et al. (2015). American Heart Association Guidelines for CPR & ECC.
Retrieved from American Heart Association. In Web-based integrated guidelines for cardiopulmonary resus-
citation and emergency cardiovascular care – Part 10: Special circumstances of resuscitation: Eccguidelines.
heart.org.
Li, Y., & Tang, W. (2012). Optimizing the timing of defibrillation: The role of ventricular fibrillation waveform
analysis during cardiopulmonary resuscitation. Crit Care Clin, 28(2), 199–210.
Link, M. S., Berkow, L. C., Kudenchuk, P. J., et al. (2015). American Heart Association Guidelines for CPR & ECC.
Retrieved from American Heart Association. In Web-based integrated guidelines for cardiopulmonary resus-
citation and emergency cardiovascular care – Part 7: Adult advanced cardiovascular life support: Eccguidelines.
heart.org.
Madder, R. D., Reynolds, J. C. (2018). Multidisciplinary management of the post–cardiac arrest patient. Cardiol
Clin, 36(1), 85–101.
Meaney, P. A., Bobrow, B. J., Mancini, M. E., et al. (2013). Cardiopulmonary resuscitation quality: [corrected]
Improving cardiac resuscitation outcomes both inside and outside the hospital: A consensus statement from
the American Heart Association. Circulation, 128(4), 417–435.
Morrison, L. J., Neumar, R. W., Zimmerman, J. L., et al. (2013). Strategies for improving survival after in-hospital
cardiac arrest in the United States: 2013 consensus recommendations. Circulation, 127(14), 1538–1563.
Myerburg, R. J. (2020). Approach to cardiac arrest and life-threatening arrhythmias. In L. Goldman, &
A. I. Schafer (Eds.), Goldman-Cecil medicine (26th ed., pp. 327–331). Philadelphia, PA: Elsevier.
Myerburg, R. J., & Goldberger, J. J. (2019). Cardiac arrest and sudden cardiac death. In D. P. Zipes, P. Libby, R.
O. Bonow, D. L. Mann, G. F. Tomaselli, & E. Braunwald (Eds.), Braunwald’s heart disease: A textbook of cardio-
vascular medicine (11th ed., pp. 807–847). Philadelphia, PA: Elsevier.
Panchal, A. R., Bartos, J. A., Cabañas, J. G., et al. (2020). Part 3: Adult basic and advanced life support: 2020
American Heart Association guidelines for cardiopulmonary resuscitation and emergency cardiovascular care.
Circulation, 142(16 suppl 2), S366–S468.
Panchal, A. R., Berg, K. M., Hirsch, K. G., et al. (2019a). 2019 American Heart Association focused update on
advanced cardiovascular life support: Use of advanced airways, vasopressors, and extracorporeal cardiopulmonary
resuscitation during cardiac arrest. Circulation, 140(24), e881–e894.
Panchal, A. R., Berg, K. M., Kudenchuk, P. J., et al. (2018). 2018 American Heart Association focused update on
advanced cardiovascular life support use of antiarrhythmic drugs during and immediately after cardiac arrest.
Circulation, 138(23), e740–e749.
Panchal, A. R., Duff, J. P., Escobedo, M. B., Pellegrino, J. L., Charlton, N., & Hazinski, M. F. (2019b, Nov). High-
lights of the 2019 focused updates to the American Heart Association guidelines for cardiopulmonary resuscitation and
emergency cardiovascular care. Retrieved from https://www.google.com/url?sa=t&rct=j&q=&esrc=s&source=we
b&cd=&cad=rja&uact=8&ved=2ahUKEwjL_PHRvcnwAhUNK80KHZj5B4oQFjAAegQIBhAD&url=http
s % 3 A % 2 F % 2 Fc p r. h e a r t . o r g % 2 F - % 2 F m e d i a % 2 Fc p r - fi l e s % 2 Fre s u s - s c i e n c e % 2 Fe c c - d i g i t a l -
digest%2Fhighlights-update.pdf%3Fla%3Den&usg=AOvVaw0lN53LXlEjWCi3WPfehy_6
Rab, T., Kern, K. B., Tamis-Holland, J. E., et al. (2015). Cardiac arrest: A treatment algorithm for emergent inva-
sive cardiac procedures in the resuscitated comatose patient. J Am Coll Cardiol, 66(1), 62–73.
Rabjohns, J., Quan, T., Boniface, K., & Pourmand, A. (2019). Pseudo-pulseless electrical activity in the emergency
department, an evidence based approach. Am J Emerg Med, 19, 30652–30657.
Salen, P., Melniker, L., Choollijian, C., Rose, J. S., Alteveer, J., & Heller, M. (2005). Does the presence or absence
of sonographically identified cardiac activity predict resuscitation outcomes of cardiac arrest patients? Am J
Emerg Med, 23(4), 459–462.
Shanmugasundaram, M., & Lotun, K. (2018). Refractory out of hospital cardiac arrest. Curr Cardiol Rev, 14(2),
109–114.
Siao, F. Y., Chiu, C. C., Chiu, C. W., et al. (2015). Managing cardiac arrest with refractory ventricular fibrillation
in the emergency department: Conventional cardiopulmonary resuscitation versus extracorporeal cardiopulmonary
resuscitation. Resuscitation, 92, 70–76.
Simard, R. D., Unger, A. G., Betz, M., Wu, A., & Chenkin, J. (2019). The POCUS pulse check: A case series on
a novel method for determining the presence of a pulse using point-of-care ultrasound. J Emerg Med, 56(6),
674–679.
Sohn, M., Brinkman, R., & Wellman, G. S. (2020). Coprescription of opioid and naloxone in office-based practice
and emergency department settings in the United States. Public Health, 180, 82–84.
Stoecklein, H., Slack, S., Tonna, J. E., & Youngquist, S. T. (2017). ECMO & ECPR. JEMS. Retrieved from
https://www.jems.com/2017/12/01/ecmo-ecpr/.
Sunde, K., & Steen, P. A. (2012). The use of vasopressor agents during cardiopulmonary resuscitation. Emerg Med
Clin North Am, 30(1), 189–198.
Yannopoulos, D., Bartos, J. A., Aufderheide, T. P., et al. (2019). The evolving role of the cardiac catheterization
laboratory in the management of patients with out-of-hospital cardiac arrest: A scientific statement from the
American Heart Association. Circulation, 139(12), e530–e552.
C H A P T E R 8
Acute Coronary Syndromes

INTRODUCTION
Acute coronary syndromes (ACSs), also called acute ischemic coronary syndromes (AICSs), are a
group of conditions that are caused by an abrupt reduction in coronary artery blood flow. The sequence
of events that occurs during an ACS results in conditions that range from myocardial ischemia (i.e.,
unstable angina [UA] pectoris) to infarction (with or without associated ST-segment elevation [STE]
on the electrocardiogram [ECG]). This chapter discusses the pathophysiology, history and clinical
presentation, patient evaluation, and initial management of the patient experiencing an ACS.

L E A R N I N G OBJECTIVES
After completing this chapter, you should be able to:
1. Explain the pathophysiology of ACSs.
2. Discuss the typical clinical presentation of the patient with suspected ACS.
3. Explain and give examples of anginal equivalents.
4. Explain atypical presentation and its significance in ACSs.
5. Identify the ECG changes associated with myocardial ischemia, injury, and infarction.
6. Identify the ECG leads that view the anterior wall, the inferior wall, the lateral wall, the
septum, the inferobasal wall, and the right ventricle.
7. Describe the initial management of a patient who is experiencing an ACS.

L E A R N I N G PLAN
• Read this chapter before class.
• Master identification of the following rhythms: sinus rhythm, sinus bradycardia, sinus
tachycardia, atrial fibrillation (AFib), atrial flutter; atrioventricular (AV) blocks: first-
degree, second-degree type I, second-degree type II, third-degree; premature atrial
complexes (PACs), and premature ventricular complexes (PVCs).
• Master the following medications: oxygen (O2), nitroglycerin (NTG), morphine sulfate,
aspirin
• Master the following skills:
n Ensure scene safety and the use of personal protective equipment.
n Assign team member roles or perform as a team member in a simulated patient
situation.
n Direct or perform a basic life support (BLS) assessment, primary assessment, and
secondary assessment.

173
174 CHAPTER 8  Acute Coronary Syndromes

n Obtain vital signs; establish vascular access; attach a pulse oximeter and blood
pressure (BP) and cardiac monitor; give supplemental O2 if indicated; and order a
12-lead ECG.
n Recognize the signs and symptoms of ACSs.
n Recognize signs of myocardial ischemia, injury, and infarction on an ECG.
n Demonstrate familiarity with the ACS algorithm.
n Develop and implement a treatment plan based on the patient’s presentation,
history, physical examination, and diagnostic test results.
n Demonstrate an understanding of the actions, indications, dosages, adverse
effects, and contraindications for the medications used in the treatment of ACSs.
n Review your performance as a team leader or team member during a postevent
debriefing.
• Develop and use flashcards, flowcharts, and mnemonics to help enhance your
retention of the information presented.
• Complete the chapter quiz and review the quiz answers provided.
• Read the case study at the end of this chapter and answer each question that follows
it. Compare your answers with the answers provided at the end of the case study.

KEY TERMS
Anginal equivalent  Symptom other than chest pain or discomfort resulting from
myocardial ischemia that may occur either alone or in combination in a patient with
ischemic heart disease (IHD).
Arteriosclerosis  A chronic disease of the arterial system characterized by abnormal
thickening and hardening of the vessel walls.
Atherosclerosis  A form of arteriosclerosis in which the thickening and hardening of the
vessel walls are caused by a buildup of fat-like deposits in the inner lining, specifically
of large- and middle-sized muscular arteries.

CORONARY HEART DISEASE


Cardiovascular disease (CVD) is a collection of conditions that involve the circulatory system, which
contains the heart (cardio) and blood vessels (vascular), including congenital CVD. It is estimated that
one person dies every 37 seconds in the United States from CVD (Heron, 2019). Coronary heart
disease (CHD) refers to disease of the coronary arteries and resulting complications, such as angina
pectoris and acute myocardial infarction (MI). CHD is the leading cause of death for men, women,
and people of most racial and ethnic groups in the United States (Heron, 2019). Coronary artery dis-
ease (CAD) affects the arteries that supply the heart muscle with blood. About two in 10 deaths from
CAD happen in adults younger than age 65 years (Benjamin et al., 2019). Coronary microvascular
disease (CMD) refers to disorders that affect the coronary microcirculation.
The main coronary arteries lie on the epicardial surface of the heart (Fig. 8.1). Narrowing or block-
age of a coronary artery can disrupt the oxygen supply to the area of the heart supplied by the affected
vessel(s). If the cause of the ischemia is not reversed and blood flow is not restored to the affected area
of the heart muscle, ischemia may lead to cellular injury and, ultimately, cellular death (i.e., infarction).
Clinical presentations of ischemic heart disease (IHD) may include angina pectoris, silent myocardial
ischemia, acute MI, or sudden cardiac death.
The walls of the ventricles consist of an outer layer (i.e., the epicardium), the middle layer (i.e., the
myocardium), and an inner layer (i.e., the endocardium). The myocardium consists of two areas. The
inner half of the myocardium is called the subendocardial area, and the outer half is called the subepi-
cardial area (Fig. 8.2). The endocardial and subendocardial areas of the myocardial wall are the least
perfused areas of the heart and the most vulnerable to ischemia because these areas have a high oxygen
demand and are fed by the most distal branches of the coronary arteries. Transmural is a term that is
used to describe ischemia, injury, or infarction that extends from the endocardium through the myo-
cardium to the epicardium. For example, an infarction that involves the entire thickness of the left
ventricular wall is called a transmural MI.
CHAPTER 8  Acute Coronary Syndromes 175

Fig. 8.1  ​Major coronary arteries and some of their branches.  (From Benjamin, I., Griggs, R.C., Wing, E.J., Fitz, J.G.,
Andreoli, T.E. [2011]. Andreoli and Carpenter’s Cecil essentials of medicine, ed 8, Philadelphia, Saunders.)

Subendocardial
infarction

Endocardium

Transmural
infarction

Epicardium

Intramural
infarction

Subepicardial
infarction

Fig. 8.2  ​Possible locations of infarctions in the ventricular wall. (From Urden, L.D., Stacy, K.M., Lough, M.E. [2018]. Critical
care nursing: diagnosis and management, ed 8, Maryland Heights, MO, Elsevier.)

The usual cause of an ACS is the rupture of an atherosclerotic plaque. Arteriosclerosis is a chronic
disease of the arterial system characterized by abnormal thickening and loss of elasticity of the vessel
walls. Atherosclerosis is a form of arteriosclerosis in which injury to the endothelial cells that line the
large and middle-sized muscular arteries causes a buildup of fat-like deposits, leading to the formation
of a lesion (i.e., plaque). Possible causes of endothelial injury include risk factors for atherosclerosis,
such as smoking, hypertension, diabetes, increased levels of low-density lipoprotein, decreased levels of
high-density lipoprotein, and autoimmunity (Brashers, 2020). Nontraditional risk factors include in-
creased serum markers for inflammation and thrombosis (e.g., C-reactive protein [CRP]), troponin I,
adipokines (i.e., hormones released from adipose cells), and infection (Brashers, 2020).
176 CHAPTER 8  Acute Coronary Syndromes

Initially, the walls of the blood vessel outwardly expand (i.e., remodel) as plaque builds up inside of
it. Vessel expansion occurs so that its size stays relatively constant, despite the increased size of the
plaque. Remodeling eventually stops because the vessel can no longer expand to make room for the
increase in plaque size. As an atherosclerotic plaque increases in size, the vessel becomes severely nar-
rowed (i.e., stenosed) (Fig. 8.3). In general, arterial stenosis of more than 70% of the vessel’s diameter
produces anginal symptoms (Kumar et al., 2018). The extent of arterial narrowing and the amount of
reduction in blood flow are critical determinants of CAD.
The pathophysiology of ACS can be divided into three separate classes: (1) obstructive atheroscle-
rosis with systemic inflammation, (2) obstructive atherosclerosis without systemic inflammation, and
(3) nonobstructive atherosclerosis (Crea & Liuzzo, 2013).

Obstructive Atherosclerosis With Systemic Inflammation


IHD is usually the result of reduced coronary blood flow caused by the disruption or erosion of an ath-
erosclerotic plaque and the subsequent blockage of an epicardial coronary artery. Contributing factors,
such as endothelial dysfunction, microvascular disease, and vasospasm, may also exist alone or in combi-
nation with coronary atherosclerosis. Any of these factors may be the dominant cause of myocardial
ischemia in some patients (Morrow & De Lemos, 2019).
Atherosclerotic plaques differ with regard to their makeup, their vulnerability to rupture, and their
tendency to form a blood clot (Figs. 8.4 and 8.5). A “stable” or “nonvulnerable” atherosclerotic plaque

Fig. 8.3  ​These cross-sections of the left anterior descending coronary artery show atherosclerosis with more pronounced
luminal narrowing at the left, the more proximal portion of this artery. Atherosclerosis is generally worse at the origin of a coronary
artery and in the first few centimeters, where turbulent blood flow is greater. This turbulent flow over many years promotes
endothelial injury that favors inflammation with insudation of lipids to promote the formation of atheromas. With lifestyle modi-
fications, this process is reversible. (From Klatt, E.C. [2015]. Robbins and Cotran atlas of pathology, ed 3, Philadelphia, Elsevier.)

A B

Fig. 8.4  ​Plaque disruption and myocardial infarction. (A) Plaque disruption. The cap of the lipid-rich plaque has become
torn, with the formation of a thrombus, mostly inside the plaque. (B) Myocardial infarction. This infarct is 6 days old. The
center is yellow and necrotic with a hemorrhagic red rim. The responsible arterial occlusion is probably in the right coronary
artery. The infarct is on the posterior wall. (From Brashers, V.L. [2020]. Alterations of cardiovascular function. In S.E. Huether,
K.L. McCance, V.L. Brashers, editors. Understanding pathophysiology, ed 7, St. Louis, Elsevier [Original source: Damjanov, I.,
Linder, J., editors. (1996). Anderson’s pathology, ed 10, St Louis, Mosby].)
CHAPTER 8  Acute Coronary Syndromes 177

Rupture of fibrous cap Superficial erosion

Intraplaque
hemorrhage

Calcium nodule
Micro
vessels
Erosion of calcium nodule Intraplaque hemorrhage
Fig. 8.5  ​Mechanisms of plaque disruption. Rupture of the fibrous cap (upper left) causes two-thirds to three-fourths of fatal
coronary thrombosis. Superficial erosion (upper right) occurs in one-fifth to one-fourth of all cases of fatal coronary thrombosis.
Certain populations, such as diabetic individuals and women, appear more susceptible to superficial erosion as a mechanism
of plaque disruption and thrombosis. Erosion of a calcium nodule may also cause plaque disruption and thrombosis (lower left).
In addition, the friable microvessels in the base of the atherosclerotic plaque may rupture and cause intraplaque hemorrhage
(lower right). The consequent local generation of thrombin may stimulate smooth muscle proliferation, migration, and collagen
synthesis, promoting fibrosis and plaque expansion on a subacute basis. Severe intraplaque hemorrhage can also cause sud-
den lesion expansion by a mass effect acutely. (From Libby, P. [2020]. Vascular medicine: a companion to Braunwald’s heart
disease, ed 3, Philadelphia, Elsevier. [Original source: Libby, P., Theroux, P. (2005). Pathophysiology of coronary artery disease.
Circulation, 111, 3481–3488].)

has a relatively thick fibrous cap that separates it from contact with the blood. The cap covers a core
that contains a large amount of collagen and smooth muscle cells but a relatively small lipid pool. A
“vulnerable” plaque is prone to rupture because it has a thin cap of fibrous tissue over a large, soft, fatty
center that separates it from the opening of the blood vessel. If the fibrous cap erodes or ruptures, the
contents of the plaque are exposed to flowing blood, activating the clotting cascade and promoting clot
formation, and disrupting blood flow.
Injured endothelial cells become inflamed. CRP is a protein synthesized in the liver that increases
when inflammation is present in the body. A high-sensitivity CRP (hs-CRP) test is an indirect measure
of atherosclerotic plaque–related inflammation to assess CAD risk and aid decision-making about
pharmacologic interventions for individuals with other risk factors for coronary disease.

Obstructive Atherosclerosis Without Systemic


Inflammation
In patients in whom coronary instability occurs without systemic evidence of inflammation, possible
mechanisms of plaque disruption may be related to extreme physical, emotional, or environmental
stressors (Crea & Liuzzo, 2013). Sympathetic nervous system activation and catecholamine release lead
to increases in heart rate, BP, and cardiac workload, and local vasoconstriction at the site of vulnerable
plaques, which can lead to plaque rupture. Sympathetic nervous system stimulation also leads to platelet
activation, hypercoagulability, and intense coronary microvascular constriction (Crea & Liuzzo, 2013).

Nonobstructive Atherosclerosis
Changes involving the epicardial coronary arteries and the coronary microcirculation are the likely
causes of ACS in patients without evidence of obstructive atherosclerosis (Crea & Liuzzo, 2013). In
some patients who present with an ACS, coronary vasospasm is the cause, and intense constriction of
the coronary microcirculation is the cause in others.
178 CHAPTER 8  Acute Coronary Syndromes

ISCHEMIC HEART DISEASE


Myocardial ischemia develops if coronary blood flow or the oxygen content of coronary blood is insuf-
ficient to meet the metabolic demands of myocardial cells (Brashers, 2020). Ischemia can occur because
of increased oxygen demand (i.e., demand ischemia), reduced myocardial oxygen supply (i.e., supply
ischemia), or both. Key factors that determine left ventricular myocardial oxygen demands are: (1) the
heart rate; (2) the strength of its contractions; and (3) the systolic pressure developed in the main
pumping chamber (Goldberger et al., 2018). Although the most common cause of myocardial ischemia
is coronary atherosclerosis, ischemia can occur in the absence of epicardial coronary artery obstruction.
When a temporary or permanent blockage occurs in a coronary artery, the region of the heart sup-
plied by the affected artery is called the area at risk because the blood supply to the heart muscle is
impaired, and myocardial cells distal to the site of the blockage are starved for oxygen and other nutri-
ents. Ischemia occurs immediately in the area supplied by the affected artery, depriving the myocardium
of a glucose source necessary for aerobic metabolism (Brashers, 2020). Anaerobic metabolism ensues,
lactic acid accumulates in the cardiac cells, the ischemic cells lose the ability to contract, and cardiac
output decreases. Ischemia also contributes to dysrhythmias, probably by causing electrical instability of
ischemic areas of the heart (Kumar et al., 2018). If blood flow can be quickly restored, it is possible to
salvage the area at risk. If the cause of the ischemia is not reversed and blood flow restored to the affected
area of the heart muscle, ischemia may lead to cellular injury and, ultimately, cellular death (i.e., infarc-
tion). Early assessment and emergency care are essential to prevent worsening ischemia.
When the cause of ischemic chest discomfort is coronary vasoconstriction, methods to increase
blood flow to the ischemic myocardium include giving medications, such as NTG or dihydropyridine
calcium blockers. Methods to reduce the heart’s oxygen demand include resting or slowing the patient’s
heart rate with medications, such as beta blockers or nondihydropyridine calcium blockers.

Clinical Features
Angina pectoris is chest discomfort that occurs when the heart muscle does not receive enough oxygen
(i.e., myocardial ischemia). The discomfort associated with angina is thought to occur because of nerve
ending irritation caused by the release of substances, such as lactate, histamine, adenosine, bradykinin,
and serotonin into the coronary circulation during periods of lactic acidosis (Tobin & Eagle, 2020).
Ischemic chest discomfort often builds in intensity over a few minutes. It may be described as a
burning feeling, squeezing, or heaviness in the center of the chest that may radiate to the neck, jaw,
shoulders, right or left arm, or epigastrium. Although chest discomfort is the classic symptom associ-
ated with IHD, anginal equivalents (i.e., symptoms of myocardial ischemia other than chest pain or
discomfort) may be present either alone or in combination (Box 8.1).

Diagnostic Studies
In addition to the patient’s history and physical examination, examples of possible diagnostic studies
to obtain in a patient with a complaint of chest discomfort include the following:
• Measurement of cardiac biomarkers (among other laboratory tests)
• A 12-lead ECG
• Noninvasive testing (e.g., echocardiography, exercise ECG testing, pharmacologic testing, exercise
imaging stress testing)

BOX 8.1 Examples of Anginal Equivalent Symptoms


• Difficulty breathing • Indigestion
• Dizziness • Isolated arm, back, jaw, or neck discomfort
• Dysrhythmias • New dyspnea on exertion
• Epigastric pain or burning • Palpitations
• Excessive sweating • Syncope or near-syncope
• Fatigue • Unexplained nausea or vomiting
• Generalized weakness
CHAPTER 8  Acute Coronary Syndromes 179

A chest radiograph may be helpful for patients with suspected heart failure or comorbid pulmonary
disease, or to exclude other causes of chest discomfort. Coronary angiography may be performed to
determine the amount of ischemic myocardium and determine an overall treatment plan if noninvasive
testing suggests the presence of high-risk coronary lesions.

ACLS Pearl
Injured myocardial cells release enzymes and proteins that pass through broken cell membranes
and leak into the bloodstream. Examples include myoglobin, cardiac troponins (cTn), creatine
kinase (CK) and its myocardial band (MB) isoform, and lactate dehydrogenase, among others. The
presence of these substances in the blood, which are called cardiac biomarkers, serum cardiac
markers, or serum biomarkers, can subsequently be measured using blood tests to verify the
presence of myocardial injury or infarction.

The 12-lead ECG is an essential diagnostic tool for patients presenting with ischemic chest discomfort
or anginal equivalent symptoms. Obtain the first 12-lead ECG within 10 minutes of patient contact.
Because ischemia affects repolarization, its effects can be viewed on the ECG as changes in ST segments
and T waves. When measured at the J point, ST-segment depression of 0.5 mm or more is suggestive of
myocardial ischemia when viewed in two or more anatomically contiguous leads (discussed later). Nega-
tive (i.e., inverted) T waves may also be present. ST segments usually return to the baseline after the
episode of chest discomfort is resolved.

ACLS Pearl
Monitoring of ST-segment changes can provide useful diagnostic and predictive information in the
patient experiencing an ACS.

Stable Angina
Stable angina, also called classic angina, remains relatively constant and predictable in terms of its qual-
ity and duration, precipitating events, and response to treatment (Box 8.2). It is characterized by brief
episodes of chest discomfort related to activities that increase the heart’s need for oxygen. Symptoms
usually begin gradually, last less than 5 minutes, and are typically relieved within 5 minutes with rest,
short-acting NTG, or both. Although ECG findings may be normal at rest, the ECG usually reveals
ST-segment depression in the leads that face the affected area of the ventricle during an anginal episode.

ACLS Pearl
In their 2019 Chronic Coronary Syndromes Clinical Practice Guidelines, the European Society of
Cardiology (ESC) describes CAD as a dynamic process of atherosclerotic plaque accumulation and
functional alterations of coronary circulation that can be modified by lifestyle, pharmacologic therapies,
and revascularization, which result in disease stabilization or regression (Knuuti et al., 2020). Recogniz-
ing that the clinical presentation of a patient who has CAD may be categorized as acute (i.e., unstable)
or chronic (i.e., stable), the ESC replaced the phrase stable CAD with chronic coronary syndromes.

BOX 8.2 Stable Angina


Common Precipitating Events Related Signs and Symptoms
• Emotional upset • Fatigue
• Exercise or exertion • Faintness
• Exposure to cold weather • Nausea or vomiting
• Heavy meal • Palpitations
• Stair climbing • Shortness of breath
• Sexual activity • Sweating
180 CHAPTER 8  Acute Coronary Syndromes

Variant Angina
Variant angina, also called Prinzmetal angina or vasospastic angina, is a form of angina resulting from
intense coronary vasospasm. Coronary vasospasm can occur in patients with or without coronary ath-
erosclerosis, can be focal or diffuse, and can affect epicardial or microvascular coronary arteries (Picard
et al., 2019). Although mental stress, central nervous system stimulants (e.g., cocaine), or exposure to
cold can precipitate an episode of coronary artery spasm, it usually occurs at rest and may awaken the
patient from sleep. Episodes typically last only a few minutes, but this may be long enough to produce
serious dysrhythmias, such as AV block, ventricular tachycardia (VT), ventricular fibrillation (VF), as
well as sudden cardiac death. If the spasm is prolonged, infarction may result.
Patients with variant angina are generally younger (usually in their 40s or 50s) and have fewer
coronary risk factors (except for smoking) compared with patients with chronic stable angina. The most
important predisposing risk factor for variant angina is smoking (Picard et al., 2019). In some patients,
migraine headaches, Raynaud disease, systemic lupus erythematosus, fibromuscular hyperplasia, and
rheumatoid arthritis are associated with variant angina, suggesting a possible association with a more
generalized vasospastic disorder (Okumus et al., 2019).
The patient with variant angina complains of chest discomfort that is often described as severe and
may be accompanied by syncope. Symptoms are usually relieved by NTG. Although typical angina pro-
duces ST-segment depression, variant angina usually produces ST segment elevation during periods of
chest discomfort or during exercise. ST segments typically return to the baseline after the episode of chest
discomfort is resolved. Because NTG is effective at relieving the coronary spasm, the ECG evidence of
variant angina may be lost if no pretreatment ECG is obtained. Treatment typically includes a calcium
blocker, alone or in combination with a long-acting nitrate. Because several studies have shown that
statins are effective at preventing coronary artery vasospasm, providers may opt to add statin therapy to
the patient’s conventional medication regimen (Picard et al., 2019). Beta blockers should be used cau-
tiously in these patients because they may exacerbate coronary vasospasm (Tobin & Eagle, 2020).

ACLS Pearl
Obtain a baseline 12-lead ECG before initiating treatment in any patient presenting with a
possible ACS.

Microvascular Angina
Microvascular angina, formerly called cardiac syndrome X, is related to disease of the small, distal
branches of the coronary arteries (i.e., CMD). Ischemia and angina occur because functional and
structural changes in the coronary microvasculature disrupt the ability of the vessels to dilate and
increase coronary blood flow in response to increased myocardial oxygen demand (Chen et al., 2016).
Microvascular angina is most commonly seen in perimenopausal and postmenopausal women. Symp-
toms often occur at rest and are frequently related to emotional stress. ST-segment depression is often
present during anginal episodes.

ACLS Pearl
Microvascular disease is present in up to 50% of symptomatic patients referred for the evaluation
of suspected or known CAD. It is associated with a worse cardiovascular prognosis even in those
without obstructive CAD (Taqueti, 2019).

Possible mechanisms of microvascular angina include the effects of cardiovascular risk factors on the
arterial wall, hypersensitivity of the vascular smooth muscle cell layer, or both (Schindler & Dilsizian,
2020) (Fig. 8.6). Examples of contributing factors include increased systemic inflammation, insulin
resistance, abnormal vasoconstriction and impaired vasodilation of the microvascular bed, and estrogen
deficiency, among others.
Positron emission tomography (PET) and transthoracic Doppler echocardiography are useful in non-
invasively identifying microvascular dysfunction. Cardiac magnetic resonance and computed tomography
CHAPTER 8  Acute Coronary Syndromes 181

Sex-specific precursors
• PCOS
• Hypoestrogenemia
• Menopause
Subendocardial or
epicardial ischemia
Hormonal alterations Base
coupled with: Sept
Inf

Pro-atherogenic factors
• Hyperlipidemia Lat Apex
• HTN
• Smoking
• Metabolic dysfunction
• Inflammation

Pro-vasculopathy

Accelerating factors Microvascular


• Early menopause dysfunction
• Risk factor clustering
Vascualr dysfunction symptoms
Atypical symptoms, including
Nonobstructive • Prolonged symptoms at rest
atheroma • Shortness of breath
• Unusual fatigue
• More frequent pattern

Fig. 8.6  ​Model of microvascular angina in women. HTN, Hypertension; PCOS, polycystic ovary syndrome. (From Zipes, D.P.,
Libby, P., Bonow, R.O., Mann, D.L., Tomaselli, G.F., Braunwald E. [2019]. Braunwald’s heart disease: a textbook of cardiovascular
medicine, ed 11, Philadelphia, Elsevier.)

may also be suitable. Although NTG can relieve symptoms in some patients, it has inconsistent beneficial
effects for others. Lifestyle modifications, beta blockers, or heart rate–lowering calcium blockers may
be useful.

ACUTE CORONARY SYNDROMES


Ischemia that is prolonged by more than just a few minutes can result in myocardial injury. Myocardial
injury can be extensive enough to produce a decrease in pump function or electrical conductivity in the
affected cells.
Death of myocardial cells occurs when blood flow to the area at risk stops or is suddenly decreased
long enough to cause myocardial cell death and necrosis of the myocardium (Thomas & Brady, 2018;
Thygesen et al., 2018) (Fig. 8.7). The resulting infarction depends on multiple factors, such as the size
of the vascular bed perfused by the obstructed vessel(s), the rate of development and duration of the
occlusion, the metabolic demands and oxygen needs of the myocardium at risk (affected, for example,
by BP and heart rate), and the extent of collateral circulation to the affected area (Kumar et al., 2018).
Without clinical intervention (i.e., reperfusion therapy), the infarction can expand to involve the entire
thickness of the myocardial wall. Because time is muscle when caring for patients with an ACS, the
benefits of reperfusion therapy are most significant when performed early.
UA and non–ST-elevation MI (NSTEMI) may occur when blood flow through a coronary artery is
partially or intermittently blocked. The clinical presentation of patients with these conditions is similar,
and it is often difficult to distinguish between them. UA and NSTEMI are often grouped as non–ST-
elevation acute coronary syndromes (NSTE-ACS) because ECG changes associated with these condi-
tions usually include ST-segment depression and T-wave inversion in the leads that face the affected
area. These ECG findings are more likely to be present while the patient is symptomatic. With
NSTEMI, cardiac injury is sufficient to result in the release of biomarkers (i.e., troponin elevation). The
diagnosis is UA when biomarkers are not elevated. NSTE-ACS is the cause of about two-thirds of all
hospital admissions for ACS in the United States each year (Bob-Manuel et al., 2017).
182 CHAPTER 8  Acute Coronary Syndromes

Aorta

Pulmonary
artery

Left circumflex coronary


artery
Right
coronary Left anterior descending
artery coronary artery
Acute coronary
arterial occlusion

Zone of perfusion
(area at risk)

Completed infarct
Cross-section involving nearly the
of myocardium entire area at risk

Obstructed
coronary
artery

Endocardium

Zone of perfusion Zone of Zone of


(area at risk) necrosis necrosis

0 hr 2 hr 24 hr

Fig. 8.7  ​Progression of myocardial necrosis after coronary artery occlusion. A transmural segment of myocardium that is
dependent on the occluded vessel for perfusion constitutes the area at risk (outlined). Necrosis begins in the subendocardial
region in the center of the ischemic zone and, with time, expands to involve the entire wall thickness. Note that a very narrow
zone of myocardium immediately beneath the endocardium is spared from necrosis because it can be oxygenated by diffu-
sion from the ventricle. (From Kumar, V., Abbas, A.K., Aster, J.C. [2018]. Robbins basic pathology, ed 10, Philadelphia, Elsevier.)

Complete blockage of a coronary artery may result in ST-elevation MI (STEMI) or sudden death.
Both NSTEMI and STEMI are life-threatening; however, they are discussed separately because their
management strategies differ, primarily in terms of the methods used and urgency of reestablishing
flow in the affected artery (Frampton et al., 2020).

ACLS Pearl
The complete blockage of a coronary artery may cause an MI. However, because a plaque usually
increases in size over months and years, other vascular pathways may enlarge as portions of a
coronary artery become blocked. These vascular pathways (i.e., collateral circulation) serve as an
alternative route for blood flow around the blocked artery to the heart muscle. Thus the presence
of collateral arteries may prevent infarction despite complete blockage of the artery.

Clinical Features
The patient experiencing an ACS typically complains of retrosternal pain, pressure, heaviness, or
squeezing lasting 10 minutes or longer that usually occurs at rest or with minimal exertion (Amsterdam
et al., 2014). Chest discomfort may be accompanied by anginal equivalents, such as unexplained new-
onset or increased exertional dyspnea (most common equivalent), unexplained fatigue, diaphoresis,
nausea and vomiting, or syncope (Amsterdam et al., 2014).
Because not all chest discomfort is cardiac-related, evaluate patients with suspected ACS rapidly
to differentiate between those with an emergent versus a less urgent condition. Seek answers to two
essential questions during the initial patient evaluation (Amsterdam et al., 2014): (1) What is the
CHAPTER 8  Acute Coronary Syndromes 183

likelihood that the patient’s signs and symptoms represent an ACS; and (2) What is the probability
of an adverse clinical outcome? Several risk assessment scores and clinical algorithms are available
that encompass the patient’s history, physical examination, ECG, and cardiac biomarkers. These tools
aid in identifying patients with ACS who are at increased risk of adverse outcomes and can help guide
clinical decision making (Amsterdam et al., 2014).

Patient History
Obtaining an accurate history is vital to help determine whether a patient’s signs and symptoms are
most likely related to ischemia as a result of CAD. It is essential to ask targeted questions to assess the
patient’s probability of an ACS and to avoid reperfusion therapy delays if such therapy is indicated.

ACLS Pearl
When obtaining the patient’s history, use the patient’s words for the discomfort. For example, the
patient may not consider their symptom “discomfort” or “pain” but instead have another appro-
priately descriptive term to describe their symptom. Whatever word the patient uses, continue to
use that term when interacting with the patient.

SAMPLE History
• Signs and Symptoms. Ask the patient what prompted them to seek medical assistance.
• Allergies. Ask the patient about allergies to medications, food, environmental elements (e.g., pollen),
and products (e.g., latex).
• Medications. Ask the patient about the prescription and over-the-counter medications they are is
currently taking. Find out if the patient has taken any medication for erectile dysfunction in the past
24 to 48 hours. Ask about the use of any herbal supplements or recreational drugs, such as cocaine.
• Past medical history. Ask if the patient has a history of a heart attack, angina, heart failure, high BP,
or abnormal heart rhythm. If the patient answers yes to this question, ask them how the current
symptoms compare to the previous episode. Ask if the patient has ever had a heart-related medical
procedure, such as a bypass (i.e., open-heart surgery), cardiac catheterization, angioplasty, transplant,
valve replacement, or pacemaker implantation. Determine whether the patient has a history of
stroke; diabetes; lung, liver, or kidney disease; or other medical condition. Find out the patient’s risk
factors for heart disease. Ask the patient if they smoke. If the answer is yes, ask how many packs per
day. Ask the patient if a history of heart disease is in the family. If the answer is yes,
ask whether anyone died of heart disease and at what age. Ask about a family history of high BP,
diabetes, and high cholesterol. Also ask about any recent hospitalizations and any recent surgeries.
• Last oral intake. Ask the patient when they last had anything to eat or drink, and if any recent
changes in eating patterns or fluid intake (or output) have occurred.
• Events leading to the incident. Try to find out what precipitated the patient’s current symptoms. For
example, did an event or activity cause the patient’s symptoms, such as strenuous exercise, sexual
activity, or unusual stress?

OPQRST History
Use the OPQRST mnemonic to explore the characteristics of the patient’s symptoms.
• Onset. When did your symptoms begin? Did they start suddenly or gradually? Have you ever had
this discomfort before? When? How long did it last? Were you seen, evaluated, or treated for it? If
so, what was the diagnosis? How does the discomfort you are feeling right now compare with that?
• Provocation/Palliation/Position. What were you doing when your symptoms started? What makes
the discomfort better or worse? What have you tried to relieve the problem? Does a change in
position lessen the discomfort?
• Quality. Can you describe your discomfort?
• Region/Radiation/Referral. Where is your discomfort? Does it stay in one area? Do you have symptoms
in a different area of your body?
• Severity. On a scale of 0 to 10, with 0 being the least and 10 being the worst, what number would
you assign your discomfort?
• Timing. Is your discomfort still present? Is it getting better, worse, or staying about the same? Does
it come and go, or is it constant?
184 CHAPTER 8  Acute Coronary Syndromes

Atypical Presentation
Not all patients experiencing an ACS present similarly. Atypical presentation refers to the uncharacteristic
signs and symptoms that are experienced by some patients. The presence of atypical findings can lead to
underrecognition, undertreatment, and higher in-hospital mortality rates (Giugliano & Braunwald, 2019;
McSweeney et al., 2016). Although typical characteristics increase the probability of CAD, features that are
not characteristic of ischemic chest pain do not exclude the possibility of ACS (Amsterdam et al., 2014).
Patients who are experiencing an ACS and who are most likely to present atypically include the
following (Amsterdam et al., 2014; Giugliano & Braunwald, 2019; Karve et al., 2007):
• Older adults
• Diabetic individuals
• Women
• Patients with chronic kidney disease
• Patients with dementia
• Patients with prior cardiac surgery
• Patients during the immediate postoperative period after noncardiac surgery
Older adults may have atypical symptoms, such as dyspnea, shoulder or back pain, weakness, fatigue,
a change in mental status, syncope, unexplained nausea, and abdominal or epigastric discomfort. They
are also more likely than younger patients to present with more severe preexisting conditions, such as
hypertension, heart failure, or a previous acute MI.
Diabetic individuals may present atypically because of autonomic neuropathy that prevents the
perception of pain (Kumar et al., 2018). Signs and symptoms may include a change in mental status,
fatigue, nausea or vomiting, dyspnea, generalized weakness, or lightheadedness.
Women who experience an ACS are less likely than men to experience chest pain or discomfort;
however, most experience prodromal symptoms (i.e., symptoms that are new and intermittent before an
acute event and resolve spontaneously after the event), such as shortness of breath or unusual fatigue for
weeks or even months before an acute cardiac event (McSweeney et al., 2016). Studies reveal that women
refer to their chest discomfort differently from men using descriptors such as sharp, stabbing, aching, or
tightness and may experience pain or discomfort in other body locations, such as that localized to the
arm(s), shoulder, middle back, jaw, or epigastrium. When experiencing an ACS, women may report symp-
toms that include unusual fatigue, anxiety, frequent indigestion, heart racing, new vision problems,
changes in thinking or remembering, loss of appetite, tingling in hands or arms, numbness or burning in
hands or fingers, cough, and increased frequency or intensity of headaches (McSweeney et al., 2016).

ACLS Pearl
About 30% of patients who present with STEMI are women (Frampton et al., 2020). Data from the
National Cardiovascular Data Registry reveals that women present later after symptom onset,
have longer door-to-fibrinolysis and door-to-balloon (or device) times, and less often receive
aspirin or beta blockers within the first 24 hours of presentation than men (O’Gara et al., 2013).

Studies show that patients with chronic kidney disease of all stages receive recommended interventions
for ACSs less frequently than do patients with normal renal function. Consider the following (O’Gara
et al., 2013):
• Patients on dialysis experience longer prehospital delays, are less often recognized as having an acute
MI, and less often have ST elevation or left bundle branch block (LBBB) on initial ECG than
patients not on dialysis.
• Only 45% of eligible patients on dialysis receive reperfusion therapy, and only 70% receive aspirin
on admission.
• At discharge, only 67% of patients on dialysis are prescribed aspirin, and only 57% are prescribed
beta blockers.

Physical Examination
The physical examination for patients who are being evaluated for possible ACS is often normal; how-
ever, it is essential to perform a physical exam for the following reasons (Anderson et al., 2007;
O’Connor et al., 2015):
• To identify potential precipitating causes of myocardial ischemia (e.g., uncontrolled hypertension,
gastrointestinal [GI] bleeding)
CHAPTER 8  Acute Coronary Syndromes 185

• To assess the hemodynamic effect of the ischemic event


• To identify coexisting conditions (e.g., pulmonary disease, malignancies) that could influence treatment
decisions
• To evaluate the patient for complications related to ACS
The physical examination should include the following:
• Measurement of vital signs (obtain BP readings in both arms if dissection is suspected)
• Auscultation of breath sounds for crackles (i.e., rales)
• Auscultation of cardiac sounds for murmurs, gallops, and friction rubs
• Assessment for jugular venous distention ( JVD), peripheral pulse deficits, and the presence of bruits
• Neurologic evaluation
• Identification of contraindications to antiplatelet or fibrinolytic therapy

Diagnosis
The diagnosis of an ACS is made based on the following: (1) the patient’s history and clinical presentation,
(2) cardiac biomarker results, and (3) the patient’s ECG findings (Fig. 8.8).

Cardiac Biomarkers
Cardiac troponins (i.e., TnI and TnT) are components of the contractile apparatus of myocardial cells and
are the biomarkers of choice for diagnosing MI. Because the ranges of normal biomarker levels vary among
laboratories, current clinical practice guidelines define an increased cTn concentration as a value that
exceeds the 99th percentile as compared with a normal reference population (Amsterdam et al., 2014).

Presentation Ischemic discomfort

ACS

Working Dx

ECG No ST elevation ST elevation

NSTE-ACS

UA NSTEMI* STEMI*

Cardiac biomarker

Unstable angina Myocardial infraction QwMI Noncardiac


etiologies

Final Dx
Fig. 8.8  ​Acute coronary syndrome (ACS). Symptomatic, morphologic, electrocardiographic, and serologic findings in
patients with various kinds of ACS. Individuals with ACS usually complain of chest pain. If the involved coronary artery is
totally occluded by a fresh thrombus (shown on the right), the patient’s electrocardiogram (ECG) reveals ST-segment eleva-
tion, cardiac biomarkers subsequently are elevated, and the patient is diagnosed with an ST-segment elevation myocardial
infarction (STEMI). In some cases, initial ST elevation can lead to non–Q-wave myocardial infarction. If the involved coronary
artery is partially occluded by a fresh thrombus (shown on the left), the patient’s ECG does not show ST-segment elevation.
If cardiac biomarkers are not elevated, the patient is diagnosed with unstable angina (UA). If cardiac biomarkers remain
negative or are not elevated during hospitalization, the final diagnosis remains unstable angina. If cardiac biomarkers are
elevated, the patient is diagnosed with a non–ST-segment elevation MI (NSTEMI). Most patients presenting with NSTEMI
develop NQMI, but a few may develop QwMI. Dx, Diagnosis; NQMI, non–Q-wave myocardial infarction; QwMI, Q-wave
myocardial infarction. (From Schafer, A.I., Goldman, L. [2020]. Goldman-Cecil medicine, ed 26, Philadelphia, Elsevier.)
186 CHAPTER 8  Acute Coronary Syndromes

TABLE 8.1 Possible Causes of Elevated Cardiac Troponin Values Because


of Myocardial Injury
Cardiac-Related Causes Other Causes
Aortic dissection Acute pulmonary embolism
Atrial fibrillation End-stage renal disease
Cardiac contusion Envenomation (e.g., jellyfish, scorpion, snake)
Cardiac infiltrative disorders (e.g., amyloidosis, Gastrointestinal bleeding (severe)
sarcoidosis, scleroderma) Hypotension/shock
Cardiac procedures (e.g., ablation, angioplasty, Hypovolemia
cardioversion, defibrillation, surgery) Kawasaki disease
Cardiomyopathy Pharmacologic agents (e.g., amphetamines,
Coronary artery spasm cardiotoxic chemotherapy, cocaine, cyanide)
Coronary embolism Respiratory failure
Endocarditis Rhabdomyolysis
Heart failure Sepsis
Left ventricular hypertrophy Severe anemia
Myocardial ischemia Strenuous exercise
Myocarditis Stroke
Sustained bradydysrhythmia or tachydysrhythmia

(From Kelley, W.E., Januzzi, J.L., Christenson, R.H. [2009]. Increases of cardiac troponin in conditions other than acute
coronary syndrome and heart failure. Clin Chem, 55[12], 2098–2112; Thygesen, K., Alpert, J.S., Jaffe, A.S., et al. [2018].
Fourth universal definition of myocardial infarction. J Am Coll Cardiol, 72[18], 2231–2264.)

Experts recommend using the term myocardial injury when there is evidence of elevated cTn values
with at least one value above the 99th percentile of the upper reference limit (URL) (Thygesen et al.,
2018). Although myocardial injury may be related to acute myocardial ischemia, many other causes of
myocardial injury can cause elevated cTn levels (Table 8.1).
Elevated cTn levels can be detected within 3 to 4 hours after the onset of myocardial injury (Chung
& Brown, 2019). Troponin levels remain elevated for several days after myocardial necrosis and may
remain elevated for up to 2 weeks with a large infarction (Amsterdam et al., 2014). CK-MB may be
used to estimate the size of an MI (Amsterdam et al., 2014) and is the preferred alternative when cTn
markers are unavailable (Thygesen et al., 2012).

Electrocardiogram Findings
Obtaining and reviewing a 12-lead ECG is important when evaluating a patient presenting with
symptoms suggestive of ACS. The 12-lead ECG is the cornerstone of the decision pathway because it
aids in the diagnosis of acute MI, suggests the distribution of the infarct-related artery, and estimates
the amount of myocardium at risk (Chung & Brown, 2019).
Indicative changes, which are ECG findings that are seen in leads that look directly at the area fed
by the blocked vessel, are significant when they are seen in two anatomically contiguous leads. Two
leads are contiguous if they look at the same or adjacent areas of the heart or if they are numerically
consecutive chest leads. Reciprocal ECG changes may be seen in leads opposite (i.e., about 180 degrees
away from) the leads that show the indicative change (Kawji & Glancy, 2017). Reciprocal changes can
be more noticeable than the ECG changes observed in the indicative lead(s). ECG findings are dis-
cussed in more detail later in this chapter.
The first 12-lead ECG should be obtained and interpreted within 10 minutes of patient contact
(Amsterdam et al., 2014). On the ECG, epicardial injury may cause elevation of the ST segment (in
the leads that face the affected area) and depression of the baseline. In contrast, endocardial injury may
cause depression of the ST segment and elevation of the baseline (Surawicz & Knilans, 2008). Because
it may be normal or initially nondiagnostic, a 12-lead ECG should be repeated every 20 to 30 minutes
until the patient’s symptoms resolve, or the diagnosis of MI is established or excluded (Giugliano &
Braunwald, 2019).

Classification of Myocardial Infarction


MI may be classified into various types based on pathologic, clinical, and prognostic differences. Five
types of MIs are recognized, with types 1 and 2 being the most common. A type 1 MI is caused by
CHAPTER 8  Acute Coronary Syndromes 187

TABLE 8.2 Diagnostic Criteria for Type 1 and Type 2 Myocardial Infarction
Type 1 Myocardial Infarction Type 2 Myocardial Infarction
Detection of a rise and/or fall of cTn values Detection of a rise and/or fall of cTn values with at
with at least one value above the 99th least one value above the 99th percentile URL, and
percentile URL and with at least one of evidence of an imbalance between myocardial
the following: oxygen supply and demand unrelated to coronary
• Symptoms of acute myocardial ischemia thrombosis, requiring at least one of the following:
• New ischemic ECG changes • Symptoms of acute myocardial ischemia
• Development of pathologic Q waves • New ischemic ECG changes
• Imaging evidence of new loss of viable • Development of pathologic Q waves
myocardium or new regional wall motion • Imaging evidence of new loss of viable myocar-
abnormality in a pattern consistent with an dium or new regional wall motion abnormality in a
ischemic etiology pattern consistent with an ischemic etiology
• Identification of a coronary thrombus by
angiography, including intracoronary imaging
or by autopsy

cTn, Cardiac troponin; ECG, electrocardiogram; MI, myocardial infarction; URL, upper reference limit
(From Thygesen, K., Alpert, J.S., Jaffe, A.S., et al. [2018]. Fourth universal definition of myocardial infarction. J Am Coll
Cardiol, 72[18], 2231–2264.)

atherothrombotic CAD and is usually triggered by atherosclerotic plaque rupture or erosion, resulting
in coronary thrombosis (Thygesen et al., 2018). With a type 2 MI, myocardial necrosis results from a
mismatch between myocardial oxygen supply and demand in the absence of acute atherothrombotic
plaque disruption (Thygesen et al., 2018). Criteria for type 1 and type 2 MIs appear in Table 8.2.

Unstable Angina and Non–ST-Elevation Myocardial


Infarction
UA, which is also known as preinfarction angina, accelerating or crescendo angina, intermediate coro-
nary syndrome, and preocclusive syndrome, is a condition of intermediate severity between stable angina
and acute MI. It is characterized by symptoms that begin suddenly at rest or with minimal exertion and
last for 10 minutes or more. The chest discomfort associated with UA may be described as pressure,
heaviness, or pain and accompanied by dyspnea, diaphoresis, nausea, syncope, or dysrhythmias.
UA and NSTEMI differ primarily by whether myocardial ischemia is severe enough to cause cellular
damage leading to detectable quantities of cardiac biomarkers. Patients with typical symptoms without per-
sistent (lasting longer than 20 minutes) STE in at least two contiguous ECG leads, but with the elevation of
cardiac biomarkers, are classified as having NSTEMI (Giugliano & Braunwald, 2019). Patients without
typical symptoms and serial negative cardiac biomarkers are classified as having UA. More than half of pa-
tients with definite NSTE-ACS may have normal or nondiagnostic ECGs (Giugliano & Braunwald, 2019).

ST-Elevation Myocardial Infarction


With STEMI, an epicardial coronary artery is completely blocked (Fig. 8.9). This occlusion can result
in myocardial ischemia that affects the entire thickness (i.e., transmural) of the left (and sometimes
right) ventricular wall. About 50% of STEMI patients have multivessel disease (Levine et al., 2016).
ECG changes associated with STEMI often occur in a predictable pattern (Fig. 8.10). The ECG
changes described here appear in leads looking at the area fed by the blocked (culprit) vessel (Power &
Seppelt, 2019).
• Hyperacute T waves. Within minutes of an interruption of coronary blood flow, hyperacute T waves
may be observed on the ECG. Hyperacute T waves are tall, positive, peaked, and broad-based. They
may be seen as early as 30 minutes after the onset of chest pain and usually appear before the eleva-
tion in cardiac biomarkers or ST changes on the ECG (Sovari et al., 2007); however, hyperacute
T waves are often not observed because they have typically resolved by the time the patient seeks
medical assistance. Possible causes of tall T waves other than myocardial ischemia and infarction
include hyperkalemia, left ventricular hypertrophy, and acute central nervous system events (e.g.,
intracranial hemorrhage), among others.
188 CHAPTER 8  Acute Coronary Syndromes

Plaque rupture with thrombus

Vasospasm or endothelial
dysfunction Causes of myocardial
oxygen supply-demand
Fixed atherosclerosis and imbalance
supply-demand imbalance

Supply-demand
imbalance alone

Presentation Ischemic discomfort

Working Supply-demand imbalance Acute coronary syndrome


diagnosis (nonthrombotic) (atherothrombotic)

ECG No ST elevation ST elevation

Biomarkers – + – + +

Final Unstable angina Non–ST elevation Unstable angina Non–ST elevation ST elevation
diagnosis (demand related) MI (type 2) (thrombotic mediated) MI (type 1) MI (type 1)

Final ECG
manifestation Non–Q-wave MI Q-wave MI

Fig. 8.9  ​Myocardial ischemia and myocardial infarction (MI) can result from various coronary disease processes, including
vasospasm, increased myocardial demand in the setting of a fixed coronary lesion, and erosion or rupture of vulnerable
atherosclerotic plaque leading to acute thrombus formation and subsequent ischemia. All result in myocardial oxygen supply-
demand mismatch and can precipitate ischemic symptoms, and all processes, when severe or prolonged, will lead to myo-
cardial necrosis or infarction. Nonthrombotically mediated events (bottom half, left side) typically occur without ST-segment
elevation (STE) on the electrocardiogram (ECG) but can have elevated levels of cardiac biomarkers if the ischemia is severe
and prolonged, in which case they are classified as having type 2 MI. The atherothrombotic lesion is the hallmark pathobio-
logic event of an acute coronary syndrome (ACS). The reduction in flow may be caused by a completely occlusive thrombus
(bottom half, right side) or by a subtotally occlusive thrombus (bottom half, middle). Ischemic discomfort may occur with or
without STE on the ECG. Of patients with STE, Q-wave MI ultimately develops in most, but not all, patients, depending on the
duration of ischemia and collateralization. Patients without STE have either unstable angina or non–ST-segment elevation MI
(NSTEMI), a distinction that is ultimately made by the presence or absence of a serum or plasma cardiac marker (e.g., cardiac
troponin) detected in the blood. Non–Q-wave MI eventually develops in most patients with NSTEMI on the ECG; Q-wave MI
may develop in a few. MI that develops as the result of the atherothrombotic lesion of an ACS is classified as type 1 MI. (From
Zipes, D.P., Libby, P., Bonow, R.O., Mann, D.L., Tomaselli, G.F., Braunwald E. [2019]. Braunwald’s heart disease: a textbook of
cardiovascular medicine, ed 11, Philadelphia, Elsevier. [Original source: Modified from Thygesen, K., Alpert, J.S., Jaffe, A.S.,
et al. [2012]. Third universal definition of myocardial infarction. J Am Coll Cardiol, 60, 1581.)

• ST-segment changes. During the acute (minutes to hours) phase, changes in the ST segment (e.g.,
elevation, depression) may be evident on the ECG. Recognizing these ECG changes and communicat-
ing these findings is essential when caring for the patient with suspected ACS. In a patient who is
experiencing an ACS, new horizontal or downsloping ST segment depression of 0.5 mm or more is
suggestive of myocardial ischemia when observed in two anatomically contiguous leads (Thygesen
et al., 2018). Progressive inversion of T waves may also be present. Evidence of myocardial injury can
be seen on the ECG as STE. New or presumed new STE of 1 mm or more at the J point in all leads
other than V2 and V3 in a patient who is experiencing an ACS is suggestive of myocardial injury when
observed in two or more anatomically contiguous leads (O’Gara et al., 2013). For leads V2 and V3, STE
CHAPTER 8  Acute Coronary Syndromes 189

Normal A. Hyperacute B. Acute C. Early D. Indeterminate E. Old

Fig. 8.10  ​Total acute coronary occlusion leads to serial electrocardiogram changes. The evolution is variable and may be
interrupted or altered by successful reperfusion. ST segment elevation is an early and relatively specific indicator of the
need for acute reperfusion in patients with an acute coronary syndrome.  (From Bersten, A.D., Handy, J.M. [2019]. Oh’s
intensive care manual, China, Elsevier.)

V1 V2 V3 V4 V5

A
V1 V2 V3 V4 V5

B
Fig. 8.11  ​Chest leads from a patient with acute anterior ST-segment elevation myocardial infarction (STEMI). (A) In the
earliest phase of the infarction, tall, positive (hyperacute) T waves are seen in leads V2 to V5. (A) Several hours later,
marked ST-segment elevations are present in the same leads, and abnormal Q waves are seen in leads in V1 and V2.
(From Goldberger, A.L., Goldberger, Z.D. [2018]. Goldberger’s clinical electrocardiography, ed 9, Philadelphia, Elsevier.)

is considered significant if it is elevated 2 mm or more in men 40 years and older, 2.5 mm or more in
men younger than 40 years, or 1.5 mm or more in women (Thygesen et al., 2018). Because ST-segment
changes can evolve quickly, continuous ST-segment monitoring is recommended.

ACLS Pearl
Experts note that ST segment depression of 1 mm or more in six leads, which may be associated
with STE in leads aVR or lead V1 and hemodynamic compromise, is suggestive of multivessel
disease or left main disease (Thygesen et al., 2018).

• QRS changes. In the early (hours to days) period after STEMI, a gradual loss of R waves and the devel-
opment of pathologic Q waves may be observed in the leads viewing the infarcted area (Fig. 8.11).
• T-wave inversion. In the indeterminate (days to weeks) period after STEMI, pathologic Q waves
and inverted T waves usually persist, while ST segments normalize. Inverted T waves may remain
inverted for varying periods ranging from days to weeks or months, or they may remain inverted
permanently (Wagner et al., 2009).

Anatomic Location of a Myocardial Infarction


The areas of the heart supplied by the three major coronary arteries are shown in Fig. 8.12. Anatomic
regions of the left ventricle include the septal, anterior, lateral, inferior, and inferobasal (i.e., posterior)
surfaces. Leads that view the same surfaces of the heart can be grouped and analyzed for ECG evi-
dence of myocardial ischemia, injury, or infarction (Fig. 8.13). Because ECG evidence must be found
in at least two contiguous leads, assessing lead groupings for indicative changes helps determine the
location of the area at risk and predict which coronary artery is affected (Table 8.3). In general, the
more leads showing indicative changes, the larger the area of myocardium involved.
190 CHAPTER 8  Acute Coronary Syndromes

RCA LAD LAD


RCA LAD Cx or or or
Cx Cx RCA

1. Four chamber 2. Two chamber 3. Long axis

4. Base 5. Mid 6. Apex


Fig. 8.12  ​Typical myocardial segments supplied by the right coronary artery (RCA), left anterior descending artery (LAD),
and circumflex (Cx) coronary arteries. The coronary anatomy is shown on the left with the corresponding wall segments in
standard echocardiographic views on the right. The arterial distribution varies among patients. Some segments have variable
coronary perfusion, as indicated by the hatched regions.  (From Otto, C.M. [2018]. Textbook of clinical echocardiography,
ed 6, Philadelphia, Elsevier. [Original source: Lang, R.M., Bierig, M., Devereux, R.B., et al. [2005]. Recommendations for
chamber quantification: a report from the American Society of Echocardiography’s Guidelines and Standards Committee and
the Chamber Quantification Writing Group, developed in conjunction with the European Association of Echocardiography, a
branch of the European Society of Cardiology. J Am Soc Echocardiogr, 18, 1440-1463].)

Fig. 8.13  ​The surfaces of the heart and facing leads. (From Phalen, T., Aehlert, B. [2019]. The 12-lead ECG in acute coronary
syndromes, ed 4, St. Louis, Elsevier.)

ACLS Pearl
Leads II, III, and aVF are contiguous leads because they view the inferior wall of the left ventricle.
Leads I, aVL, V5, and V6 are contiguous because they all look at adjoining tissue in the lateral wall
of the left ventricle. Numerically consecutive chest leads are also contiguous leads.
CHAPTER 8  Acute Coronary Syndromes 191

TABLE 8.3 Relationships Among Ventricular Surfaces, Facing Leads,


and Coronary Arteries
Ventricular Surface Indicative Changes (Facing Leads) Affected (Culprit) Coronary Artery
Anterior V3, V4 LAD
Inferior II, III, aVF RCA (most common) or Cx
Lateral I, aVL, V5, V6 Cx
Septal V1, V2 LAD
Inferobasal (posterior) V7, V8, V9 RCA or Cx
Right ventricle V1R-V6R RCA
Cx, Circumflex; LAD, left anterior descending; RCA, right coronary artery

Localization of an infarction works reasonably well for STEMI. However, ST-segment depression and
T-wave changes that suggest the presence of myocardial ischemia, as in NSTE-ACS, are less reliable in
localizing the culprit vessel because these ECG changes reflect subendocardial rather than transmural
ischemia (Halim et al., 2010). Factors including the anatomic position and size of the heart, the patient’s
unique pattern of coronary artery distribution, the location of the occlusion along the length of the coro-
nary artery, the presence of collateral circulation, previous infarctions, and drug- and electrolyte-related
ECG changes may also affect the perceived location of an infarction versus its actual location.

Anterior Infarction
The left anterior descending (LAD) artery supplies the anterior wall of the heart through its di-
agonal branches, and the anterior two-thirds of the interventricular septum through its septal
perforating branches (Fig. 8.14). Evidence of an anterior infarction can be seen in leads V3
and V4, which face the anterior wall of the left ventricle. Septal involvement is evidenced
by changes in leads V1 and V2 (Fig. 8.15). If an infarction involves the anterior wall and septum,

Aorta

Left main
occlusion
Proximal
LAD occlusion
Septal artery
Circumflex artery

Obtuse
marginal artery Diagonal artery

Mid-LAD
Left anterior occlusion
descending artery (LAD)

I V1 V4
Lateral aVR Septum Anterior

II aVL V2 V5
Inferior Lateral Septum Lateral

III aVF V3 V6
Inferior Inferior Anterior Lateral

Fig. 8.14  ​Anterior wall infarction. Occlusion of the midportion of the left anterior descending (LAD) artery results in an
anterior infarction. Proximal occlusion of the LAD may become an anteroseptal infarction if the septal branch is involved or
an anterolateral infarction if the marginal branch is involved. If the occlusion occurs proximal to both the septal and diago-
nal branches, an extensive anterior infarction will result.  (From Phalen, T., Aehlert, B. [2019]. The 12-lead ECG in acute
coronary syndromes, ed 4, St. Louis, Elsevier.)
192 CHAPTER 8  Acute Coronary Syndromes

Aorta

Left main
coronary artery
Right coronary
artery (RCA)

Circumflex
artery

Left anterior
descending
artery (LAD)

I V1 V4
aVR
Lateral Septum Anterior
II aVL V2 V5
Inferior Lateral Septum Lateral
III aVF V3 V6
Inferior Inferior Anterior Lateral

Fig. 8.15  ​Septal infarction. (From Phalen, T., Aehlert, B. [2019]. The 12-lead ECG in acute coronary syndromes, ed 4,
St. Louis, Elsevier.)

I aVR V1 V4

aVL V2 V5
II

V3 V6
III aVF

Fig. 8.16  ​Anteroseptal infarction. (From Phalen, T., Aehlert, B. [2019]. The 12-lead ECG in acute coronary syndromes, ed 4,
St. Louis, Elsevier.)

ECG changes will be visible in V1, V2, V3, and V4 and the descriptive name anteroseptal MI is
used (Fig. 8.16).
Because the LAD artery supplies a large portion of the left ventricle, a blockage in this area can lead
to complications, such as left ventricular dysfunction, including left-sided heart failure and cardiogenic
shock. An anterior MI may cause dysrhythmias, including PVCs, atrial flutter, or AFib. A blockage in
the area of the septum, which contains the bundle branches, may result in right or left BBB, second-
degree AV block type II, and third-degree AV block.

Lateral Infarction
Lateral wall infarctions often occur as extensions of anterior or inferior infarctions because the lateral
wall of the left ventricle may be supplied by the circumflex (Cx) artery, the LAD artery, or a branch of
the right coronary artery (RCA) (Fig. 8.17). Because the lateral wall of the left ventricle is viewed by
a combination of chest (V5 and V6) and limb (I and aVL) leads, evidence of a lateral wall infarction
may be seen in some or all of the following leads: I, aVL, V5, and V6. An example of an infarction
involving the lateral wall is shown in Fig. 8.18.
CHAPTER 8  Acute Coronary Syndromes 193

Aorta
Left main
coronary artery
Right coronary Circumflex
artery (RCA) artery
a Obtuse marginal

b c

Right ventricular Diagonal artery


marginal branch Left anterior
Posterior descending
descending artery artery (LAD)

I V1 V4
aVR
Lateral Septum Anterior

II aVL V2 V5
Inferior Lateral Septum Lateral

III aVF V3 V6
Inferior Inferior Anterior Lateral

Fig. 8.17  ​Lateral wall infarction. Coronary artery anatomy shows (a) blockage of the circumflex artery, (b) blockage of the
proximal left anterior descending artery, and (c) blockage of the diagonal artery. (From Phalen, T., Aehlert, B. [2019]. The
12-lead ECG in acute coronary syndromes, ed 4, St. Louis, Elsevier.)

I aVR V1 V4

II aVL V2 V5

III aVF V3 V6

Fig. 8.18  ​Lateral wall infarction. Lead I shows a small Q wave with ST-segment elevation (STE). A larger Q wave with STE
can be seen in lead aVL. This patient had an anterior non–ST-elevation myocardial infarction (NSTEMI) 4 days earlier with
STE and T-wave inversion in leads V2 through V6. A coronary arteriogram at that time showed a blocked LAD artery distal
to its first large septal perforator. The STE evolved and the T waves in all of the chest leads had become upright the day
before this tracing was recorded. The patient then had another episode of chest pain associated with the appearance of
signs of acute lateral infarction, as shown in this tracing. A repeat coronary arteriogram showed a new blockage of the
obtuse marginal branch of the circumflex artery. (From Surawicz, B., Knilans, T.K. [2001]. Chou’s electrocardiography in
clinical practice: adult and pediatric, ed 5, Philadelphia, Saunders.)
194 CHAPTER 8  Acute Coronary Syndromes

Inferior Infarction
The RCA supplies the inferior wall of the left ventricle in most individuals (Fig. 8.19); however, in some
patients, the Cx artery supplies the inferior wall through the posterior descending artery (Fig. 8.20).
Examine limb leads II, III, and aVF for ECG evidence of an ACS involving the inferior wall. Leads
aVR or V1 may show STE of 1 mm or more in patients with inferior and suspected right ventricular
infarction (RVI) (Thygesen et al., 2018).
Parasympathetic nervous system hyperactivity is common with inferior wall MIs, resulting in bra-
dydysrhythmias, hypotension, or both (Scirica et al., 2019). Conduction delays, such as first-degree
AV block and second-degree AV block type I are common and usually transient. An example of an
infarction involving the inferior wall is shown in Fig. 8.21.

Aorta

Left main
coronary artery
Dominant right
coronary artery (RCA) Septal artery

a Circumflex artery

b Obtuse
marginal artery
Right ventricular
marginal branch Diagonal artery

Posterior Left anterior


descending artery descending
artery (LAD)
Posterolateral branch
of the circumflex artery
Fig. 8.19  ​Inferior wall infarction. Coronary anatomy shows a dominant right coronary artery (RCA). A blockage at point “a”
results in an inferior infarction and right ventricular infarction. A blockage at point “b” involves only the inferior wall, sparing
the right ventricle. (From Phalen, T., Aehlert, B. [2019]. The 12-lead ECG in acute coronary syndromes, ed 4, St. Louis, Elsevier.)

Aorta

Left main
coronary artery
Nondominant
right coronary Dominant
artery (RCA) circumflex artery I V1 V4
Lateral aVR Septum Anterior
b Obtuse marginal
artery II aVL V2 V5
a Inferior Lateral Septum Lateral
Diagonal artery
Right ventricular III aVF V3 V6
marginal branch Inferior Inferior Anterior Lateral
Left anterior
descending
Posterior artery (LAD)
descending artery
Fig. 8.20  ​Inferior wall infarction. Coronary anatomy shows a dominant circumflex artery. A blockage at point “a” results
in an inferior infarction. A blockage at “b” may result in a lateral and inferobasal infarction. (From Phalen, T., Aehlert, B.
[2019]. The 12-lead ECG in acute coronary syndromes, ed 4, St. Louis, Elsevier.)

Inferobasal Infarction
About 15% to 30% of MIs that involve the posterior or posteroseptal wall also extend into the right
ventricle (Kumar et al., 2018). Experts recommend that the term inferobasal wall be used instead of
posterior wall (Thygesen et al., 2018). The inferobasal wall of the left ventricle is supplied by the Cx
coronary artery in most patients; however, in some patients, it is supplied by the RCA (Fig. 8.22).
Although isolated inferobasal infarctions do occur, an inferobasal infarction more commonly occurs
with lateral wall or inferior wall infarctions. If the RCA supplies the inferobasal wall, complications
may include dysrhythmias that involve the sinoatrial node, the AV node, and the bundle of His.
CHAPTER 8  Acute Coronary Syndromes 195

I aVR V1 V4

II aVL V2 V5

III aVF V3 V6

Fig. 8.21  ​ST elevation in leads II, III, and aVF suggest an inferior wall injury pattern. Reciprocal ST depression is seen in leads
I and aVL. (From Johnson, R., Schwartz, M. [1986]. A simplified approach to electrocardiography, Philadelphia, Saunders.)

Aorta

Left coronary
artery
Circumflex
artery
Obtuse Posterior
marginal artery descending
artery
Left anterior
descending Right coronary
A branch artery

Aorta

Left coronary
artery
Circumflex
artery I V1 V4 V7
Posterior aVR
Lateral Septum Anterior Posterior
Obtuse descending
marginal artery artery II aVL V2 V5 V8
Inferior Lateral Septum Lateral Posterior
Left anterior
descending III aVF V3 V6 V9
branch Right Inferior Inferior Anterior Lateral Posterior
coronary
B artery
Fig. 8.22  ​Inferobasal (posterior) infarction. (A) Coronary anatomy shows a dominant right coronary artery (RCA). Blockage
of the RCA commonly results in an inferior and inferobasal infarction. (B) Coronary anatomy shows a dominant circumflex
artery. Blockage of a marginal branch is the cause of most isolated inferobasal infarctions. (From Phalen, T., Aehlert, B.
[2019]. The 12-lead ECG in acute coronary syndromes, ed 4, St. Louis, Elsevier.)

Because no leads of a standard 12-lead ECG directly view the inferobasal wall of the left ventricle,
posterior chest leads V7, V8, and V9 should be used to detect evidence of an inferobasal infarction.
Indicative changes of an inferobasal infarction include STE in these leads. Placement of additional
posterior chest leads in the right midscapular line (V10), right paraspinal line (V11), and left scapular
line (V12) has been suggested and may increase the likelihood of identifying an inferobasal infarction
(Vasaiwala & Schreiber, 2008). An example of an inferobasal infarction is shown in Fig. 8.23.
196 CHAPTER 8  Acute Coronary Syndromes

I aVR V1 V4

RV4

V5
II aVL V1
V8

III aVF V1 V6
V9

Fig. 8.23  ​Fifteen-lead electrocardiogram with inferior, lateral, posterior, and right ventricular acute myocardial infarction
(AMI). The standard 12-lead ECG reveals the typical ST-segment elevation (STE) in the inferior and lateral leads, as well as
ST-segment depression with prominent R wave in the right precordial leads. Posterior AMI is indicated by both the right
precordial ST-segment depression with prominent R wave and the STE in posterior leads V8 and V9. Note that the degree of
STE is less pronounced than that seen in the inferior leads because of a relatively longer distance from the posterior
epicardium to surface leads. The right ventricular infarction is noted in this case, using the simplified approach with only
RV4, which demonstrates STE of relatively small magnitude. (From Walls, R.M., Hockberger, R.S., Gausche-Hill. M. [2018].
Rosen’s emergency medicine: concepts and clinical practice, ed 9, Philadelphia, Elsevier.)

ACLS Pearl
If a patient presents with a possible ACS and the only ST-segment change seen on a standard
12-lead ECG is depression (particularly in leads V1 through V4), consider obtaining posterior chest
leads V7 through V9 to assess for a possible inferobasal (i.e., posterior) infarction.

Right Ventricular Infarction


When an RVI occurs, it is most often the result of an occlusion of the RCA (Fig. 8.24). All patients
with inferior STEMI should be evaluated for evidence of RVI because about one-third of patients with
inferior STEMI have some involvement of the right ventricle (O’Gara et al., 2013). The right ventricle
is a volume-dependent chamber; therefore early recognition of RVI is essential to avoid interventions
that are likely to lower right heart preload (e.g., nitrates, diuretics) (Namana et al., 2018).
The most sensitive ECG signs of right ventricular injury include 1-mm ST elevation in lead V1 and lead
V4R (O’Gara et al., 2013). An example of an infarction involving the right ventricle is shown in Fig. 8.25.
In addition to the typical ACS signs and symptoms, patients may present with, or subsequently develop, a
hemodynamic triad that includes hypotension, jugular venous distention, and clear lungs.

ACLS Pearl
In addition to RVI, elevation of the ST segment in lead V4R may be seen in conditions such as
acute pulmonary embolism and pericarditis.

Complications include bradydysrhythmias, which can precipitate severe hemodynamic compromise


in patients with RVI (Namana et al., 2018). Temporary pacing is often required. Additional complica-
tions may include AV blocks, atrial and ventricular dysrhythmias, hypotension that can progress to
cardiogenic shock, and right ventricular thrombi.
CHAPTER 8  Acute Coronary Syndromes 197

Aorta

Left main
coronary artery
Right coronary
artery (RCA)
Circumflex
a artery
b

Right ventricular
marginal branch Left anterior
descending
Posterior artery (LAD)
descending artery

Posterolateral branch
of the circumflex artery

I V1 V4 V4R
aVR
Lateral Septum Anterior Rt ventricle

II aVL V2 V5 V5R
Inferior Lateral Septum Lateral Rt ventricle
III aVF V3 V6 V6R
Inferior Inferior Anterior Lateral Rt ventricle

Fig. 8.24  ​Right ventricular infarction (RVI). At “a,” blockage of the right coronary artery proximal to the right ventricular
marginal branch results in an inferior infarction and RVI. At “b,” blockage of the right ventricular marginal branch results
in an isolated RVI.  (From Phalen, T., Aehlert, B. [2019]. The 12-lead ECG in acute coronary syndromes, ed 4, St. Louis,
Elsevier.)

Right-sided leads

I aVR V1 V4

II aVL V2 V5

III aVF V3 V6

V1

II

V5

Fig. 8.25  ​The right-sided chest leads in a patient with acute inferior ST-segment elevation myocardial infarction show ST
segment elevation in leads V4R and V5R, consistent with concomitant right ventricular infarction. (From Adams, J.G. [2013].
Emergency medicine, ed 2, Philadelphia, Saunders.)

Lead aVR
Some clinicians believe that lead aVR reflects reciprocal changes from leads aVL, II, V5, and V6 (Gorgels
et al., 2001). Research has shown value in the use of lead aVR in evaluating CAD and clinical situations,
including pericarditis, pulmonary embolism, tricyclic antidepressant toxicity, tension pneumothorax,
stress-induced cardiomyopathy, and as a means of differentiating atrial tachydysrhythmias (Vorobiof &
Ellestad, 2011). Lead aVR has also been used to distinguish between left main coronary artery (LMCA)
disease and disease of the proximal LAD. STE in lead aVR that is greater than or equal to that in V1
suggests LMCA disease; conversely, STE that is greater in V1 than in aVR suggests disease of the
proximal LAD artery (Matos et al., 2019; Yamaji et al., 2001).
198 CHAPTER 8  Acute Coronary Syndromes

Imaging Studies
Obtain a portable chest radiograph for patients with suspected ACS within 30 minutes of patient
presentation. Two-dimensional transthoracic echocardiography is useful for the evaluation of left and
right ventricular function, including the assessment of myocardial thickness, thickening, and motion at
rest. Echocardiography is also helpful for detecting mechanical complications of acute MI, including
pericardial effusion, myocardial free wall rupture, acute ventricular septal defect, acute mitral regurgita-
tion, and intracardiac thrombus formation.
Other imaging studies, such as transesophageal echocardiography, a contrast-enhanced computed
tomography scan of the chest, or magnetic resonance imaging are useful for excluding some of the
nonischemic causes of acute chest pain, such as valvular heart disease, aortic dissection, and pulmonary
embolism.

INITIAL MANAGEMENT OF ACUTE CORONARY


SYNDROMES
Treatment of the patient with a suspected ACS is time-sensitive, and interventions must be performed
efficiently. Therapeutic interventions are aimed at improving myocardial tissue oxygen supply and reduc-
ing myocardial oxygen demand, protecting ischemic myocardium, relieving ischemic chest discomfort,
restoring coronary blood flow, preventing major adverse cardiovascular events including reocclusion of
the artery, and managing complications of ACS, including cardiac dysrhythmias.

Prehospital Management
When arriving on the scene of a patient who is complaining of chest discomfort or an anginal equivalent,
quickly perform a primary assessment and stabilize the patient’s airway, breathing, and circulation
(ABCs) as necessary. Allow the patient to assume a position of comfort. Assess vital signs and oxygen
saturation. Because evidence shows that the routine administration of supplemental oxygen may be harm-
ful, oxygen may be withheld from patients with a possible ACS who have a normal oxygen saturation
(i.e., who are not hypoxemic) in the prehospital, emergency department, and hospital settings (O’Connor
et al., 2015). If the oxygen saturation is less than 90%, give oxygen starting at 4 L/min and titrate.

ACLS Pearl
Research has shown that high-flow (6 L/min) oxygen administration reduces epicardial coronary
blood flow and increases coronary vascular resistance (Farquhar et al., 2009). More recently, the
Air Versus Oxygen in ST-Elevation Myocardial Infarction (AVOID) trial demonstrated that supple-
mental high-flow oxygen therapy in patients with STEMI but without hypoxia was associated with
larger myocardial infarct size assessed at 6 months (Stub et al., 2015).

Obtain a focused history, including the time of symptom onset. Assess and document the location,
character, duration, and severity of the patient’s pain or discomfort using a 0-to-10 scale. Give aspirin if
no contraindications are present. Establish cardiac monitoring and obtain a diagnostic-quality 12-lead
ECG as soon as possible (O’Connor et al., 2015). Perform a secondary assessment during transport as
dictated by the patient’s condition.
STEMI alert programs exist in many emergency medical services (EMS) systems and hospitals
across the country in an attempt to minimize total ischemic time, which is defined as the time from
onset of symptoms of STEMI to successful reperfusion (O’Gara et al., 2013).
If the prehospital 12-lead ECG clearly shows evidence of STEMI, alert the receiving hospital, and
begin completing a fibrinolytic checklist. Establish intravenous (IV) lines in transit and give medications
for pain control per local or system protocol. Experts encourage the development of local protocols that
allow preregistration and direct transport to the catheterization laboratory of a percutaneous coronary
intervention (PCI)-capable hospital (bypassing the emergency department) for patients who do not re-
quire emergent stabilization upon arrival (O’Gara et al., 2013). Although prehospital fibrinolytic therapy
is not used in most U.S. communities, multiple studies have demonstrated its safety (O’Gara et al., 2013).
CHAPTER 8  Acute Coronary Syndromes 199

In communities where prehospital fibrinolysis is part of a STEMI system of care, current guidelines state
that prehospital fibrinolysis is reasonable when transport time is more than 30 minutes, and in-hospital
fibrinolysis is the alternative treatment strategy (O’Connor et al., 2015). In communities where prehos-
pital fibrinolysis is available, and transport directly to a PCI-capable hospital is available, transport
directly to the PCI facility may be preferred because the incidence of intracranial hemorrhage, although
relatively rare, is greater with fibrinolysis (O’Connor et al., 2015).

Emergency Department Management


Although patients experiencing ischemic chest pain symptoms may arrive in the emergency department by
ambulance, many arrive using a private vehicle. Triage patients who arrive by private vehicle immediately.
Quickly assess the patient’s ABCs and ensure that the patient has a secure airway and adequate
breathing. Frequent assessment of the patient’s mental status, vital signs, and oxygen saturation level
are important, and continuous ECG monitoring is essential. Administer supplemental oxygen if indi-
cated. If not already done by EMS, obtain a 12-lead ECG and give aspirin if no contraindications are
present. Establish IV access and complete a fibrinolytic checklist. Draw initial laboratory tests, includ-
ing cardiac biomarkers, electrolytes, and coagulation studies within 10 minutes, and obtain a chest
radiograph within 30 minutes of the patient’s arrival.
Obtain a targeted history and physical examination while performing other procedures. Assess and
document the character of the patient’s chest discomfort, the presence of risk factors for CAD, and the
presence of associated signs and symptoms. Consider the possibility of other conditions that can mimic
acute MI (e.g., aortic dissection, acute pericarditis, acute myocarditis, panic attack, and pulmonary em-
bolism). Continually reassess the degree of the patient’s pain or discomfort using a 0-to-10 scale and
reassess the patient’s response to medications given. Consider antiplatelet therapy with P2Y12 inhibitors.
Use risk assessment tools to ascertain the patient’s risk of an adverse outcome, determine the need
for hospital admission, and make decisions about potential treatment options. Within 10 minutes of
the patient’s arrival, carefully review the 12-lead for ECG evidence of an ACS. Patients experiencing
a STEMI are considered the most emergent, followed by those with NSTE-ACS, and then persons
experiencing chest pain of probable cardiac origin. Based on the 12-lead ECG findings, categorize the
patient into one of the following groups:
1. STE. Patients with STE in two or more contiguous leads are classified as having a STEMI. They
should be evaluated for immediate pharmacologic (i.e., fibrinolytics) or mechanical (i.e., PCI) reperfu-
sion therapy (discussed later in this chapter). The goals of reperfusion are to administer fibrinolytics
within 30 minutes of arrival or to provide PCI within 90 minutes of arrival (O’Connor et al., 2015).
Evaluate patients with obvious STE in leads II, III, and/or aVF for a possible RVI.
2. ST-segment depression. ST depression or transient ST-segment/T-wave changes that occur with
chest discomfort suggest myocardial ischemia. Evaluate patients with apparent ST-segment depres-
sion in leads V1 and V2 for possible inferobasal MI. Admit patients presenting with NSTE-ACS,
including those with recurrent symptoms, ischemic ECG changes, or positive cTn, to a monitored
bed for further evaluation (Amsterdam et al., 2014). Admit stabilized patients with NSTE-ACS to
an intermediate (or step-down) care unit (Amsterdam et al., 2014). Admit patients with continuing
angina, hemodynamic instability, uncontrolled dysrhythmias, or a large MI to a coronary care unit
(Amsterdam et al., 2014). Treatment options for NSTE-ACS are based on risk stratification and
include antianginal, antiplatelet, and anticoagulant therapy (O’Connor et al., 2015). Anterolateral
ST-segment depression is a predictor of poor prognosis with an NSTE-ACS, partly because of
an increased incidence of left main or three-vessel disease compared with individuals without ST
depression (Bob-Manuel et al., 2017).
3. Normal or nondiagnostic ECG. A normal ECG or nonspecific ST- and T-wave changes are nondi-
agnostic and should prompt consideration for further evaluation. Consider admission of the patient
with signs and symptoms suggesting an ACS and a nondiagnostic ECG to the emergency depart-
ment chest pain unit or an appropriate bed (O’Connor et al., 2015). If the initial ECG is not diag-
nostic of STEMI, but the patient remains symptomatic and there is a high clinical suspicion of
STEMI, obtain serial ECGs at 5- to 10-minute intervals or perform continuous ST segment
monitoring to detect the potential development of STE. Noninvasive tests (e.g., computed tomog-
raphy angiography, cardiac magnetic resonance, myocardial scintigraphy, stress echocardiography)
can be useful in identifying patients suitable for discharge from the emergency department
(O’Connor et al., 2015). The ACS algorithm appears in Fig. 8.26.
200 CHAPTER 8  Acute Coronary Syndromes

Symptoms suggestive of myocardial ischemia or infarction

EMS assessment and care

• Assess, monitor, and support ABCs; assess vital signs


• Give aspirin if no contraindications, give oxygen if oxygen saturation is less than 90%
• Obtain 12-lead ECG; if STEMI, notify receiving hospital/activate the cardiac cath lab
• Give nitroglycerin and morphine if needed
• Use fibrinolytic checklist if considering prehospital fibrinolysis

ED assessment and care (assessment within 10 minutes of ED arrival)

• Stabilize ABCs, obtain 12-lead ECG (if not done by EMS), assess vital signs and oxygen saturation,
give oxygen if oxygen saturation is less than 90% (start at 4 L/min and titrate)
• Establish IV access, obtain targeted history, perform physical exam, review/complete fibrinolytic
checklist
• Obtain initial laboratory studies within 10 min; obtain portable chest radiograph within 30 min
• Give aspirin if not given by EMS, give nitroglycerin and morphine if needed, consider antiplatelet
therapy with P2Y12 inhibitors

Evaluate initial 12-lead ECG

ST depression or transient Normal ECG or nonspecific


ST elevation or new left BBB?
ST/T wave changes? ST/T wave changes?

STEMI High-risk NSTE-ACS Low/intermediate risk ACS


(NSTEMI/UA)
Begin adjunctive therapies (e.g., Begin adjunctive therapies Consider admission to ED
nitroglycerin, heparin) as (e.g., nitroglycerin, heparin) chest pain unit or appropriate
indicated (do not delay as indicated bed for ongoing monitoring and
reperfusion) possible intervention

Symptom Symptom
onset 12 hours onset more Troponin elevated or
or less than 12 hours high-risk patient

Consider early invasive


strategy if:
• Refractory ischemic chest
Determine pharmacologic or
discomfort
mechanical reperfusion strategy
• Recurrent/persistent ST
Goals
deviation
• Pharmacologic (fibrinolysis):
• Ventricular tachycardia
ED door-to-needle time 30 min
• Hemodynamic instability
or less
• Signs of heart failure
• Mechanical (PCI): Door-to-
balloon inflation 90 min or less
from first medical contact
ABCs = Airway, breathing, circulation; ACS = acute coronary syndrome; BBB = bundle branch block; ECG = electrocardio-
gram; ED = emergency department; EMS = emergency medical services; IV = intravenous; MI = myocardial infarction;
NSTE-ACS = non–ST-elevation acute coronary syndrome; PCI = percutaneous coronary intervention; STEMI = ST-elevation
myocardial infarction; UA = unstable angina.
(Sources: Amsterdam et al., 2014; O'Connor et al., 2015; O'Gara et al., 2013.)
Fig. 8.26  Acute coronary syndromes algorithm.
CHAPTER 8  Acute Coronary Syndromes 201

Pharmacologic Therapies
Relief of cardiac-related discomfort is a priority for the management of a patient who is experiencing
an ACS and often requires a combination of therapies, such as bedrest, NTG, and opioid analgesics.
The relief of pain decreases anxiety, myocardial oxygen demand, and the risk of dysrhythmias.

Nitroglycerin
NTG produces venous vasodilation (Table 8.4). Dilation of the capacitance vessels (i.e., veins) decreases
venous return to the heart, reducing ventricular volume and decreasing ventricular wall tension. Decreasing
cardiac preload decreases myocardial oxygen demand. NTG also dilates normal and atherosclerotic epicar-
dial coronary arteries and increases coronary collateral flow (Amsterdam et al., 2014).

TABLE 8.4 Nitroglycerin


Nitroglycerin (NTG) NSTE-ACS STEMI
Sublingual Indications Class I recommendation Patients with ongoing ischemic
and Dosage (Amsterdam et al., 2014): discomfort should receive
Sublingual NTG may be given at 5-min sublingual NTG (0.4 mg) every
intervals to a maximum of three doses 5 min up to three doses as BP
allows (O’Gara et al., 2013)
Intravenous Indications Class I recommendation May be useful to treat patients with
(Amsterdam et al., 2014): STEMI and hypertension or HF
IV NTG is indicated for patients with (O’Gara et al., 2013)
NSTE-ACS for the treatment of
persistent ischemia, HF, or hypertension
Notes Nitrates should not be administered to patients with a systolic BP less than
90 mm Hg or 30 mm Hg or more below baseline, marked bradycardia or
tachycardia, phosphodiesterase inhibitor use within the previous 24 to
48 hours, or suspected RVI (O’Gara et al., 2013)
BP, Blood pressure; HF, heart failure; IV, intravenous; mm Hg, millimeters of mercury; NSTE-ACS, non–ST-elevation acute
coronary syndrome; NTG, nitroglycerin; RVI, right ventricular infarction; STEMI, ST-elevation myocardial infarction.

Before giving NTG, assess the degree of the patient’s ischemic discomfort with the use of a 0-to-
10 scale. Also record the pain’s duration, the time of onset, the activity that was being performed, and
the pain quality. Reassess and document the patient’s vital signs and level of discomfort after each
dose. If ischemic discomfort persists after three sublingual NTG doses, consider the use of IV NTG.
Common adverse effects of NTG administration include headache, flushing, tachycardia, dizziness,
and orthostatic hypotension. Hypotension usually responds to supine positioning and the administration
of IV fluids. Make sure that the patient has not used a phosphodiesterase inhibitor, such as sildenafil (e.g.,
Viagra) within 24 hours or tadalafil (e.g., Cialis) within 48 hours before NTG administration. The com-
bination of a phosphodiesterase inhibitor and nitrates may result in severe hypotension. Avoid NTG in
inferior wall MI with a possible associated RVI. Consider the presence of RVI if the patient with an
inferior wall MI becomes hypotensive after nitrate administration.

Analgesic Therapy
Morphine sulfate is a potent narcotic analgesic and anxiolytic (Table 8.5). It causes venodilation, and it can
lower heart rate (through increased vagal tone) and systolic BP, thereby reducing myocardial oxygen de-
mand. Morphine administration is associated with nausea and vomiting, delayed gastric emptying, and re-
duced intestinal mobility. These effects may lead to decreased absorption and delayed onset of action of oral
antiplatelet agents, which can result in decreased peak plasma levels and subsequent early treatment failure
(Eitel et al., 2020; Parodi et al., 2015). Other adverse effects of morphine administration include bradycardia
and respiratory depression. Hypotension may occur, particularly among patients who are volume-depleted
or who have received vasodilators. Use supine positioning or IV boluses of normal saline to restore BP. Use
of a narcotic antagonist (e.g., naloxone) may be necessary to manage respiratory depression or excessive
morphine-related bradycardia. Consider other narcotics for patients who are allergic to morphine.
Before giving morphine, assess the degree of the patient’s pain or discomfort with the use of a 0-to-
10 scale. Determine the duration, the time of onset, the activity being performed, and the quality of
202 CHAPTER 8  Acute Coronary Syndromes

TABLE 8.5 Morphine Sulfate


Non–ST-Elevation Acute Coronary
Morphine Syndrome ST-Elevation Myocardial Infarction
Indications Class IIb recommendation: Morphine sulfate (4 to 8 mg IV initially
and Dosage Morphine sulfate (1 to 5 mg IV) may be [use lower doses in the elderly] with
reasonable for patients with NSTE-ACS increments of 2 to 8 mg IV repeated at
if there is continued ischemic chest pain 5- to 15-min intervals if needed) is the
despite maximally tolerated antiischemic analgesic of choice for patients with
therapy (Amsterdam et al., 2014). Repeat STEMI, especially those whose course
every 5 to 30 minutes as needed to relieve is complicated by acute pulmonary
symptoms and maintain patient comfort edema (O’Gara et al., 2013)
Notes • Ensure that a narcotic antagonist and airway equipment is readily available before
administration
• Factors, such as patient age, body size, BP, and heart influence the dose of morphine
needed to achieve adequate pain control (O’Gara et al., 2013)
• When indicated, administer to reverse the narcotic effects of morphine
• Excessive morphine-related bradycardia may require the administration of atropine
BP, Blood pressure; IV, intravenous; NSTE-ACS, non–ST-elevation acute coronary syndrome; STEMI, ST-elevation myocardial
infarction.

the patient’s discomfort. Reassess and document the patient’s vital signs and level of discomfort after
each morphine dose.
Because of the increased risk of MACEs (e.g., reinfarction, hypertension, heart failure, myocardial
rupture) associated with the use of nonsteroidal antiinflammatory drugs, these drugs (except for aspirin)
should not be initiated in the acute phase of care and should be discontinued in patients using them
before hospitalization (Amsterdam et al., 2014; O’Connor et al., 2015; O’Gara et al., 2013).
Some clinicians prefer to use fentanyl (i.e., Sublimaze) for pain relief, as well as vasodilation in place
of morphine in patients experiencing an ACS. Fentanyl is a synthetic opioid that has minimal cardio-
vascular effects and more rapid onset and shorter duration of action than morphine. The adverse effects
of fentanyl are like those of morphine.

Beta Blockers
The inhibition of beta1-adrenergic receptor sites reduces systolic BP, decreases heart rate and the force
of myocardial contraction, and decreases cardiac workload and myocardial oxygen demand (Table 8.6).
The simultaneous IV administration of beta blockers and IV calcium channel blockers (CCBs) (e.g.,
verapamil, diltiazem) can cause severe hypotension.

Calcium Channel Blockers


Nondihydropyridine CCBs (e.g., verapamil, diltiazem) decrease heart rate and myocardial contractility,
slow conduction through the AV node, and have some peripheral arterial dilatory effects (Table 8.7).
Because they cause coronary vasodilation, these agents are useful in patients with variant angina, in
which ischemia results from coronary artery vasoconstriction.

Lipid Management
Several studies have demonstrated that in patients stabilized after an ACS, statin therapy lowers the
risk of CHD death, stroke, recurrent MI, and the need for coronary revascularization (Amsterdam
et al., 2014; O’Gara et al., 2013) (Table 8.8).

Renin-Angiotensin-Aldosterone System Inhibitors


Angiotensin-converting enzyme (ACE) inhibitors produce vasodilation by blocking the conversion of
angiotensin I into angiotensin II (Table 8.9). Because angiotensin is a potent vasoconstrictor, limiting
its production decreases peripheral vascular resistance, thereby reducing the pressure that the heart
must pump against and decreasing myocardial workload. ACE inhibitors also increase renal blood flow,
which helps rid the body of excess sodium and fluid accumulation. ACE inhibitors have been shown
to reduce fatal and nonfatal major cardiovascular events in patients with STEMI (O’Gara et al., 2013).
An angiotensin receptor blocker (e.g., valsartan) may be substituted for patients who are intolerant of
ACE inhibitors (O’Gara et al., 2013).
CHAPTER 8  Acute Coronary Syndromes 203

TABLE 8.6 Beta Blockers


Non–ST-Elevation Acute
Beta Blockers Coronary Syndrome ST-Elevation Myocardial Infarction
Indications Class I recommendation Class I recommendation (O’Gara et al., 2013):
(Amsterdam et al., 2014): • Oral beta blockers should be initiated in the first
• Initiate oral beta blockers within 24 hours in patients with STEMI who do not
the first 24 hours in the absence have any of the following: signs of HF, evidence of
of HF, low output state, the risk a low output state, increased risk for cardiogenic
for cardiogenic shock, or other shock, or other contraindications to use of oral
contraindications to beta blockade beta blockers (PR interval more than 0.24 seconds,
• Use of sustained-release second- or third-degree heart block, active
metoprolol succinate, carvedilol, asthma, or reactive airway disease)
or bisoprolol is recommended for • Beta blockers should be continued during and
beta blocker therapy with after hospitalization for all patients with STEMI
concomitant NSTE-ACS, stabilized and with no contraindications to their use
HF, and reduced systolic function • Patients with initial contraindications to the use
• Reevaluate to determine of beta blockers in the first 24 hours after
subsequent eligibility in patients STEMI should be reevaluated to determine their
with initial contraindications to subsequent eligibility
beta blockers Class IIa recommendation (O’Gara et al., 2013):
Class IIa recommendation • It is reasonable to administer IV beta blockers
(Amsterdam et al., 2014): at the time of presentation to patients with
• It is reasonable to continue beta STEMI and no contraindications to their use
blocker therapy in patients with who are hypertensive or have ongoing ischemia
normal LV function with NSTE-ACS
Notes • Risk factors for cardiogenic shock are age greater than 70 years, systolic BP less than
120 mm Hg, heart rate greater than 110 beats/min, or increased time since onset of
STEMI symptoms (O’Gara et al., 2013)
• Carefully monitor the patient’s blood pressure, heart rate, and cardiac rhythm after
beta blocker administration
BP, Blood pressure; HF, heart failure; IV, intravenous; LV, left ventricular; mm Hg, millimeters of mercury; NSTE-ACS, non–
ST-elevation acute coronary syndrome; STEMI, ST-elevation myocardial infarction.

TABLE 8.7 Calcium Channel Blockers


Calcium Channel Non–ST Elevation Acute Coronary ST-Elevation Myocardial
Blockers Syndrome Infarction
Indications Class I recommendations (Amsterdam et al., 2014): May be useful to relieve ischemia,
• In patients with NSTE-ACS, continuing or frequently lower BP, or control the
recurring ischemia, and a contraindication to beta ventricular response rate to
blockers, a nondihydropyridine CCB should be AFib in patients who are
given as initial therapy in the absence of clinically intolerant of beta blockers
significant LV dysfunction, increased risk for (O’Gara et al., 2013)
cardiogenic shock, PR interval greater than
0.24 second, or second- or third-degree AV block
without a cardiac pacemaker
• Oral nondihydropyridine calcium antagonists are
recommended in patients with NSTE-ACS who
have recurrent ischemia in the absence of
contraindications, after appropriate use of beta
blockers and nitrates
• CCBs are recommended for ischemic symptoms
when beta blockers are not successful, are contra-
indicated, or cause unacceptable side effects
• Long-acting CCBs and nitrates are recommended in
patients with coronary artery spasm
AFib, Atrial fibrillation; AV, atrioventricular; BP, blood pressure; CCB, calcium channel blocker; HF, heart failure; IV, intrave-
nous; LV, left ventricular; NSTE-ACS, non–ST-elevation acute coronary syndrome; STEMI, ST-elevation myocardial infarction.
204 CHAPTER 8  Acute Coronary Syndromes

TABLE 8.8 Lipid Management


Statin Non–ST-Elevation Acute ST-Elevation Myocardial
Therapy Coronary Syndrome Infarction
Indications Class I recommendation (Amsterdam et al., 2014): Class I and Class IIa recommenda-
High-intensity statin therapy should be initiated or tions same as for NSTE-ACS
continued in all patients with NSTE-ACS and no (O’Gara et al., 2013)
contraindications to its use
Class IIa recommendation (Amsterdam et al., 2014):
It is reasonable to obtain a fasting lipid profile in
patients with NSTE-ACS, preferably within 24 hours
of presentation

NSTE-ACS, Non–ST-elevation acute coronary syndrome; STEMI, ST-elevation myocardial infarction.

TABLE 8.9 Renin-Angiotensin-Aldosterone System Inhibitors


Renin-Angiotensin-Aldosterone Non–ST-Elevation Acute ST-Elevation Myocardial
System Inhibitors Coronary Syndrome Infarction
ACE inhibitors (e.g., lisinopril, Class I recommendation Class I recommendation
captopril, ramipril) (Amsterdam et al., 2014): (O’Gara et al., 2013):
An ACE inhibitor should be An ACE inhibitor should be
started and continued administered within the first
indefinitely in all patients with 24 hours to all patients with
an LVEF less than 0.40 and STEMI with anterior location,
those with hypertension, HF, or an LVEF of 0.40 or less,
diabetes mellitus, or stable unless contraindicated
chronic kidney disease unless Class IIa recommendation
contraindicated (O’Gara et al., 2013):
ACE inhibitors are reasonable for
all patients with STEMI and no
contraindications to their use
ARBs (e.g., Valsartan) Class I recommendation Class I recommendation
(Amsterdam et al., 2014): (O’Gara et al., 2013):
ARBs are recommended in An ARB should be given to
patients with HF or MI with patients with STEMI who
LVEF less than 0.40, who have indications for but are
are ACE-inhibitor intolerant intolerant of ACE inhibitors
Aldosterone antagonists Class I recommendation Class I recommendation
(e.g., eplerenone) (Amsterdam et al., 2014): (O’Gara et al., 2013):
Aldosterone blockade is An aldosterone antagonist should
recommended in patients be given to patients with
post–MI without significant STEMI and no contraindications
renal dysfunction or hyperka- who are already receiving an
lemia who are receiving ACE inhibitor and beta blocker
therapeutic doses of an ACE and who have an LVEF of 0.40
inhibitor and beta blocker or less and either symptomatic
and have an LVEF of0.40 or HF or diabetes mellitus
less, diabetes mellitus, or HF
Notes ACE inhibitors may cause a profound drop in BP after the first dose or
if used with diuretics. Avoid ACE inhibitors and ARBs in patients
with hypotension, renal failure, or hyperkalemia

ACE, Angiotensin-converting enzyme; ARB, angiotensin receptor blocker; BP, blood pressure; HF, heart failure; LVEF, left
ventricular ejection fraction; MI, myocardial infarction; NSTE-ACS, non–ST-elevation acute coronary syndrome; STEMI, ST-
elevation myocardial infarction.
CHAPTER 8  Acute Coronary Syndromes 205

Antiplatelet Therapy
Antiplatelet and anticoagulant therapies are important components of ACS patient management because
exposure of a ruptured plaque’s contents triggers activation of the coagulation cascade. Antiplatelet medica-
tions target specific platelet functions at different levels in the pathway of platelet aggregation (Fig. 8.27,
Table 8.10).
Aspirin is an antiplatelet agent that inhibits cyclooxygenase, an enzyme required by platelets to syn-
thesize thromboxane A2. Nonenteric-coated aspirin (162 to 325 mg) should be administered to patients

Plaque disruption

Platelet adhesion
and aggregation
A

Activation of
coagulation
cascade

B
Thrombin
formation

Stable thrombus

Fibrinogen Fibrin

Clot dissolution

Plasminogen Plasmin

C
Degradation
products

Fig. 8.27  ​Site of action of medications used in the treatment of acute coronary syndromes. (A) Site of action of antiplate-
let agents, such as aspirin, thienopyridines, and glycoprotein IIb/IIIa inhibitors. (B) Heparin bonds with antithrombin III and
thrombin to create an inactive complex. (C) Fibrinolytic agents convert plasminogen to plasmin, an enzyme responsible for
degradation of fibrin clots. (From Urden, L.D., Stacy, K.M., Lough, M.E. [2018]. Critical care nursing: diagnosis and manage-
ment, ed 8, Maryland Heights, MO, Elsevier.)

TABLE 8.10 Antiplatelet Medications


Category Action Example(s)
Cyclooxygenase Inhibit cyclooxygenase, an enzyme required by platelets Aspirin
inhibitors to synthesize thromboxane A2 Naproxen
Ibuprofen
ADP P2Y12 receptor Bind to ADP P2Y12 receptors on the platelet surface, Clopidogrel (Plavix)
inhibitors thereby inhibiting ADP from activating more platelets Prasugrel (Effient)
Ticagrelor (Brilinta)
Cangrelor (Kengreal)
GP IIb/IIIa receptor Block the fibrinogen binding site of the GP IIb/IIIa Abciximab (ReoPro)
inhibitors receptor on the platelet membrane and inhibit Eptifibatide (Integrilin)
platelet aggregation Tirofiban (Aggrastat)
Protease-activated Blocks thrombin-induced platelet aggregation by Vorapaxar (Zontivity)
receptor (PAR-1) blocking the binding site of PAR-1 receptors on
antagonist platelets

ADP, Adenosine diphosphate; GP, glycoprotein.


206 CHAPTER 8  Acute Coronary Syndromes

TABLE 8.11 Aspirin


Non–ST-Elevation Acute Coronary
Aspirin Syndrome ST-Elevation Myocardial Infarction
Indications and Class I recommendations (Amsterdam et al., Class I recommendations
Dosage 2014): (O’Gara et al., 2013):
• Nonenteric-coated, chewable aspirin • Aspirin 162 to 325 mg should be
(162 to 325 mg) should be given to given before primary PCI
all patients with NSTE-ACS without • After PCI, aspirin should be
contraindications as soon as possible after continued indefinitely
presentation, and a maintenance dose of • A loading dose of a P2Y12 receptor
aspirin (81 mg/d to 162 mg/d) should be inhibitor should be given as early
continued indefinitely as possible or at the time of
• In patients with NSTE-ACS who are unable primary PCI to patients with STEMI
to take aspirin because of hypersensitivity • P2Y12 inhibitor therapy
or significant GI intolerance, a loading dose (maintenance doses) should be
of clopidogrel followed by a daily given for 1 year to patients with
maintenance dose should be administered STEMI who receive a stent during
• A P2Y12 inhibitor (either clopidogrel or primary PCI
ticagrelor) in addition to aspirin should be
administered for up to 12 months to all
patients with NSTE-ACS without
contraindications who are treated with either
an early invasive or ischemia-guided strategy

GI, Gastrointestinal; NSTE-ACS, non–ST-elevation acute coronary syndrome; PCI, percutaneous coronary intervention;
STEMI, ST-elevation myocardial infarction.

experiencing an ACS as soon as possible after symptom onset, unless contraindicated, and continued
indefinitely (Amsterdam et al., 2014; O’Gara et al., 2013). Recommendations about the use of aspirin
in NSTE-ACS and STEMI appear in Table 8.11.
Clopidogrel, prasugrel, ticagrelor, and cangrelor are thienopyridines that target P2Y12 receptors,
which are adenosine diphosphate (ADP) receptors on the platelet surface. By blocking P2Y12 receptor
sites, ADP is inhibited from activating additional platelets. Clopidogrel and prasugrel are irreversible
platelet inhibitors that impede platelet function for the life of the platelet. Ticagrelor and cangrelor
bind reversibly to P2Y12 receptors. ADP inhibitors have a synergistic effect when used with aspirin
because they inhibit different platelet-activating pathways.
Glycoprotein (GP) IIb/IIIa receptors are found on the platelet surface. GP IIb/IIIa inhibitors pre-
vent the binding of fibrinogen and limit platelet aggregation. These agents are administered IV and
used in patients undergoing PCIs. When the use of any of these medications is planned, minimize
arterial and venous punctures; intramuscular injections; and the use of urinary catheters, nasotracheal
intubation, and nasogastric tubes. When establishing IV access, avoid noncompressible sites (e.g., the
subclavian or jugular veins).

Anticoagulant Therapy
Anticoagulants have been a mainstay in the management of patients with ACS, in the prevention of stroke
in patients with AFib, and the prevention and treatment of venous thromboembolism, among other condi-
tions (Garg & Halperin, 2013). Older anticoagulants typically require frequent coagulation monitoring
to ensure that a therapeutic response is achieved. For example, warfarin requires coagulation monitoring
because its anticoagulant effects are influenced by dietary vitamin K intake, other medications, and various
disease states (Weitz, 2013). Dabigatran (Pradaxa), rivaroxaban (Xarelto), and apixaban (Eliquis) are oral
anticoagulants that have a wide therapeutic window, fewer drug–drug interactions, an absence of signifi-
cant dietary effects, and less risk of intracranial bleeding than warfarin. Routine coagulation monitoring is
not required in the majority of patients with these new agents; however, strict compliance is critical because
missing even one dose could result in a period without protection from thromboembolism ( January et al.,
2014). Examples of anticoagulants appear in Table 8.12.
For patients with NSTE-ACS, anticoagulation, in addition to antiplatelet therapy, is recom-
mended for all patients regardless of whether an invasive or conservative treatment strategy is planned
CHAPTER 8  Acute Coronary Syndromes 207

TABLE 8.12 Anticoagulants


Anticoagulant Action Route
Apixaban (Eliquis) Factor Xa inhibitor Oral
Argatroban (Acova) Direct thrombin inhibitor IV
Bivalirudin (Angiomax) Direct thrombin inhibitor IV
Dabigatran (Pradaxa) Direct thrombin inhibitor Oral
Dalteparin (Fragmin)a Indirect thrombin inhibitor Subcut; IV if rapid anticoagulant
response needed
Desirudin (Iprivask) Direct thrombin inhibitor Subcut
Enoxaparin (Lovenox)a Indirect thrombin inhibitor Subcut; IV if rapid anticoagulant
response needed
Fondaparinux (Arixtra) Factor Xa inhibitor Subcut
Rivaroxaban (Xarelto) Factor Xa inhibitor Oral
Unfractionated heparin Indirect thrombin inhibitor IV or Subcut
Warfarin (Coumadin) Vitamin K antagonist Oral
a
Low-molecular-weight heparin
IV, Intravenous; Subcut, subcutaneous.

(Amsterdam et al., 2014). For patients with STEMI who are undergoing primary PCI, Class I antico-
agulant therapy recommendations include the use of unfractionated heparin (UFH) with or without a
GP IIb/IIIa inhibitor, or bivalirudin (O’Gara et al., 2013). Current resuscitation guidelines recommend
that EMS systems that do not currently administer heparin to suspected STEMI patients should not
add this treatment, and those that do administer it may continue their current practice (O’Connor et al.,
2015). Administration of UFH can occur either in the prehospital or in-hospital setting for STEMI
patients for whom there is a planned primary PCI reperfusion strategy (O’Connor et al., 2015).

Reperfusion Therapies
Patients who are experiencing STEMI are most likely to benefit from reperfusion therapy. The primary
choices for reperfusion therapy are fibrinolysis and PCI. Fibrinolytics (“clot-busters”) are medications
that work by activating the conversion of plasminogen to plasmin, which then breaks down fibrinogen
and fibrin clots. Fibrinolytic therapy is not indicated in patients with NSTE-ACS. PCI is a procedure
in which a catheter is used to open a coronary artery that has been blocked or narrowed by CAD. The
term primary PCI is used when PCI is performed as the initial reperfusion approach. It is the recom-
mended reperfusion strategy when it can be performed promptly by experienced personnel. Although
several studies have proved that mechanical catheter-based interventions produce better outcomes
when performed promptly, fibrinolytic therapy continues to play a significant role in the treatment of
STEMI because only a minority of U.S. hospitals have primary PCI capabilities.
The first physician who encounters a patient with STEMI should assess the need for reperfusion
therapy and determine how it should be performed (pharmacologic versus mechanical) given the
clinical scenario, the patient’s condition, and the availability of facilities for primary PCI (Werns,
2019). Because consultation delays therapy, routine consultation with a cardiologist or other physician
is not recommended except in equivocal or uncertain cases (O’Connor et al., 2015).
Examples of factors to consider when deciding to use fibrinolytic therapy versus primary PCI include
the following (O’Connor et al., 2015; O’Gara et al., 2013):
• The time from onset of symptoms
• The patient’s clinical presentation and hemodynamic status
• The patient’s age
• The location of the infarction
• The duration of STEMI at the time of initial emergency department presentation
• The presence of patient comorbidities
• The risk of bleeding
• The presence of contraindications
• The time delay to PCI
• The abilities of the PCI cardiologist and hospital
208 CHAPTER 8  Acute Coronary Syndromes

For adult patients who present with STEMI at a PCI-capable hospital, primary PCI should be
performed within 90 minutes of arrival. For adult patients who present with STEMI at a non–PCI-
capable hospital, it is reasonable to transfer the patient to a PCI center if the time from first medical
contact to device is expected to occur within 120 minutes (Frampton et al., 2020).
In the absence of contraindications, fibrinolytic therapy should be administered to patients with
STEMI and onset of ischemic symptoms within the previous 12 hours when it is anticipated that
primary PCI cannot be performed within 120 minutes of first medical contact (O’Gara et al., 2013).
Fibrinolytic therapy is generally not recommended for patients presenting between 12 and 24 hours
after onset of symptoms unless ischemic pain persists with continuing STE, and should not be admin-
istered to patients who present more than 24 hours after symptom onset (O’Connor et al., 2015). PCI
or a coronary artery bypass graft is the preferred reperfusion strategy for STEMI patients who present
in shock (O’Connor et al., 2015). There is no role for fibrinolytic therapy in patients with NSTE-ACS
(Amsterdam et al., 2014; O’Connor et al., 2015).
Before starting fibrinolytic therapy, choose an ECG monitoring lead that shows clear evidence
of STE. During fibrinolytic therapy, monitor the ECG and the patient’s vital signs closely. Pay careful
attention to all potential bleeding sites (including catheter insertion sites, arterial and venous puncture
sites, cutdown sites, and needle puncture sites). Watch for ST changes, dysrhythmias, and hypotension,
and question the patient about ischemic discomfort. When reperfusion occurs, the patient’s chest dis-
comfort typically stops abruptly as blood flow to the ischemic myocardium is restored. Watch for reper-
fusion dysrhythmias (e.g., PVCs, bradycardias, heart block, VT, VF) as blood flow is reestablished
through the infarct-related artery. Previously elevated ST segments should quickly return to baseline as
blood flow is restored to the affected myocardium.
About 20% of patients who achieve reperfusion with fibrinolytic therapy will experience reocclusion
(Frampton et al., 2020). Patients who have evidence of failed reperfusion therapy should be transferred
to a PCI-capable center for “rescue” PCI.
CHAPTER 8  Acute Coronary Syndromes 209

STOP AND REVIEW


Identify one or more choices that best complete the statement or answer the question.

____ 1. Which of the following is the most common cause of the blockage of a coronary
artery?
a. A thrombus
b. Coronary artery spasm
c. Coronary artery trauma
d. Coronary artery dissection

_ ___ 2. ECG changes characteristic of myocardial ischemia include temporary changes in the:
a. P wave.
b. T wave.
c. ST segment.
d. QRS complex.

____ 3. Indicative changes, which are ECG findings that are seen in leads that look directly
at the area fed by a blocked coronary artery, are significant when they are seen in two
anatomically contiguous leads. Which of the following reflects a pair of contiguous
leads?
a. I and aVF
b. V1 and V6
c. V2 and V3
d. II and aVL

____ 4. A 52-year-old woman is complaining of chest pain. The cardiac monitor reveals
a sinus rhythm at 68 beats per minute (beats/min). Her blood pressure is
88/60 mm Hg, and her ventilatory rate is 14 breaths/min. Breath sounds are
clear. There are no signs of pedal edema. A standard 12-lead ECG is obtained
that reveals 3-mm STE in leads II, III, and aVF. The patient is being given
oxygen at 2 L/min by nasal cannula. An IV has been established. You should
now:
a. Give sublingual nitroglycerin and aspirin.
b. Give morphine sulfate and a calcium blocker.
c. Give a beta blocker and complete a fibrinolytic checklist.
d. Attach right-sided chest leads to rule out right ventricular infarction.

____ 5. Which of the following is (are) the preferred cardiac biomarker(s) for diagnosing
MI?
a. Myoglobin
b. Creatine kinase
c. Cardiac troponins
d. Ischemia-modified albumin

_ ___ 6. Transmural ischemia, injury, or infarction affects:


a. Only the myocardium.
b. Only the endocardium.
c. The endocardium and myocardium.
d. The endocardium, myocardium, and epicardium.

_ ___ 7. Which of the following patients is most likely to present atypically with an ACS?
a. A 34-year-old man with no history of heart disease
b. A 56-year-old woman with a history of type I diabetes
c. A 65-year-old man with a history of two previous MIs
d. A 58-year-old man with angina and a strong family history of CAD
210 CHAPTER 8  Acute Coronary Syndromes

____ 8. A 66-year-old woman presents in acute distress. She describes a sudden onset of
severe chest discomfort and nausea that have been present for 2 hours. An initial
12-lead ECG should be obtained within __ minutes of contact with this patient.
a. 10
b. 30
c. 60
d. 90

_ ___ 9. Beta blockers:


a. Increase heart rate.
b. Decrease the force of myocardial contraction.
c. Block the conversion of angiotensin I into angiotensin II.
d. Are contraindicated in patients experiencing an acute coronary syndrome.

____ 10. Studies reveal that women experiencing an ACS may report symptoms that include
which of the following?
a. Anxiety
b. Frequent indigestion
c. Tingling in hands or arms
d. Changes in thinking or remembering
e. Numbness or burning in hands or fingers
f. Increased frequency or intensity of headaches

M AT C H I N G
a. Region of the heart supplied by a blocked coronary artery
b. Patients with obvious STE in these leads should be evaluated for a possible RVI
c. Type of angina related to disease of the small, distal branches of the coronary arteries
d. The period from STEMI symptom onset to successful reperfusion
e. Updated phrase for stable coronary artery disease
f. Used to ascertain the patient’s risk of an adverse outcome, determine the need for hospital admis-
sion, and to decide on potential treatment options
g. PR interval more than 0.24 second, second- or third-degree heart block, reactive airway disease
h. Components of the treatment plan for non–ST-elevation acute coronary syndromes
i. Evaluate patients with apparent ST-segment depression in these leads for possible inferobasal MI
j. Aortic dissection, acute pericarditis, panic attack, pulmonary embolism
k. Type of angina that is the result of intense spasm of a segment of a coronary artery
l. Essential diagnostic tool for patients presenting with ischemic chest discomfort or anginal equiva-
lent symptoms
m. Phosphodiesterase inhibitor use within the previous 24 to 48 hours
n. Symptom other than chest pain or discomfort resulting from myocardial ischemia that may occur
either alone or in combination in a patient with ischemic heart disease
____ 11. Contraindication for nitrate administration
____ 12. Examples of conditions that can mimic acute MI
____ 13. V1 and V2
____ 14. 12-lead ECG
____ 15. Antianginal, antiplatelet, and anticoagulant therapy
____ 16. Area at risk
____ 17. Variant
____ 18. Total ischemic time
____ 19. Chronic coronary syndromes
____ 20. Anginal equivalent
____ 21. II, III, and/or aVF
____ 22. Risk assessment tools
____ 23. Microvascular
____ 24. Contraindications for beta blocker administration
CHAPTER 8  Acute Coronary Syndromes 211

CASE STUDY 8.1


Identify one or more choices that best complete the statement or answer the question.

Your patient is a 68-year-old man who is complaining of chest discomfort. The patient is hospital-
ized at a PCI- capable facility. You have a sufficient number of advanced life support personnel avail-
able to assist you and carry out your instructions. Emergency equipment, including a biphasic manual
defibrillator, is available. As you work through this case study, remember that there may be alternative
actions that are perfectly acceptable, yet not discussed here.

____ 1. As you approach the patient, you observe the patient sitting upright on a stretcher
with beads of sweat visible on his forehead. He is awake and watches as you approach.
The patient appears anxious, and his skin is pale. His breathing does not appear to be
labored. Select the correct statements about this patient’s general impression findings.
a. Appearance is normal.
b. Appearance is abnormal.
c. Breathing is normal.
d. Breathing is abnormal.
e. Circulation is normal.
f. Circulation is abnormal.

____ 2. In addition to performing a primary and secondary assessment, which team member
actions should occur at the same time?
a. Obtain the patient’s vital signs.
b. Complete a fibrinolytic checklist.
c. Attach a pulse oximeter and a cardiac monitor.
d. Establish IV access and obtain a 12-lead ECG.
e. Obtain initial laboratory studies and a portable chest radiograph.

____ 3. The patient’s blood pressure is 168/94 millimeters of mercury (mm Hg) and his
ventilatory rate is 22 breaths/min. Breath sounds are clear and equal and his skin is
cool, pale, and moist. The patient’s oxygen saturation (SpO2) on room air is 95%. He
has been placed on the cardiac monitor, which reveals a sinus tachycardia at
110 beats/min. The following information has been obtained from the patient:
Signs/Symptoms: Discomfort is located in the center of his chest and radiates to his
left arm; rates his chest discomfort at 9/10
Allergies: None
Medications: Aspirin 81 mg daily
Past history: Heart attack at age 45 years, RCA stent inserted
Last oral intake: Lunch 2 hours ago
Events prior: Reading the newspaper when discomfort began about 30 min ago

The physical examination reveals no abnormalities. Should aspirin be admin-


istered to this patient?
a. No. The patient is already taking 81 mg of aspirin daily.
b. Yes. If there are no contraindications, aspirin should be given as early as pos-
sible after presentation to patients with an ACS.

____ 4. IV access has been established and a 12-lead ECG has been obtained. Which
components of the ECG should be carefully examined to determine the most
appropriate treatment course for this patient?
a. P waves
b. Q waves
c. T waves
d. ST segments
212 CHAPTER 8  Acute Coronary Syndromes

____ 5. The patient’s 12-lead ECG is shown here (Fig. 8.28).


Which of the following significant findings are present on this ECG?
a. STE is present in leads II, III, and aVF.
b. STE is present in leads V1, V2, V3, and V4.
c. ST depression is seen in leads V3, V4, V5, and V6.
d. ST depression is seen in leads III, aVF, V1, and V2.

_ ___ 6. These ECG findings suggest that this patient is experiencing:


a. A lateral infarction.
b. An inferior infarction.
c. An inferobasal infarction.
d. An anteroseptal infarction.

____ 7. Which of the following complications should be reasonably anticipated with this
type of infarction?
a. BBB
b. AFib
c. Cardiogenic shock
d. Left-sided heart failure

____ 8. Sublingual NTG is ordered for this patient. What is the rationale for giving NTG
in this situation?
a. NTG slows conduction through the AV node.
b. NTG causes venous dilation, decreasing venous return.
c. NTG decreases heart rate and the force of myocardial contraction.
d. NTG dilates normal and atherosclerotic epicardial coronary arteries.

_ ___ 9. Which of the following are true regarding NTG administration?


a. Avoid NTG in inferior wall MI with a possible associated RVI.
b. Reassess the patient’s vital signs and level of discomfort after each dose.
c. Do not administer NTG if the patient’s systolic BP is less than 110 mm Hg.
d. Ensure that the patient has not used a phosphodiesterase inhibitor in the last
24 to 48 hours.

I aVR V1 V4

II aVL V2 V5

III aVF V3 V6

x1.0 0.05-150Hz 25mm/sec


I Lateral aVR --------- V1 Septum V4 Anterior
II Inferior aVL Lateral V2 Septum V5 Lateral
III Inferior aVF Inferior V3 Anterior V6 Lateral

Fig. 8.28  (From Phalen, T., Aehlert, B. [2012]. The 12-lead ECG in acute coronary syndromes, ed 3, St. Louis, Elsevier.)
CHAPTER 8  Acute Coronary Syndromes 213

____ 10. After three doses of sublingual NTG, the patient rates his discomfort as 7/10. His
vital signs are essentially unchanged. What should be done now?
a. Give morphine for pain relief and reassess.
b. Give up to three additional doses of sublingual NTG as needed.
c. Give a beta blocker and reassess the patient’s level of discomfort.
d. Give another sublingual NTG in addition to a long-acting calcium blocker.

____ 11. Cardiac biomarkers and the 12-lead ECG confirm STEMI. It has been determined
that the patient is a reasonable candidate for reperfusion therapy. Which of the
following reflects your best course of action at this time?
a. Admit the patient to a monitored bed for further evaluation.
b. Admit the patient to the emergency department chest pain unit for ongoing
monitoring.
c. Activate the cardiac catheterization laboratory team and arrange the patient’s
transfer for primary PCI.
d. Admit the patient to an intermediate care unit, perform continuous ST segment
monitoring for 3 hours, and then reassess treatment options.

STOP AND REVIEW  ANSWERS


1. A. A thrombus is the most common cause of the blockage of a coronary artery. Less common
causes of an ACS include coronary artery spasm (e.g., with cocaine abuse), severe luminal narrow-
ing from atherosclerosis or restenosis after PCI, coronary dissection, hypercoagulation, trauma to
the coronary arteries, or coronary artery emboli (rare).

2. B, C. The effects of myocardial ischemia can be viewed on the ECG as ST depression and T wave
changes in the leads that face the affected area of the ventricle.

3. C. Two leads are contiguous if they look at the same or adjacent areas of the heart or if they are
numerically consecutive chest leads. Examples of contiguous leads include V1 and V2, V2 and V3,
V3 and V4, V4 and V5, V5 and V6, I and aVL, II and aVF, and II and III, among others.

4. D. RVI should be suspected when ECG changes suggesting an inferior infarction (ST elevation
in leads II, III, and/or aVF) are observed. The most sensitive ECG signs of right ventricular injury
include 1-mm ST elevation in lead V1 and lead V4R. Patients with RVI may present with, or
subsequently develop, hypotension caused by bradydysrhythmias or caused by a reduction in pre-
load after the administration of vasodilators, such as nitroglycerin.

5. C. Cardiac biomarkers include myoglobin, cardiac troponins (cTn), creatine kinase (CK) and its
myocardial band (MB) isoform, and lactate dehydrogenase, among others. Cardiac troponins (i.e.,
TnI and TnT) are the biomarkers of choice for diagnosing MI.

6. D. Transmural is a term that is used to describe ischemia, injury, or infarction that extends from
the endocardium through the myocardium to the epicardium. For example, an infarction that in-
volves the entire thickness of the left ventricular wall is called a transmural MI.

7. B. Patients who are experiencing an ACS who are most likely to present atypically include older adults,
diabetic individuals, women, patients with impaired renal function, patients with dementia, patients with
prior cardiac surgery, and patients during the immediate postoperative period after noncardiac surgery.

8. A. Obtaining and reviewing a 12-lead ECG is part of the initial assessment of the patient present-
ing with ischemic chest discomfort and important in determining an appropriate treatment plan.
Obtain the first 12-lead ECG with 10 minutes of patient contact.
214 CHAPTER 8  Acute Coronary Syndromes

9. B. The inhibition of beta1-adrenergic receptor sites decreases heart rate and the force of myocardial
contraction, thereby reducing myocardial oxygen demand. Angiotensin-converting enzyme (ACE)
inhibitors block the conversion of angiotensin I into angiotensin II. In the absence of contraindications
to their use, oral beta blockers should be started in the first 24 hours in patients experiencing an ACS.

10. A, B, C, D, E, F. Studies reveal that women experiencing an ACS may report symptoms that in-
clude unusual fatigue, anxiety, frequent indigestion, heart racing, new vision problems, changes in
thinking or remembering, loss of appetite, tingling in hands or arms, numbness or burning
in hands or fingers, cough, and increased frequency or intensity of headaches.

M AT C H I N G
11. m 18. d

12. j 19. e

13. i 20. n

14. l 21. b

15. h 22. f

16. a 23. c

17. k 24. g

C A S E S T U DY 8.1 ANSWERS
1. A, C, F. The general impression findings are abnormal (Appearance: normal; Breathing: normal;
Circulation: abnormal skin color).

2. A, B, C, D, E. Ask your team members to attach a pulse oximeter and cardiac monitor, and obtain
the patient’s baseline vital signs and oxygen saturation. Obtain a 12-lead ECG and instruct a team
member to establish IV access. Complete a fibrinolytic checklist, obtain initial laboratory studies
within 10 minutes (e.g., initial cardiac biomarkers, complete blood count, electrolytes, coagulation
studies), and obtain a portable chest radiograph within 30 minutes.

3. B. Yes. Nonenteric-coated chewable aspirin should be given as early as possible after presentation
to patients with an ACS, assuming there are no contraindications to its use. Although the patient
currently takes 81 mg of aspirin daily, the recommended initial dose is 162 to 325 mg.

4. B, C, D. Review the 12-lead ECG carefully after it is obtained. Look at each lead for the presence
of ST-segment displacement (i.e., elevation or depression). If ST-segment displacement is present,
note its displacement in mm. Inspect the T waves for any changes in orientation, shape, and size.
Examine each lead for the presence of a Q wave. If a Q wave is present, measure its duration.
Assess for areas of ischemia or injury by assessing lead groupings. Remember: ECG evidence must
be found in at least two contiguous leads.

5. B. STE is seen in leads V1, V2, V3, and V4. ST depression is seen in leads II, III, aVF, V5, and V6.

6. D. The STE seen in leads V1, V2, V3, and V4. suggests an anteroseptal MI.
CHAPTER 8  Acute Coronary Syndromes 215

7. A, B, C, D. Because the LAD supplies a large portion of the left ventricle, a blockage in this area
can lead to complications, such as left ventricular dysfunction, including left-sided heart failure
and cardiogenic shock. An anterior infarction may cause dysrhythmias including PVCs, atrial flut-
ter, or AFib. A blockage in the area of the septum, which contains the bundle branches, may result
in right BBB, left BBB (this is more common), second-degree AV block type II, and third-degree
AV block.

8. B, D. NTG produces venous vasodilation. Dilation of the capacitance vessels (i.e., veins) decreases
venous return to the heart, reducing ventricular volume, and decreasing ventricular wall tension.
Decreasing cardiac preload decreases myocardial oxygen demand. NTG also dilates normal and
atherosclerotic epicardial coronary arteries and increases coronary collateral flow.

9. A, B, D. Before giving NTG, assess the degree of the patient’s ischemic discomfort with the
use of a 0-to-10 scale. Also record the pain’s duration, time of onset, the activity that was being
performed, and the pain quality. Reassess and document the patient’s vital signs and level of dis-
comfort after each dose. If ischemic discomfort persists after three sublingual NTG doses, consider
the use of IV NTG. Make sure that the patient has not used a phosphodiesterase inhibitor, such
as sildenafil (Viagra) within 24 hours or tadalafil (Cialis) within 48 hours before NTG administra-
tion. Avoid NTG in inferior wall MI with a possible associated RVI. Nitrates should not be
administered to patients with a systolic BP less than 90 mm Hg or 30 mm Hg or more below
baseline, marked bradycardia, or marked tachycardia.

10. A. Morphine, which is generally given in 2-mg increments, should be administered for pain relief.
Give additional doses at 5- to 15-minute intervals. Reassess and document the patient’s vital signs
and level of discomfort after each morphine dose. Some clinicians prefer to use fentanyl for pain
relief instead of morphine in patients experiencing an ACS.

11. C. Reperfusion therapy is recommended for all eligible patients with STEMI who present within
12 hours of symptom onset. Primary PCI is the recommended method of reperfusion when it can
be performed in a timely fashion by experienced professionals. When the patient has a STEMI, is
a candidate for reperfusion, and is initially seen at a PCI-capable hospital, he or she should be sent
to the cardiac catheterization laboratory for primary PCI, which should be accomplished within
90 minutes of first medical contact.

REFERENCES
Amsterdam, E. A., Wenger, N. K., Brindis, R. G., et al. (2014). 2014 AHA/ACC guideline for the management
of patients with non-ST-elevation acute coronary syndromes. J Am Coll Cardiol, 64(24), 1–150.
Anderson, J. L., Adams, C. D., Antman, E. M., et al. (2007). ACC/AHA 2007 guidelines for the management of
patients with unstable angina/non–ST-elevation myocardial infarction. J Am Coll Cardiol, 50(7), e1–e157.
Benjamin, E. J., Muntner, P., Alonso, A., et al. (2019). Heart disease and stroke statistics-2019 update: A report
from the American Heart Association. Circulation, 139(10), e56–e528.
Bob-Manuel, T., Ifedili, I., Reed, G., Ibebuogu, U. N., & Khouzam, R. N. (2017). Non-ST elevation acute coro-
nary syndromes: A comprehensive review. Curr Probl Cardiol, 42(9), 266–305.
Brashers, V. L. (2020). Alterations of cardiovascular function. In S. E. Huether, K. L. McCance, & V. L. Brashers
(Eds.), Understanding pathophysiology (7th ed., pp. 591–638). St. Louis, MO: Elsevier.
Chen, C., Wei, J., AlBadri, A., Zarrini, P., & Bairey Merz, C. N. (2016). Coronary microvascular dysfunction -
epidemiology, pathogenesis, prognosis, diagnosis, risk factors and therapy. Circ J, 81(1), 3–11.
Chung, M. J., & Brown, D. L. (2019). Diagnosis of acute myocardial infarction. In D. L. Brown (Ed.), Cardiac
intensive care (3rd ed., pp. 91–98.e3). Philadelphia, PA: Elsevier.
Crea, F., & Liuzzo, G. (2013). Pathogenesis of acute coronary syndromes. J Am Coll Cardiol, 61(1), 1–11.
Eitel, I., Wang, J., Stiermaier, T., et al. (2020). Impact of morphine treatment on infarct size and reperfusion injury
in acute reperfused ST-elevation myocardial infarction. J Clin Med, 9(3), 1–12.
Farquhar, H., Weatherall, M., Wijesinghe, M., Perrin, K., Ranchord, A., Simmonds, M., & Beasley, R. (2009).
Systematic review of studies of the effect of hyperoxia on coronary blood flow. Am Heart J, 158(3), 371–377.
216 CHAPTER 8  Acute Coronary Syndromes

Frampton, J., Devries, J. T., Welch, T. D., & Gersh, B. J. (2020). Modern management of ST-segment elevation
myocardial infarction. Curr Probl Cardiol, 45(3), 100393.
Garg, V. P., & Halperin, J. L. (2013). Novel antiplatelet and anticoagulant agents in the cardiac care unit. Cardiol
Clin, 31(4), 533–544.
Giugliano, R. P., & Braunwald, E. (2019). Non-ST-elevation acute coronary syndromes. In D. P. Zipes, P. Libby,
R. O. Bonow, D. L. Mann, G. F. Tomaselli, & E. Braunwald (Eds.), Braunwald’s heart disease: A textbook of
cardiovascular medicine (11th ed., pp. 1181–1208). Philadelphia, PA: Elsevier.
Goldberger, A. L., Goldberger, Z. D., & Shvilkin, A. (2018). Myocardial ischemia and infarction, Part I: ST
segment elevation and Q wave syndromes. In Goldberger’s clinical electrocardiography (9th ed., pp. 73–91).
Philadelphia, PA: Elsevier.
Gorgels, A. P., Engelen, D. J., & Wellens, H. J. (2001). Lead aVR, a mostly ignored but very valuable lead in
clinical electrocardiography. J Am Coll Cardiol, 38(5), 1355–1356.
Halim, S. A., Newby, K., & Ohman, E. M. (2010). Diagnosis of acute myocardial ischemia and infarction. In
M. H. Crawford, J. P. DiMarco, & W. J. Paulus (Eds.), Cardiology (3rd ed., pp. 345–360). Philadelphia, PA:
Elsevier.
Heron, M. (2019). Deaths: Leading causes for 2017. Natl Vital Stat Rep, 68(6), 1–77.
January, C. T., Wann, L. S., Alpert, J. S., et al. (2014). 2014 AHA/ACC/HRS guideline for the management of
patients with atrial fibrillation: A report of the American College of Cardiology/American Heart Association
Task Force on Practice Guidelines and the Heart Rhythm Society. J Am Coll Cardiol, 64(21), e1–e76.
Karve, A. M., Bossone, E., & Mehta, R. H. (2007). Acute ST-segment elevation myocardial infarction: Critical
care perspective. Crit Care Clin, 23(4), 685–707.
Kawji, M. M., & Glancy, D. L. (2017). The value of reciprocal electrocardiographic leads. Am J Cardiol, 119(9),
1490–1491.
Kelley, W. E., Januzzi, J. L., & Christenson, R. H. (2009). Increases of cardiac troponin in conditions other than
acute coronary syndrome and heart failure. Clin Chem, 55(12), 2098–2112.
Knuuti, J., Wijns, W., Saraste, A., et al. (2020). 2019 ESC guidelines for the diagnosis and management of chronic
coronary syndromes. Eur Heart J, 41(3), 407–477.
Kumar, V., Abbas, A. K., & Aster, J. C. (2018). Heart. In Robbins basic pathology (10th ed., pp. 399–440).
Philadelphia, PA: Elsevier.
Levine, G. N., Bates, E. R., Blankenship, J. C., et al. (2016). 2015 ACC/AHA/SCAI focused update on primary
percutaneous coronary intervention for patients with ST-elevation myocardial infarction. J Am Coll Cardiol,
67(10), 1235–1250.
Matos, J., Tung, R., & Zimetbaum, P. (2019). Use of the electrocardiogram in acute myocardial infarction. In D.
L. Brown (Ed.), Cardiac intensive care (3rd ed., pp. 99–102). St. Louis, MO: Elsevier.
McSweeney, J. C., Rosenfeld, A. G., Abel, W. M., et al. (2016). Preventing and experiencing ischemic heart disease
as a woman: State of the science: A scientific statement from the American Heart Association. Circulation,
133(13), 1302–1331.
Morrow, D. A., & De Lemos, J. A. (2019). Stable ischemic heart disease. In D. P. Zipes, P. Libby, R. O. Bonow,
D. L. Mann, G. F. Tomaselli, & E. Braunwald (Eds.), Braunwald’s heart disease: A textbook of cardiovascular
medicine (11th ed., pp. 1209–1270). Philadelphia, PA: Elsevier.
Namana, V., Gupta, S. S., Abbasi, A. A., Raheja, H., Shani, J., & Hollander, G. (2018). Right ventricular infarction.
Cardiovasc Revasc Med, 19(1 Pt A), 43–50.
O’Connor, R. E., Alali, A. S., Brady, W. J., Ghaemmaghami, C. A., Menon, V., Welsford, M., & Shuster, M. (2015).
American Heart Association Guidelines for CPR & ECC. Retrieved from American Heart Association. In Web-based
integrated guidelines for cardiopulmonary resuscitation and emergency cardiovascular care – Part 9: Acute coronary
syndromes: Eccguidelines.heart.org
O’Gara, P. T., Kushner, F. G., Ascheim, D. D., et al. (2013). 2013 ACCF/AHA guideline for the management of
ST-elevation myocardial infarction. J Am Coll Cardiol, 61(4), e78–e140.
Okumus, N., Soma, D., & Kondapaneni, M. (2019). Risk factors for Prinzmetal angina and coronary atherosclerosis:
How different are they? JACC, 73(9), 162–162.
Parodi, G., Bellandi, B., Xanthopoulou, I., et al. (2015). Morphine is associated with a delayed activity of oral
antiplatelet agents in patients with ST-elevation acute myocardial infarction undergoing primary percutaneous
coronary intervention. Circ Cardiovasc Interv, 8(1), e001593.
Picard, F., Sayah, N., Spagnoli, V., Adjedj, J., & Varenne, O. (2019). Vasospastic angina: A literature review of
current evidence. Arch Cardiovasc Dis, 112(1), 44–55.
Power, B., Seppelt, I. (2019). Acute cardiac syndromes, investigations and interventions. In A. D. Bersten, & J. M.
Handy (Eds.), Oh’s intensive care manual (8th ed., pp. 181–203). China: Elsevier.
Schindler, T. H., & Dilsizian, V. (2020). Coronary microvascular dysfunction: Clinical considerations and nonin-
vasive diagnosis. JACC Cardiovasc Imaging, 13(1), 140–155.
Scirica, B. M., Libby, P., & Morrow, D. A. (2019). ST-elevation myocardial infarction: Pathophysiology, and clinical
evolution. In D. P. Zipes, P. Libby, R. O. Bonow, D. L. Mann, G. F. Tomaselli, & E. Braunwald (Eds.), Braunwald’s
heart disease: A textbook of cardiovascular medicine (11th ed., pp. 1095–1122). Philadelphia, PA: Elsevier.
CHAPTER 8  Acute Coronary Syndromes 217

Sovari, A. A., Assadi, R., Lakshminarayanan, B., & Kocheril, A. G. (2007). Hyperacute T wave, the early sign of
myocardial infarction. Am J Emerg Med, 25(7), 859.e1–859.e7.
Stub, D., Smith, K., Bernard, S., et al. (2015). Air versus oxygen in ST-segment elevation myocardial infarction.
Circulation, 131(24), 2143–2150.
Surawicz, B., & Knilans, T. K. (2008). Acute ischemia. In Chou’s electrocardiography in clinical practice (6th ed.,
pp. 124–161). Philadelphia, PA: Saunders.
Taqueti, V. R. (2019). Coronary microvascular dysfunction in vasospastic angina: Provocative role for the micro-
circulation in macrovessel disease prognosis. J Am Coll Cardiol, 74(19), 2361–2364.
Thomas, J. J., & Brady, W. J. (2018). Acute coronary syndrome. In R. M. Walls, R. S. Hockberger, & M. Gausche-Hill
(Eds.), Rosen’s emergency medicine: Concepts and clinical practice (9th ed., pp. 891–928.e4). Philadelphia, PA: Elsevier.
Thygesen, K., Alpert, J. S., Jaffe, A. S., et al. (2018). Fourth universal definition of myocardial infarction. J Am Coll
Cardiol, 72(18), 2231–2264.
Tobin, K., & Eagle, K. (2020). Angina pectoris. In R. D. Kellerman, & D. P. Rakel (Eds.), Conn’s current therapy
2020 (pp. 94–101). Philadelphia, PA: Elsevier.
Vasaiwala, S. C., & Schreiber, R. (2008). Posterior myocardial infarction: Unique diagnosis to an elusive problem.
Am J Emerg Med, 26(4), 520.e5-e6.
Vorobiof, G., & Ellestad, M. H. (2011). Lead aVR: dead or simply forgotten? JACC Cardiovasc Imaging, 4(2),
187–190.
Wagner, G. S., Macfarlane, P., Wellens, H., et al. (2009). AHA/ACCF/HRS recommendations for the standard-
ization and interpretation of the electrocardiogram: Part VI: Acute ischemia/infarction; a scientific statement
from the American Heart Association Electrocardiography and Arrhythmias Committee. J Am Coll Cardiol,
53(11), 1003–1011.
Weitz, J. I. (2013). Antithrombotic drugs. In R. Hoffman, E. J. Benz Jr., L. E. Silberstein, H. E. Heslop, J. I. Weitz,
& J. Anastasi (Eds.), Hematology: Basic principles and practice (6th ed., pp. 2102–2119). Philadelphia, PA: Elsevier.
Werns, S. (2019). Acute coronary syndromes and acute myocardial infarction. In J. E. Parrillo, & R. P. Dellinger
(Eds.), Critical care medicine: principles of diagnosis and management in the adult (5th ed., pp. 424–469). Phila-
delphia, PA: Elsevier.
Yamaji, H., Iwasaki, K., Kusachi, S., et al. (2001). Prediction of acute left main coronary artery obstruction by
12 lead electrocardiography: ST segment elevation in lead aVR with less ST segment elevation in V1. J Am
Coll Cardiol, 38(5), 1348–1354.
This page intentionally left blank
CHAPTER 9
Acute Ischemic Stroke

INTRODUCTION
Stroke is the fifth leading cause of death and a leading cause of disability in the United States
(Benjamin et al., 2019). On average, someone in the United States experiences a stroke every 40 seconds,
and someone dies of a stroke every 4 minutes (Benjamin et al., 2019). Risk factors for stroke are simi-
lar to those for cardiovascular disease. They include hypertension, diabetes mellitus, atrial fibrillation
(AFib), elevated cholesterol and lipid levels, smoking/tobacco use, physical inactivity, kidney disease,
and family history/genetic predisposition. Types of strokes, stroke systems of care, and the initial emer-
gency care for acute ischemic stroke (AIS) are discussed in this chapter.

L E A R N I N G OBJECTIVES
After completing this chapter, you should be able to:
1. Describe the major types of stroke.
2. Discuss the 8 Ds of stroke care.
3. Compare elements of acute stroke care facilities in the United States.
4. Describe the initial emergency care for AIS.

L E A R N I N G PLAN
• Read this chapter before class.
• Master identification of the following rhythms: sinus rhythm and atrial fibrillation.
• Master the following medications: oxygen (O2), dextrose, intravenous (IV) alteplase.
• Master the following skills:
n Ensure scene safety and the use of personal protective equipment.
n Assign team member roles or perform as a team member in a simulated patient
situation.
n Direct or perform a basic life support (BLS) assessment, primary assessment, and
secondary assessment.
n Obtain vital signs; establish vascular access; attach a pulse oximeter and blood
pressure (BP) and cardiac monitor; give supplemental O2 if indicated; and order a
12-lead electrocardiogram (ECG).
n Recognize the signs and symptoms of AIS.
n Develop and implement a treatment plan based on the patient’s presentation,
history, physical examination, and diagnostic test results.
n Know the actions, indications, dosages, adverse effects, and contraindications for
the medications used in the treatment of AIS.
219
220 CHAPTER 9  Acute Ischemic Stroke

Review your performance as a team leader or team member during a postevent


n

debriefing.
• Develop and use flashcards, flowcharts, and mnemonics to help enhance your
retention of the information presented.
• Complete the chapter quiz and review the quiz answers provided.
• Read the case study at the end of this chapter and answer the questions within the
case study. Compare your answers with the answers provided.

KEY TERMS
Stroke system of care  A comprehensive spectrum of care that addresses all aspects of
stroke care in a coordinated fashion.
Transient ischemic attack (TIA)  A transient episode of neurologic dysfunction caused by
focal brain, spinal cord, or retinal ischemia, without acute infarction.

ACLS Pearl
The term cerebrovascular accident, which was used for many years as a synonym for the word
stroke, has lost favor because strokes are not accidents (Zivin, 2012).

ANATOMY REVIEW
The brain is supplied with blood by the internal carotid and vertebral arteries. The internal carotid
arteries provide most of the brain’s blood supply. These arteries branch into the anterior and middle
cerebral arteries. Strokes involving the carotid arteries are called anterior circulation strokes or
carotid territory strokes. They usually involve the cerebral hemispheres. After passing through the
foramen magnum, the vertebral arteries join to form the basilar artery, which divides into right and
left posterior cerebral arteries. Strokes affecting the vertebral arteries are called posterior circulation
strokes or vertebrobasilar territory strokes. They usually affect the brainstem or cerebellum. The
anterior and posterior circulations form a circular connection of arteries called the circle of Willis
(COW ), which is located at the base of the brain (Fig. 9.1). The anterior cerebral arteries, the ante-
rior communicating artery, the posterior cerebral arteries, and the posterior communicating arteries
also contribute to the COW blood supply. Possible neurologic deficits related to cerebral perfusion
are shown in Table 9.1.

ACLS Pearl
The clinical presentation of stroke varies and depends on the vessel involved, the location and
extent of the damage, and collateral blood flow. Because anatomic variations are frequent, par-
ticularly in the vertebral artery system, the area supplied with blood by a given artery is not entirely
predictable; as a result, stroke syndromes do not always correlate well with the location of the
vascular injury (Zivin, 2012).

STROKE TYPES
For many years, the primary types of stroke were categorized as either ischemic or hemorrhagic.
Experts now recommend that the term hemorrhagic stroke be discontinued because the term is
confusing; it can refer to primary subarachnoid hemorrhage (SAH), primary intracerebral hemorrhage
(ICH), or hemorrhage after infarction that occurs spontaneously or because of antithrombotic or
fibrinolytic therapy (Sacco et al., 2013).
CHAPTER 9  Acute Ischemic Stroke 221

Anterior communicating artery


Anterior cerebral artery

Middle cerebral artery Opthalmic artery

Anterior choroidal artery


Internal corotid artery

Posterior
communicating artery Posterior cerebral artery

Superior cerebellar artery


Pontine arteries
Basilar artery

Anterior inferior
cerebellar artery

Vertebral artery

Posterior inferior
cerebellar artery

Anterior spinal artery


Fig. 9.1  ​Circle of Willis. (From Layne, L.J., Petrie, L. [2019]. Vascular, cardiac, and interventional radiography. In B.W. Long,
J.H. Rollins, B.J. Smith, editors. Merrill’s atlas of radiographic positioning and procedures, (vol 3, ed 14, St. Louis, Elsevier.)

TABLE 9.1 Cerebral Vessels and Neurologic Deficits


Affected Artery Possible Neurologic Deficit
Anterior cerebral Behavioral changes, emotional lability, impaired decision-making ability (especially
if bilateral infarction); contralateral hemiparesis, contralateral sensory loss, loss
of coordination, urinary incontinence
Basilar Ataxia, coma, confusion, cranial nerve deficits, dysarthria, dysconjugate gaze,
dysphagia, locked-in syndrome, nausea, quadriplegia or hemiplegia/paresis,
vertigo, vomiting
Internal carotid Aphasia (dominant hemisphere); contralateral loss of motor and sensory function
affecting the face, arm, and leg; ipsilateral visual loss
Middle cerebral Aphasia (dominant hemisphere); contralateral hemiparesis affecting face and arm
more than the leg; contralateral sensory deficit affecting face and arm more
than the leg; eye (gaze) deviation toward the affected side
Posterior cerebral Contralateral hemiplegia, contralateral sensory deficits, contralateral vision
changes, inability to read, inability to recognize familiar faces, memory deficits
Vertebral Ataxia, contralateral hemiparesis, contralateral hemisensory loss, dizziness,
dysarthria, dysphagia, facial weakness, nausea, vertigo, vomiting

Subarachnoid Hemorrhage
SAH is bleeding into the subarachnoid space, which may be the result of trauma or nontraumatic
causes, such as a ruptured cerebral aneurysm or an arteriovenous malformation (Fig. 9.2). About 3% of
all strokes are the result of SAH (Benjamin et al., 2019).
Patients often report a sudden onset of a severe “thunderclap” headache or describe the feeling
as “the worst headache of my life.” Associated signs and symptoms vary and may include nausea,
222 CHAPTER 9  Acute Ischemic Stroke

Subarachnoid Intracerebral
hemorrhage hemorrhage

Ruptured cerebral Ruptured Cerebral thrombosis Cerebral embolism


A aneurysm blood vessel B
Fig. 9.2  ​(A) Subarachnoid hemorrhage, intracerebral hemorrhage. (B) Ischemic stroke. (From Brooks, M.L., Brooks, D.L.
[2014]. Exploring medical language, a student-directed approach, ed 9, St. Louis, Mosby.)

vomiting, focal neurologic deficits, neck stiffness, dizziness, visual disturbances (e.g., blurry or double
vision), loss of consciousness, and seizures. Warning signs and symptoms can occur minutes to weeks
before a rupture because of blood leakage or because of nerve compression as the aneurysm expands.
Initial misdiagnosis of SAH occurs in about 15% of patients, especially patients with the mildest
symptoms; about 40% of misdiagnosed patients experience subsequent neurologic deterioration
(Mayer, 2020).
The patient with suspected SAH should be sent immediately for neuroimaging. Because the sensi-
tivity of computed tomography (CT) declines to about 75% by 48 hours after symptom onset, a lum-
bar puncture should be performed if a CT is negative, but the index of suspicion remains high (Mayer,
2020). Frequent vital sign checks, including oxygen saturation readings, ECG monitoring, and neuro-
logic assessments, are essential. ECG changes that may be observed in the acute phase of SAH include
peaked or deeply inverted T waves and increased U wave amplitude. The patient should be admitted
to a neurologic intensive care unit (NICU) for continuous monitoring for bleeding, hydrocephalus,
vasospasm, and other potential complications.

Intracerebral Hemorrhage
About 10% of all strokes are the result of an ICH (Benjamin et al., 2019). Chronic hypertension and
aging are among the risk factors associated with ICH. ICH is most often caused by the spontaneous
rupture of small arteries within the substance of the brain. Less common causes of ICH include aneu-
rysm, arteriovenous malformation, hemorrhagic transformation of ischemic stroke, and neoplasms
(Brouwers & Goldstein, 2012). Signs and symptoms may consist of a severe headache, vomiting, neck
stiffness, seizures, and coma or decreased level of consciousness. Neuroimaging is essential to establish
a definitive diagnosis and distinguish ICH from AIS.

ACLS Pearl
It is estimated that more than 20% of patients will experience a decrease in the Glasgow Coma
Scale (GCS) score of 2 points or more between the prehospital assessment and the patient’s
initial evaluation in the emergency department (ED). Another 15% to 23% of patients will demon-
strate continued deterioration within the first hours after hospital arrival (Hemphill et al., 2015).
CHAPTER 9  Acute Ischemic Stroke 223

Admission to a stroke unit or NICU is recommended with frequent vital sign checks, neurologic
assessments, and continuous ECG, BP, and oxygen saturation monitoring (Hemphill et al., 2015).
Neurologic evaluations should be performed using a validated stroke scale. Although the National
Institutes of Health Stroke Scale (NIHSS) score may be useful, ICH patients often have a decreased
level of consciousness on initial presentation, which may diminish the efficacy of the NIHSS
(Hemphill et al., 2015). Of the several stroke scales that exist specifically for ICH, the most widely
used and validated is the ICH Score (Hemphill et al., 2015). The ICH Score is a clinical grading scale
composed of factors related to the patient’s GCS score, age, and initial neuroimaging (ICH volume,
presence or absence of intraventricular hemorrhage [IVH], and infratentorial/supratentorial origin).
IVH occurs in about 45% of patients with spontaneous ICH and is an independent factor associated
with poor outcome (Hemphill et al., 2015).
Active bleeding may continue for hours after symptom onset. Experts note that among patients
undergoing head CT within 3 hours of ICH onset, 28% to 38% have hematoma expansion of greater
than one-third of the initial hematoma volume on follow-up CT (Hemphill et al., 2015). During the
acute phase of care, strategies to minimize ongoing bleeding may include reversal of anticoagulation,
and modest BP reduction to improve functional outcomes (Hemphill et al., 2015). Experts note that
although patients with cerebellar hemorrhage who are deteriorating neurologically or who have brain-
stem compression and/or hydrocephalus from ventricular obstruction should undergo surgical removal
of the hemorrhage as soon as possible, the usefulness of surgery is not well established for most patients
with supratentorial ICH (Hemphill et al., 2015).

Ischemic Stroke
Ischemic cerebrovascular disease is caused by a reduction of blood supply to the brain. The decrease in
blood supply may be related to blockage of a single artery (i.e., focal), several arteries (i.e., multifocal),
or diffuse (Goldstein, 2020). An ischemic stroke, also described as an occlusive stroke, is brain, spinal
cord, or retinal cell death attributable to ischemia with neuropathologic, neuroimaging, and/or clinical
evidence of permanent injury. About 87% of all strokes are ischemic (Benjamin et al., 2019). An AIS
may be symptomatic or silent (i.e., asymptomatic) (Easton et al., 2009). Ischemic stroke is a heritable
disease; stroke heritability appears to play a more significant role in strokes that occur in younger
people (Benjamin et al., 2019). A family history of stroke is associated with an increased risk of isch-
emic stroke and carotid atherosclerosis (Benjamin et al., 2019).
A thrombotic stroke is the most common cause of ischemic stroke. With a thrombotic stroke, a
thrombus (i.e., blood clot) develops in arteries that perfuse the brain (see Fig. 9.2). When the blood
clots are of sufficient size to block blood flow through the artery, the area that was previously supplied
by that artery becomes ischemic. The brain poorly tolerates ischemia because the brain is unable to
store the glucose it needs to function.
With an embolic stroke, material from an area outside of the brain (e.g., heart, aorta, other major
artery) becomes dislodged and travels through the bloodstream to the brain (i.e., cerebral embolism).
Embolic material may consist of fragments of tumors or plaques, air, fat, amniotic fluid, a foreign body,
or a blood clot. An embolus tends to become lodged where arteries branch because blood flow is most
turbulent in these areas. Fragments of the embolus may become lodged in smaller vessels.
Lacunar strokes, also called lacunar infarcts, are small infarctions caused by the blockage of a pen-
etrating branch of a large cerebral artery. Lacunar strokes are usually associated with chronic hyperten-
sion, diabetes, and hyperlipidemia, and they most often occur in the basal ganglia, thalamus, white
matter of the internal capsule, and pons.
A large vessel occlusion (LVO) is an occlusion of the anterior cerebral artery, basilar artery, internal
carotid artery, internal carotid artery terminus, middle cerebral artery, and/or the vertebral artery. LVOs
account for about 24% to 46% of AISs (Rennert et al., 2019). Before the advent of endovascular
therapies, LVOs more than doubled the risk of death or dependence compared with non-LVO AISs
(Rennert et al., 2019). Research shows that LVOs affecting the intracranial internal carotid artery and
basilar artery are associated with higher NIHSS scores and worse outcomes compared with occlusions
of more distal vessels (Rennert et al., 2019).
The blockage of a cerebral artery usually results in a core area (i.e., the infarct core) that is irrevers-
ibly damaged within minutes or hours. The area of dead tissue is typically surrounded by an area of
hypoperfused tissue called the ischemic penumbra or the transitional zone (Fig. 9.3). Ischemic changes
occur within minutes. The penumbra is supplied with blood by collateral arteries that connect with
224 CHAPTER 9  Acute Ischemic Stroke

Area of permanent damage

Penumbra, area of salvageable damage


Normal brain external to penumbra

Fig. 9.3  ​After an occlusive stroke, the penumbra is an interface between a region of permanent tissue damage and an
area that will most likely survive. Rapid and appropriate treatment, with reperfusion of the penumbra, may salvage this
region and reduce the neurologic deficits experienced by the patient. (From Haines, D.E. [2013]. Fundamental neuroscience
for basic and clinical applications, ed 4, Philadelphia, Saunders.)

branches of the blocked vessel. The ultimate volume of infarcted tissue is determined by the degree and
length of hypoperfusion, with the level of collateral blood flow to the penumbra playing a significant
role in stroke progression (Rennert et al., 2019). Because brain cells in the penumbra may be salvaged
depending on how quickly blood flow is restored, the goal of treatments for AIS, such as the adminis-
tration of intravenous (IV) fibrinolytic agents or endovascular therapy, is to restore blood flow to these
cells as quickly as possible.

Transient Ischemic Attack


A transient ischemic attack (TIA), also called a ministroke, a warning stroke, or a transient stroke, is
a transient episode of neurologic dysfunction caused by focal brain, spinal cord, or retinal ischemia,
without acute infarction (Easton et al., 2009). About 12% of all strokes are preceded by a TIA, and
12% of patients who experience a TIA die within 1 year (Benjamin et al., 2019). Most TIAs last less
than 1 hour, but symptoms may persist for up to 24 hours. Research shows that nearly 15% of patients
with TIAs have underlying LVOs, with the absence of fixed neurologic deficits or strokes in these
patients presumably because of adequate collateral perfusion (Rennert et al., 2019). A TIA should be
treated with the same urgency as a completed stroke.

STROKE SYSTEMS OF CARE


The “8 Ds of stroke care” reflect key steps in the diagnosis and treatment of stroke: detection, dispatch,
delivery, door, data, decision, drug/device, and disposition ( Jauch et al., 2013) (Table 9.2).
Similar to the systems of care developed for trauma patients and for individuals who experience an
ST-elevation myocardial infarction (STEMI), a stroke system of care is a comprehensive spectrum of
care that addresses all aspects of stroke care in a coordinated fashion. Stroke systems of care goals in-
clude stroke prevention, community stroke education, optimal use of emergency medical services
(EMS), effective acute and subacute stroke care, rehabilitation, and performance review of stroke care
delivery ( Jauch et al., 2013).
CHAPTER 9  Acute Ischemic Stroke 225

TABLE 9.2 The 8 Ds of Stroke Care


Step Description
Detection Recognition of stroke signs and symptoms by the patient, family, or bystanders
Dispatch Activation of the EMSS, priority EMSS dispatch, and prompt EMSS response
Delivery Prompt triage and transport to an appropriate stroke hospital while providing prehospital
assessment and care; prearrival notification
Door ED triage and immediate stroke team assessment
Data Prompt patient assessment, laboratory studies, and brain imaging
Decision Decision about potential therapies made based on the data gathered and stroke type,
location (e.g., carotid, vertebrobasilar), and stroke severity; discussion with patient
and family
Drug/device Administration of appropriate medications and/or endovascular therapy (if eligible)
Disposition Prompt admission to a stroke unit or intensive care unit, or emergent transfer for endo-
vascular therapy, ongoing care, and monitoring
ED, Emergency department; EMSS, emergency medical services system.
(Modified from Jauch, E.C., Saver, J.L., Adams Jr., H.P., et al. [2013]. Guidelines for the early management of patients with
acute ischemic stroke: a guideline for healthcare professionals from the American Heart Association/American Stroke
Association. Stroke, 44[3], 870–947.)

Stroke Centers
Because many of the therapeutic interventions for acute stroke are time-sensitive, eligible patients must
be rapidly identified and transported to an appropriate facility for emergency care. The Joint Commis-
sion (TJC), in collaboration with the American Heart Association/American Stroke Association,
offers certification for four levels of hospital stroke programs (Box 9.1). All levels of certified stroke
centers have an acute stroke team available 24/7. The stroke team is responsible for responding to
patients with acute stroke and initiating diagnostic testing and immediate care.
Because they have differing levels of resources, certified stroke centers have transfer protocols in
place to ensure the prompt transfer of eligible patients to more advanced care. Several organizational
models have been developed for the timely transfer of patients for endovascular therapies. Factors
affecting the model used are specific to an area, such as geography, local infrastructure, population
density, transportation times, and the distribution of stroke centers (Urdaneta & Bhalia, 2019).
Nonstroke hospitals transfer patients to stroke centers that can initiate fibrinolytic therapy. With the
mothership model, patients are transported directly to the nearest thrombectomy–capable stroke center
(TSC) or comprehensive stroke center (CSC), bypassing lower-level stroke centers. With the “drip-
and-ship” model, the patient is transported to the nearest stroke center, treatment with IV fibrinolytics
is begun, and the patient is then transferred to a TSC or CSC for endovascular therapy if an LVO is

BOX 9.1 Key Requirements of Acute Stroke Care Facilities


• Acute stroke–ready hospital (ASRH) certification is for hospitals or emergency centers with
a dedicated stroke-focused program. These facilities have an acute stroke team available
24/7, a neurologist available 24/7 in person or via telemedicine (telemedicine available within
20 minutes of it being necessary), diagnostic services, and the ability to provide intravenous
(IV) fibrinolytics.
• Primary stroke centers (PSCs) meet the same requirements as ASRHs and have a designated
stroke unit.
• Thrombectomy–capable stroke centers (TSCs) meet the same requirements as PSCs and can
also provide endovascular procedures and postprocedural care.
• Comprehensive stroke centers (CSCs) are able to receive and treat the most complex stroke
patients. They meet the same requirements as TSCs and also have a dedicated neurologic
intensive care unit (NICU) for complex stroke patients, comprehensive diagnostic services,
the ability to meet concurrent needs for two complex stroke patients, neurosurgical services
available 24/7, and on-site NICU coverage by a neurospecialist.
226 CHAPTER 9  Acute Ischemic Stroke

identified on neuroimaging. A mobile stroke unit is used in some areas. The unit is essentially an am-
bulance equipped with a CT scanner, telemedicine capabilities, and a point-of-care laboratory system
(Urdaneta & Bhalia, 2019), enabling triage to an appropriate stroke center and initiation of stroke-
specific therapy.

Public Education
About 62% of stroke deaths in 2016 occurred outside of an acute care hospital (Benjamin et al., 2019).
Recognition of stroke signs and symptoms by the patient, family, or bystanders is critical. Despite
stroke education initiatives, the public’s awareness of stroke symptoms remains poor. Of the warning
signs of stroke (Box 9.2), sudden confusion or difficulty speaking, and sudden numbness or weakness
of the face, arm, or leg are the most correctly identified stroke symptoms. In contrast, sudden headache
is the least (Benjamin et al., 2019).

ACLS Pearl
Although advance notification of stroke patient arrival by EMS personnel shortens the time to be
seen for an initial evaluation by an emergency physician, reduces the time to brain imaging, and
increases the use of IV fibrinolytics for eligible patients, only about 60% of all stroke patients use
EMS (Powers et al., 2019). Many patients with an acute stroke present at a hospital using a private
car, taxi, or another mode of transportation (other than an ambulance) (Higashida et al., 2013).

Emergency Medical Services


The primary goals of EMS assessment and management are rapid evaluation, early stabilization, neu-
rologic evaluation, and rapid transport and triage to a stroke-ready hospital ( Jauch et al., 2013). Several
prehospital stroke screening tools have been developed and studied. Current stroke guidelines note that
no strong recommendation can be made for the use of one tool over another (Powers et al., 2019).

ACLS Pearl
The American Heart Association/American Stroke Association commissioned a systematic review
to answer the question, “What is the diagnostic accuracy of LVO prediction instruments for iden-
tifying LVO in individuals with suspected stroke or confirmed to have an AIS in the prehospital or
hospital emergency room settings?” The NIHSS, Cincinnati Prehospital Stroke Severity Scale
(CPSSS), Los Angeles Motor Scale (LAMS), Rapid Arterial oCclusion Evaluation (RACE), and the
3-Item Stroke Scale were among the screening tools reviewed. The experts concluded that no
scale predicted LVO with both high sensitivity and high specificity (Smith et al., 2018). The authors
noted that the NIHSS is the optimal LVO prediction instrument in the hospital ED. In contrast, in
the prehospital setting, a variety of scales, including the CPSSS, LAMS, and RACE, could be used
without clear evidence for the superiority of one scale over the others (Smith et al., 2018).

Assessment and Management


Prehospital providers should quickly perform a primary assessment and stabilize the patient’s airway,
breathing, and circulation (ABCs) as necessary. A focused history should be obtained, and the patient’s
normal baseline mental status determined. Because families often confuse the time of symptom onset

BOX 9.2 Stroke Warning Signs: The Five “Suddens”


Sudden confusion, difficulty speaking (e.g., an inability to say what is meant, slurred speech) or
difficulty understanding speech
Sudden dizziness, difficulty walking, loss of balance or coordination
Sudden severe headache with no known cause
Sudden visual changes in one or both eyes
Sudden weakness or numbness of the face, arm, or leg (especially on one side of the body)
CHAPTER 9  Acute Ischemic Stroke 227

with the time the patient was found, the patient, patient’s family, coworkers, or others at the scene
should be asked when the patient was last known to be symptom-free (i.e., last known normal or last
known well time) (Demaerschalk et al., 2016). Determining and documenting the time of symptom
onset is critical so that the patient’s eligibility for time-limited fibrinolytic and/or endovascular thera-
pies can be determined.
All medications that the patient is currently taking should be collected and documented. Particu-
larly important medications include anticoagulants, antiplatelet agents, antihypertensives, insulin, oral
hypoglycemics, and sympathomimetics. Ascertain if the patient has a history of conditions that in-
crease the likelihood that their symptoms are caused by a stroke, such as previous TIAs, prior stroke,
seizures, diabetes mellitus, hypertension, and AFib.
Perform a neurologic assessment using a validated stroke screening tool. If the patient’s assessment
findings and symptoms suggest an acute stroke, begin rapid transport to the closest healthcare facility
that can administer IV fibrinolytics (Powers et al., 2019). In areas where several IV fibrinolytic–capable
hospital options exist, the benefit of bypassing the closest IV fibrinolytic–capable facility to bring the
patient to one that offers a higher level of stroke care, including mechanical thrombectomy, is uncertain
(Powers et al., 2019).
Perform a secondary assessment during transport as dictated by the patient’s condition. Monitor the
patient’s breathing effort and be prepared to assist ventilations. Attach the patient to a pulse oximeter
and a cardiac monitor. Maintain the patient’s oxygen saturation above 94% (Powers et al., 2019). Ob-
tain a 12-lead ECG and establish IV access. Check the patient’s serum glucose level and give dextrose
if they are hypoglycemic. If consistent with local protocols, obtain blood samples for laboratory testing,
and transfer the samples to receiving facility staff on arrival. Do not delay transport to perform these
procedures.
Monitor vital signs at least every 15 minutes, and more frequently if any vital sign is abnormal. In
general, hypertension should not be treated in the prehospital setting. Hypotension should be treated
in accordance with the underlying cause of the hypotension.
Encourage family members or bystanders to accompany the patient to the hospital so they can
provide historical information to the stroke team and provide support to the patient. If the patient’s
family cannot go to the hospital, obtain a telephone number where they can be contacted, preferably a
cell phone number, and be sure to document this information for subsequent retrieval by other mem-
bers of the healthcare team.
Because strokes are dynamic processes, reassess the patient often during transport. Document any
changes in the patient’s presentation from your initial assessment findings and relay this information
to the appropriate staff on arrival at the receiving facility.

Hospital Initial Evaluation and Treatment


Proper triage of stroke patients requires that emergency nurses be familiar with both typical and un-
usual stroke presentations (Summers et al., 2009). A patient presenting with a possible stroke should
be seen by a physician, stroke team, or other expert within 10 minutes of their arrival. Reassess the
patient’s ABCs and ensure that the patient has a secure airway and adequate breathing. Assess the
patient’s vital signs, including oxygen saturation, and give oxygen if needed to maintain the patient’s
oxygen saturation above 94%; supplemental oxygen is not recommended in nonhypoxic patients with
AIS (Powers et al., 2019).

History and Physical Examination


Attempt to identify the patient’s symptoms and progression of symptoms. Verify the patient’s last
known well time. Was anyone with the patient when their symptoms started? What was the
patient doing when the symptoms began? Did the patient complain of a headache? Did they have
a seizure? Has there been a change in their level of responsiveness? Is there a history of any recent
trauma?
Review the patient’s pertinent medical history and determine the presence of stroke risk factors. Ask
if there is any history of alcohol or illicit drug use, migraine, seizure, infection, trauma, or pregnancy
( Jauch et al., 2013). Because they may be associated with coagulopathy, ask if the patient has liver
disease, cancer, or a hematologic disorder. Find out the medications the patient is currently taking and
their allergies to medications.
228 CHAPTER 9  Acute Ischemic Stroke

BOX 9.3 Conditions That Mimic Stroke


Bell’s palsy Encephalitis, meningitis
Central nervous system abscess or tumor Hypertensive encephalopathy
Complicated migraine Metabolic disorders (e.g., hyperglycemia,
Concussion with head injury hypoglycemia, hyponatremia)
Conversion disorder Positional vertigo
Drug toxicity (e.g., carbamazepine, lithium, Seizures
phenytoin) Subdural hematoma
Eclampsia Wernicke’s encephalopathy

When performing a physical examination, consider the presence of conditions that mimic stroke
(Box 9.3). Examine the head and face for signs of trauma or recent seizure activity (e.g., contusions,
tongue laceration). Auscultate the neck for carotid bruits, which suggests the presence of carotid
atherosclerotic disease. Assess for jugular venous distention, which may be a sign of heart failure. Aus-
cultate heart sounds, which may reveal murmurs or gallops, and lung sounds. Examine the extremities
for asymmetric strength and movement and asymmetric or diminished pulses. Inspect the skin for
petechiae, purpura, or ecchymoses, which may be the result of trauma, a platelet disorder, or a coagula-
tion disorder.

Neurologic Examination
Perform a brief neurologic screening assessment using a validated stroke scale, preferably the NIHSS
(Powers et al., 2019). This scale is widely used and takes less than 10 minutes to perform. Training is
required to use the scale accurately and to ensure interrater reliability. The NIHSS assigns points for
neurologic deficits with possible scores ranging from 0 to 42; the lower the score, the less the impair-
ment. A score of 0 indicates no impairment, a score between 1 and 20 indicates mild to moderate
impairment, and a score of more than 20 indicates severe impairment. An increase of 2 or more points
on NIHSS when serially administered suggests stroke progression, although smaller changes may be
equally significant. If the initial history, physical examination, and neurologic examination are sugges-
tive of stroke, mobilize the stroke team.

Intravenous Fluids
A minimum of two IV lines should be established if it is anticipated that the patient will receive fibri-
nolytic therapy. One site is used for infusing IV fluids and medications, and the other is used for the
administration of the fibrinolytic. Current stroke guidelines note that there are no data to guide the
volume and duration of parenteral fluid delivery, and no studies have compared different isotonic fluids
(Powers et al., 2019).

Diagnostic Studies
A noncontrast brain CT scan (or brain magnetic resonance imaging [MRI]) should be obtained for all
patients with suspected AIS to confirm or exclude the presence of cerebral hemorrhage (Powers et al.,
2019). In patients presenting with a history and clinical examination consistent with acute stroke, brain
imaging is useful in determining stroke location and vascular distribution, the presence of bleeding, the
severity of an ischemic stroke, and the presence of LVO. Studies have shown that median or mean
door-to-imaging times of 20 minutes or less can be achieved in a variety of different hospital settings
(Powers et al., 2019). Brain imaging should be interpreted within 45 minutes of patient arrival.
Diagnostic laboratory tests should be drawn immediately and before IV fluids are started. Of the
laboratory tests recommended during the initial emergency evaluation (Table 9.3), only the assess-
ment of blood glucose must precede the initiation of IV alteplase in all patients (Powers et al., 2019).
Other tests (e.g., international normalized ratio, activated partial thromboplastin time, platelet count)
may be necessary if there is suspicion of coagulopathy (Powers et al., 2019). Additional diagnostic
studies should be obtained in selected cases. In the hyperacute stroke setting, the usefulness of chest
radiography in the absence of evidence of acute pulmonary, cardiac, or pulmonary vascular disease is
unclear; if chest radiographs are obtained, they should not unnecessarily delay the administration of
IV alteplase (Powers et al., 2019).
CHAPTER 9  Acute Ischemic Stroke 229

TABLE 9.3 Immediate Diagnostic Studies for Suspected Stroke


All Patients Selected Patients
• Noncontrast brain CT or brain MRI • Arterial blood gas (if hypoxia is suspected)
• Activated partial thromboplastin time • Blood alcohol level
• Cardiac biomarkers, troponin is preferred • Chest radiography (if lung disease is suspected)
• Complete blood count, including platelet count • Electroencephalogram (if seizures are suspected)
• ECG • Hepatic function tests
• Oxygen saturation • Lumbar puncture (if subarachnoid hemorrhage is
• Prothrombin time or INR suspected and CT scan is negative for blood
• Renal function tests • Pregnancy test
• Serum electrolytes • TT and/or ECT if it is suspected the patient is
• Serum glucose taking direct thrombin inhibitors or direct factor
Xa inhibitors
• Toxicology screen

CT, Computed tomography; ECG, electrocardiogram; ECT, ecarin clotting time; INR, international normalized ratio; MRI,
magnetic resonance imaging; TT, thrombin time.
(Modified from Jauch, E.C., Saver, J.L., Adams Jr., H.P., et al. [2013]. Guidelines for the early management of patients with
acute ischemic stroke: a guideline for healthcare professionals from the American Heart Association/American Stroke
Association. Stroke, 44[3], 870–947.)

Blood Glucose
Perform a fingerstick glucose test to assess for hypoglycemia and hyperglycemia. Administer dextrose
if the blood glucose is less than 60 mg/dL (Powers et al., 2019). Because persistent in-hospital hyper-
glycemia during the first 24 hours after AIS is associated with worse functional outcomes than nor-
moglycemia, current guidelines note that it is reasonable to treat hyperglycemia to achieve blood
glucose levels in a range of 140 to 180 mg/dL and to closely monitor to prevent hypoglycemia in
patients with AIS (Powers et al., 2019).

Blood Pressure
About 75% of patients who present with AIS have a systolic BP (SBP) greater than 140 mm Hg
(Urdaneta & Bhalia, 2019). Strict BP recommendations exist for patients with AIS who are otherwise
eligible for IV fibrinolytic or endovascular therapy. Patients who have an elevated BP and are otherwise
eligible for treatment with IV alteplase should have their BP carefully lowered so that their SBP is
less than 185 mm Hg and their diastolic BP (DBP) is less than 110 mm Hg before therapy is started
(Powers et al., 2019). In patients for whom mechanical thrombectomy is planned and who have not
received treatment with IV fibrinolytics, it is reasonable to maintain the BP at 185/110 mm Hg or less
before the procedure (Powers et al., 2019).
For patients who undergo mechanical thrombectomy, it is reasonable to maintain the BP at 180/105 mm
Hg or less during and for 24 hours after the procedure (Powers et al., 2019). For patients who undergo
mechanical thrombectomy with successful reperfusion, it might be reasonable to maintain the BP at a level
below 180/105 mm Hg (Powers et al., 2019).

Electrocardiogram Monitoring
All patients with suspected acute stroke should receive continuous ECG monitoring to detect myocar-
dial ischemia and cardiac dysrhythmias, such as AFib. Monitoring should be continued for at least the
first 24 hours after stroke ( Jauch et al., 2013). A 12-lead ECG should be obtained to evaluate for
preexisting cardiac disease and concurrent myocardial injury.

Temperature
Sources of hyperthermia (temperature higher than 100.4° F [38° C]) should be identified and treated,
and antipyretic medications should be administered if indicated (Powers et al., 2019).

Patient Positioning
Several small studies have shown that the lying-flat position may improve cerebral perfusion in patients
with AIS caused by a large artery occlusion when the intervention is initiated early after stroke onset;
230 CHAPTER 9  Acute Ischemic Stroke

however, the benefit of flat head positioning soon after hospitalization for stroke is uncertain (Powers
et al., 2019).

Intravenous Fibrinolysis
Fibrinolytic therapy with IV alteplase is recommended for selected patients within 3 hours of the
onset of AIS. IV alteplase is recommended for administration to a select group of eligible patients who
present within a 3- to 4.5-hour window after the onset of acute stroke symptoms. The eligibility crite-
ria for treatment in this time frame are similar to those for patients treated within 3 hours of symptom
onset, but with additional exclusion criteria. For patients who awake with stroke symptoms or have an
unclear time of onset of more than 4.5 hours from their last known normal, obtaining an MRI can be
useful for identifying those individuals who can benefit from IV alteplase administration (Powers et al.,
2019).
IV alteplase is not recommended for otherwise eligible patients with mild nondisabling stroke
symptoms (NIHSS score 0 to 5), who could be treated within 3 hours, or within 3 to 4.5 hours, of
ischemic stroke symptom onset, or the patient’s last known well or at-baseline state (Powers et al.,
2019).
The IV alteplase dose is 0.9 mg/kg, not to exceed 90 mg. Ten percent of the dose is given as an
initial IV bolus over 1 minute, followed by the remaining 90% of the dose infused using an infusion
pump during the next hour. Calculate the desired dose, withdraw any excess amount from the vial, and
then discard the excess amount to prevent accidental overdose (Summers et al., 2009). IV alteplase
should not be given to patients who have received a full treatment dose of low-molecular-weight
heparin (LMWH) within the previous 24 hours (Powers et al., 2019).
Current guidelines note that it may be reasonable to choose tenecteplase (single IV bolus of
0.25 mg/kg, maximum 25 mg) over IV alteplase in patients without contraindications for IV
fibrinolysis who are also eligible to undergo mechanical thrombectomy (Powers et al., 2019).
Tenecteplase administered as a 0.4-mg/kg single IV bolus might be considered as an alternative to
alteplase in patients with minor neurologic impairment and no significant intracranial occlusion
(Powers et al., 2019).
Bleeding and angioedema are two critical complications of treatment with IV fibrinolytics.
Although bleeding may occur from any site, intracranial bleeding is of particular concern. Cerebral
microbleeds occur in 15% to 27% of patients who receive IV alteplase (Powers et al., 2019). Close
monitoring of the patient is critical. In addition to using the NIHSS to assess neurologic deficits, assess
pupil size, and use the GCS to monitor the patient’s level of responsiveness. These assessments should
be performed every hour for the first 24 hours after fibrinolytic administration and more often if indi-
cated (Summers et al., 2009). The physician should be notified, the alteplase infusion stopped (if it is
still infusing), and an emergent CT scan obtained if the patient develops acute hypertension, nausea
or vomiting, or severe headache, or if the patient has a worsening neurologic examination (Powers
et al., 2019)
Carefully observe for swelling of the tongue, lips, or oropharynx (i.e., orolingual angioedema). Al-
though this complication of IV alteplase administration occurs in a small number of patients, it can
lead to airway obstruction. Patients taking angiotensin-converting enzyme (ACE) inhibitors and those
with infarctions that involve the insular and frontal cortex appear to be at the highest risk ( Jauch et al.,
2013). Treatment includes discontinuing the alteplase infusion and holding ACE inhibitors; adminis-
tering IV methylprednisolone, IV diphenhydramine, and IV ranitidine or famotidine; and supportive
care (Powers et al., 2019).
Measure the patient’s BP every 15 minutes during and after the IV alteplase infusion for 2 hours,
then every 30 minutes for 6 hours, and then every hour until 24 hours after the infusion (Powers et al.,
2019). Assess the patient’s BP more frequently if their SBP is more than 180 mm Hg or if the
DBP is more than 105 mm Hg (Powers et al., 2019). Administer antihypertensive medications per
physician’s orders to maintain the patient’s BP at or below these levels (Powers et al., 2019). Obtain a
brain CT or MRI scan 24 hours postinfusion before starting anticoagulants or antiplatelet agents
(Powers et al., 2019).

Endovascular Therapies
The results of several clinical trials published in 2014, 2015, and 2018 conclusively demonstrated the
efficacy of endovascular therapy for selected patients with acute stroke with improved functional
CHAPTER 9  Acute Ischemic Stroke 231

outcomes at 90 days. The primary goal of endovascular stroke therapy is to restore blood flow to the
brain tissue within the penumbra and territory at potential risk while avoiding hemorrhagic transfor-
mation of unsalvageable tissue (Fargen et al., 2012).
In selected symptomatic patients with an LVO, endovascular therapy is recommended if the proce-
dure can begin within 6 hours of symptom onset, and they meet specific eligibility criteria. Patients
eligible for IV alteplase should receive IV alteplase even if endovascular therapy is being considered
(Powers et al., 2019). For patients being considered for endovascular therapy, observation after IV al-
teplase to assess for a clinical response should not be performed (Powers et al., 2019). Patients with an
LVO in the anterior circulation may be candidates for endovascular therapy if their last known normal
was up to 24 hours after symptom onset, and specific eligibility criteria are met (Powers et al., 2019).

Antiplatelet Therapy
Administration of aspirin is recommended in patients with AIS within 24 to 48 hours after onset
(Powers et al., 2019). Aspirin administration is generally delayed until 24 hours after IV alteplase
therapy (Powers et al., 2019). The rectal or nasogastric routes should be used for aspirin administration
if the patient is unable to swallow. Aspirin is not recommended as a substitute for acute stroke treat-
ment in patients who are otherwise eligible for IV alteplase or mechanical thrombectomy (Powers
et al., 2019).
For patients presenting with minor noncardioembolic ischemic stroke (NIHSS score of 3 or less)
who did not receive IV alteplase, treatment with aspirin and clopidogrel that starts within 24 hours
after symptom onset and is continued for 21 days has proved effective in reducing recurrent ischemic
stroke for a period of up to 90 days from symptom onset (Powers et al., 2019).

ACLS Pearl
Poststroke depression (PSD) is one of the most common complications after stroke occurring in
about one-third of stroke survivors (Towfighi et al., 2017). It has also been reported after TIA and
minor stroke (NIHSS score of 5 or less at discharge). Symptoms most often develop in the first
year. PSD is associated with poorer functional outcomes, higher healthcare use, and higher mor-
tality (Towfighi et al., 2017). Research suggests that PSD has underlying biologic causes and is
not merely a psychologic response to a new disability or a life-threatening event (Towfighi et al.,
2017). Consistent predictors of PSD include physical disability, stroke severity, history of depres-
sion before the stroke, and cognitive impairment (Towfighi et al., 2017). Screening for PSD is
recommended. Patients diagnosed with PSD should be treated with antidepressants, if not con-
traindicated, and closely monitored to confirm their effectiveness (Powers et al., 2019).
232 CHAPTER 9  Acute Ischemic Stroke

STOP AND REVIEW


Identify one or more choices that best complete the statement or answer the question.

_ ___ 1. What is the most common cause of stroke?


a. A thrombus
b. An embolus
c. A ruptured cerebral aneurysm
d. An arteriovenous malformation

____ 2. Paramedics are at the home of a 62-year-old man presenting with signs and symp-
toms suggestive of stroke. Which of the following is the most important question
that should be asked of this patient, family members, or others at the scene?
a. “When did you last see a physician?”
b. “When did your symptoms begin?”
c. “Do you have a history of hypertension?”
d. “Are you currently taking any blood thinners?”

_ ___ 3. Which of the following are true of ASRHs in the United States?
a. ASRHs have a stroke team available 24/7.
b. Neurosurgical coverage is available in-house 24/7.
c. ASRHs can care for two complex stroke patients at the same time.
d. ASRHs are equipped to provide endovascular procedures and postprocedural
care.

____ 4. Which of the following diagnostic studies should be obtained for all patients with
suspected stroke?
a. Lumbar puncture
b. Chest radiograph
c. Serum electrolytes
d. Toxicology screen
e. Oxygen saturation
f. Cardiac biomarkers
g. Noncontrast brain CT
h. Complete blood count

____ 5. A 52-year-old woman presents with a sudden onset of numbness and weakness
in her right arm and leg. Family members state her signs and symptoms began
while the patient was preparing breakfast 1 hour ago. Examination reveals un-
equal grips with marked weakness on the patient’s right side. Her blood pressure
is 174/86 mm Hg, pulse 88 beats per minute (beats/min), and ventilatory rate 16.
Her oxygen saturation on room air is 96%. As you establish vascular access, you
note an improvement in the patient’s symptoms. After 25 minutes, her grips are
equal, and there is no weakness on the patient’s right side. You
suspect:
a. Hypoglycemia
b. AIS
c. TIA
d. SAH
CHAPTER 9  Acute Ischemic Stroke 233

____ 6. Which of the following must be performed before an IV fibrinolytic is administered


for AIS?
a. Serum glucose
b. Serum electrolytes
c. Cardiac biomarkers
d. Activated partial thromboplastin time

____ 7. During which of the following steps of the 8 Ds of stroke care is immediate ED
triage performed?
a. Data
b. Drug
c. Door
d. Delivery
234 CHAPTER 9  Acute Ischemic Stroke

C A S E S T U DY 9.1
Identify one or more choices that best complete the statement or answer the question.

Paramedics are called to a private residence for a 78-year-old man with a “possible stroke.” The
patient’s wife is present. Emergency equipment, including a biphasic manual defibrillator, is available
to the crew. As you work through this case study, remember that there may be alternative actions that
are perfectly acceptable, yet not discussed here.

____ 1. The patient is sitting upright in a recliner. He is awake and aware of your approach.
His breathing is unlabored, and equal chest rise and fall is observed. The patient’s
skin is pink. Select the correct statements about this patient’s general impression
findings.
a. Appearance is normal.
b. Appearance is abnormal.
c. Breathing is normal.
d. Breathing is abnormal.
e. Circulation is normal.
f. Circulation is abnormal.

_ ___ 2. Which of the following actions should be performed at this time?


a. Obtain a focused history.
b. Obtain the patient’s vital signs.
c. Perform a primary and secondary assessment.
d. Document the medications the patient is currently taking.
e. Determine when the patient was last known to be symptom-free.

____ 3. The patient’s breathing is quiet and unlabored at a rate of 16 breaths/min. His
oxygen saturation (SpO2) on room air is 95%, and his radial and carotid pulses are
strong but irregular. The patient’s skin is warm, pink, and dry. The patient’s wife
relays that her husband dropped a coffee cup about 20 minutes ago. When she
asked what happened, her husband attempted to answer her. Noticing that his
speech was garbled, she immediately called 9-1-1. Which of the following should
be done next?
a. Establish IV access.
b. Check the patient’s blood glucose level.
c. Complete a prehospital stroke assessment.
d. Attach the patient to a cardiac monitor and obtain a 12-lead ECG.

____ 4. The cardiac monitor reveals AFib with a ventricular rate of 90 beats/min. Your
assessment reveals that the patient appears anxious and has a left facial droop, left
motor weakness, and garbled speech. His BP is 160/90 mm Hg, and his serum
glucose is 125 mg/dL. The patient has a history of hypertension, for which he takes
lisinopril and hydrochlorothiazide daily, and has no known allergies. IV access is
established, and a primary stroke center (PSC), which is 8 minutes away, is notified
that the patient is en route. Which of the following should be performed by the
ED staff when the patient arrives?
a. Activate the stroke team.
b. Order laboratory studies.
c. Verify the patient’s last known well time.
d. Administer oxygen by nonrebreather mask.
e. Perform a neurologic assessment using the NIHSS.
f. Consider the presence of conditions that mimic stroke.
CHAPTER 9  Acute Ischemic Stroke 235

_ ___ 5. Brain imaging should be interpreted within __ minutes of the patient’s arrival.
a. 10
b. 15
c. 45
d. 90

____ 6. The patient has an NIHSS score of 8. The brain CT reveals no evidence of bleed-
ing. After reviewing the inclusion and exclusion criteria for treatment with IV
alteplase, the decision is made to begin fibrinolytic therapy. Which of the
following statements is true regarding IV alteplase?
a. Aspirin should be administered simultaneously with IV alteplase.
b. A standard dose of 90 mg is administered to all patients with AIS.
c. IV alteplase is given as a continuous IV infusion using an infusion pump.
d. Bleeding and angioedema are two critical complications of treatment with IV
alteplase.

____ 7. Which of the following are recommended interventions if a patient develops


angioedema during IV alteplase administration?
a. Discontinue IV alteplase.
b. Administer IV diphenhydramine.
c. Administer IV methylprednisolone.
d. Administer IV ranitidine or famotidine.

_ ___ 8. Aspirin administration is generally delayed until __ after IV alteplase therapy.


a. 2 hours
b. 6 hours
c. 24 hours
d. 48 hours

STOP AND REVIEW  ANSWERS


1. A. Most strokes are the result of blockages caused by blood clots that develop within a cerebral
artery (i.e., cerebral thrombosis) or clots that arise elsewhere in the body and then migrate to the
brain (i.e., cerebral embolism).

2. B. Determining and documenting the time of symptom onset is critical and the single most impor-
tant determinant of treatment options during the hyperacute phase of stroke care. The patient,
patient’s family, coworkers, or others at the scene should be asked when the patient was last known
to be symptom-free (i.e., last known normal or last known well time).

3. A. ASRH certification is for hospitals or emergency centers with a dedicated stroke-focused pro-
gram. These facilities have an acute stroke team available 24/7, a neurologist (not a neurosurgeon)
available 24/7 in person or via telemedicine (telemedicine available within 20 minutes of it being
necessary), diagnostic services, and the ability to provide IV fibrinolytics.

4. C, E, F, G, H. The following diagnostic studies should be obtained for all patients with suspected
stroke: noncontrast brain CT or brain MRI, activated partial thromboplastin time, cardiac biomark-
ers (troponin is preferred), complete blood count (including platelet count), ECG, oxygen satura-
tion, prothrombin time or INR, renal function tests, serum electrolytes, and serum glucose.

5. C. A TIA is a transient episode of neurologic dysfunction caused by focal brain, spinal cord, or
retinal ischemia, without acute infarction. A TIA should be treated with the same urgency as a
completed stroke.
236 CHAPTER 9  Acute Ischemic Stroke

6. A. Of the laboratory tests recommended during the initial emergency evaluation of a patient
with a possible stroke, only the assessment of blood glucose must precede the administration of
IV fibrinolytics.

7. C. The door step in the 8 Ds of Stroke Care refers to immediate ED triage upon the patient’s arrival.

C A S E S T U DY 9.1 ANSWERS
1. A, C, E. The general impression findings are normal (Appearance: normal; Breathing: normal;
Circulation: normal skin color).

2. A, B, C, D, E. Perform a primary and secondary assessment, obtain a focused history, and determine
when the patient was last known to be symptom-free (i.e., last known normal or last known well
time). All medications that the patient is currently taking should be collected and documented.
Ascertain if the patient has a history of conditions that increase the likelihood that his symptoms
are caused by a stroke, such as previous TIAs, prior stroke, seizures, diabetes mellitus, hypertension,
and AFib.

3. A, B, C, D. Attach the patient to a cardiac monitor. Obtain a 12-lead ECG and establish IV access.
Check the patient’s serum glucose level and give dextrose if he is hypoglycemic. If consistent with
local protocols, obtain blood samples for laboratory testing, and transfer the samples to receiving
facility staff on arrival.

4. A, B, C, E, F. Within minutes of the patient’s arrival in the ED, reassess the patient’s ABCs and
ensure that the patient has a secure airway and adequate breathing. Assess the patient’s vital
signs, including oxygen saturation, and give oxygen if needed to maintain oxygen saturation
above 94%; supplemental oxygen is not recommended in nonhypoxic patients with AIS. When
performing a physical examination, consider the presence of conditions that mimic stroke.
Perform a brief neurologic screening assessment using a validated stroke scale, preferably the
NIHSS.

5. C. Studies have shown that median or mean door-to-imaging times of 20 minutes or less can be
achieved in a variety of different hospital settings. Brain imaging should be interpreted within
45 minutes of the patient’s arrival.

6. D. IV alteplase is a weight-based therapy. The dose is 0.9 mg/kg, not to exceed 90 mg. Ten
percent of the dose is given as an initial IV bolus over one minute, followed by the remaining
90% of the dose infused using an infusion pump during the next hour. Calculate the desired
dose, withdraw any excess amount from the vial, and then discard the excess amount to prevent
accidental overdose. Administration of aspirin is recommended in patients with AIS within
24 to 48 hours after onset. Aspirin administration is generally delayed until 24 hours after IV
alteplase therapy. Bleeding and angioedema are two critical complications of treatment with IV
fibrinolytics. Although bleeding may occur from any site, intracranial bleeding is of particular
concern.

7. A, B, C, D. Although angioedema occurs in a small number of patients who receive IV alteplase, it


can lead to airway obstruction. Treatment includes discontinuing the alteplase infusion and holding
ACE inhibitors; administering IV methylprednisolone, IV diphenhydramine, and IV ranitidine or
famotidine; and supportive care.

8. C. Administration of aspirin is recommended in patients with AIS within 24 to 48 hours after


onset. Aspirin administration is generally delayed until 24 hours after IV alteplase therapy.
CHAPTER 9  Acute Ischemic Stroke 237

REFERENCES
Benjamin, E. J., Muntner, P., Alonso, A., et al. (2019). Heart disease and stroke statistics-2019 update: A report
from the American Heart Association. Circulation, 139(10), e56–e528.
Brouwers, H. B., & Goldstein, J. N. (2012). Therapeutic strategies in acute intracerebral hemorrhage. Neurothera-
peutics, 9(1), 87–98.
Demaerschalk, B. M., Kleindorfer, D. O., Adeoye, O. M., et al. (2016). Scientific rationale for the inclusion and
exclusion criteria for intravenous alteplase in acute ischemic stroke: A statement for healthcare professionals
from the American Heart Association/American Stroke Association. Stroke, 47(2), 581–641.
Easton, J. D., Saver, J. L., Albers, G. W., et al. (2009). Definition and evaluation of transient ischemic attack. Stroke,
40(6), 2276–2293.
Fargen, K. M., Velat, G. J., Waters, M. F., Hoh, B. L., & Mocco, J. (2012). Endovascular treatment of stroke. In A.
Quiñones-Hinojosa (Ed.), Schmidek and Sweet’s operative neurosurgical techniques (6th ed., pp. 1029–1036).
Philadelphia, PA: Saunders.
Goldstein, L. B. (2020). Ischemic cerebrovascular disease. In L. Goldman, & A. I. Schafer (Eds.), Goldman-Cecil
medicine (26th ed., pp. 2396–2407). Philadelphia, PA: Elsevier.
Hemphill, J. C., Greenberg, S. M., Anderson, C. S., et al. (2015). Guidelines for the management of spontaneous
intracerebral hemorrhage: A guideline for healthcare professionals from the American Heart Association/
American Stroke Association. Stroke, 46(7), 2032–2060.
Higashida, R., Alberts, M. J., Alexander, D. N., et al. (2013). Interactions within stroke systems of care: A policy
statement from the American Heart Association/American Stroke Association. Stroke, 44(10), 2961–2984.
Jauch, E. C., Saver, J. L., Adams Jr., H. P., et al. (2013). Guidelines for the early management of patients with acute
ischemic stroke: A guideline for healthcare professionals from the American Heart Association/American
Stroke Association. Stroke, 44(3), 870–947.
Mayer, S. A. (2020). Hemorrhagic cerebrovascular disease. In L. Goldman, & A. I. Schafer (Eds.), Goldman-Cecil
medicine (26th ed., pp. 2408–2416). Philadelphia, PA: Elsevier.
Powers, W. J., Rabinstein, A. A., Ackerson, T., al. (2019). Guidelines for the early management of patients with
acute ischemic stroke: 2019 update to the 2018 guidelines for the early management of acute ischemic stroke:
A guideline for healthcare professionals from the American Heart Assoc/American Stroke Assoc. Stroke,
50(12), e344–e418.
Rennert, R. C., Wali, A. R., Steinberg, J. A., et al. (2019). Epidemiology, natural history, and clinical presentation
of large vessel ischemic stroke. Neurosurgery, 85(suppl 1), S4–S8.
Sacco, R. L., Kasner, S. E., Broderick, J. P., et al. (2013). An updated definition of stroke for the 21st century: A
statement for healthcare professionals from the American Heart Association/American Stroke Association.
Stroke, 44(7), 2064–2089.
Smith, E. E., Kent, D. M., Bulsara, K. R., et al. (2018). Accuracy of prediction instruments for diagnosing large
vessel occlusion in individuals with suspected stroke: A systematic review for the 2018 guidelines for the early
management of patients with acute ischemic stroke. Stroke, 49(3), e111–e122.
Summers, D., Leonard, A., Wentworth, D., et al. (2009). Comprehensive overview of nursing and interdisciplinary
care of the acute ischemic stroke patient. Stroke, 40(8), 2911–2944.
Towfighi, A., Ovbiagele, B., El Husseini, N., et al. (2017). Poststroke depression: A scientific statement for health-
care professionals from the American Heart Association. Stroke, 48(2), e30–e43.
Urdaneta, A. E., & Bhalia, P. (2019). Cutting edge acute ischemic stroke management. Emerg Med Clin North Am,
37(3), 365–379.
Zivin, J. A. (2012). Approach to cerebrovascular diseases. In L. Goldman, & A. I. Schafer (Eds.), Goldman’s Cecil
medicine (24th ed., pp. 2304–2310). Philadelphia, PA: Saunders.
This page intentionally left blank
C H A P T E R 10
Posttest

Identify one or more choices that best complete the statement or answer the question.

Questions 1 through 7 pertain to the following scenario.


A 66-year-old man has experienced a cardiac arrest. You are the resuscitation team leader. You and your
team have arrived at the patient’s bedside, where cardiopulmonary resuscitation (CPR) is in progress.

_ ___ 1. Elements of high-quality CPR include:


a. Avoiding excessive ventilation.
b. Minimizing pauses in chest compression.
c. Ensuring full chest recoil after each compression.
d. Compressing the chest at a depth of at least 2.5 to 3 inches.

_ ___ 2. During this resuscitation effort:


a. Use team member names, if known.
b. State instructions to team members one at a time.
c. Expect all team members to be knowledgeable about current resuscitation
algorithms.
d. Expect team members to clearly acknowledge when procedures and medications
are complete.
e. Expect each team member to know their roles and responsibilities and
limitations.
f. Encourage team members to confer among themselves throughout the
resuscitation effort.

____ 3. The cardiac monitor reveals asystole. A team member is attempting to establish
intravenous (IV) access. Which of the following statements is correct regarding
this situation?
a. Rotate chest compressors every 2 minutes.
b. Administer oxygen using a bag-mask device (BMD).
c. Attempt defibrillation as soon as a defibrillator is available.
d. Attempt to achieve a chest compression fraction (CCF) of 40%.

239
240 CHAPTER 10  Posttest

____ 4. A BMD that is used with supplemental oxygen set at a flow rate of 10 to
15 L/min with an attached reservoir delivers about ___% oxygen to the patient.
a. 21%
b. 40% to 60%
c. 60% to 90%
d. 90% to 100%

____ 5. Multiple attempts to establish a peripheral IV have proved unsuccessful. Your best
course of action at this time will be to:
a. Insert a central line.
b. Attempt intraosseous access.
c. Discontinue resuscitation efforts.
d. Continue peripheral IV attempts until successful.

____ 6. Which of the following medications should be administered during this resuscitation
effort?
a. Atropine
b. Lidocaine
c. Epinephrine
d. Amiodarone
e. Vasopressin

___
_ 7. Which of the following are true with regard to a postevent debriefing?
a. The facilitator should use open-ended questions to encourage discussion.
b. Team members are encouraged to identify lessons learned in a nonpunitive
environment.
c. The gather phase of the debriefing includes a comparison of the team’s actions
with current resuscitation algorithms.
d. Team members are given an opportunity to reflect on their performance and
how their performance can be improved.

Questions 8 through 21 pertain to the following scenario.


Paramedics are on the scene with a 55-year-old man who is complaining of severe chest discomfort.
He describes his discomfort as a “heavy pressure” in the middle of his chest that has been present for
about 1 hour.

___
_ 8. Acute coronary syndromes (ACSs) include which of the following?
a. Stable angina
b. Variant angina
c. Unstable angina (UA)
d. ST-elevation myocardial infarction (STEMI)
e. Non–ST-elevation myocardial infarction (NSTEMI)

____ 9. Which of the following actions performed at the scene is most likely to reduce
subsequent treatment delays at the hospital?
a. Giving aspirin.
b. Obtaining a 12-lead electrocardiogram (ECG).
c. Assessing vital signs and oxygen saturation.
d. Assessing the patient’s degree of discomfort.

____ 10. The patient rates his discomfort 9/10. His blood pressure (BP) is 126/72 mm Hg and
ventilations 14 breaths/min. His oxygen saturation level (SpO2) on room air is 94%.
The cardiac monitor shows a sinus rhythm at 60 beats/min. Immediate management
of this patient should include which of the following?
a. Giving aspirin
b. Assessing vital signs
c. Administering oxygen
d. Establishing IV access
e. Obtaining a targeted history
CHAPTER 10  Posttest   241

_ ___ 11. Conditions that can mimic an acute myocardial infarction (MI) include:
a. Bell’s palsy
b. Panic attack
c. Aortic dissection
d. Acute pericarditis
e. Complicated migraine
f. Pulmonary embolism

_ ___ 12. Atypical symptoms of ACSs are more common in:


a. Women.
b. Older adults.
c. Diabetic individuals.
d. Individuals who have liver disease.
e. Patients who have impaired renal function.
f. Patients who have a history of hypertension.

____ 13. When the patient’s 12-lead ECG is reviewed, the results should be used to classify
the patient into one of three categories. Identify all three categories.
a. ST elevation (STE)
b. ST depression
c. Q waves present
d. Inconclusive ECG
e. Normal or nondiagnostic ECG

____ 14. A 12-lead ECG has been obtained (Fig. 10.1).


The patient’s 12-lead ECG shows:
a. STE in leads II, III, and aVF.
b. STE in leads I, aVL, and V2 to V6.
c. ST depression in leads I, II, III, and aVL.
d. ST depression in leads V1, V4, V5, and V6.

I aVR V1 V4

II aVL V2 V5

III aVF V3 V6

x1.0 0.05-150Hz 25mm/sec


I Lateral aVR --------- V1 Septum V4 Anterior V4R Right ventricle
II Inferior aVL Lateral V2 Septum V5 Lateral V5R Right ventricle
III Inferior aVF Inferior V3 Septum V6 Lateral V6R Right ventricle

Fig. 10.1 (From Phalen, T., Aehlert, B. [2019]. The 12-lead ECG in acute coronary syndromes, ed 4,
St. Louis, Mosby.)
242 CHAPTER 10  Posttest

____ 15. To be considered significant, ECG findings, such as STE or ST depression,


need to be viewed in two or more contiguous leads. Which of the following are
contiguous leads?
a. V1, V4, and V5
b. V2, V3, and V4
c. III, aVF, and V1
d. I, II, III, and aVL

_ ___ 16. The patient’s 12-lead ECG findings suggest a(n) _____ myocardial infarction.
a. Posterior
b. Inferolateral
c. Anterolateral
d. Isolated septal

_ ___ 17. Which of the following complications should be anticipated with this type of MI?
a. Left heart failure
b. Cardiogenic shock
c. Pulmonary embolism
d. Atrial flutter or atrial fibrillation (AFib)

_ ___ 18. Based on the patient’s 12-lead ECG findings:


a. The patient should be classified as having a nondiagnostic ECG and discharged
with follow-up instructions.
b. The patient should be classified as having a STEMI and should be evaluated for
immediate reperfusion therapy.
c. The patient should be classified as having a normal ECG; serial ECGs should
be obtained at 30-minute intervals to detect the development of STE.
d. The patient should be classified as having a non–ST elevation ACS (NSTE-ACS)
and should be admitted to a monitored bed for further evaluation.

____ 19. Vascular access has been established. The patient’s BP is 130/70 mm Hg, his pulse
is 60 beats/min, and his ventilatory rate is 14 breaths/min. Assuming there are no
contraindications for any of the following medications, which of the following
would be appropriate for this patient at this time?
a. Nitroglycerin (NTG)
b. Aspirin
c. An oral beta-blocker
d. A nonsteroidal antiinflammatory drug (NSAID)

_ ___ 20. NTG has been ordered for administration to this patient. NTG:
a. Is contraindicated in hypotensive patients.
b. Should be administered via the IV route for maximum benefit.
c. Should be used with caution in patients with anterior infarction.
d. Should be given every 15 to 20 minutes until chest discomfort is relieved.

____ 21. The patient’s chest discomfort was unrelieved after the maximum recommended
dosage of NTG tablets. Morphine sulfate was ordered, and a 4-mg dose was given
IV. The patient’s BP is now 80/60 mm Hg and his skin is cool, moist, and pale.
His breath sounds are clear. You should:
a. Prepare a lidocaine infusion at 1 to 4 mg/min.
b. Prepare an epinephrine infusion at 2 mcg/min.
c. Give a 250-mL IV fluid bolus of normal saline.
d. Prepare a dopamine infusion at 5 to 20 mcg/kg/min.
CHAPTER 10  Posttest   243

Questions 22 through 26 pertain to the following scenario.


Paramedics are called to a private residence for a 35-year-old woman who complained of dizziness and
feeling faint while preparing dinner. Her husband helped her to the couch and called 9-1-1.

_ ___ 22. Which of the following actions should occur while caring for this patient?
a. Obtain the patient’s vital signs.
b. Attach a pulse oximeter and cardiac monitor.
c. Perform a primary and secondary assessment.
d. Ask a qualified team member to insert an advanced airway.

____ 23. The patient is alert and oriented only to person and place. Her husband indicates
that she has a history of hypertension, high cholesterol, and is prediabetic. Her skin
is cool and moist and her lungs are clear. The patient’s BP is 72/44 mm Hg and
her ventilatory rate is 16 breaths/min. The patient’s SpO2 on room air is 96%. The
cardiac monitor reveals the rhythm here (Fig. 10.2). This rhythm is:
a. Junctional rhythm.
b. Sinus bradycardia.
c. Ventricular escape rhythm.
d. Pulseless electrical activity (PEA).

II

Fig. 10.2 (From Aehlert, B. [2004]. ECG study cards, St. Louis, Mosby.)

____ 24. Which of the following ACLS treatment guidelines should be used during the
initial management of this patient?
a. Bradycardia with a pulse
b. Tachycardia with a pulse
c. PEA
d. ACSs

____ 25. An IV has been established. A glucose check reveals a value of 130 mg/dL. Your
next action should be to:
a. Complete a reperfusion checklist.
b. Give atropine IV and reassess.
c. Make preparations for transcutaneous pacing (TCP).
d. Administer sedation and perform synchronized cardioversion.

____ 26. As patient care continues, you ask your team members to watch for signs of improved
cardiac output, which may include which of the following?
a. Increased heart rate
b. Improved mental status
c. Improved skin condition
d. Increased blood pressure
244 CHAPTER 10  Posttest

Questions 27 through 43 pertain to the following scenario.


A 52-year-old man is complaining of palpitations that came on suddenly after walking up a short flight
of stairs. His symptoms have been present for about 20 minutes.

_ ___ 27. Which of the following actions should occur while caring for this patient?
a. Establish IV access.
b. Prepare to defibrillate.
c. Obtain the patient’s vital signs.
d. Attach a pulse oximeter and cardiac monitor.
e. Administer sedation and prepare for synchronized cardioversion.

____ 28. The patient is alert and oriented to person, place, time, and event. He denies chest
pain and is not short of breath. His skin is warm and dry, breath sounds are clear,
BP is 144/88 mm Hg, heart rate is 186 beats/min, and ventilatory rate is
18 breaths/min. His SpO2 on room air is 97%. The cardiac monitor reveals the
rhythm shown here (Fig. 10.3). This rhythm is:
a. Sinus tachycardia
b. Narrow-QRS tachycardia
c. Monomorphic ventricular tachycardia (VT)
d. Polymorphic VT

Fig. 10.3 (From Aehlert, B. [2004]. ECG study cards, St. Louis, Mosby.)

_ ___ 29. Based on the information provided, how would you categorize this patient’s condition?
a. Asymptomatic
b. Stable but symptomatic
c. Symptomatic and unstable
d. Pulseless

____ 30. Which of the following medications would be appropriate to consider in this
situation?
a. NTG
b. Sotalol
c. Atropine
d. Morphine
e. Amiodarone
f. Procainamide

____ 31. A 49-year-old man is found unresponsive, not breathing, and pulseless. The cardiac
monitor reveals monomorphic VT. The two most important actions in the manage-
ment of this patient are:
a. Performing CPR.
b. Performing defibrillation.
c. Promptly inserting an advanced airway.
d. Performing synchronized cardioversion.
e. Administering resuscitation medications.
CHAPTER 10  Posttest   245

____ 32. A 35-year-old woman presents with a narrow-QRS tachycardia with a ventricular
rate of 210 beats/min. She is stable but symptomatic. If the patient remains stable,
but the rhythm persists, which of the following actions would be appropriate to
consider?
a. Establish vascular access.
b. Attempt a vagal maneuver.
c. Administer adenosine rapid IV push.
d. Perform synchronized cardioversion.
e. Administer sedation and defibrillate with 360 J.

____ 33. An 84-year-old man presents with an acute onset of altered mental status. The car-
diac monitor shows the rhythm here (Fig. 10.4). The patient’s BP is 58/30 mm Hg
and his ventilatory rate is 14 breaths/min. His skin is cool, moist, and pale. His
SpO2 on room air is 95%. An IV has been established.
On the basis of the information provided, your best course of action will be to:
a. Give amiodarone 300 mg IV push.
b. Give epinephrine 1 mg IV bolus and reassess.
c. Give atropine 0.5 to 1 mg IV and prepare for TCP.
d. Closely observe the patient’s response to the interventions provided.

II

Fig. 10.4 (From Aehlert, B. [2004]. ECG study cards, St. Louis, Mosby.)

____ 34. Persistent hypotension (i.e., a systolic BP of less than 90 mm Hg) after the return
of spontaneous circulation (ROSC) may necessitate the use of which of the
following?
a. Dopamine
b. Epinephrine
c. Fluid boluses
d. Procainamide
e. Isoproterenol
f. Norepinephrine

_ ___ 35. Synchronized cardioversion:


a. Is used only for atrial dysrhythmias.
b. Delivers a shock during ventricular depolarization.
c. Delivers a shock between the peak and end of the T wave.
d. Is used only for rhythms with a ventricular rate of less than 60 beats/min.

____ 36. A 48-year-old man became unresponsive shortly after presenting with nausea and
generalized chest discomfort. You observe gasping breathing and are unsure if you
feel a pulse. You should now:
a. Call for help and begin chest compressions.
b. Wait until breathing stops and then check again for a pulse.
c. Begin chest compressions only if you are certain a pulse is absent.
d. Observe the patient for 2 minutes, then reassess his breathing and pulse.
246 CHAPTER 10  Posttest

____ 37. If a patient wakes from sleep or is found with symptoms of a stroke, the time of
onset of symptoms is defined as the time:
a. Of awakening.
b. The patient retired for sleep.
c. The patient was last known to be symptom-free.
d. The patient was last seen by a healthcare professional.

____ 38. The preferred method used to verify the proper placement of an endotracheal
tube is:
a. Obtaining a chest radiograph.
b. Using continuous waveform capnography.
c. Auscultating the presence of bilateral breath sounds.
d. Observing adequate chest rise with positive pressure ventilation.

____ 39. A patient is unresponsive with spontaneous ventilations at a rate of 4 per minute.
Chest movement is barely visible with each breath. A pulse is present. Which of
the following oxygen delivery devices would be most appropriate to use in this
situation?
a. A nasal cannula at 4 L/min
b. A simple face mask at 6 L/min
c. A nonrebreather mask at 12 L/min
d. A BMD with a reservoir at 15 L/min

____ 40. A 53-year-old woman is unresponsive. The cardiac monitor initially showed a
narrow-QRS tachycardia at 220 beats/min. Her BP was 50 mm Hg by
palpation and her ventilatory rate was 10 breaths/min. Supplemental oxygen ther-
apy was initiated and an IV established before the patient’s collapse. You promptly
delivered a synchronized shock. Reassessment reveals the patient is not breathing
and has no pulse. The cardiac monitor now reveals the rhythm shown in Fig. 10.5.
What course of action should you take at this time?
a. Defibrillate immediately.
b. Perform CPR for 2 minutes and then prepare to defibrillate.
c. Place an advanced airway and then begin TCP.
d. Press the “Sync” control and deliver another synchronized shock.

Fig, 10.5 (From Aehlert, B. [2013]. ECGs made easy, ed 5, St. Louis, Mosby.)

_ ___ 41. Which of the following are recommended when performing defibrillation?
a. Check for a pulse immediately after defibrillation to determine next steps.
b. Visually check and ensure that everyone is clear of the patient before shock
delivery.
c. Quickly remove transdermal medication patches or ointment from the patient’s
chest before the procedure.
d. All team members with the exception of the chest compressor should clear the
patient as the machine charges.
CHAPTER 10  Posttest   247

____ 42. Your general impression of a 78-year-old woman reveals that her eyes are closed,
she is not moving, you can see no rise and fall of her chest or abdomen, and her
skin color is pale. When you arrive at the patient’s side, you confirm that she is
unresponsive. Your best action in this situation will be to:
a. Open her airway and give two breaths.
b. Apply an automated external defibrillator.
c. Assess breathing and determine whether she has a pulse.
d. Prepare the necessary equipment to insert an advanced airway.

____ 43. A 62-year-old man is presenting with signs and symptoms suggesting a stroke.
Realizing that the benefits of IV or intraarterial fibrinolytics are time-dependent,
which of the following is the most important question that you should ask this
patient, family, and/or bystanders?
a. “When did your symptoms begin?”
b. “When did you last see a physician?”
c. “Do you have a history of hypertension?”
d. “What were you doing when your symptoms began?”

Questions 44 through 46 pertain to the following scenario.


You and your team are summoned to the room of a 45-year-old man who was admitted yesterday for
abdominal pain and lower gastrointestinal (GI) bleeding. A laboratory technician says that she found
the patient unresponsive in bed upon entering the room minutes ago. An IV is in place. Emergency
equipment, including a biphasic manual defibrillator with TCP capability, is available.

____ 44. Your assessment reveals that the patient is unresponsive, apneic, and pulseless.
Which of the following actions should occur at this time?
a. Direct your team to begin CPR.
b. Attach the patient to a cardiac monitor.
c. Ask a team member to prepare to perform TCP.
d. Instruct a team member to administer epinephrine IV.

____ 45. The cardiac monitor reveals the rhythm shown here (Fig. 10.6). Which of the
following actions should be taken at this time?
a. Obtain a cardiology consult.
b. Request bedside sonography.
c. Clear the patient and defibrillate.
d. Ask a team member to administer adenosine IV.
e. Ask a team member to administer epinephrine IV.
f. Ask a qualified team member to insert an advanced airway.

Fig. 10.6 (From Aehlert, B. [2004]. ECG study cards, St. Louis, Mosby.)
248 CHAPTER 10  Posttest

____ 46. Considering the information available so far, which of the following is the most
likely cause of the patient’s cardiac arrest?
a. Hypoxia
b. Hypothermia
c. Hypovolemia
d. Hyperkalemia
e. Toxic ingestion

Questions 47 and 48 pertain to the following scenario.


An 89-year-old man is complaining of a “racing heart.” He states his symptoms began while playing a
card game with friends. He had an MI 15 years ago and a coronary artery bypass graft 5 years ago. His
BP is 140/90 mm Hg and his ventilatory rate is 16 breaths/min. Breath sounds are clear and his tidal
volume is adequate. His SpO2 on room air is 88%.

_ ___ 47. Based on the information provided, supplemental oxygen:


a. Is unnecessary at this time.
b. Is indicated and should be delivered using a nasal cannula.
c. Is indicated for all patients who are experiencing a tachycardia.
d. Should ideally be administered only after placement of an advanced airway.

____ 48. You have started an IV and placed the patient on the cardiac monitor, which
reveals a regular, monomorphic, wide-QRS tachycardia. Which of the following
statements is true with regard to the management of this patient?
a. The patient is unstable. Sedate the patient and defibrillate as quickly as possible.
b. The patient is stable. Administration of IV verapamil is recommended for
termination of the rhythm.
c. The patient is stable. Administration of IV adenosine can be used as a therapeutic
and diagnostic maneuver.
d. The patient is unstable. Because there are recognizable QRS complexes on the
monitor, synchronized cardioversion should be performed.

Questions 49 and 50 pertain to the following scenario.


A 65-year-old man is complaining of a sudden onset of dizziness. He is awake, alert, and diaphoretic.
The patient states that his symptoms began 45 minutes ago while cleaning his garage.

____ 49. The patient denies chest pain, shortness of breath, and nausea. His breath sounds
are clear, his BP is 78/50 mm Hg, and his ventilatory rate is 18 breaths/min. His
SpO2 on room air is 96%. The cardiac monitor reveals the following rhythm
(Fig 10.7). This rhythm is:
a. Junctional rhythm.
b. Sinus bradycardia.
c. Third-degree atrioventricular (AV) block.
d. Second-degree AV block (2:1 AV block).

Fig. 10.7 (From Aehlert, B. [2004]. ECG study cards, St. Louis, Mosby.)
CHAPTER 10  Posttest   249

_ ___ 50. An IV is in place. Your best course of action will be to:


a. Defibrillate immediately.
b. Administer atropine 0.5 to 1 mg IV.
c. Administer amiodarone 300 mg IV.
d. Administer vasopressin 40 units IV.

POSTTEST    ANSWERS 
1. A, B, C. Elements of high-quality CPR include maximizing the CCF (i.e., at least 60%) by
minimizing pauses in chest compressions, performing chest compressions at a rate of 100 to 120
compressions per minute, compressing the adult chest at a depth of at least 2 inches (5 cm), not to
exceed 2.4 inches (6 cm), avoiding leaning on the chest between compressions to allow full chest
wall recoil, changing chest compressors about every 2 minutes (or earlier if warranted), and avoid-
ing excessive ventilation.

2. A, B, C, D, E. Each member of the resuscitation team must have clear roles and responsibilities,
must know his or her limitations, must be knowledgeable about current resuscitation algorithms,
must be practiced in resuscitation skills, and must be prepared to question other team members if
an action is about to occur that may be inappropriate. To avoid information overload and to help
ensure that what is said by the team leader is what is heard by the team members, the team leader
should state his or her instructions one at a time using terms that are known and shared by all team
members. The team member’s name should be used, if known. A good team leader values his or
her team members, fosters an environment in which team members feel comfortable speaking up,
and encourages a respectful exchange of ideas. Team members must clearly acknowledge when
procedures and medications are complete. Because there are often a large number of persons pres-
ent during a code, sidebar conversations among team members that can be distracting to other
team members must be avoided.

3. A, B. Rescuers delivering chest compressions should rotate every 2 minutes, or sooner if fatigued.
Ideally, the switch should be accomplished in less than 5 seconds. Administer oxygen using a
BMD. The CCF is the proportion of time spent performing chest compressions during a cardiac
arrest. Experts recommend a CCF of at least 60% for optimal outcome. Defibrillation is not
indicated because asystole is a nonshockable rhythm.

4. D. A BMD that is used with supplemental oxygen set at a flow rate of 10 to 15 L/min with an
attached reservoir delivers about 90% to 100% oxygen to the patient.

5. B. If peripheral IV access is unsuccessful during cardiac arrest, consider an intraosseous infusion


before considering placement of a central line.

6. C. Epinephrine should be given as soon as feasible after the onset of cardiac arrest associated with
a nonshockable rhythm. Amiodarone and lidocaine are antiarrhythmics that can be used for
shockable cardiac arrest rhythms (i.e., ventricular fibrillation [VF], pulseless ventricular tachycardia
[pVT]). They are not indicated for nonshockable cardiac arrest rhythms (i.e., asystole and PEA).
The efficacy of vasopressin is similar to that of epinephrine, and research has shown no benefit
from administering both drugs when compared with epinephrine administered alone. Atropine is
not recommended in the management of cardiac arrest.

7. A, B, D. During a debriefing, each member of the code team has an opportunity to engage in
honest dialogue to gain understanding and to identify lessons learned in a nonpunitive environ-
ment. An opportunity is provided for each team member to reflect on what they did, when they
did it, how they did it, why they did it, and how they can improve. The facilitator uses open-ended
questions to encourage discussion and listens to the team members describe their perceptions of
their behaviors. The actions of the team can be compared with current resuscitation algorithms,
250 CHAPTER 10  Posttest

professional standards, institution policies, best evidence, and local protocols to enhance under-
standing and support discussion during the analysis phase of the debriefing.

8. C, D, E. ACSs are a group of conditions that are caused by an abrupt reduction in coronary artery
blood flow. UA and NSTEMI are often grouped as non–ST-elevation acute coronary syndromes
(NSTE-ACS) because ECG changes associated with these conditions usually include ST-segment
depression and T-wave inversion in the leads that face the affected area. With NSTEMI, cardiac
injury is sufficient to result in the release of biomarkers (i.e., troponin elevation). The diagnosis is UA
when biomarkers are not elevated. Complete blockage of a coronary artery may result in STEMI.

9. B. Although it is not yet known if this patient is experiencing a STEMI or if he is a candidate for
reperfusion therapy, obtaining a prehospital 12-lead ECG is associated with shorter reperfusion
times and lower mortality rates from STEMI.

10. A, B, D, E. Frequent assessment of the patient’s mental status, vital signs, and oxygen saturation
levels is important, and continuous ECG monitoring is essential. Supplemental oxygen is war-
ranted if the patient is having difficulty breathing, having obvious signs of heart failure or shock,
or if his or her oxygen saturation level declines to less than 90%. Establish IV access, obtain a
targeted history and physical examination, and consider the possibility of other conditions that
mimic acute MI. Give aspirin if no contraindications are present. NTG should not be administered
until a 12-lead ECG has been obtained and a right ventricular infarction has been ruled out.
In addition, obtain initial cardiac biomarkers, electrolytes, and coagulation studies, and obtain a
portable chest radiograph.

11. B, C, D, F. Examples of conditions that can mimic acute MI include aortic dissection, acute peri-
carditis, acute myocarditis, panic attack, and pulmonary embolism. Bell’s palsy and complicated
migraine are examples of conditions that can mimic an acute stroke.

12. A, B, C, E. Patients who are experiencing an ACS who are most likely to present atypically include
older adults, diabetic individuals, women, patients with impaired renal function, patients with
dementia, patients with prior cardiac surgery, and patients during the immediate postoperative
period after noncardiac surgery.

13. A, B, E. The patient’s initial 12-lead ECG should be reviewed, and the patient classified into one
of three categories: STE, ST depression, or normal or nondiagnostic ECG.

14. B. The patient’s 12-lead ECG shows STE in leads I, aVL, and V2 to V6, ST depression in III and
aVF, and tall T waves in V2 to V5.

15. B. Two leads are contiguous if they look at the same or adjacent area of the heart or if they are
numerically consecutive chest leads. V2, V3, and V4 are numerically consecutive chest leads.

16. C. The patient’s 12-lead ECG shows STE in lead I, aVL, and V2 through V6. Because these leads
view the lateral and anterior surfaces of the left ventricle, an extensive anterolateral infarction is
suspected.

17. A, B, D. The left anterior descending (LAD) artery supplies the anterior wall of the heart through
its diagonal branches, and the anterior two-thirds of the interventricular septum through its septal
perforating branches. Because the LAD artery supplies a large portion of the left ventricle, a block-
age in this area can lead to complications including left heart failure and cardiogenic shock and
may cause dysrhythmias, including premature ventricular contractions, atrial flutter, or AFib.

18. B. Patients with STE in two or more contiguous leads are classified as having a STEMI and
should be evaluated for immediate reperfusion therapy.

19. A, B. Aspirin should be administered as soon as possible after symptom onset to patients with
suspected ACSs (if there are no contraindications). NTG relaxes vascular smooth muscle and
CHAPTER 10  Posttest   251

decreases myocardial oxygen consumption. An oral beta blocker should be started within the first
24 hours after hospitalization in the absence of contraindications to beta blockade. Because of the
increased risk of major adverse cardiac events associated with the use of NSAIDs, these drugs
should not be initiated in the acute phase of care and should be discontinued in patients using
them before hospitalization.

20. A. Patients with ischemic discomfort should receive up to three doses of sublingual NTG
tablets or spray at 3- to 5-minute intervals until chest discomfort is relieved or hypotension
limits its use. Nitrates are contraindicated in patients with hypotension (systolic BP below 90 mm
Hg or 30 mm Hg or more below baseline), marked bradycardia or tachycardia, phosphodiesterase
inhibitor use within the previous 24 to 48 hours, or suspected right ventricular infarction.

21. C. Your best course of action will be to place the patient supine and give a 250-mL IV fluid bolus
of normal saline. Reassess his BP (and other vital signs) and breath sounds after administration.

22. A, B, C. Ask a team member to attach a pulse oximeter and cardiac monitor, and obtain the pa-
tient’s baseline vital signs while you perform (or direct a team member to perform) a primary and
secondary assessment. Based on the information provided so far, insertion of an advanced airway
is premature at this time.

23. B. The rhythm shown is a sinus bradycardia.

24. A. Because the patient has a pulse, the monitor shows a sinus bradycardia, and the patient is
experiencing symptoms that appear to be related to the bradycardia, the bradycardia with a pulse
(i.e., symptomatic bradycardia) algorithm should be used in this situation.

25. B. Atropine is the drug of choice for symptomatic bradycardia if the dysrhythmia persists despite
adequate oxygenation and ventilation. TCP or a dopamine or epinephrine IV infusion are alternative
therapies that may be used if atropine is ineffective; however, sinus bradycardia typically responds well
to atropine administration. Synchronized cardioversion is used to treat rhythms that have a clearly
identifiable QRS complex and a rapid ventricular rate in a patient who has a pulse and signs of
hemodynamic compromise. It is not used for symptomatic bradycardias.

26. A, B, C, D. After administering atropine, look for signs of improved cardiac output, such as
improvement in the patient’s mental status, improved skin condition (e.g., color, temperature,
moisture), an increase in heart rate, and an increase in BP.

27. A, C, D. Ask your team members to attach a pulse oximeter and cardiac monitor, and obtain the
patient’s baseline vital signs. At this point, we do not yet know what the patient’s cardiac rhythm
is, so it is premature to administer sedation in preparation for synchronized cardioversion. Defibril-
lation is not indicated because the patient is awake (i.e., he has a pulse).

28. C. The rhythm shown is monomorphic VT.

29. B. On the basis of the patient’s history and physical findings, the patient is stable but symptomatic
at this time.

30. B, E, F. Procainamide would be appropriate to consider in this situation. Acceptable alternatives


include amiodarone and sotalol. The patient has no complaint of chest discomfort, so NTG and
morphine are not indicated. Atropine is not indicated because the patient has a tachycardia, not a
symptomatic bradycardia.

31. A, B. CPR and defibrillation are the most important treatments for the patient in cardiac arrest
associated with pVT or VF. Insertion of an advanced airway and administration of resuscitation
medications are of secondary importance. Although synchronized cardioversion may be used
in the treatment of an unstable patient in monomorphic VT with a pulse, it is not indicated
for pVT.
252 CHAPTER 10  Posttest

32. A, B, C. Establish vascular access and obtain a 12-lead ECG (but do not delay therapy to obtain
the ECG). Attempt a vagal maneuver to try to stop the rhythm or slow conduction through the
AV node. If vagal maneuvers fail, antiarrhythmic medications should be tried. Adenosine is the
drug of choice. The initial dose is 6 mg rapid IV push over 1 to 3 seconds. If no response within
1 to 2 minutes, give 12 mg rapid IV push. The 12 mg dose may be repeated once in 1 to 2 minutes.
If needed, calcium blockers or beta-blockers may be used to slow the ventricular rate. If the tachy-
cardia is sustained and causing persistent signs of hemodynamic compromise, synchronized car-
dioversion should be performed. Defibrillation is not indicated for a patient who is stable with a
narrow-QRS tachycardia.

33. C, D. The rhythm shown is a third-degree AV block at a rate of about 30 beats/min. This patient
is clearly symptomatic and needs immediate emergency care. A reasonable course of action will be
to administer atropine 0.5 to 1 mg IV and prepare for TCP if the response to atropine is ineffec-
tive. Closely observe the patient’s response to the interventions provided. Amiodarone is not indi-
cated in the management of a symptomatic bradycardia. Although epinephrine may be used in the
management of a symptomatic bradycardia, it is given as a continuous IV infusion, not as an IV
bolus.

34. A, B, C, F. IV fluid boluses can be considered if the patient is hypotensive after the ROSC. Vaso-
pressor IV infusions, such as epinephrine, dopamine, or norepinephrine, may be started if necessary
and titrated to achieve a minimum systolic BP of less than 90 mm Hg. Isoproterenol is an alterna-
tive agent that is primarily used to increase heart rate in a patient with a symptomatic bradycardia.
Because it is not a vasopressor, it is not used to treat hypotension. Procainamide is an antiarrhyth-
mic used to treat many atrial and ventricular dysrhythmias. Procainamide is not a vasopressor and
because an adverse effect of procainamide administration is hypotension, it would not be used to
treat hypotension.

35. B. Synchronized cardioversion is a type of electrical therapy in which a shock is “timed” or


“programmed” for delivery during ventricular depolarization (i.e., the QRS complex). It is indi-
cated in the management of a patient who is exhibiting serious signs and symptoms related to a
tachycardia. Because the machine must be able to detect a QRS complex to “sync,” synchronized
cardioversion is used to treat rhythms that have a clearly identifiable QRS complex and a rapid
ventricular rate (such as some narrow-QRS tachycardias and VT).

36. A. Gasping breathing is not effective breathing. After recognizing that the patient is unresponsive
and is not breathing normally, activate the emergency response system and check for a pulse for
no more than 10 seconds. If you do not feel a pulse or unsure if you feel a pulse during that period,
begin chest compressions.

37. C. For a patient with symptoms of stroke on awakening, the time of onset is assumed to be the
time the patient was last known to be symptom-free before retiring (last known well time). If a
patient had mild impairments but then had worsening over the subsequent hours, the time the first
symptom began is assumed to be the time of onset.

38. B. In addition to clinical assessment, continuous quantitative waveform capnography is recommended


as the most reliable method for confirmation and monitoring of endotracheal tube placement.

39. D. Remember that an open airway does not ensure adequate ventilation. This patient’s breathing
is inadequate as evidenced by his rate and depth of ventilations. The patient with inadequate
breathing requires positive-pressure ventilation with supplemental oxygen. Of the choices listed,
the only device that can provide positive-pressure ventilation is the BMD. If readily available, an
oropharyngeal airway should be inserted before beginning bag-mask ventilation (if the patient
does not have a gag or cough reflex).

40. A. The cardiac monitor shows VF. Appropriate care includes immediate defibrillation. TCP can be
used in the management of symptomatic bradycardia; it is not used for cardiac arrest rhythms, such
CHAPTER 10  Posttest   253

as VF. Synchronized cardioversion is not used to treat disorganized rhythms (e.g., polymorphic
VT) or those that do not have a clearly identifiable QRS complex (e.g., VF).

41. B, C, D. Be sure that high-quality CPR is continued as the defibrillator is readied for use. While
CPR continues, instruct a team member to expose the patient’s chest and to remove any transder-
mal medication patches or ointment from the patient’s chest, if present. All team members except
for the chest compressor should clear the patient as the machine charges. When the defibrillator
is charged, the chest compressor should immediately clear the patient. Call “Clear!” Look around
you (360 degrees) to be sure that everyone—including you—is clear of the patient, the bed, and
any equipment that is connected to the patient. Be sure oxygen is not flowing over the patient’s
chest. Press the “Shock” control to defibrillate the patient. Release the “Shock” control after the
energy dose has been delivered. Instruct the team to resume chest compressions.

42. C. After forming a general impression, approach the patient and assess her level of responsiveness.
If the patient is unresponsive, quickly determine whether the patient is not breathing (or only
gasping) and simultaneously check for a pulse for up to 10 seconds. If there is no pulse, begin chest
compressions.

43. A. The time from onset of stroke symptoms until treatment is a key factor for success of any
therapy. The earlier the treatment for stroke, the more favorable the results are likely to be.

44. A, B. Ask your team to begin high-quality CPR. Ask another team member to attach the patient
to a cardiac monitor. Although an IV is in place, it is premature to administer epinephrine until
you know if the patient’s cardiac rhythm is shockable or nonshockable. If it is a shockable rhythm,
defibrillation takes priority over epinephrine administration. TCP is not appropriate at this time
because: (1) the patient’s cardiac rhythm is not known, and (2) TCP is not used in the management
of cardiac arrest rhythms.

45. A, B, E, F. Sinus tachycardia is present on the monitor, but the patient has no palpable pulse. The
clinical situation is PEA. Reasonable actions at this time include obtaining a cardiology consult
and requesting bedside sonography to differentiate between true PEA and pseudo-PEA. While
CPR continues, additional actions that should be taken include administering IV epinephrine and
asking a qualified team member to insert an advanced airway. Defibrillation is not indicated for
PEA. Adenosine, which is an antiarrhythmic used in the management of stable narrow-QRS
tachycardias, is not used in the management of cardiac arrest rhythms.

46. C. Based on the patient’s admission complaint of abdominal pain and history of lower GI bleeding,
hypovolemia is the most likely cause of the patient’s cardiac arrest.

47. B. Administer supplemental oxygen as needed to maintain an oxygen saturation of 95% to 98%.
Because it is generally better tolerated than a mask, it is reasonable to use a nasal cannula. If the
patient’s oxygen saturation does not adequately improve with the use of the cannula, it may be
necessary to switch to oxygen delivery by mask. Because the patient’s breathing is adequate, ad-
vanced airway placement and positive pressure ventilation is not necessary at this time; however, if
the patient becomes unresponsive or his breathing becomes inadequate, administer oxygen by
positive-pressure ventilation.

48. C. On the basis of the information provided, the patient is stable at this time. Administration of
IV adenosine can be used as a therapeutic and diagnostic maneuver. Verapamil is a calcium blocker
and should only be given to patients with a narrow-QRS tachycardia. It should not be given to
patients with a wide-complex tachycardia. Because electrical therapy is used for unstable patients,
neither synchronized cardioversion nor defibrillation is indicated for this patient. If he were un-
stable, synchronized cardioversion would be used because the patient has a pulse and there are
recognizable QRS complexes on the monitor. Defibrillation would be performed if the rhythm
observed was polymorphic VT, pulseless monomorphic VT, or VF.
254 CHAPTER 10  Posttest

49. D. The rhythm shown is second-degree AV block (2:1 AV block).

50. B. The drug of choice for symptomatic bradycardia is atropine. The dose is 0.5 to 1 mg, which
may be repeated every 3 to 5 minutes to a maximum dose of 3 mg. If atropine is ineffective,
second-line interventions include TCP or administering dopamine, epinephrine, or isoproterenol
IV. Defibrillation, amiodarone, and vasopressin are not indicated in the management of symp-
tomatic bradycardia.
G L O S S A RY
Absolute bradycardia  A heart rate of less than 60 beats/min.
Accessory pathway  An extra bundle of working myocardial tissue that forms a connection between
the atria and ventricles outside the normal conduction system.
Anginal equivalent  Symptom other than chest pain or discomfort resulting from myocardial isch-
emia that may occur either alone or in combination in a patient with ischemic heart disease (IHD).
Arteriosclerosis  A chronic disease of the arterial system characterized by abnormal thickening and
hardening of the vessel walls.
Atherosclerosis  A form of arteriosclerosis in which the thickening and hardening of the vessel walls
are caused by a buildup of fat-like deposits in the inner lining, specifically of large- and middle-
sized muscular arteries.
Automated external defibrillation  The placement of pads on a patient’s chest and interpretation of
the patient’s cardiac rhythm by the defibrillator’s computerized analysis system. Depending on the
type of automated external defibrillator (AED) used, the machine will deliver a shock (if a shock-
able rhythm is detected) or instruct the operator to deliver a shock.
Automated external defibrillator (AED)  A machine with a sophisticated computer system that ana-
lyzes a patient’s heart rhythm using an algorithm to distinguish shockable rhythms from nonshock-
able rhythms and provide visual and auditory instructions to the rescuer to deliver an electrical
shock, if a shock is indicated.

Bundle of His  Fibers located in the upper portion of the interventricular septum that conduct an
electrical impulse through the heart.

Capnography  The graphic representation of the exhaled carbon dioxide concentration in respiratory
gases.
Capnometer  A monitoring device that measures and numerically displays the concentration of
exhaled carbon dioxide.
Cardiopulmonary (cardiac) arrest  The absence of cardiac mechanical activity, which is confirmed by
the absence of a detectable pulse, unresponsiveness, and apnea or agonal, gasping breathing.
Cardiovascular collapse  A sudden loss of effective blood flow that is caused by cardiac and/or periph-
eral vascular factors that may reverse spontaneously (e.g., syncope) or require interventions (e.g.,
cardiac arrest).
Chain of Survival  The essential elements of a system of care that are necessary to link the victim of
sudden cardiac arrest with survival.
Conduction system  A system of pathways in the heart composed of specialized electrical (i.e.,
pacemaker) cells.

Defibrillation  Delivery of an electrical current across the heart muscle over a very brief period to
terminate an abnormal heart rhythm; also called unsynchronized countershock or asynchronous counter-
shock because the delivery of current has no relationship to the cardiac cycle.

Electrocardiogram (ECG)  A recording of the heart’s electrical activity from the body surface that
appears on ECG paper as specific waveforms and complexes.

His-Purkinje system  Portion of the conduction system consisting of the bundle of His, bundle
branches, and Purkinje fibers. 255
256 Glossary

Lead  A record (i.e., tracing) of electrical activity between two electrodes.

Manual defibrillation  The placement of adhesive pads on a patient’s chest, interpretation of the
patient’s cardiac rhythm by a trained healthcare professional, and the healthcare professional’s
decision to deliver a shock (if indicated).

Oxygenation  The process of getting oxygen into the body and to its tissues for metabolism.

Relative bradycardia  A term that refers to a situation in which a patient’s heart rate may be more than
60 beats/min but, physiologically, the patient needs a more rapid rate (as in hypovolemia) and is
unable to increase their heart rate because of sinoatrial (SA) node disease, beta-blockers, or other
medications.
Respiration  The exchange of oxygen and carbon dioxide during cellular metabolism.

Stroke system of care  A comprehensive spectrum of care that addresses all aspects of stroke care in
a coordinated fashion.
Sudden cardiac death (SCD)  A natural death of cardiac cause that is preceded by an abrupt loss of
consciousness within 1 hour of the onset of an acute change in cardiovascular status; sudden cardiac
arrest is a term commonly applied to such an event when the patient survives.
Supraventricular arrhythmias  Rhythms that begin in the sinoatrial (SA) node, atrial tissue, or the
atrioventricular (AV) junction.
Symptomatic bradycardia  A term used to describe a patient who experiences signs and symptoms of
hemodynamic compromise related to a slow heart rate.
Synchronized cardioversion  A type of electrical therapy during which a shock is timed or pro-
grammed for delivery during ventricular depolarization (i.e., the QRS complex).

Transient ischemic attack (TIA)  A transient episode of neurologic dysfunction caused by focal brain,
spinal cord, or retinal ischemia, without acute infarction.

Ventilation  The mechanical movement of gas or air into and out of the lungs.
INDEX
Note: Page numbers followed by f indicate figures, t indicate tables, b indicate boxes.

A Acute stroke, 225. See also Stroke Anticoagulant therapy, 206–207, 207t
Absolute bradycardia, 98 Acute stroke teams (ASTs), 225 Antiplatelet therapy, 205–206, 205f, 205t
Accelerating angina, 187 Acute stroke–ready hospitals (ASRHs), 225b Area at risk, myocardial necrosis, 181, 182f
Accessory pathway, 60 Adenosine, 74–79t, 128–129, 128b Arteriosclerosis, 175
ACE inhibitors. See Angiotensin-converting Adhesive pad Aspirin, 205–206, 205f, 206t
enzyme (ACE) inhibitors position, 65–66, 66f ASRHs. See Acute stroke–ready hospitals (ASRHs)
ACLS. See Advanced cardiac life support (ACLS) size, 65 Asthma, 89
Active compression-decompression CPR ADP P2Y12 receptor inhibitors, 205t, 206 ASTs. See Acute stroke teams (ASTs)
(ACD-CPR), 9 Advanced airway, 54–56, 54b Asynchronous countershock. See Defibrillation
Acute coronary syndromes (ACSs), 173–218 endotracheal tube placement, confirming, Asystole, 147–148, 147f, 148f
algorithm, 200f 55–56, 55b cardiac arrest rhythms, 146, 147–148, 147f, 148f
analgesic therapy in, 201–202, 202t Advanced cardiac life support (ACLS), 1, 20, 22b, characteristics, 147
anticoagulant therapy in, 206–207, 207t 60, 145 P-wave, 147, 148f
atypical presentation of, 184 Advanced life support, effective, 5 ventricular, 147
cause of, 175 AED. See Automated external defibrillator (AED) AT. See Atrial tachycardia (AT)
clinical features, 182–185 AFib. See Atrial fibrillation (AFib) Atherosclerosis, 175, 176f
coronary artery blockage, 182 Agency for Healthcare Research and Quality Atherosclerotic plaques, 176–177, 176f, 177f
diagnosis (AHRQ), 17 Atrial fibrillation (AFib), 118f, 123–124
cardiac biomarker results, 185–186, 185f, 186t AHA. See American Heart Association (AHA) asymptomatic, 124
electrocardiogram findings of, 185f, 186 AHA/ASA. See American Heart Association/ ECG characteristics of, 123–124, 124f
imaging studies for, 198 American Stroke Association (AHS/ASA) with a rapid ventricular response, 124, 124f
initial management of, 198–208 AICSs. See Acute ischemic coronary syndromes signs and symptoms, 124
emergency department, 199 (AICSs) treatment of, 129
pharmacologic therapies, 201–207 Airway(s) Atrial flutter, 123
prehospital, 198–199, 198b advanced, 54–56 anteroposterior electrode placement, 129
myocardial infarction management, 40–59 anticoagulation, 129
anatomic location of, 189–196, 190f, 191t manual, maneuvers for, 40–42, 42t ECG characteristics, 123, 123f
anterior infarction, 191–192, 191f, 192f oxygenation and ventilation assessing tools, signs and symptoms, 123
classification, 186–187 40, 41f treatment of, 129
diagnostic criteria for type 1 and type 2, primary assessment, 36, 37t Atrial tachycardia (AT), 118–119, 118f
186–187, 187t Airway adjuncts, 44 ECG characteristics of, 118, 119f
inferior infarction, 194, 194f, 195f Alert, Responds to verbal stimuli, Responds to focal, 118, 119
inferobasal infarction, 194–196, 195f, 196f painful stimuli, Unresponsive (AVPU), 36 multifocal, 118, 118f, 122–123, 123f
lateral infarction, 192, 193f Ambulance drone, 5, 5f nonsustained rhythm, 119
lead aVR, 197 American Heart Association (AHA), 154 paroxysmal supraventricular tachycardia, 118
right ventricular infarction, 196, 197f American Heart Association/American Stroke sustained rhythm, 119
ST elevation, 187–197, 188f, 189f Association (AHS/ASA), 225, 226b Atrioventricular blocks, 100–103
unstable angina and non–ST-elevation, 187 Amiodarone (Cordarone), 74–79t, 128–129, 128b, first-degree, 100, 101f
myocardial necrosis, 181, 182f 154–155 second-degree, 100
non–ST-elevation MI (NSTEMI), 181 Analgesic therapy, 201–202, 202t 2:1, 102, 102f
pathophysiology of Angina type I, 101, 101f
nonobstructive atherosclerosis, 177 microvascular, 180–181, 180b type II, 101–102, 102f
obstructive atherosclerosis with systemic pectoris, 174, 178, 178b third-degree, 102–103, 103f
inflammation, 176–177, 176f, 177f Prinzmetal, 180, 180b Atrioventricular bundle. See Bundle of His
obstructive atherosclerosis without systemic stable, 179, 179b Atrioventricular nodal reentrant tachycardia (AVNRT)
inflammation, 177 unstable, 187 atypical, 119–120
patient history, 183, 183b variant, 180 characteristic of, 118
OPQRST history, 183 Anginal equivalents, 178 ECG characteristics of, 119–120, 119f
SAMPLE history, 183 examples of, 178b mechanism for, 119
physical examination of, 184–185 Angiotensin-converting enzyme (ACE) inhibitors, orthodromic, 118f
stable angina, 179, 179b 202 pathways, 119
unstable angina, 181 Anterior circulation strokes, 220 premature atrial complexes, 119
variant angina, 180 Anterior myocardial infarction, 191–192, 191f, 192f signs and symptoms, 120
Acute ischemic coronary syndromes (AICSs), 173 Anteroseptal myocardial infarction, 191–192, 192f treatment, 120
Acute ischemic stroke, 219–238, 220b, 221t Antiarrhythmics medications, 154–155, 159–160 typical, 119
257
258 Index

Atrioventricular node (AVN), 118f C Chain of Survival (Continued)


Atrioventricular reentrant tachycardia (AVRT) CAD. See Coronary artery disease (CAD) links in, 3
antidromic, 118f Calcium blockers, 74–79t, 128, 129, 151t out-of-hospital, 3–6
characteristic of, 118 Calcium channel blockers, 202, 203t advanced life support, effective, 5
mechanism for, 119 Capnography, 11f, 39b, 40 delivery of high-quality CPR, 4–5
normal and abnormal conduction pathways, Capnometers, digital, 40 early recognition, emergency response
120, 120f Cardiac arrest, 2–3 activation, and prevention, 3–4
signs and symptoms, 121 algorithm, 150–156, 150b, 151t, 152f postcardiac arrest care, 5–6
Wolff-Parkinson-White (WPW) pattern, chest compressions during, 153b intraarrest and, 7
121, 121f heart rhythms in, 145–172 recovery, 6
Atropine sulfate, 70, 74–79t, 105 Hs and Ts, causes of, 150, 151t CHD. See Coronary heart disease (CHD)
Atypical presentation, of acute coronary syndromes, in-hospital, 6b, 7, 150, 150b Chest compressions
184 out-of-hospital, 4, 149, 149b, 149f, 149t during cardiac arrest, 153b
Augmented limb leads, 61, 61f phases of, 2–3, 3t fraction, 7–8
Automated external defibrillation, 64–65 pregnancy and, 157–158 high-quality, 8, 8b
Automated external defibrillator (AED), 4, 5, 5f primary, 2–3 manual, 10
Automatic atrial tachycardia, 118 refractory, 156 mechanical device, 6, 9–10
Automatic cells. See Pacemaker cells sudden, 3 Chest leads, 61
AVNRT. See Atrioventricular nodal reentrant Cardiac Arrest Management station, 22b augmented limb, 61, 61f
tachycardia (AVNRT) Cardiac arrest rhythms, 145–172 frontal plane, 61, 61f
AVPU (Alert, Responds to verbal stimuli, Responds asystole, 147–148, 147f, 148f and heart surfaces viewed, 61, 62t
to painful stimuli, Unresponsive), 36 pulseless electrical activity, 148, 148b, 148f horizontal plane, 61, 61f
AVRT. See Atrioventricular reentrant tachycardia ventricular fibrillation, 147, 147f posterior surface of heart, 61, 62f
(AVRT) ventricular tachycardia, 146, 146f right ventricle evaluation, 61, 62f
Cardiac biomarkers, 185–186, 185f, 186t standard limb, 61, 61f
B Cardiac catheterization laboratory (CCL), 161–162 Chest radiograph
Bad news, conveying, resuscitation efforts and, Cardiac output, 98 for COVID-19 patient, 39
27–29, 28b, 28f Cardiac tamponade, 148b, 151t for ischemic chest discomfort, 179
Bag-mask device (BMD), 51, 51f Cardiac troponins, 185, 186, 186t Chronic obstructive pulmonary disease (COPD),
supplemental oxygen, with/without, 51 Cardiopulmonary arrest, 2–3. See also Cardiac arrest 89, 90
ventilating with, 52–53, 52f Cardiopulmonary resuscitation (CPR) Cincinnati Prehospital Stroke Severity Scale
Bag-mask resuscitator, 51 bystander, 4 (CPSSS), 226b
Bag-mask ventilation (BMV), 49f, 51–53, 90, 91 cough, 73 Circle of Willis (COW), 220
advantages, 52b dispatcher-assisted, 4 Circulation, primary assessment of, 37t, 38
disadvantages, 52b high-quality, 7–10, 7b Circumflex (CX) artery, 190f, 192
oxygen delivery, 51 active compression-decompression CPR, 9 blockage of, 192, 193f
single-rescuer, 52, 52f barriers to effective, 9 Classic angina, 179, 179b
troubleshooting, 53 chest compressions, 8, 8b Closed-loop communication, resuscitation team,
Bag-valve-mask device. See Bag-mask device delivery in out-of-hospital Chain of Survival, 4–5 21, 25
(BMD) elements of, 7–8, 8b CMD. See Coronary microvascular disease (CMD)
Basic life support (BLS), 1 impedance threshold device, 9 “Coarse” ventricular fibrillation, 147
assessment, 35–36, 35b, 36b indication, 7 Code director, 20
cardiopulmonary resuscitation and, 4–5 mechanical chest compression devices, 9–10 Code team. See Resuscitation team
Berman airway, 44, 45f monitoring during, 10–12 Cold debriefings, 26
Beta blockers, 74–79t, 117, 128, 129, 130, 151t, patient’s response, 10, 11f Colorimetric capnometer, 40
202, 203t performance in-hospital Chain of Survival, 7 Colorimetric exhaled carbon dioxide detector, 40, 41f
Bilevel positive airway pressure (BiPAP), 53–54, rescuer’s performance, 11–12, 12f Common bundle, 60
54b techniques and adjuncts of, 9–10 Communication, resuscitation team
Biphasic defibrillation, 65 ventilating during cardiac arrest, 8, 8b closed-loop, 21, 25
Biphasic defibrillators, 66, 150–153 telecommunicator, 4 team, effectiveness of, 24–25, 24f
Biphasic waveforms, 65 Cardiovascular care, after ROSC, 159–162 Complexes of electrocardiogram, 63, 64t
BLS. See Basic life support (BLS) emergent coronary angiography, 161–162 Comprehensive stroke centers (CSCs), 225–226,
BMD. See Bag-mask device (BMD) extracorporeal cardiopulmonary resuscitation, 161 225b
Bradycardias, 97. See also specific types Cardiovascular collapse, 2–3 Computed tomography (CT)
absolute, 98 Cardiovascular disease (CVD), 174 intracerebral hemorrhage, 223
algorithm of, 104–105, 104f Cardioversion, synchronized, 68–70, 126, 127, 128, noncontrast brain, 228, 229t
patient assessment, 103–104, 103b 129, 130 subarachnoid hemorrhage, 222
relative, 98 electrical therapy, 68 Conducting cells. See Pacemaker cells
sinus, 98–99, 99f possible complications, 70 Conduction system, 60
symptomatic, 103–104, 103b procedure, 68–70, 69f accessory pathway, 60
therapeutic interventions, 104–105, 104f, 105b treating rhythms, 68 bundle branches, right and left, 60
Brain, arterial blood supply to, 221f Caregivers, assistance by resuscitation team, 29, 29f bundle of His, 60
Brain imaging, 228 Carotid sinus, location of, 127f of heart, 60, 60f
Breathing Carotid sinus massage (CSM), 127, 127f, 128f His-Purkinje system, 60
management of, 40–59 Carotid territory strokes, 220 sinoatrial node, 60, 60f
primary assessment of, 37–38, 37t Chain of Survival, 3–7 Continuous positive airway pressure (CPAP), 53–54
adequate and inadequate, 37, 38t defibrillation Contributing risk factors, 91, 176, 180
chest expansion, 37 prompt, 7 COPD. See Chronic obstructive pulmonary disease
for COVID-19, 37–38 rapid, 5, 5f (COPD)
noninvasive positive pressure ventilation, 38 definition, 3 Coronary angiography
oxygenation, 37–38 in-hospital, 6–7 emergent, 161–162
respiration, 37 early recognition and prevention, 6 for ischemic chest discomfort, 179
ventilation, 37 high-quality CPR performance, 7 Coronary arteries, 174, 175f
Bundle branch block (BBB), 121 notification and response, 6 Coronary artery disease (CAD), 125b, 174, 179b
Bundle of His, 60 recovery, 7 prevalence of, 161
Index 259

Coronary heart disease (CHD), 174–177, E Flutter waves, 123


175f, 176f E-C clamp technique, 49–50, 49f Focal atrial tachycardia (focal AT), 118, 119
Coronary microvascular disease (CMD), 174 E-C grip, 49–50, 49f Forming a general impression, 35, 36b
Coronary perfusion pressure, 7–8, 154 ECG. See Electrocardiogram (ECG) French suction catheters, 43
Coronavirus disease 2019 (COVID-19), 6b, 10b, Ectopic atrial tachycardia (ectopic AT), 118 Frontal plane leads, 61, 61f
39, 156b EDDs. See Esophageal detector devices (EDDs)
known/suspected patients Effective team performance, 23–27 G
activating rapid response system, 20b decision making, 24f, 26 Gasping, 2–3
acute decompensation risk in pregnancy, 157 situation awareness, 24f, 25–26 Gastric distention, 50b
advance directives, 150b task management, 24, 24f Glasgow Coma Scale (GSC), 222b
advanced airways, 55 team communication, 24–25, 24f Global positioning system (GPS), 5
assessment and care for, 89–90 teamwork, 23–24, 24f Glycoprotein (GP) IIb/IIIa receptor inhibitors,
BMD with high-efficiency particulate air filter Electrical capture, 71–72, 71f 205f, 205t, 206
for, 38 Electrical therapies, 64–73 Guedel airway, 44, 45f
breathing assessment for, 37–38 defibrillation, 64–68
in cardiac arrest, 55 synchronized cardioversion, 68–70 H
chest radiograph for, 39 transcutaneous pacing, 70–72 Head tilt–chin lift, 37t, 41–42, 41f, 42t
definitive care for, 5 Electrocardiogram (ECG) Heart attack, 3–5, 183
management in prone position, 91 atrioventricular block Heart disease, 1, 1f
mechanical compression device, 153, 155 2:1, 102f Heart rhythms. See also specific types
nasal cannula over surgical mask for, 37–38 first-degree, 101f in cardiac arrest, 145–172
perimortem delivery preparations, 158 third-degree, 102–103, 103f asystole, 147–148, 147f, 148f
personal protective equipment, 89–90 type I, second-degree, 101f pulseless electrical activity, 148, 148b, 148f
personnel essential for patient care, 21 type II, second-degree, 102f ventricular fibrillation, 147, 147f
PPE for rescuer, 157 bradycardia algorithm, 104–105, 104f ventricular tachycardia, 146, 146f
respiratory complaint in, 35 chest leads for, 61 nonshockable, 155, 155b
responding to resuscitation team, 20b components of, 63 shockable, 150–155, 153b, 153f
special resuscitation situations, 157 recording, 63, 63f Heart, surfaces of, 189, 190f
tight face-to-mask seal for, 52 segments and intervals, 64t Hemorrhagic stroke, 220
risk factors for, 4 waveforms and complexes, 64t HFNC systems. See High-flow nasal cannula
CPR. See Cardiopulmonary resuscitation (CPR) conduction system, 60 (HFNC) systems
CPSSS. See Cincinnati Prehospital Stroke Severity electrodes, location for, 61, 62f High-efficiency particulate air (HEPA) filter, 157
Scale (CPSSS) junctional escape rhythm, 99f High-flow nasal cannula (HFNC) systems, 57
Crash cart, 23 15-lead, 61b High-quality cardiopulmonary resuscitation, 1–16
C-reactive protein (CRP), 177 multiple-lead, 61b High-sensitivity CRP (hs-CRP) test, 177
Crescendo angina, 187 polymorphic ventricular tachycardia (PMVT), His-Purkinje system, 60
Crew resource management (CRM), 21 124–125, 124f Horizontal plane leads, 61, 61f
CSCs. See Comprehensive stroke centers (CSCs) sinus bradycardia, 98, 99f Hot debriefings, 26
CT. See Computed tomography (CT) ventricular escape rhythm, 100, 100f Hs and Ts
CUS (Concern, Uncomfortable, and Safety) tool, 25 Electrocardiography paper, 63, 63f cause of dysrhythmia using, 105, 105b
CVD. See Cardiovascular disease (CVD) Electrodes reversible causes of cardiac arrest, 150, 151t,
CX artery. See Circumflex (CX) artery locations for electrocardiogram recording, 61, 62f 154, 158
Cyclooxygenase inhibitors, 205t transcutaneous pacing, 72 Hydrogen ion, 151t
Electromechanical dissociation, 148 Hyperacute T waves, 187
D Embolic ischemic stroke, 223 Hypercarbic respiratory failure, 89
Debriefing, of team members, 26–27, 26f Emergency medical services (EMS), 3–4, 198 Hyperkalemia, 151t
Decision making, effective team performance on stroke care, 226–227 Hyperventilation, 10
element, 24f, 26 Emergent coronary angiography, 161–162 Hypokalemia, 151t
Defibrillation, 64–68, 158 EMS. See Emergency medical services (EMS) Hypothermia, 151t, 158, 161
adhesive pad Endotracheal intubation, 54 Hypoxemic respiratory failure, 89
position, 65–66, 66f Endotracheal tube (ETT), 43 Hypoxia, 151t
size, 65 confirm correct placement of, 55–56
automated external, 64–65 Epiglottis, 44, 45f I
definition, 64 Epinephrine, 74–79t, 105, 153, 153b, 154 ICH. See Intracerebral hemorrhage (ICH)
manual, 64–65 Esophageal detector devices (EDDs), 40, 55–56 Idioventricular rhythm, 100. See also Ventricular
monophasic vs. biphasic, 65 Esophageal intubation detectors, 55 escape rhythm
procedure for, 66–68, 67f Exposure, assessment of patient’s, 37t, 38 IHCA. See In-hospital cardiac arrest (IHCA)
prompt, 7 Extracorporeal cardiopulmonary resuscitation, 161 IHD. See Ischemic heart disease (IHD)
rapid, 5, 5f Extracorporeal membrane oxygenation (ECMO), 161 Impedance threshold device (ITD), 9
selected energy, 66 Extraglottic airway devices, 54 Inferior myocardial infarction, 194, 194f, 195f
transthoracic impedance, 65–66, 65b Inferior wall infarction, 194, 194f
Defibrillation waveforms, 65 F Inferobasal myocardial infarction, 194–196, 195f,
biphasic waveform, 65 Family notification, resuscitation team, 27–29 196b, 196f
monophasic waveform, 65 Feedback devices, 11 Inferobasal wall myocardial infarction, 194
Defibrillators. See also Defibrillation Fibrinolytics, 207 In-hospital cardiac arrest (IHCA), 20
automated external, 4, 5, 5f “Fine” ventricular fibrillation, 147 adult surviors, 150
biphasic, 66, 150–153 First-degree atrioventricular blocks, 100, 101f causes of, 150
monophasic, 47, 66 Flexible suction catheters, 43, 44f in hospital setting, 150
Delta wave, 120f, 121 Flow rates initial rhythm associated with, 150
Digital capnometers, 40 of bag-mask device, 51 In-hospital Chain of Survival, 6–7
Digoxin, 151t of mouth-to-mask ventilation, 49 Intermediate coronary syndrome, 187
Disability, primary assessment of, 37t, 38 of nasal cannula, 56 Intervals, 63, 64t
Dispatcher-assisted bystander CPR (DA-CPR), 4 of nonrebreather mask, 59 Intracerebral hemorrhage (ICH), 220, 222f
Dopamine, 74–79t, 105 of partial rebreather mask, 58 active bleeding, 223
Dysrhythmias, 159–160 of simple face mask, 57 causes of, 222
260 Index

Intracerebral hemorrhage (Continued) MAT. See Multifocal atrial tachycardia (MAT) Neurologic care, after ROSC, 162
head computed tomography, 223 Mechanical capture, 72 NIHSS. See National Institutes of Health Stroke
neurologic evaluations, 223 Mechanical chest compression devices, 6, 9–10 Scale (NIHSS)
score, 223 Medical emergency team (MET), 18 Nitroglycerin (NTG), 201, 201t
signs and symptoms, 222 Mega Code station, 22b NIV. See Noninvasive ventilation (NIV)
Intravenous fibrinolysis, 230 MET. See Medical emergency team (MET) Noninvasive pacing. See Transcutaneous pacing
Introduction, Situation, Background, Assessment, MI. See Myocardial infarction (MI) (TCP)
Recommendation, and Repeat/Read back Microvascular angina, 180–181, 181f Noninvasive positive pressure ventilation (NPPV),
(ISBARR), 18b Ministroke. See Transient ischemic attack (TIA) 38, 53–54, 54b, 90, 91
Inverted T waves, 179, 189 Modified jaw thrust, 42 Noninvasive ventilation (NIV), 53
Irregular tachycardias, 122–125 Monomorphic ventricular tachycardia, 122, 122f Nonrebreather mask, 58f, 59, 59b, 59t
atrial fibrillation, 123–124, 124f Monophasic defibrillation, 65 Nonshockable rhythms, 155, 155b
atrial flutter, 123, 123f Monophasic defibrillators, 47, 66 Non–ST-elevation acute coronary syndromes
multifocal atrial tachycardia, 122–123, 123f Monophasic waveform, 65 (NSTE-ACSs), 181
polymorphic ventricular tachycardia (PMVT), Mouth-to-mask ventilation, 48–50, 49f, 51t, 90 Nonsustained rhythm, 119
124–125, 124f MRI. See Magnetic resonance imaging (MRI) Nonsustained ventricular tachycardia, 121–122, 121f
signs and symptoms, 122 Multifocal atrial tachycardia (MAT), 118, 118f, Nonverbal communication, 23, 24f
treatment of, 129–130 122–123 Norepinephrine, 74–79t
ISBARR (Introduction, Situation, Background, ECG characteristics of, 122–123, 123f NPPV. See Noninvasive positive pressure ventilation
Assessment, Recommendation, and Repeat/ in older adults, 123 (NPPV)
Read back), 18b treatment of, 123, 129 NSTE-ACSs. See non–ST-elevation acute coronary
Ischemic heart disease (IHD), 174, 178–181 Myocardial blood flow, 9, 147 syndromes (NSTE-ACSs)
anginal equivalents symptom, 178, 178b Myocardial cells, 65, 178, 179b, 181, 185 NTG. See Nitroglycerin (NTG)
cause of, 178 Myocardial infarction (MI) Nursing supervisor, 23
clinical features, 178 anatomic location of, 189–196, 190f, 191t
diagnostic studies, 178–179 anterior, 191–192, 191f, 192f O
microvascular angina, 180–181, 181f inferior, 194, 194f, 195f Occlusive stroke, 223, 224f
stable angina, 179, 179b inferobasal, 194–196, 195f, 196f OHCA. See Out-of-hospital cardiac arrest
variant angina, 180 lateral, 192, 193f (OHCA)
Ischemic penumbra, 223–224, 224f right ventricular, 196, 197f Opioid overdose, 36, 38, 90, 91, 156–157
Ischemic stroke, 222f, 223–224 anteroseptal, 191–192, 192f Oral airway, 44
acute, 219–238 classification, 186–187 Oropharyngeal airway (OPA), 44–46
asymptomatic, 223 diagnostic criteria for type 1 and type 2, Berman airway, 44, 45f
caused by, 223 186–187, 187t complications, 44, 45f, 46f
embolic, 223 lead aVR, 197 Guedel airway, 44, 45f
heritable disease, 223 ST-elevation, 187–197, 188f, 189f insertion method, 46, 46f
lacunar, 223 hyperacute T waves, 187 J-shaped plastic device, 44
thrombotic, 223, 224f QRS changes, 189, 189f size determination, 44, 45f
Isoproterenol, 74–79t, 105 ST-segment changes, 188–189, 189b in varying sizes, 44
ITD. See Impedance threshold device (ITD) T-wave inversion, 189 Out-of-hospital cardiac arrest (OHCA), 54, 149
transmural, 174 adult nontraumatic
J unstable angina and non–ST-elevation, 187 initial arrest rhythms, 149, 149b
J point, 64t Myocardial injury, 181, 186, 186t, 188–189 survival rates for, 149, 149t
Jaw thrust, 42, 42f, 42t Myocardial ischemia, 173, 178, 178b, 187, 188–189, emergency medical service, 149
Junctional escape rhythm, 99, 99f 188f, 191 in home/residence, 149, 149f
Out-of-hospital Chain of Survival, 3–6
L N Oxygen delivery devices, 56–59
Lacunar infarcts, 223 Naloxone, 74–79t, 91, 91b bag-mask ventilation, 51–53
Lacunar strokes, 223 Narrow-QRS tachycardias, 116–121 nasal cannula, 56–57, 56b, 56f, 59t
LAD artery. See Left anterior descending (LAD) atrial tachycardia, 118–119, 119f nonrebreather mask, 58f, 59, 59b, 59t
artery atrioventricular nodal reentrant tachycardia partial rebreather mask, 58–59, 58b, 58f, 59t
Lateral myocardial infarction, 192, 193f (AVNRT), 119–120, 119f simple face mask, 57, 57b, 57f, 59t
Lead aVR, 197 atrioventricular reentrant tachycardia (AVRT), Oxygenation, 37–38, 157
Leads. See Chest leads 120–121, 120f pulse oximetry, 39b
Left anterior descending (LAD) artery, 191–192 sinus tachycardia, 116–117, 117b, 117f ROSC and, 158–159
Left bundle branch block (LBBB), 184 supraventricular tachycardias, 117–121, 117b, 118f tools for assessing, 40
Left main coronary artery (LMCA), 197 treatment for, 127–128
Lidocaine (Xylocaine), 74–79t, 154–155, 159–160 adenosine, 128, 128b P
Life support, advanced. See also Basic life support (BLS) beta blockers, 128 PAC. See Premature atrial complex (PAC)
effective, 5 carotid sinus massage (CSM), 127, 127f Pacemaker cells, 60
Lipid management, 202, 204t diltiazem, 128, 128b Pad, adhesive
LMCA. See Left main coronary artery (LMCA) pharmacologic therapy, 128 position, 65–66, 66f
Los Angeles Motor Scale (LAMS), 226b postural modifications, 127, 128f size, 65
Valsalva maneuver, 127 Paroxysmal supraventricular tachycardia (PSVT),
M verapamil, 128, 128b 118, 119f
Magnesium, 155 Nasal airway, 46–48, 47f Partial rebreather mask, 58–59, 58b, 58f, 59t
for PMVT, 130 Nasal cannula, 56–57, 56b, 56f, 59t Patient assessment, 35
Magnesium sulfate, 74–79t Nasal prongs, 56 basic life support assessment, 35–36, 35b, 36b
Magnetic resonance imaging (MRI), 228, 230 Nasopharyngeal airway (NPA), 46–48 primary assessment, 36–39, 37t
Major adverse cardiac event (MACE), 202 insertion method, 47, 47f with respiratory complaint, 89–90
Manual airway maneuvers, 40–42, 42t placement of, 48 secondary assessment, 39, 39b
head tilt–chin lift, 37t, 41–42, 41f, 42t of proper size determination, 47, 47f tachycardias, 125, 125b
jaw thrust, 42, 42f, 42t in varying size, 47, 47f Patient history, of acute coronary syndromes,
Manual defibrillation, 64–65 National Institutes of Health Stroke Scale 183, 183b
Mask, eye protection, a gown, and disposable gloves (NIHSS), 223, 226b, 228, 230 OPQRST history, 183
(MEGG), 35 Negative T waves, 179 SAMPLE history, 183
Index 261

PCI. See Percutaneous coronary intervention (PCI) Purkinje fibers, 60, 60f Resuscitation therapies (Continued)
PEA. See Pulseless electrical activity (PEA) PVCs. See Premature ventricular complexes (PVCs) suctioning, 43, 43b, 43f, 44b, 44f
Percussion pacing, 73 pVT. See Pulseless ventricular tachycardia (pVT) advanced airways, 54–56, 54b
Percutaneous coronary intervention (PCI), 156, P-wave, asystole, 147–148, 148f oxygen delivery devices, 56–59
206–207 P2Y12 receptor inhibitors, 205, 205t, 206 pharyngeal airways, 44–48, 48t
Perimortem cesarean delivery (PMCD), 157, 158 positive pressure ventilation, 48–54
Personal protective equipment (PPE), 35, 89–90, Q vascular access, 73
150, 157, 158 QRS complexes, 64t Resuscitative efforts
Pharyngeal airways, 44–48, 48t of polymorphic ventricular tachycardia, 122, 124f, patient transfer, 156
nasopharyngeal airways, 46–48, 47f 130 special resuscitation situations, 156–158
oropharyngeal airways, 44–46, 45f, 46f of ventricular tachycardia, 122, 124f cardiac arrest and pregnancy, 157–158
Physical examination, of acute coronary syndromes, QT interval, 64t, 125, 130 known/suspected COVID-19, 157
184–185 known/suspected opioid overdose, 156–157
Pit crew approach, 21 support roles in, 23
R
PMCD. See Perimortem cesarean delivery (PMCD) Return of spontaneous circulation (ROSC),
Rapid Arterial oCclusion Evaluation (RACE), 226b
PMVT. See Polymorphic ventricular tachycardia 156, 156b
Rapid Response System (RRS), 18–20, 18f
(PMVT) Right chest leads, 61, 62f
barriers to activation of, 19, 19b
Pocket face mask, 48–49 Right coronary artery (RCA), 190f
calling criteria, 18–19, 19b
Pocket mask, 48–49, 49f blockage of, 101, 197f
concept of, 6
Point-of-care ultrasound (POCUS), 148, 148b, 151 inferior wall infarction, 194, 194f
resuscitation guidelines, 20
Polymorphic ventricular tachycardia (PMVT), inferobasal (posterior) infarction, 194, 195f
scoring systems, 18–19
124–125 lateral wall infarction, 192, 193f
use, 20
ECG characteristics, 124–125, 124f Right ventricular infarction (RVI), 196, 197f
Rapid response team (RRT), 18
QRS complexes of, 122, 130 Rigid suction catheters, 43, 43f
Rapid ventricular rates (RVRs)
signs and symptoms, 125 ROSC. See Return of spontaneous circulation (ROSC)
atrial fibrillation with, 124, 124f
treatment, 130 RRS. See Rapid Response System (RRS)
atrioventricular reentrant tachycardia and, 121
types of, 125 RRT. See Rapid response team (RRT)
dysrhythmias with, 117b
Positive pressure ventilation, 48–54 RVI. See Right ventricular infarction (RVI)
signs and symptoms, 125
bag-mask ventilation, 51–53, 51f, 52b RVRs. See Rapid ventricular rates (RVRs)
RCA. See Right coronary artery (RCA)
mouth-to-mask ventilation, 48–50, 49f, 51t
Reentrant tachycardias
noninvasive positive pressure ventilation,
atrioventricular nodal reentrant tachycardia S
53–54, 54b SAH. See Subarachnoid hemorrhage (SAH)
(AVNRT), 119–120, 119f
Postcardiac arrest care, 5–6, 158–162 SAMPLE (signs and symptoms, allergies,
atrioventricular reentrant tachycardia (AVRT),
algorithm, 158, 160f medications, past history, last oral intake,
120–121, 120f, 121f
body temperature monitoring, 162 events prior), 39, 39b, 125
Reentry, 119
cardiovascular care, 159–162 Saturation of peripheral oxygen (SpO2), 40
Refractory cardiac arrest, 156
insulin therapy, 162 SBAR (Situation, Background, Assessment, and
Relative bradycardia, 98
intraarrest and, 7 Recommendation), 18b
Relative refractory period, 68
neurologic care, 162 SCD. See Sudden cardiac death (SCD)
Renin-angiotensin-aldosterone system inhibitors,
oxygenation and ventilation, 158–159 Scout approach, 35
202, 204t
steroids, use of, 162 Secondary assessment, on patient, 39, 39b
Reperfusion therapies, 207–208
Postcardiac arrest syndrome, 158 Second-degree atrioventricular blocks, 100
Respiration, 37
components, 158, 159t 2:1, 102, 102f
Respiratory arrest, 89
Posterior chest lead, 62f type I, 101, 101f
therapeutic interventions for, 91
Posterior circulation strokes, 220 type II, 101–102, 102f
Respiratory compromise, patient with, 87–96, 88f
Posterior wall myocardial infarction, 194 Segments, 63, 64t
Respiratory distress
Postresuscitation support. See Postcardiac arrest care Selected energy, 66
causes of, 90
Poststroke depression (PSD), 231b Serum biomarkers, 179b
characteristics, 88
PPE. See Personal protective equipment (PPE) Serum cardiac markers, 105, 179b
signs of, 88, 88b
Pregnancy Severe sinus bradycardia, 98
therapeutic interventions for, 90, 91f
acute decompensation in COVID-19 pregnancy, SGAs. See Supraglottic airways (SGAs)
uncorrected, 88
157 Shockable rhythms, 150–155, 153f
Respiratory failure, 88
cardiac arrest and, 157–158 medications, 154–155
acute, 89
Preinfarction angina, 187 antiarrhythmic, 154–155
hypercarbic, 89
Premature atrial complexe (PAC), 119 vasopressors, 154
hypoxemic, 89
Premature ventricular complexes (PVCs), 121–122 Signs and symptoms, allergies, medications, past
signs of, 89, 89b
Preocclusive syndrome, 187 history, last oral intake, events prior
therapeutic interventions for, 91
Primary assessment, on patient, 36–39, 37t, 39b (SAMPLE), 39, 39b, 125
Resuscitation mask, 48–49
Primary cardiac arrest, 2–3 Simple face mask, 57, 57b, 57f, 59t
Resuscitation, phases of, 3t
Primary percutaneous coronary intervention, 207, Sinoatrial node, 118f
Resuscitation team, 20–23, 20b
208 Sinus bradycardia, 98–99, 99f
caregivers, assisting, 29
Primary stroke centers (PSCs), 225b Sinus tachycardia, 116–117, 117b
crew resource management, 21
Prinzmetal angina, 180, 180b causes of, 117, 117b
debriefing, 26–27
Procainamide, 74–79t, 128–129, 128b definition, 116–117
family notification, 27–29
PSCs. See Primary stroke centers (PSCs) ECG characteristics of, 116–117, 117f
conveying bad news, 27–29, 28b
Pseudo-pulseless electrical activity, 148 signs and symptoms, 117
goals, 20
PSVT. See Paroxysmal supraventricular tachycardia treatment for, 117
pit crew approach, 21
(PSVT) Situation awareness (SA), effective team
roles and responsibilities, 22–23, 22t
Public access defibrillation, 5 performance element, 24f, 25–26
support roles, 23
Pulmonary compliance, 53 Situation, Background, Assessment, and
team arrival, 21–22
Pulse ox. See Pulse oximeter Recommendation (SBAR), 18b, 156
Resuscitation therapies, 40
Pulse oximeter, 159 Soft suction catheters, 43
airway and breathing management, 40–59
Pulse oximetry, 37–38, 39b, 40 Sotalol, 74–79t, 128–129, 128b
ECG monitoring, 60–63
Pulseless electrical activity (PEA), 104–105, 148, Speaking up, in resuscitation effort, 25
electrical therapies, 64–73
148b, 148f SPIKES protocol, 28–29, 28b
medications in cardiovascular emergencies, 73,
Pulseless ventricular tachycardia (pVT), 5, 64 SpO2. See Saturation of peripheral oxygen (SpO2)
74–79t
262 Index

ST segment, changes of, with acute coronary Symptomatic bradycardia (Continued) Total ischemic time, 198
syndrome, 188–189, 189b isoproterenol for, 105 Tracheal tube, 43b
Stable angina, 179, 179b with a narrow-QRS complex, 105 placement, 40
Standard limb leads, 61, 61f signs and symptoms, 103–104, 103b Transcutaneous pacing (TCP), 70–72, 105
Standard mask, 57. See also Simple face mask Synchronized cardioversion, 68–70, 126, 127, 128, indications for, 70
Stress, warning signs of, 29, 29f 129, 130 limitations of, 72
Stroke, 219, 220b. See also specific types electrical therapy, 68 possible complications in, 72, 72b
anatomy review, 220, 220b, 221f, 221t possible complications, 70 procedure for, 70–72, 71f, 72b
anterior circulation, 220 procedure, 68–70, 69f Transient ischemic attack (TIA), 224
associated with atrial flutter, 129 treating rhythms, 68 Transitional zone, 223–224, 224f
carotid territory, 220 Syndrome X, 180–181, 181f Transmural myocardial infarction, 174
conditions mimicking, 228b Transthoracic impedance, 65–66, 65b
hemorrhagic, 220 Tricyclic antidepressants, 151t
occlusive, 223, 224f
T TTM. See Targeted temperature management (TTM)
T waves
posterior circulation, 220 12-lead ECG, 105, 180b
hyperacute, 187
types of, 220–224 acute ST-segment elevation, 161
inversion, 189
intracerebral hemorrhage, 222–223, 222b frontal and horizontal planes, 61
Tachycardias, 115–144
ischemic stroke, 222f, 223–224, 224f for ischemic chest discomfort, 179
algorithm for, 115, 126, 126f
subarachnoid hemorrhage, 221–222, 222f respiratory compromise management, 90
irregular, 122–125
transient ischemic attack, 224 STEMI, 198–199
narrow-QRS, 116–121, 118f
vertebrobasilar territory, 220 synchronized cardioversion and, 68
patient assessment, 125, 125b
warning signs of, 226b tachycardia, 125
signs and symptoms, 116
Stroke centers, 225–226, 225b VT vs. SVT, 122b
therapeutic interventions, 125–130, 127b
Stroke systems of care, 224–231, 225t
wide-QRS, 121–122, 122b
acute phase of, 225, 225b U
Tamponade, cardiac, 148b, 151t
emergency medical services on, 226–227 Unstable angina (UA), 187
Targeted temperature management (TTM), 162
hospital initial evaluation and treatment, 227–231 Unsynchronized countershock. See Defibrillation
Task management, 24, 24f
antiplatelet therapy, 231, 231b
TCP. See Transcutaneous pacing (TCP)
blood glucose test, 229
blood pressure, 229
TCPR. See Telecommunicator cardiopulmonary V
resuscitation (TCPR) Vagal maneuvers, 118, 125, 127, 128
diagnostic studies for, 228–229, 229t
TdP. See Torsades de pointes (TdP) Valsalvamaneuver, 127, 128f
electrocardiogram monitoring, 229
TE grip, 50, 50f Variant angina, 180
endovascular therapies, 231
Team(s) Vasopressin, 74–79t
history examination, 227–228, 228b
definition, 17 Vasopressors, 153, 154
intravenous fibrinolysis, 230
effectiveness, 17 Ventilation, 37
intravenous fluids, 228
key elements of effective team performance, ROSC and, 158–159
neurologic examination, 228
23–27 Ventilation face mask, 48–49
patient positioning, 230
rapid response systems, 18–20, 18b, 18f, 19b Ventricular asystole, 147
physical examination, 227–228
resuscitation, 20–23, 22t Ventricular escape rhythm, 100, 100f
temperature, 229
Team communication, 24–25, 24f Ventricular fibrillation (VF), 147
prehospital assessment and management, 227
Team leader, 20 coarse, 147
public education, 226, 226b
ACLS algorithms, 22b ECG characteristics of, 147
Subarachnoid hemorrhage (SAH), 221–222, 222f
in direct patient care activities, 21 fine, 147
Subendocardial area, 174, 175f
responsibilities in resuscitation team, 22–23, 22t Ventricular tachycardia (VT), 121–122, 146
Subepicardial area, 174, 175f
Team member, in resuscitation team in implantable cardioverter-defibrillator, 122
Suction catheters
airway, 22t monomorphic, 122, 122f, 146, 146f
flexible, 43, 44f
cardiopulmonary resuscitation, 22t nonsustained, 121–122, 121f
French, 43
communication, 24–25 vs. supraventricular tachycardias, 122b
passage of, 44, 45f
COVID-19 patient’s status to, 20b sustained, 121–122
rigid, 43, 43f
direct patient care activities, 21 syncope/near-syncope of, 122
soft, 43
support roles, 23 Ventricular tachydysrhythmias, 147f
tonsil tip, 43
and teamwork, 23 Verbal communication, 23, 24f
whistle tip, 43
vascular access and medication administration, Vertebrobasilar territory strokes, 220
Yankauer, 43
21, 22t VF. See Ventricular fibrillation (VF)
Suctioning, 43
Teamwork, 23–24, 24f Vital signs, assessment of patient’s, 37t, 38–39
possible complications, 43, 44b
Telecommunicator cardiopulmonary resuscitation VT. See Ventricular tachycardia (VT)
purpose of, 43
(TCPR), 4 “Vulnerable” plaque, 176–177
Sudden cardiac arrest, 3, 5
Temporary external pacing. See Transcutaneous
Sudden cardiac death (SCD), 3
pacing (TCP)
Support roles, in resuscitation team, 23
The Joint Commission (TJC), 225, 225b
W
Support staff, 23 Warm debriefings, 26
Thenar eminence (TE) technique, 50, 50f
Supraglottic airways (SGAs), 54 Warning stroke. See Transient ischemic attack
Therapeutic interventions
Supraventricular arrhythmias, 116 (TIA)
bradycardias, 104–105, 104f, 105b
Supraventricular tachycardias (SVT), 117–121, Waveforms, 63, 64t
for patient with respiratory compromise, 90–91,
117b biphasic, 65
91b, 91f
atrial tachycardia, 118–119, 119f defibrillation, 65
tachycardias, 125–130, 127b
examples of, 117, 118f monophasic, 65
Thienopyridines, 205f, 206
reentrant tachycardias, 119. See also Whistle tip suction catheters, 43
Third-degree atrioventricular block, 102–103, 103f
Atrioventricular nodal reentrant tachycardia Wide-QRS tachycardias, 121–122
Thrombectomy–capable stroke centers (TSCs),
(AVNRT); Atrioventricular reentrant treatment for, 128–129
225–226, 225b
tachycardia (AVRT) ventricular tachycardia, 121–122, 121f, 122f
Thrombosis, 151t
Sustained rhythm, 119 Wolff-Parkinson-White (WPW) pattern, 120f,
Thrombotic ischemic stroke, 223
SVT. See Supraventricular tachycardias (SVT) 121, 121f
TIA. See Transient ischemic attack (TIA)
Symptomatic bradycardia, 70, 97
Tidal volume, 37
atropine for, 105
causes, 105
Tonsil tip suction catheters, 43 Y
Torsades de pointes (TdP), 125 Yankauer catheters, 43

You might also like